Sei sulla pagina 1di 644

Comprehensive

Board examinations
HIGHLIGHT
SLIDES
OSCE
MCQs

OTOLARYNGOLOGY
Head and Neck Surgery
Dr.Aqeel Alyasiery First edition /2017
Comprehensive
Board examinations
for ENT ,H&N surgery

edited by
dr. Aqeel Ch.alyasiery
Specialist ENT surgeon
m.b.ch.b /md-cabs(orl-hns)

First edition/2017
‫`````‬

‫الرحيم‬ ‫الرمحن‬
‫بسم اهلل الرحمن الرحيم‬
‫اهلل‬ ‫بسم‬
‫قالو سبحنك العلم لنا اال ماعلمتنا انك انت‬
‫العليم الحكيم‬
‫صدق اهلل العلي العظيم‬

‫سورة البقرة اية ‪32‬‬


This book is organized into four chapters:

1- Highlight
2- SLIDES
3- OSCE
4- MCQs
I
Contributors

- Dr.sajad Alhelo
consultant ORL-HN surgeon
Chief of Arabic Board..Najaf center-Iraq
DLO-FICMS-MD.CABS (ORL-HNS)

-Prof.Dr.Ismail Zohdi
ORL-HNS department/Cairo university

-Dr.Sundus Al-Sedra
Specialist ENT surgeon/Al-Yarmuk hospital
II

DEDICATION

To my parents:
With out their abundant support,I would
never have succeeded in my life.

To my wife:
my life’s projects would not have been
possible without her patience ,love and
support.

Also to my three sons :


Dr.Ali ,Dr.Osama ,Hussein : they made me
proud.
III

To my teacher:
I sincerely would like to express my deep
gratitude to my teacher
Dr.sajad yonis Al-Helo (consultant ENT
surgeon) ,for his keen and support and great
advice to performing this book.
IV

Preface
-This first edition of comprehensive board examinations ENT ,head and
neck surgery has high yield Review for the Otolaryngology in-service
and Board Exams .undergoing ENT, head and neck surgery training to
find and understand both essential and advanced information, and
orient clinical decisions and surgical approaches.
-This book is organized into four chapters including ,
highlight ,OSCE, slids ,MCQs.
-This will prepare you for written and osce examinations in the field of
otolaryngology in a systematic and logical way, excelling on clinical
rounds and on board examinations is a skill that improves with
familiarity of oral testing formats and compartmentalizing your fund of
knowledge in an organized, easily accessible manner. When we studied
for our Board exam, we spent hours poring over textbooks and loss of
time to know the answer for many questions or searching the internet
for answers this book give you comprehensive informations for many
questions not needs long time to answers. I wish for a directed
Comprehensive ENT book to use as our source; brief enough to get
through in the 4-6 weeks leading up to the exam, not need to go back
to another source for more information ,it covers all the topics noted
above.
Thanks

Dr.Aqeel Al-yasiery
Specialist ENT surgeon
MD-CABS (ORL-HNS)
V

Abbreviations :

ABR : Auditory brainstem response.


AN : acoustic neuroma.
AC : Air conduction.
ACE :Angiotensin converting enzyme .
AOM : Acute otitis media.
AP :Action potential .
AFRS:allergic fungal rhinosionsitis.
BC :Bone conduction.
BAHA:Bone anchored hearing aid.
BPPV:Benign paroxysmal positional vertigo.
CI:cochlear implant.
CN:cranial nerve.
CNS:centeral nervous system.
CPA:cerebellopontine angle.
CPAP:continuous postive airways pressure.
CSF:cerebrospinal fluid.
CSOM:chronic suppurative otitis media.
CT:computed tomography.
CWD:canal wall dawon.
CHL:conductive hearing loss.
COME:chronic otitis media with effusion.
CRS:chronic rhinosinositis.
CF:cystic fibrosis.
CLNM:cervical lymph node metastasis.
DCR:Dacryocystorhinostomy.
DSA:digital subtraction angiography.
EAC:external auditory canal.
ECoG:electrocochleography.
EMG:electromyography.
ESR:erythrocyte sedimentation rate.
ETD:Eustachian tube disfunction.
ECS:extracapsular seprad.
ESS:endoscopic sinus surgery.
VI

FESS:functional endoscopic sinus surgery.


FNA:fine needle aspiration.
FN:facial nerve.
GJB2:gap junction beta 2.
GERD:gastroesophageal reflex disease.
HPV:human papilloma virus.
HSCC:horizontal semicercular canal.
IAM:internal auditory meatus.
IMRT:intensity modulated radiothrapy.
IHC:inner hair cell.
IJV:internal jugular vein.
JNA:juvenile nasal angiofibroma.
MRI:magnetic resonsnce imaging.
MRND:modified radical neck dissection
MTC:medullary thyroid carcinoma.
MOE:malignant otitis externa
NIHL:noise induced hearing loss.
NPC:nasopharyngeal carcinoma.
OAE:otoacoustic emission.
OME:otitis media with effusion.
OSA:obstructive sleep apnoea.
OW:oval window.
OHC:outer hair cell.
OS:otoseclerosis.
OCT:optical coherene tomography.
PTA:pure tone audiogram.
PET:postron emission tomography.
RND:radical neck dissection.
RND:radical neck dissection.
RRP:recurrent respitetory papillomatosis.
RT:Raiotherapy.
.
VII

SCM:sternocleidomastoid.
SP:summating potential.
SRT:speech reception threshold.
SRT:speech reception threshold.
SNHL:sensorineural hearing loss.
SSCD:superior semicircular canal dihescence.
TM:tympanic membrane.
TE:tracheoesophageal.
TLM:transoral laser microsurgery.
VC:vocal cord.
VS:vestibular schwanomma.
VHL:vertical hemilaryngectomy.
.
Table of contents

Contributors :-------------------------------- I
Dedication:----------------------------------- II-III
Preface :-------------------------------------- IV
Abbreviations-------------------------------- V-VII

Chapter 1 : Highlight
Part 1 :Ear-------- --------------------------1– 50
Part 2 :Nose-------------------------------- 51-78
Part 3 :Throat--------------------------------79-126
Part 4 :Head and neck-------------------- 127- 210

Chapter 2 : Slides---------------------------211-288

Chapter 3 :
OSCE ------------------------------------------290-474

Chapter 4 :
MCQs ---------------------------------------475–611
Answers of MCQs ------------------------612-615

References :--------------------------------- 616


Chapter one

Highlight
Highlight

EAR: 577
NOSE: 330
THROAT: 558
HEAD AND NECK: 1035
Part I

EAR
EAR
EAR
1

1-Absolutely contraindication to cochlear implantation would include


severe Cochlear dysplasia, absence of the cochlear nerve, and severe
ossific labyrinthitis.

2-High-pitched, nonpulsatile binaural tinnitus from presbycusis, noise


Exposure, or accompanied by a highfrequency sensorineural hearing loss
needs no imaging.

3-In the presence of a normal physical exam and CHL, pathologies to


investigate radiologically include a congenital cholesteatoma that cannot
be seen through the tympanic membrane, congenital stapes gusher,
enlarged vestibular aqueduct, congenital middle ear anomaly.

4-Acute labyrinthitis is a self-limited disease usually viral in etiology. It


presents with acute onset of hearing loss or vertigo.

5-Classic petrous apicitis manifests as a deep temporal headache, sixth


nerve palsy, and a draining ear (Gradenigo’s syndrome).

6-X-linked stapes gusher. In this anomaly, the absence of a bony


partition at the fundus of the IAC results in transmission of CSF pressure
to the inner ear structures resulting in “gushing” of endolymph at
stapedectomy.

7-Syndromes that may be associated with EAC atresia include the


Goldenhar and Treacher Collins syndromes, PierreRobin sequence,
Klippel–Feil anomaly, and hemifacial microsomia among others.

8-Prussak’s space is located in the epitympanum and is bounded


laterally by the pars flaccida and a bony spur called the scutum. The
medial wall is formed by the neck and head of the malleus.

9-If an air-bone gap did not exist, the loss was thought to be purely
sensorineural.
2

10-Human hearing ranges from the threshold of audibility, around 0 dB


HL, to the threshold of pain, around 140 dB HL. Normal conversational
speech occurs at around 40 to 50 dB HL, and the point of discomfort is
approximately 90 dB HL .

11-Aquino sign is the cessation of pulsation and blanching of the mass


in the middle ear with manual compression of the ipsilateral carotid
artery.

12-Brown sign is the cessation of tumor pulsation and tumor blanching


with positive pressure using the pneumatic otoscope;

13-Vestibular schwannomas (VS) (acoustic neuromas) are nerve sheath


tumors of the superior and inferior vestibular nerves (cranial nerve VIII).

14-Tuning fork tests should always be performed on patients with a


temporal bone fracture.

15-Battle sign, which is a postauricular ecchymosis resulting from


extravasated blood from the postauricular artery or mastoid emissary
vein.

16-The presence of stapedius reflex with low-frequency conductive


hearing loss should prompt radiological imaging of the inner ear to
exclude the possibility of dehiscence of the inner ear.

17-Vestibular neuronitis is the third most common cause of peripheral


vestibular vertigo after BPPV and Meniere disease.

18-A delayed onset of vertigo may be the result of a perilymphatic


fistula, an excessively long prosthesis, or labyrinthitis.in stapedotomy.

19-Fluoride reduces osteoclastic bone resorption and increases


osteoblastic bone formation.
3

20-One of the earliest signs of otosclerosis is an abnormal acoustic


reflex pattern often preceding the development of an air-bone gap.

21-The SRT is the lowest intensity level at which a patient can


correctly repeat 50% of common bisyllabic words such as “hotdog” or
“baseball.”

22-The ABR is used routinely to screen hearing of newborns, and for


intraoperative monitoring of hearing function during skull base surgery .

23-EOAEs can be used to assess cochlear function in patients who


present with sudden-onset idiopathic hearing loss.

24-Otoacoustic emissions (OAEs) are sounds generated in the cochlea


and recorded in the ear canal. They can be used to assess the status of
cochlear outer hair cells.

25-Stenger testing is strongly recommended for every patient with a


unilateral or asymmetric hearing loss.

26-Moller and Jannetta’s study also indicates that wave V is associated


primarily with the activation of the lateral lemniscus and not the inferior
colliculus as was previouslyconsidered.

27-It is believed that the presence of hydrops affects the elasticity of


the basilar membrane and contributes to the increased amplitude of the
SP relative to that of the AP. A relatively large SP/AP ratio is considered
diagnostic of endolymphatic hydrops.

28-One of the main clinical applications of ECOG is in the differential


diagnosis of hydropic conditions of the cochlea that may be associated
with Meniere disease or other diseases .

29-frequencies of 500 and 1000 Hz are most often used in acoustic


reflex decay tests.Abnormal reflex decay may be indicative of
retrocochlear disease.

30-Stapedius reflex measurement should be included routinely in the


evaluation of patients with facial paralysis.
4
31- Speech discrimination testing is an essential component of testing
to evaluate candidacy for a cochlear implant.
32-Threshold is defined as the lowest signal intensity at which multiple
presentations are detected 50% of the time.
33-Pure-tone bone-conduction thresholds provide auditory threshold
information when the cochlea is stimulated more or less directly, with
stimuli that bypass external- and middle-ear structures.
-Pure-tone air-conduction thresholds measure the function of the total
hearing system, including the external, middle, and inner ear.
34-Treatment for MOE : Ceftazidime given intravenously combined
with oral ciprofloxacin and topical aminoglycoside/steroid drops .
35-Bacterial and fungal cultures should be collected if MOE is
suspected. Erythrocyte sedimentation rate (ESR) is generally the only
laboratory abnormality, and it may be markedly elevated. ESR indicates
a state of inflammation in the body, and it can be used to follow
treatment response and recurrence.
36-MOE:The infection begins in the soft tissues of the EAC and
spreads to the skull base via the fissures of Santorini and to the
stylomastoid foramen and jugular foramen via the tympanomastoid
suture.
37-Malignant O E:Common findings on physical examination include
granulation tissue within the external auditory canal, preauricular and
auricular edema and erythema,tympanic membrane necrosis, and facial
nerve paralysis.
38-All patients with unilateral otitis media should have their
nasopharynx inspected for possible nasopharyngeal masses.
39-Schwabach test compares bone conduction of the patient with that
of the examiner.
40-Midline Weber result is referred to as “negative.” “Weber right” and
“Weber left” refer to the direction to which the sound lateralized.
41-Weber test is performed by placing the vibrating 512-Hz tuning
fork in the center of the patient’s forehead, at the bridge of the nose,
or on the central incisors with the patient’s teeth tightly clenched.
42-Tuning fork tests, usually done with a 512-Hz fork, allow the
otolaryngologist to distinguish between sensorineural and conductive
hearing loss .
5

43-Vascular masses should prompt consideration of middle ear glomus


tumor; the clinician may also note a Brown sign, in which the mass
blanches with pneumatic otoscopy.

44-White masses behind an intact tympanic membrane, most often in


the anterosuperior quadrant, are consistent with cholesteatoma.

45- Serous effusions often appear as amber fluid, sometimes with air-
fluid levels or air bubbles. Mucoid effusions will appear to be a dull gray
color,

46-Perforations and middle ear effusions are common causes for


immobile tympanic membranes.

47-Lacerations of EAC may be present in the setting of trauma, which


may include temporal bone fractures.

48-The presence of granulation tissue at the junction of the


cartilaginous and bony canal should raise concern for malignant otitis
externa,

49-Thin, atelectatic membrane draped closely over the under lying


middle ear structures may indicate adhesive otitis media

50-prominent radial blood vessels can indicate a chronic middle ear


effusion.

51-Preoperatively, the platelet count should be greater than 50,000/μL;


at levels below 20,000/μL, spontaneous bleeding may occur.

52-The pars flaccida of the tympanic membrane is critical to examine


for retraction pockets, which may develop into cholesteatomas.

53-Meniere’s disease(idiopathic).Meniere’s syndrome(secondery to


other disease like trauma,ear surgery,viral inf.,syphlis,autoimmune.

54-Remember that a negative Rinne for 256,512 and 1024 Hz indicates


a minimum AB gap of 15, 30, 45 dB respectively.
6

55-A negative Rinne indicates a minimum air - bone gap of 15—20 dB .

56-Single frequency sound is called a pure tone.

57-Recurrent episodes of acute otitis media treated by: period of 2–3


months of once daily prophylactic antibiotics, such as amoxicillin and
Grommet insertion

58-Carthart’s notch is an artefactual dip in bone conduction


(BC)/cochlear threshold at 2KHz seen in conductive hearing loss, which
resolves/improves on correction of the conductive defect, but may be
explained as follows. When attempting to test only cochlear function
with BC thresholds, some of the sound inadvertently also travels via the
tympanic membrane and ossicles to the inner ear. The natural
resonance frequency of the ear canal/TM/middle ear/ossicles is around
2KHz meaning this effect is greatest at this frequency.Therefore, if a
conductive hearing loss occurs, this will inadvertently also reduce the
apparent BC threshold across all frequencies but most markedly at
2KHz.

59-Masking is always through air conduction.never bone conduction.

60-The auditory brainstem implant makes direct contact with the dorsal
cochlear nucleus .

61- Low frequency SNHL more in menire’s dis.,benign intracranial


HT.basilar migraine, syphilis.

62-Sensorineural hearing loss can complicate radiation therapy and


cisplatin treatment for nasopharyngeal carcinoma in up to 30% of
patients.

63-Tinnitus retraining therapy is effective regardless of the etiology of


tinnitus.

64-The prevalence of tinnitus is approximately the same in children as


in adults.

.
7

65-One of the first indicators of age-related hearing loss is loss of


auditory sensitivity at frequencies higher than 8 kHz.

66-Atrophy of the stria vascularis is the predominant lesion of the aging


ear.

67-Vestibular suppressants should be avoided except for management


of acute vertigo.

68-The antigen target in autoimmune inner ear disease is a 68 kD inner


ear protein.

69-The vestibulo-spinal response to fistula testing is more sensitive


than the vestibulo-ocular response.

70-Diffuse gadolinium enhancement of the cochlea on magnetic


resonance imaging in apatient with sudden sensorineural hearing loss
indicates a leaky
blood-brain barrier.

71-The outer hair cells are not primarily transducers of sound.

72-Mutations in Cx26 are responsible for over one half of moderate-to-


profound hearing impairment in many world populations.

73-Mitochondrial hearing loss may be associated with diabetes mellitus.

74-Alport’s syndrome can be inherited as an X-linked or autosomal


disorder.

75-Usher’s syndrome is the most common autosomal recessive


syndromic hearing loss.

76-Goiter may be apparent at birth in Pendred’s syndrome, it typically


presents in midchildhood.

77-A total ossicular replacement prosthesis is used only when the


stapes superstructure is absent..
8

78-The most effective treatment for auricular hematoma is adequate


incision and drainage with suture-secured bolsters.

79-Electronystagmography and the rotational chair are primarily


confirmatory for the suspected site of lesion.

80-The most popular hearing aid style used by Children :behind the
ear.

81-Autologous costal cartilage is the gold standard for auricular


reconstruction .

82-The first stage of auricular reconstruction involves harvest of costal


cartilage and creation of an auricular framework. This usually occurs at
6 years of age or later.

83-Similar elevations in the SP/AP ratio have, however, been reported


in perilymph fistula, autoimmune inner ear disease, and superior
semicircular canal dehiscence .

84-Dorsal cochlear nucleus is the stimulation site for an auditory


brainstem.

85-Schuknecht identified four categories of presbycusis based on


clinical and histopathologic changes within the cochlea. They are
sensory, conductive, strial, and neural.

86-If the fistula involves one of the semicircular canals and if the
mastoid is small, a CWD mastoidectomy, leaving the matrix on the
fistula, is appropriate.

87-A prelingual deaf adult with no oral language is a poor cochlear-


implant

88-Vertical downbeating nystagmus suggests a disorder in the


brainstem.

89-BPPV is considered to be the most common cause of dizziness in


the elderly.
9

90-Most common ossicular abnormality in congenital aural atresia is


Fused malleus-incus.

91-OAEs are generally not detected in patients with middle ear


pathology .

92-The most significant risk factor for meningitis after temporal bone
fracture is duration of CSF leak.

93-Prediction of hearing levels is not possible by measuring OAEs.

94-Pendred syndrome is associated with abnormal iodine metabolism.

95-B2-transferrin. When present, a CSF leak is confirmed; however, a


negative test result does not exclude a diagnosis of CSF leak.

96-Vestibular evoked myogenic potentials (for diagnosing semicircular


canal dehiscence),

97-Evaluation under Frenzel goggles can include the head-shake test


and Valsalva-, hyperventilation-, or sound-induced nystagmus.

98-Rotary nystagmus may be more common with semicircular


dysfunction, and abnormal sensations of tilt or sudden drop attacks can
be seen with otolith dysfunction.

99-Pendred syndrome (thyroid organification defect and hearing loss,


Alport syndrome (renal disease and hearing loss, Jervell and Lange–
Nielsen syndromes(prolonged QT interval and hearing loss).

100-Cerebrospinal fluid leak (otorrhea with longitudinal fracture,


rhinorrhea with transverse).

101-Acute coalescent mastoiditis is the most common intratemporal


complication of otitis media.
.
10

102-Sudden sensorineural hearing loss is classified as unilateral


hearing loss of at least 30 dB in at least 3 frequencies occurring within
3 days.

103-Distortion product OAE:useful for testing for noise-induced hearing


loss and medication ototoxicity.

104-Internal auditory canal anomalies:Narrow (<3 mm) associated


with absent or hypoplastic CN VIII,, Widened (>10 mm) associated
with stapes gusher.

105-Schiebe (cochleosaccular aplasia), Alexander(cochlear duct


aplasia, especially affecting basal turn) dysplasia.

106- Contraindications CI: narrow IAC, cochlear nerve aplasia, Michel


aplasia.

107-Indications for tympanostomy tube:Recurrent AOM (>3 episodes


in 6 months, >4 episodes in 12 months), Bilateral COME >3 months,
unilateral COME >6 months, severe retraction pocket,

108-von Hippel Lindau syndrome, Associated with endolymphatic sac


tumors.

109-Aural atresia:Jahrsdoerfer grading system: 10-point grading


system, Six favorable for surgical intervention. 2 points for stapes; 1
point mastoid pneumati, OW status, RW status, malleus, incus, FN
course, status of ME, and external ear appearance.

110-Ossicles: adult-sized at birth.

111-Hemangiomas of the temporal bone often involve the geniculate


ganglion and the internal auditory meatus.

112-Epidermoid cysts have imaging characteristics similar to CSF on


MRI (hypointense on T1-weighted imaging and hyperintense on T2-
weighted imaging) and do not enhance with gadolinium contrast.
.
11

113-distinguishing characteristic relative to vestibular schwannomas


and meningiomas is that epidermoid lesions show no enhancement
with intravenous contrast.

114-The vestibular neurectomy may be approached via a


retrosigmoidal or middle fossa approach.

115-Meniere disease is a clinical diagnosis.

116-Nonsyndromic hearing loss , in which hearing loss is the only


clinical abnormality, or syndromic hearing loss , in which hearing loss
is associated with anomalies in other organ systems.

117-Aplasia or hypoplasia of the round window may be associated with


endemic cretinism and mandibulofacial dysostosis.

118-Round window aplasia is commonly associated with stapes


ankylosis and results in unsuccessful stapedectomy.

119-Irrigation with 1 :1 : 1 distilled white vinegar, distilled water, and


70% isopropyl alcohol may keep some cholesteatomas stable if their
opening into the ear canal is sufficiently large.

120-Recurrent or bilateral auricular swelling should prompt a


rheumatologic workup for relapsing polychondritis.

121-The facial nerve is the most common cranialnerve involved, with


paralysis resulting from involvement at the stylomastoid foramen.
inMOE.

122-Most cases of external otitis should be managed with topical


medications.

123-Microbiology of otitis externa:Pseudomonasaeruginosa was the


most common bacteria responsible for infections. Staphylococcus sp
were the next most common pathogens. Fungi wereresponsible for only
2% of cases.

124-The most commonly used technique for correction of an absent


antihelical fold, originally described by Mustarde,
12

125-Ideal age for otoplasty is between 5 and 6 years.

126-Prominent ears occurs in approximately 5% of the population .

127-The translabyrinthine approach provides direct exposure of the


CPA through the petrous pyramid, the shortest route from the surface,

128-The middle fossa approach provides exposure to the IAC from


above and limited access to the CPA.

129-The retrosigmoidal approach is a classic means of exposing the


CPA.

130-The most common primary sites of malignant growth that spreads


to the temporal bone are the breasts (25%),

131-Facial nerve schwannomas :intermediate signal intensity on both


T1- and T2-weighted MRI.

132-The typical feeding vessels for a glomus jugulare tumor are the
ascending pharyngeal artery and the stylomastoid branch of the
occipital artery.

133-Aquino sign is the blanching of the mass with manual compression


of the ipsilateral carotid artery.

134-Brown sign is the cessation of tumor pulsation and tumor


blanching with positive pressure using the pneumatic otoscope;

135-Inherited forms of Paget’s disease have been associated with


mutations on chromosome 5 as well as chromosome 10.

136-Paget’s disease, or osteitis deformans, is a disorder of excessive


bone remodeling, primarily of the axial skeleton.

137-Fibrous dysplasia is perhaps the most common benign


fibroosseous disorder of the temporal bone. has three major
classifications: (1) monostotic, (2) polyostotic, and (3) the McCune–
Albright syndrome.
.
13

138-ABR has a sensitivity >90%and a specificity of 90% in detecting


VS.

139-Hearing loss is present in 95% of patients with VS.

140-Ninety-five percent of VS occur in a sporadic fashion. no gender


bias and the age of presentation is between 40 and 60 y of age.

141-VS make up 80% of CPA tumors and 8% of all intracranial tumors.

142-The only definitive way to make the diagnosis of a perilymphatic


fistula is surgical exploration.

143-Combined middle fossa-transmastoid approach . This approach is


used for facial nerve exploration in patients with normal hearing.

144-β 2 transferrin is a protein found only in CSF. and perilymph.

145-2% incidence of CSF leak in all skull fractures and a 20%


incidence in temporal bone fractures.

146-Immediate facial nerve injury is highly suggestive of facial nerve


transection.

147-Patients with delayed-onset palsy present to the emergency room


with normal facial nerve function.

148-The most common form of ossicular discontinuity after temporal


bone trauma is incudostapedial joint dislocation.

149-Transverse fractures begin from the foramen magnum, run


through the otic capsule bone that surrounds the inner ear, and then
turn anteriorly toward the foramen lacerum..

150-Longitudinal fractures begin at the squamous portion of the


temporal bone, run through the external auditory canal, and then turn
anteriorly toward the foramen lacerum.
14

151-Tuning fork tests should always be performed on patients with a


temporal bone fracture.

152-Battle sign, which is a postauricular ecchymosis resulting from


extra vasated blood from the postauricular artery or mastoid emissary
vein.

153-blow to the occipital skull may go through the foramen magnum


and result in a transverse fracture of the temporal bone.

154-Blunt trauma to the lateral surface of the skull (the squamous


portion of the temporal bone) often results in a longitudinal fracture.

155-Ossicular chain dislocation with an intact eardrum manifests as a


maximal (60 dB) conductive hearing loss.

156-No medical or surgical treatments available to reverse the effects


of NIHL, prevention is key.

157-Typically, the left ear has poorer thresholds in right-handed


people.

158-Risk of NIHL is considered to increase significantly with chronic


exposures above 85 dBA for an 8 hour time-weighted average.

159-Stenger test is the classic behavioral test for estimating a


functional unilateral hearing loss.

160-NIHL is thought to result from metabolic depletion of the sensory


epith elium of the cochlea, mainly the outer hair cells and associated
neurons.

161-The stapedius muscle contracts reflexively (acoustic reflex) in


response to noise >90 dB.

162-Noise-induced hearing loss(NIHL) due to chronic overexposure to


hazardous levels of noise in the workplace.
15

163-Stapedius reflexes are present in SSCD.

164-Hearing loss is artifactual and mimics otosclerosis(low-frequency


conductive hearing loss); inSSCD.

165-Patients report increased sensitivity to bone-conducted sounds,


hearing their pulse sound, hearing their eye movements, and
autophony.inSSCD.

166-Inner ear conductive hearing loss in SSCD.

167-Purely vertical or purely torsional nystagmus is highly suggestive


of a central disorder.

168-Central nystagmus is not affected by visual fixation and may


change directions with changes in gaze.

169-The primary central cause for acute vertigo lasting days is a


brainstem or cerebellar stroke.

170-Vestibular neuronitis is the third most common cause of peripheral


vestibular vertigo after BPPV and Meniere disease.

171-BPPV:basophilic deposits adherent to the cupula; this finding was


termed cupulolithiasis , free floating debris in the endolymph termed
canalolithiasis.

172-The central compensation for vestibular injury occurs via the


cerebellum.

173-One single gene, GJB2 ( Gap-Junction Beta 2 encoding for


connexin 26), has emerged to be the most common cause of recessive
deafness,

174-Presbycusis is a type of sensorineural hearing loss that is both


progressive (1–2 dB/year) and irreversible.
16

175-Nonsyndromic hearing loss , in which hearing loss is the only


clinical abnormality, or syndromic hearing loss , in which hearing loss
is associated with anomalies in other organ systems.

176-Possible causes of early hearing loss post-stapedectomy include


intraoperative trauma, labyrinthitis, postoperative infection, granuloma
formation, and a perilymphatic fistula.

177-High perilymphatic outflow after performing a stapedectomy or


staped otomy is termed as perlymphatic “gusher.” is the result of either
an abnor mally patent cochlear aqueduct or malformation of the lateral
end of the internal auditory canal.

178-BC level of 0–25 dB in the speech range (250–4000 Hz) and an AC


level of 45–65 dB is a suitable candidate for surgery; an air-bone gap of
at least 15 dB and speech discrimination scores of 60% or better are
preferred.in indication for stapedectomy.

179-Sodium fluoride is also thought to inhibit proteolytic enzymes that


are cytotoxic to the cochlea and that may lead to SNHL

180-Fluoride reduces osteoclastic bone resorption and increases


osteoblastic bone formation.

181-The diagnosis of OS is confirmed at the time of surgery or upon


histologic study of the temporal bone.

182-One of the earliest signs of otosclerosis is an abnormal acoustic


reflex pattern, often preceding the development of an air-bone gap.

183-Carhart notch is thought to result from the disruption of normal


ossicular resonance, which is approximately 2000 Hz. It is therefore a
mechanical phenomenon and not a true reflection of cochlear reserve
since it reverses after successful surgery.

184-Halo sign or double ring sign seen on CT as a low-density zone


outlining the basal turn of the cochlea.in cochlear OS.
17

185-CT ,initial imaging modality of choice when the OS is in question.


It is valuable in assessing the pathology of the oval window and
footplate, and the extent of otic capsule involvement.

186-Rinne test at 512 Hz usually indicates an air-bone gap of at least


25 dB. In otoseclerosis.OS

187-Cong.cholesteatoma,defined as an embryonic rest of epithelial


tissue in the ear without T.M perforation and without a history of ear
infection.

188-Otitic hydrocephalus typically presents as headaches and lethargy


without evidence of meningeal signs or intracranial abscess. Often
associated with papilledema.

189-Meningitis secondary to AOM should be treated urgently with a


myringotomy.

190-The term “masked mastoiditis” is used to describe granulation


tissue and bony erosion of the mastoid in the absence of otorrhea.

191-Tympanostomy tubes were first introduced in 1954 by Armstrong,

192-Once commonly practiced, tonsillectomy has been shown to


demonstrate little to no benefit on resolution of middle ear effusions.

193-Congenital cholesteatomas recur in approximately 30–55% of


cases after surgical removal.

194-Management of congenital cholesteatoma is timely surgical


removal. Nonoperative intervention or observation may result in
progressive growth of the lesion with progressive erosion of the
ossicles.

195-Congenital cholesteatoma average age at presentation is 2–4


years for anterior lesions and 12 years for posterior lesions.
.
18

196-VEMP:are short latency electromyograms that are evoked by


acoustic stimuli in high intensity and recorded from surface electrodes
over the

197-Bilateral congenital cholesteatoma (3% of cases), male


predilection,

198-Smooth pursuit is the term used to describe eye movement that is


created when the eyes track moving objects.

199-Saccades are rapid eye movements that bring objects in the


periphery of the visual field onto the fovea.

200-OAE:emissions typically are not detected if there is a conductive or


sensorineural hearing loss greater than 25–30 dB HL.

201-Otoacoustic emissions (OAEs) are objective, noninvasive, and


rapid measures (typically less than 2 minutes) used to determine
cochlear outer hair cell function.

202-The amplitude of the summating potential (reflecting activityof the


hair cells) is compared with that of the compound action potential
(reflecting whole nerve activity). If the ratio is larger than normal (0.3–
0.5), it is considered indicative of Meniere disease

203-ECOG :evaluates the electrical activity generated by the cochlea


and the eighth cranial nerve, occurring during the first 2–3 ms
subsequent to a stimulus.

204-ABR:The absolute latencies of all waves are prolonged, but the


interwave latencies are not substantially affected.in conductive hearing
loss.

205- ABR:interaural (between ears) latencies are equal(within 0.2–0.3


ms).in normal hearing.

206-ABR is frequently used for newborn hearing screening, because it


provides accurate information in a relatively short amount of time.
19

207-The measurement of acoustic reflex decay may be useful when a


retrocochlear lesion is suspected.

208-When hearing loss exceeds 70 dB HL, it becomes difficult to


determine whether absent acoustic reflexes are due to a cochlear or a
retrocochlear hearing loss.

209-Speech discrimination testing assesses a patient’s ability to


identify monosyllabic words.

210- The SRT is the lowest intensity level at which a patient can
correctly repeat 50% of common bisyllabic words such as “hotdog” or
“baseball.”

211-Masking is required for AC whenever the difference between the


AC presentation level and the non-test ear BC thresholds exceeds
approximately 40 dB for the lower frequencies and 60 dB for the higher
frequencies.

212-Click” into the ear, a large number of auditory nerve fibers are
excited simultaneously. This is called the compound action potential,
and is Wave 1 of the ABR.

213-displacement of the cupula toward the canal provides an


excitatory response (ampullofugal endolymph flow).

214-displacement of the cupula toward the vestibule provides an


excitatory response (ampullopetal endolymph flow).

215-Hair cells Their name derives from the fact that they have about
100 stereocilia at their apical end.

216-Inner or membranous chamber is filled with a high potassium salt


solution called endolymph , which resembles intracellular fluid.

217-Outer or bony chamber is filled with a sodium salt solution called


perilymph , which resembles cerebrospinal fluid..
20

218-The total middle ear gain is between 20 and 35 dB.

219-large surface area of the tympanic membrane, compared with the


small surface area of the stapes (14:1).

220-The retrotympanum includes the sinus tympani and facial recess.

221-the shape of the external ear provides approximately 20 dB of


gain to sounds in the middle frequency range (2–4 kHz).

222-People with normal hearing can tell the difference between two
sounds that differ in frequency by less than 0.5%.

223-Most ear anomalies are inherited as autosomal dominant traits


with incomplete penetrance.

224-ABR.wave V component arises from the lateral lemniscus fibers


just caudal to their arrival at the inferior colliculus.

225-SSCCD hearing not benefit from stapedectomy.

226-Menier’s absolute contraindection for stapedectomy,sacule injuery.

227-The ear generally reaches 85% of its ultimate vertical height, 5


cm, by 3 years of age and is nearly full size,6 cm, by 5 years of age.

228-Inherited mitochondrial susceptibility can experience SNHL with


aminoglycoside whether it is given intravenously or by transtympanic
delivery.

229-Similar elevations in the SP/AP ratio have, however, been reported


in perilymph fistula, autoimmune inner ear disease, and superior
semicircular canal dehiscence.and Meniere disease.

230-Oxaliplatin is a third-generation cisplatin analogue that is not


associated with nephrotoxicity or ototoxicity.

231-Meniere disease is an absolute contraindication for stapedectomy/


stapedotomy.
21

232-Pendred syndrome:Mutations in SLC2 6A4 gene.

233-Autoimmune inner ear disease:progressive bilateral SHNL that


responds to immunosuppressant therapy.

234-Aggressive papillary adenocarcinoma of the endolymphatic sac


may erode the posterior face of the petrous bone and may be
associated with von Hippel-Lindau disease and renal cyst or tumors.

235-The most common locations of origin of cholesteatomas in


decreasing frequency are the posterior epitympanum, the posterior
mesotympanum, and the anterior epitympanum.

236-The proper use of ear protection can provide over 20 dB of


protection.

237-NIHL is not accelerating and does not progress once exposure has
been terminated.

238-The auditory brainstem implant makes direct contact with the


dorsal cochlear nucleus .

239-Calcification within the tumor or associated hyperostosis supports


the diagnosis of meningioma.

240-Head-impulse test:Involves rotational head movements.

241-Surgical identification of the endolymphatic sac Posterior and


inferior to the posterior semicircular canal.

242-The facial nerve is dehiscent approximately 50 % of the time just


superior to the oval window in its tympanic segment.

243-In an extensive fistula with a contacted mastoid, a canal-wall-


down (leaving the matrix on the fistula, is appropriate)procedure is
best. matrix can be removed if the defect is small and localized.

.
22

244-A prelingual deaf adult with no oral language is a poor cochlear-


implant candidate.

245-Paget disease affects the otic capsule creating bone remodeling


and causes both sensorineural and conductive hearing loss.

246-Size of middle ear is the most important determinant of surgical


candidacy by CT scan in congenital aural atresia.

247-principle measure is used to determine cochlear-implant


candidacy: Hearing in noise test (HINT).

248-Direction of nystagmus with posterior canal BPPV is up beating


(toward the forehead), geotropic (beating toward the ground), and
torsional.

249-BPPV is considered to be the most common cause of dizziness in


the elderly.

250-In aural atresia cases, the expected finding is a fused and


deformed malleus -incus complex.

251-OAEs are generally not detected in patients with middle-ear


pathology and conductive hearing loss.

252-With bone-conduction stimulation, interaural attenuation is < 1 0


dB.

253-Factors that may contribute to the cochlear amplifier include


motility of outer hair cells and the mechanical properties of the
stereocilia and tectorial membrane.

254-Peripheral vestibular nystagmus is suppressed with visual fixation,


is generally horizontal-rotary, can be more easily detected after
headshaking.

255-Clinically, ECochG may be used to predict postoperative audition


and is sensitive to changes in cochlear blood supply.
23

256-low-frequency conductive hearing loss and Stapedial reflexes are


present suggests a third-window disorder.

257-Mondini deformity has a higher likelihood of trauma causing


leakage of CSF and perilymph from the inner to middle ear.

258-The direction of nystagmus with right posterior canal BPPV is


upbeating (toward the forehead), geotropic (beating toward the
ground), and torsional (counterclockwise).

259-arachnoid cyst. This is characterized as being hypointense in Tl


and hyperintense on T2 .

260-Atresia surgery is best delayed until age 6 to 7 years, with use of


a bone-vibrator hearing aid before this age is reached.

261-Prediction of hearing levels is not possible by measuring OAEs.

262-In addition to the ossicular chain, the tympanic membrane is


adult-sized at birth.

263-In children developmentally at 5 to 6 months of age, it is possible


to measure hearing threshold levels using VRA.(visual reinforcement
audiometry).

264-The hearing impairment in Pendred syndrome is associated with


abnormal iodine metabolism, which typically results in a euthyroid
goiter.

265-CO 2 and argon lasers, which have been used for the treatment of
benign paroxysmal positional vertigo (BPPV).

266-All aminoglycosides can cause ototoxicity and nephrotoxicity.


Ototoxicity can be irreversible and is cumulative .Nephrotoxicity is more
common and is frequently reversible.
24

267- Auricular reconstruction precedes hearing restoration surgery by


at least 2 months to optimize the blood supply for the autologous rib
graft and complex flaps.

268-The usual distance from the lateral surface of the incus to the
footplate is 4.5 mm. Because the piston prosthesis is usually measured
from the medial surface of the incus, 0.25 mm is subtracted to allow for
this distance (incus - 0.5 mm+ extension into vestibule of 0.25) .The
most common piston size is 4.25 mm.

269-acoustic reflex is the contraction of the middle ear stapedius


muscle, attached to the posterior part of the stapes, in response to
medium to high intensity sounds.

270-A major advantage that the ABR has over other auditoryevoked
potentials is that it is not strongly affected by attention, sleep, or
sedation.

271-Thus, otoacoustic emissions and cochlear microphonics are


preneural phenomena that reflect the integrity of the outer hair cells of
the cochlea,

272-Distortionproduct otoacoustic emissio. are more sensitive than


transient-evokedotoacoustic emissions for the detection of early signs
of ototoxicity.

273-Vestibular aqueduct radiologically wider than 1.5 mm at its


midpoint or wider than 2 mm at the operculum is defined as enlarged.

274-Betahistine, an oral preparation of histamine, is one such


medication. It has proved to be effective in treatment of Meniere’s
disease.

275-The most common type of vertigo after head trauma is benign


paroxysmal position vertigo, which manifests with transient rotatory
nystagmus with the Halpike maneuver.
25

276-An enlarged vestibular aqueduct, defined as a diameter larger than


1.5 mm at the vestibular aqueduct midpoint or greater than 2 mm at the
operculum, may lead to sudden and progressive sensorineural hearing
loss in children, particularly after even minor head trauma.

277-The incidence of meningitis following a temporal bone fracture


without a CSF fistula is 1%.

278- Bilateral endolymphatic sac tumor associated with von Hippel-


Lindau disease.VHL.

279-labyrinthine fistula has been reported in 10% of cholesteatomas in


adults and children,

280-Otosclerotic Patients are typically treated with Florical, 8 mg three


times per day, until hearing loss stabilizes.

281-CSF leakage occurs through a defect in the lamina cribrosa of the


internal auditory canal in association with a defect in the stapes
footplate.

282-The most common labyrinthine anomaly associated with


spontaneous CSF leak is a Mondini malformation.

283-Paget’s disease, which may be inherited in an autosomal dominant


manner with high penetrance.

284-The incidence of meningitis following a temporal bone fracture


without a CSF fistula is 1%. • The incidence of CSF fistulas with
temporal bone fractures is 17%.

285-The most important landmarks for the facial nerve are the HSCC,
the short process of the incus, and the posterior bony external
auditorycanal; the digastric ridge is also a landmark,

286-Identifying the short process of the incus provides an important


landmark for facial nerve dissection.
26

287-The nystagmus with lateral canal BPPV is horizontal and may beat
toward (geotropic) or away from (ageotropic) the downward ear. It
often begins with a shorter latency, increases in magnitude while
maintaining the test position, and is less susceptible to fatigue with
repetitive testing than the vertical torsional nystagmus of posterior
canal BPPV.

288-A relatively large SP/AP ratio is considered diagnostic of


endolymphatic hydrops.

289-giant cholesterol granulomas, but not congenital cholesteatomas,


tend to show capsular enhancement.

290-Facial twitching may occur in the presence of congenital


cholesteatomas and with facial nerve neuromas.

291-Congenital cholesteatomas of the temporal bone may be divided


into four anatomic groups: middle ear, perigeniculate area, petrous
apex, and cerebellopontine angle.

292-The most important landmarks for the facial nerve are the HSCC,
the short process of the incus, and the posterior bony external
auditorycanal;

293-The fistula test results are reported as positive in only 55% to


70%of patients with lateral canal erosion,

294-Hennebert’s sign (a positive fistula test without middle ear


disease) and Tullio’s phenomenon (vertigo or nystagmus on exposure
to high-intensity sound) have been strongly associated with otosyphilis.

295-In otosclerosis we never operate on the only-hearing ear or better


hearing ear.

296-cholesterol granuloma is high signal on T1WI and T2WI from the


methemoglobin.

297-High-resolution T2-weighted MRI is the study of choice for


preoperative evaluation for cochlear implantation.
27

298-The Baha was also approved for treatment of unilateral


sensorineural hearing loss or single-sided deafness.

299-Indications for BAHA include a pure tone average bone-conduction


threshold better than or equal to 45 dB.

300-The cochleostomy is made at the anteroinferior aspect of the


round window.

301-Usually, removing the chorda tympani is unnecessary for adequate


visualization of the round window niche area.

302-Superior semicircular canal dehiscence also can be identified


radiographic -ally and has been associated with conductive hearing loss
in children,

303-Mondini dysplasia reflects abnormal development of the cochlea


and is associated with both Pendred’s and branchio-oto-renal
syndromes.

304-Pendred’s syndrome represents 10% of the cases of syndromic


hearing loss and is the most common cause of syndromic hearing loss.
It is associated with a dilated vestibular aqueduct on temporal
Ctscanning.

305-Dilated vestibular aqueduct is defined as one with a diameter


greater than 1.5 mm at its middle third or greater than2.0 mm
anywhere along its length.

306-The space between the mastoid segment of the facialnerve and


the chorda tympani is a space through which a posterior tympanotomy
(or facial recess) can be surgically created to access the middle ear.

307-Type II accounts for 10% of cases, and is characterized by


congenital moderate to severe stable hearing loss, normal vestibular
responses, and onset of retinitis pigmentosa in patients 17 to 23 years
old..
28

308-Usher’s syndrome type I accounts for 85% of all cases, and is


characterized by profound congenital hearing loss, absent vestibular
response, and the development of retinitis pigmentosa by age 10 years.

309-A major advantage that the ABR has over other auditory evoked
potentials is that it is not strongly affected by attention, sleep, or
sedation.

310-ECoG:ratio between the two (SP/AP ratio) greater than 0.4 and an
AP duration longer than 3 msec are indicative of endolymphatic hydrops

311-Audiometric evaluation must always precede the ABR test to


ascertain that the level of stimuli used to elicit the ABR is within the
patient’s auditory dynamic range. It is also possible to record ECoG and
ABR simultaneously..

312-Oral fluoroquinolones have become the treatment of choice in


bacterial MOE with cure rates near 90%. Serial gallium-67 bone scans
may help monitor the response to therapy.

313-Bell’s palsy base their decisions for surgery primarily on the NET or
ENoG, but also require confirmatory EMG.

314-EMG is the recording of spontaneous and voluntary muscle


potentials using needles introduced into the muscle. Its role in the early
phase of Bell’s palsy is limited,

315-Stapedial reflex . particularly useful for differentiation of cochlear


and retrocochlear lesion sites .

316-The saccule is the second most common site affected by hydrops,.

317-The most common pathologic interconnection between the


subarachnoid space and the inner ear is through the fundus of the IAC.

318- Large vestibular equduct:It is seen in isolation, as part of the


Mondini malformation, and in patients with branchio-otorenal syndrome
and Pendred syndrome. It is well shown on high resolution CT imaging
of the temporal bone.
29

319-landmarks in the middle fossa are the middle meningeal artery at


the foramen spinosum, the greater superficial petrosal nerve at the
facial hiatus, and the arcuate eminence.

320-The site of injury of the facial nerve in temporal bone fractures is


in the perigeniculate region in 80% to 93% of patients.

321-only patients with complete paralysis of immediate or unknown


onset are considered for surgical exploration in temporal bone trauma
causing VII palsy.

322-type C tympanogram also may be present in the early stages of


otitis media without effusion.

323-A type C tympanogram indicates negative middle ear pressure as


reflected by a negative pressure peak and may indicate eustachian tube
dysfunction.

324-Type B tympanograms may be seen in cases of otitis media with


effusion, space-occupying lesions of the tympanic cavity, and tympanic
membrane perforations.

325-It is believed that the presence of hydrops affects the elasticity of


the basilar membrane and contributes to the increased amplitude of the
SP relative to that of the AP.

326-Relatively large SP/AP ratio is considered diagnostic of


endolymphatic hydrops.

327- vertical crest (Bill's bar) is exposed. This bony projection is a key
surgical landmark, in acoustic neuroma.

328-Cholesteatoma and cholesterol granuloma can be differentiated


from each other by MRI, since in the T1 sequence the cholesterol
granuloma is bright but the cholesteatoma appears less bright because
of a longer T1 relaxation time. in petrous apex.
30

329-CT findings are identical to those of a cholesterol granuloma and


mucocele of the petrous apex. Cholesterol granul. are indistinguishable
from congenital cholesteatomas
and mucoceles of the petrous apex.

330-In congenital atresia the absence of any identifiable ear canal in


an atretic ear, drilling should always begin at the level of the linea
temporalis, just posterior to the glenoid fossa.

331-Because the primary goal of surgery for acquired atresia is


improvement in hearing, preoperative audiologic testing will help
confirm the nature and extent of loss in the affected ear.

332-Atelectatic ear is one of the most important indications for


cartilage tympanoplasty.

333-Acoustic reflex decay is superiortest of auditory fetigue and less


time consuming.

334-Tone decay tests useful as a screen for VIII nerve lesion.

335-The varicella zoster virus that causes Ramsay Hunt syndrome is


the same virus that causes chickenpox (varicella) and shingles.

336-Battle sign (ecchymosis of postauricular skin) Raccoon sign


(ecchymosis of periorbital area) .

337-Temporal bone fracture comprises 15-25% of all skull fractures .

338-Contrast-enhanced MRI is the study of choice to detect VS,


whereas air contrast CT cisternography is reserved for patients who are
unable to undergo MRI.

339-CT scanning is insensitive to the detection of vestibular


schwannomas, even when performed with contrast medium..
31

340-Type As tympanogram:Otoseclerosis,tympanoseclerosis &malleus


fixation .

341- OHC Contain Actin (contractile protein). OHC Reduce Length with
Depolarization.

342- In G J tumor treatment the most common site of lower cranial


nerve involvement was the pars nervosa of the jugular bulb, and this
involvement was typically multiple.

343-Baha consists of three parts: an implanted titanium flange fixture,


a sound processor, and an external abutment that couples the titanium
fixture to the sound processor .

344- Paraganglioma or glomus tumor is a tumor of the neuroendocrine


system derived from neural crest cells and chemoreceptor cells. The
tumor was termed a chemodectoma by Mulligan in 1950.

345-Glomus tumors represent 0.6% of neoplasms of the head and


neck and 0.03% of all neoplasms; they are the most common tumors
of the inner ear and the second most common tumors of the temporal
bone after schwannomas.

346-Developmental failure of the first branchial groove results in


stenosis or atresia of the external auditory canal.

347-Microtia, anotia, and aberrant positioning of the pinna derive from


abnormal development of the first and second branchial arches.

348- The landmark for opening into the facial recess is the fossa
incudis.

349-Enlarged vestibular aqueduct if it is larger than one millimeter,


roughly the size of the head of a pin.
32

350-Prolonged wave V & wave I latency in ABR suggest a conductive


loss.

351-Mondini deformety:autosomal dominant

352-Levenson criteria of congenital cholesteatoma,White mass medial


to normal TM-Normal pars flaccida and tensa-No history of otorrhea or
perforations-No prior otologic procedures-Prior bouts of otitis media not
grounds for exclusion.

353-The presence of an aural polyp in a chronically infected ear should


be considered to be a cholesteatoma until proved otherwise.

354-The diagnosis of aural cholesteatoma is made on otoscopic


examination, including endoscopic and microscopic evaluation or
surgical exploration.

355-Osteoclastic bone resorption in chronic otitis media with or without


cholesteatoma is stimulated by various factors, including inflammation,
local pressure, keratin, specific cytokines, and bacterial toxins.

356-A fistula of the bony labyrinth is the most common complication of


COM and is reported to occur in 4–13 percent of cases.

357-Tympanosclerosis refers to hyaline deposits of acellular material


visible as white plaques in the tympanic membrane and as white
nodular deposits in the submucosal layers of the middle ear on
otoscopy.

358-Frenzel’s glasses are generally worn to prevent ocular fixation and


suppression of nystagmus by vision.

359-The OHCs’ main function is to modify the acoustic information that


reaches the IHCs.

360-The sensory receptor for the auditory nerve is the IHC, which
synapses with type I spiral ganglion..
33

361-The preferred method for detection of OHC damage is the


measurement of OAEs.

362-Ototoxic substances enter the cochlear tissues via the vascular


system, the CSF (via either the cochlear aqueduct or the internal
auditory meatus), or via the middle ear, across the round window
membrane.

363-Complication of grommate:purulant otorrhoea,early tube


extrution, granuloma, permanent perforation,tympanosclerosis and
cholesteatoma.

364-SNHL,tinnitus,vertigo and feeling of fullness. is detected in


patients with perilymphatic fistula.surgical grafting is the only definitive
treatment.

365-Chemotherapy does not have a role in treatment of acoustic


neuroma.

366-Unilateral HL. Is the classic presentation of acoustic neuroma.

367-20% of patients with Bell’s palsy are actually suffering from Rams
H S.

368-Vestibular Neuronitis it effect only vestibular n. its has no effect on


hearing.

369-The cause of vestibular neuronitis(VN) is still largely unknown.


F=m.

370-Labyrinthitis follows an URT infection in over 50% of cases.

371-Recruitment is positive in cochlear disorders such as Meniere’s


disease.

372-Argon laser is useful in middle ear surgery such as


stapedectomy,ME adhesions and tympanoplasty.

373-Gold standard to diagnose acoustic neuroma is contrast enhanced


MRI with T1 images .
34

374-Donaldson’s line is the landmark of endolymphatic sac.

375-SNHL due to quinine,salicylates and furosemide is reversible.

376-Kanamycin is cochleotoxic and most likely to cause unilateral HL.

377-Hearing aid consists of three parts microphone, amplifier and


receiver.

378-CI:two components external (microphone,speech processor and


coil or transmitter).internal(single or multiple electrodes and coil or
receiver).

379-Heterochromia iridis,two eyes are of different coluor or two


different colours in one eye.

380-Dystopia canthorum is lateral displacement of medial canthi.


differentiating from hypertelorism in which orbits are laterally displaced.

381-Hirschsprung’s disease may be associated with both WS-I and WS-


II.

382-Waardenburg’s syndrome is Autsomal dominant.(WS type I with


dystopia canthorm and type II no such feature)

383-Unilateral SNHL is commonly seen in acoustic neuroma.(20%


SSNHL).

384-Episodic vertigo is seen in only 20% of acoustic neuroma.

385-BPPV(free-floating particles).is the most common cause of


vertigo(20%). Followed by Meniere’s and vestibular neuronitis.

386-Notch at 4kHz both in AC and BC is seen in noise –induce HL.

387-Hitzelberger sign its diminished sensation in posteriosuperior wall


of EAC, due to CN VII sensory fiberes affected by acoustic neuroma..
35

388-Otolithic crisis of Tumarkin or drop attacks are seen early or late


Meniere’s.

389-Hennebert’s sign it’s a positive fistula test without presence of


fistula.its seen in cong.syphilis or Meniere’s disease(25%).

390-Tullio phenomenon is loud sounds or noise produce giddiness.its


seen in congenital syphlis.meniere’s and sup.SCCD(three functioning
windows).

391-Carhart’s notch is loss of bone conduction(2kHz) in audiogram in


otosclerosis.can be reversed by stapedectomy.

392-paracusis of Willisi:better hearing in noisy surroundings in


otospongiosis.

393-Schwartze’s sign is reddish hue seen over promontory.its due to


increased vascularity of bone and is seen in early or active
otospongiosis.

394-most common site for otosclerosis is fossula ante-fenestram.

395-Otosclerosisis autosomal dominant disease.

396-Aquino’s sign is blanching of glomus t. on compres. of ipsilateral


carotid

397-Griesinger’s sign is edema over the masoid due to thrombosis of


masoid emissary vein.its seen in lat.sinus thrombosis.

398-Glomus tumor. is five time more common in women.its very slow


growing tumor.

399-petrous apex is pneumatised only in 30% of cases.


36

400-cholesteatoma destroy bone by proteolytic enzymes_protease,


hyaluronidase and acid phosphatase.

401-Brown’s sign (pulsation sign) in glomus tumor.when pressure is


raised in the ear canal with siegel’s speculum,tumor first pulsates
vigorously and then blanches.

402-phelps sign is destruction of bone between carotid canal and


jugular foramen in glomus jugulare tumors.

403-nystsgmus of central origin has: appears immediately as soon as


the head is put in critical position without latent period,change
direction,lasts as long as the head position is maintained.is not
fatigable.

404-Dix-Hallpike test used in episodic positional vertigo.

405-Vestibular neuronitis produce nystagmus in opposite side.

406-serous labryrinthitis produce nystagmus in same side.

407-labyrinthectomy produce nystagmus in opposite side.

408-Purulent labyrinthitis produce nystagmus in opposite side.

409-Cogan’s syndrome :episodic vertigo,interstitial keratitis and hearing


loss.

410-Caloric test determines function of lat.SCC.

411-spondee words are used to measure speech reception threshold


(SRT).

412-Phonetically balanced words are used to measure discrimination


score.

413-Tone decay test is positive in acoustic neuroma.

414-A sound of 20 dB is 100-fold increase in sound energy.


.
37

415-Recruitment is a feature of cochlear deafness.

416-Vowels are low frequency sounds, consonants are high ferquency


sounds.

417-Speech frequencies include: 500.1000,2000 Hz.

418-Invesigation of choice in assessing hearing loss in neonates is


ABR.

419-Costen’s syndrome is characterised by otalgia, vertigo, tinnitus.

420-Dorello’s canal transmits abducens n.(cn VI ).

421-Lat.SCC.stimulated during caloric test in a sitting position with


head tilted 60” .

422-Jacobson’s n. supplies ME,mastoid,secrtomotor to parotid.

423-Furosemide ototoxicity on stria vascularis.

424-Interpretation of ABR is affected by age of the child.

425-Vancomycin induced augmentation of aminoglycoside ototoxicity.

426-The dominant ototoxic effect of salicylates appears to be the


production of tinnitus as well a reversible mild to moderate symmetric
SNHL.

427-Surgical grafting is the only definitive treatment in perilymphatic


fistula.

428-perilymphatic fistula present with tinnitus,vertigo,hearing loss and


feeling of fullness within the ear its difficult to differentiate fro
meniere’s disease on clinical basis alone.

.
38

429-BPPV result in vertigo,but dose not cause hearing loss.

430-unilateral hearing loss is the classic presentaion of acoustic


neuroma.

431-little evidence that viral infection cause vestibular neuronitis is still


largely unknown.

432-Vestibular neuronitis is a self limiting,short term illness.usually


lasts for a few weeks.main symptoms are nausea, vomting and
vertigo.no effect on hearing.

433-Sever headache, vomiting, dysphagia ,&visual field defects in a


patien with cholesteatoma indicate: temporal lobe abcess.

434-Ottitic barotraumas is charactarised by middle ear effusion .

435-Bilateral cauliflower ear deformity, saddle nose deformity, and


stridor.. ..relapsing polychondritis…Rx- dapsone .

436-most consistent landmark that a surgeon can easily use in


identifying the facial nerve near the skull base is the tympanomastoid
suture.

437-Sensorineural hearing loss can complicate radiation therapy and


cisplatin treatment for nasopharyngeal carcinoma in up to 30% of
patients.

438-The outer cartilage bites of Mustarde sutures are described as 1


cm separated by 2 mm.

439-Most ear anomalies are inherited as autosomal dominant traits


with incomplete penetrance.

440-The auricle is 90 to 95% of adult size by 5 to 6 years of age.


39

441-Recent studies have linked the cause of Bell’s palsy to herpes


simplex virus.

442-Fibrillation potentials found on facial EMG indicate intact motor


end plates but no evidence of reinnervation.

443-Adults with prelingual deafness are generally not considered good


candidates for cochlear implantation.

444-Nonauditory centers (the limbic and autonomic nervous systems)


are always involved in clinically significant tinnitus.

445-About 20 to 30% of tinnitus patients have normal hearing.

446-first indicators of age-related hearing loss is loss of auditory


sensitivity at frequencies higher than 8 kHz.

447-atrophy of the stria vascularis is the predominant lesion of the


aging ear.

448-Vestibular neuronitis is characterized by an acute attack of vertigo


that may last for a few days. Some patients may later develop BPPV.

449-The superior vestibular nerve innervates the superior and lateral


semicircular canals and the utricle, whereas the posterior semicircular
canal and the saccule are innervated by the inferior vestibular nerve.

450-A 75% rate of hearing improvement was seen with corticosteroids


in patients with a positive Western blot for a 68 kD antigen.

451-antigen target in autoimmune inner ear disease is a 68 kD inner


ear protein.

452-nystagmic eye movements of Hennebert’s sign sometimes Require


the use of infrared nystagmography.
.
40

453-clinical diagnostic criteria for perilymphatic fistula Include triad (1)


positive fistula test (Hennebert’s sign/symptom); (2) constant
dysequilibrium, however mild (eg, eyes closed walking); and (3)
postural nystagmus and/or vertigo.

454-Mumps deafness is usually unilateral.

455-Hearing loss associated with carboplatin treatment of brain tumors


can be prevented by administration of sodium thiosulfate.

456-Cisplatin damages the outer hair cells most frequently, whereas


carboplatin damages the inner hair cells in animal experiments.

457-The outer hair cells of the basal turn of the cochlea are damaged
first following exposure to ototoxic doses of aminoglycoside antibiotics.

458-Aminoglycoside ototoxicity can be prevented in experimental


animals by pretreatment with salicylates.by chelate iron and prevent it
from reacting with the aminoglycoside.

459-Benign paroxysmal positional vertigo occurs in 25% of all head


injuries whether there is a fracture or not.

460-perilymphatic fistula should be suspected,in progressive hearing


loss and episodic vertigo after a temporal bone fracture.

461-most common traumatic ossicular abnormality , incudostapedial


joint disarticulation.

462-percentage of traumatic perforations of the tympanic membrane


heal spontaneously: 80%.

463-Mutations in Cx26 are responsible for over one half of moderate-


to-profound hearing impairment in many world populations.

464-The typical autosomal dominant phenotype is one of postlingual


hearing loss. Autosomal recessive hearing loss is more severe than
autosomal dominant hearing loss.
41

465-Mitochondrial hearing loss, It may be associated with diabetes


mellitus.

466-Alport’s syndrome:Hematuria is the most common mode of


presentation.

467-Usher’s syndrome, It is the most common autosomal recessive


syndromic hearing loss.

468-The most reliable means of improving hearing in the patient with


otosclerosis is a hearing aid.

469-The principal objective in stapes surgery is to restore sound


pressure transmission to the inner ear.

470-The most frequent site of first occurrence of otosclerosis is in the


fissula ante fenestram.

471-Otosclerosis is a disease affecting endochondral bone.

472-The radiologic study of choice to diagnose lateral sinus thrombosis


is enhanced magnetic resonance imaging.

473-A total ossicular replacement prosthesis is used only when the


stapes superstructure.

474-The most critical factor in avoiding injury to the facial nerve during
mastoid operations is drilling parallel to the nerve.

475-Exostosis is common in cold-water swimmers.

476-Keratosis obturans is associated with bronchiectasis or sinusitis.

477-The most commonly isolated pathogens in acute diffuse otitis


externa are Pseudomonas aeruginosa and Staphylococcus aureus.

478-Cerumen:forms an acidic coat that aids in the prevention of


external auditory canal infection.

.
42

479-Otoacoustic emissions are produced by the outer hair cell motility.

480-Major anatomic generator of the wave V component of the


auditory brainstem response is the lateral lemniscus.

481- The anterior portion of the Incudomalleal Fold and the Anterior
portion of the TTF are the most superior portion of the Tympanic
Diaphragm.

482-TTF separates the AER from the Supra-Tubal Recess .

483-the size of the AER increases as the angle of the TTF increases .

484-Two openings into the tympanic diaphragm insure the ventilation


of the Epitympanum :Ant. Tympanic Isthmus: Between the tensor
tympani muscle & stapes,: Post. Tympanic Isthmus: Between the
stapedial muscle & the Incus

485- Tympanic diaphragm. Formed by:Ant. Malleal Fold, Lateral Incudo-


Malleal Fold, TTF, Ossicles.

486- Prussack’s space: Vent: PP Von Troltsch + Lower Lateral Attic.

487- Epitympanum divided: Ant. + Post by superior Malleal Fold.

488-Koِrner’s septum, the embryologic junction between petrosal and


squamosal portions of the temporal bone,

489-The tympanic segment is the most common site of congenital


dehiscence of the bony canal, especially above the oval window.

490-The facial nerve is most vulnerable to iatrogenic injury just


proximal to the second genu and in its vertical path.

491-In 55% of cases, however, the facial nerve is partially exposed


because of bony dehiscence.
4
43

492-Supratubal recess. The posterior opening of this recess is marked


by the cochleariformprocess inferiorly and the “cog” superiorly,

493-Shrapnell’s membrane serves as the lateral wall of Prussak’s space.


the cochlear aqueduct can be used as a guide to the lower limits of IAC
dissection.

494-Boyette’s solution is( 4% lidocaine, phenylephrine, and sodium


chloride, with sterile water added to make 200 mL.).

495-The most vulnerable portion of the ossicular chain is the long


process of the incus, which possesses a single nutrient vessel and lacks
collateral circulation.

496-The arcuate eminence is the prominence of the superior


semicircular canal on the middle fossa floor.

497-About 50% of children with otitis media will ultimately be


diagnosed with allergic or infectious sinonasal disease.

498-Referred otalgia with dysphagia is a sinister symptom and poor


prognostic sign

499-Acoustic neuromas originate at the junction of the glial and


Schwann cells, that are bilateral (5%) are seen in(NF2). This is an
autosomal dominant disease due to an aberration on the long arm of
chromosome 22.

500-Prerequisites for Type I tympanoplasty are: 1. Presence of mobile


ossicles 2. Patent eustachean tube 3. Perforated ear drum .

501- Derlacki and Clemis criteria for the diagnosis of congenital


cholesteatoma:
1. The patient should not have previous episodes of middle ear
disease
2. Ear drum must be intact and normal
3. It is purely an incidental finding .
44

502-laddergram ABLB test indicates complete recruitment..


Recruitment is common in cochlear deafness. It is not seen in
retrocochlear deafness.

503-osteogenesis imperfecta :characterized by brittle bones, blue


sclera and fixation of foot plate of stapes .

504-drugs cause Microtia . Warfarin, Folic acid antagonists like


methotrexate and aminopterin .

505-Kertosis obturans occur due to faulty epithelial migration of


external canal skin.

506-Glomus jugulare - associated with pulsatile tinnitus, conductive


deafness, positive Brown's sign. CT scan shows eorsion of jugular
foramen.

507-Levenson's criteria for malignant otitis externa:


1. Refractory otitis externa.
2. Severe nocturnal otalgia.
3. Purulent otorrhoea.
4. Granulation tissue in external canal.
5. Growth of pseudomonas aeruginosa.
6. Presence of diabetes mellitus / other immunocom- promised states.

508-Siegle's pneumatic aural speculum:Testing the mobility of the ear


drum, magnification 2.5 times.

509-familial glomus tumors may occur in 20% of patients with glomus


tumors.

510-Glomus tumors of the temporal bone are neoplasms arising from


normally occurring paraganglionic bodies or formations that are located
on the dome of the jugular bulb and along the course of Jacobson's
nerve (ninthcranial nerve) onto the cochlear promontory.

511-The largest concentration of paraganglionic cells is with the


adrenal medulla. The other predominant locations of paraganglionic
tissue are found in the branchiomeric chemoreceptor system of the
aortic arch, carotid bifurcation, and temporal bone.
.
45

512-28 % - 60% of patients with severe Tinnitus are clinically


depressed .

513- Aggravate Tinnitus:Caffeine, Cola, Tea, Smoking.

514- Tinnitus + HL → 63% noise exposure. 80 % of Otosclerosis. 80


% of Accoustic Neuroma. First sign of ototoxicity.

515- Postural Unsteadiness: Tandem Test, Romberg Test.

516- Fast phase is usually away of the lesion side, Slow phase looks to
the lesion side. Hypofunction.

517- Beat to the right when the eyes look to the right, Beat to the
right when the eyes look on central gaze, Beat to the right when the
eyes look to the left: Sign of a Labyrinthine Disorder.

518- beat to the right on Gaze Right , beat to the left on Gaze Left:
Suspect a brain disease.

519- Positionnal Nystagmus: Suppression by visual fixation .

520- Gaze Nystagmus: Never occur with peripheral vestibular lesion.

521- Congenital Nystagmus: When beats on direct forward gaze.

522- First degree nystagmus: beats in one direction of lateral gaze.

523-Second degree nystagmus: nystagmus still present in straight


ahead gaze.

524- Third degree nystagmus: nystagmus present in all 3 directions of


gaze.

525- Induced nystagmus: occurs with caloric or rotationnal tests.


.
.
46

526- Spontaneous nystagmus: occurs without stimulation.

527- Positional nystagmus: nystagmus occurs with certain head or


body position.

528-Gaze-evoked nystagmus: If the nystagmus begins when the


patient look to the left and right or up and down.

529- Progressive SN hearing loss : Acoustic Neuroma.

530- Sudden hearing loss: Labyrinthe or VIII nerve.

531- Tullio Phenomenon:Loud noise may induce VERTIGO.

532- Facial Nerve Neurinoma Commonest site is geniculate region,


20% of cases of recurrent Bell’s.

533- FN paralysis in Malignant otitis externa 40 % , Only 20 % of the


cases show complete recovery after intensive treatment.

534- Mobeius syndrome Partial or complete agenesis of FN nucleus &


nerve roots Usually bilateral Associated with other cranial nerves
affection

535- Lacrimation test (Shirmer’s test): 25% reduction from normal


side.

536- Stapedial reflex text: absent in 1% of normal individuals .

537-The facial recess is, therefore, bounded medially by the facial


nerve and laterally by the tympanic annulus, with the chorda tympani
nerve running obliquely through the wall between the two.

538-The facial recess is, therefore, bounded medially by the facial


nerve and laterally by the tympanic annulus, with the chorda tympani
nerve running obliquely through the wall between the two.
.
.
47

539-oval window Its size naturally varies with the size of the footplate,
but on average it is 3.25 mm long and 1.75 mm wide.

540-EMG -Primary used in acute phase of facial paralysis (Ist 2 wk) is


confirmatory for other tests.

541-THE DISTRACTION TEST:is suitable for babies from 6 to 18


months.

542-Severity of HL not parallel to disease’s severity(discharging ear).

543- The most common germ of otorrhea is pseudomonas aerogenosa.

544- The most common cause of otorrhea is external otitis.

545- Serum IgE: high values in patients with discharging ears.

546- TB otomastoiditis;Multiple perforations, creamy


discharge,Extensive bony destruction of mastoid and Pulmonary T.B

547- Transcanal(Rosen incision), Endaural(Lempert incision), Post


auricular (Wildes incision).

548-G.Jug:only 1% to 3% of these tumors actively secrete


norepinephrine. Catecholamines are much more likely to be secreted by
glomus jugulare tumors than by glomus tympanicum tumors.

549-Glomus tumors may secrete vasoactive substances or be


associated with other tumors, such as pheochromocytomas.

550-high jugular bulb is present in about 25% of cases.

551-High-tone sensorineural hearing loss, perhaps secondary to


bacterial toxins migrating across the round window.as a seguelae to
ASOM..
48

552-ABR has only a 3% false-negative rate. It demonstrates a


retrocochlear lesion by an increased latency between N1 and N5
waves.in Acoustic N.

553-Acoustic neuroma :The most accurate means of identifying small


intra- canalicular tumours is (MRI) with gadolinium enhancement. used
for screening. CT should only be used when MRI is unavailable.

554-Loss of the corneal reflex is an early sign of trigeminal nerve


impairment. In acoustic neuroma.

555-Acoustic neuroma:. Hypaesthesia of the posterior EAC on the side


of the hearing loss should raise the index of suspicion (Hitselberger
sign).

556-The posterior auricular appears to be the dominant artery and


supplies the medial surface (except the lobule) of the auricle.

557-Role of 55 in the ear :vibrating area of the tympanic memberane


55 sq.mm :wieght of ossiculatar chain 55 mg(incus 27 mg,malleus 25
mg and stapes 3mg)/, TM in obliqe position 55` degree
anteroinferiorlly. In 55% of cases, however, the facial nerve is partially
exposed because of bony dehiscence..

558-One of the major reasons for the failure of a mastoid cavity


to become a dry skin-lined bowl is poor meatoplasty technique.

559-Apert syndrome is a rare autosomal dominant craniosynostosis.


Similar to Crouzon's syndrome, midface hypoplasia is prominent with
relative class III occlusion. The most characteristic features are: ‘tower
skull .

560-Crouzon syndrome is an autosomal dominant craniosynostosis.


The syndrome is characterized by midface hypoplasia with relative
mandibular prognathism with an anterior bite and class III occlusion.
.
49

561-Aural cholesteatomas are epidermal inclusion cysts of the middle


ear or mastoid .

562-presence of granulation tissue at the junction of the cartilaginous


and bony junction. This may signify that a malignant otitis externa is
present, which, as an osteomyelitis of the temporal bone.

563-Marginal perforations with thin keratin migrating laterally from the


edge (trail sign) suggests squamous epithelium or cholesteatoma in the
middle ear

564-The diagnosis of otosclerosis must be considered when there is a


history of slowly progressive hearing loss, bilateral disease is present,
and the family history is positive.

565-Indications for reconstruction of the middle ear and tympanic


membrane are to eliminate recurrent disease, provide a dry ear canal
and middle ear space with an intact tympanic membrane, and maintain
or improve hearing.

566-Failure to close the air-bone gap after surgery(stapedectomy) for


presumed otosclerosis may be due to fixation of the lateral ossicular
chain or dehiscence of the superior semicircular canal.

567- If a floating footplate develops and it sinks into the vestibule,


place a tissue graft over the oval window and reconstruct with a wire or
bucket-handle prosthesis.

568- Palpate the lateral ossicular chain after the incudostapedial joint
has been separated, confirm that the lateral chain is not fixed, and then
check the mobility of the stapes.

569-Almost all middle ear prosthetics in use today are


nonferromagnetic .The Robinson prosthesis is included in the group of
devices that can be safely subjected to MRI.
.
50

570- Most of the pathology occurring in the middle ear and mastoid
results from eustachian tube dysfunction.

571-Cholesterol granuloma is a comprehensive term describing a


loculated cystic or solid foreign body reactive process directed against
cholesterol crystals resulting from the by-products of degraded blood.

572-Laboratory markers for identifying perilymph, such as β2-


transferrin or fluorescein, can be unreliable and their absence
misleading.

573- Patients with sudden hearing loss or dizziness who have a history
of recent barotrauma, physical ear injury, or otologic surgery should be
suspected of
having a.perilymphatic fistula.

574-Cholesteatoma definition:destructive and expanding keratinizing


equamous cell cyst(epidermal inclusion cyst).

575-The absence of OAE can be associated with hearing loss of mild to


moderate degree, and presence does not ensure normal hearing.

576-surgical correction of conductive loss may close the air-bone gap,


a sensorineural component may still require amplification.

577-90% of patients suffering from keratosis obturans also have a past


history of
bronchiectasis or sinusitis.
Part 1I

NOSE
Part 2

Nose
51

1-Choanal atresia is usually repaired via an endoscopic transnasal


approach, which results in low recurrence rate and few complications.

2-While sinusitis is generally a bilateral disease, adults with unilateral


nasal obstruction must be evaluated carefully for a tumor.

3-The mainstay of treatment for both acute and chronic sinusitis is


medical.

4-On MRI, an inverted papilloma may demonstrate a convoluted so-


called cerebriform appearance on postcontrast images.

5-Three types of papillomas arise from the Schneiderian epithelium of


the nasal cavity:inverted, fungiform, and cylindrical. Inverted
papillomas arise from the lateral nasal walls near the middle turbinate
and demonstrate an endophytic pattern of growth into the ethmoid and
maxillary sinuses.

6-The vascular supply of angiofibromas is from the internal maxillary


and ascending pharyngeal arteries, and preoperative embolization is
common practice.

7-Pott’s puffy tumor is the term applied to a collection of pus under the
frontal pericranium arising from a frontal sinus mucopyocele.

8-The black turbinate sign, which reflects non-enhancing, ischemic


middle turbinate mucosa, may be an early indicator of invasive fungal
sinusitis.

9-A classic radiographic sign of acute sinusitis is a fluid level, but fluid
levels may also be seen in a setting of trauma, prolonged nasogastric
intubation, barotrauma, or CSF leak.

10-Onodi cells are the most posterior ethmoid air cells and are
intimately related to the optic canal; they can completely surround the
optic nerve, increasing the risk of optic nerve injury at surgery.

.
52

11- nasopharyngeal Ca.risk factores:genetic predisposition, viral


infection with the Epstein–Barr virus (EBV); dietary factors, including
salt-cured fish; and environmental factors such as sawdust and smoke
inhalation.

12-Nasal glioma presents as a mass over the nasal dorsum without


direct intracranial extension.

13-Nasal encephalocele. Intacranial contents are protruding through


foramen cecum toward the nasal dorsum.

14-Nasal dermoids are firm, slow- growing, noncompressible masses


that do not transilluminate. They do not enlarge with crying or Valsalva
maneuvers and demonstrate a negative Furstenberg test by not
expanding with the compression of ipsilateral jugular veins.

15-Up to 45% of nasal dermoid cysts have an intracranial connection.

16-Nasal dermoid cysts are found in the midline of the nose as masses,
sinus tracts, or as a combination of the two. They are derived from
ectoderm and mesoderm, are lined by keratinized stratified squamous
epithelium, and can contain hair follicles, sweat glands, sebaceous
glands.

17- Up to 60% of nasal pyriform aperture stenosis is associated with a


single central maxillary incisor.

18-Cysts in the superior portion of the nasopharynx may represent a


benign Tornwaldt cyst or a malignant craniopharyngioma.

19-Headache is not considered to be a cardinal symptom of chronic


sinusitis.

20-Imaging is not indicated in uncomplicated acute sinusitis.

21-The degree of sphenoid pneumatization is classified into three


types: sellar( 86 % ) , presellar ( 11 %) , and conchal ( 3 % ) .

.
53

22-Large Onodi cells may cause the surgeon to become disoriented


and put anatomic structures such at the optic nerve and carotid at risk.

23-Removal of the anterior wall of the maxillary sinus does not weaken
the structural integrity of the midface, nor does it result in cosmetic
deformity. Therefore reconstruction is not required.

24-The frontal sinus opens into the middle meatus medial to UP in 88


% of patients and lateral to the uncinate in 12% of patients .

25-The most common cause of epiphora in elderly women is lacrimal


duct stenosis.

26-local transposition flaps (bilobed, rhomboid, and Z-plasty) provide


the best options for complex nasal defect repair.

27-The most appropriate imaging technique for the assessment of the


obliterated frontal sinus is magnetic resonance imaging.

28-Cytokines responsible for the migration of eosinophils in chronic


sinusitis include interleukin-4 and interleukin-5.

29-In HHT , the primary defect is an absence of muscle and elastic


tissue in the tunica media of the telangiectasia.

30-most common cause of permanent loss of the sense of smell is


upper respiratory infections.

31-Normal sweat chloride less than 50 mEq/L,,,60 or more = CF.

32-Preseptal cellulitis Anterior to the orbital septum.

33-Skull base invasion from NPC can be detected earlier with MRI than
with CT.

34-The current treatment recommendation for T1 nasopharyngeal


tumors includes Definitive RT and elective neck irradiation.
54

35-In nasophar. cancer the most common site of metastases is the


Bone.

36-The nerve supply to the nasopharyngeal mucosa is derived from the


maxillary division of the trigeminal nerve, which arises in the
pterygopalatine fossa.

37-Fibrous dysplasia:Progressive replacement of normal cancellous


bone by fibrous tissue and immature woven bone.

38-Surgery is the mainstay of therapy for tumors of the nose and


paranasal sinuses.

39-The Caldwell-Luc approach to biopsy of a maxillary infrastructure


mass should be avoided because it can lead to cancer seeding of cheek
soft tissues and complicate subsequent definitive surgery.

40-Only 10% of malignant tumors of the paranasal sinuses present are


seen with neck nodal metastasis.

41-Adenocarcinoma represents the second most common malignancy


of the sinonasal cavities. It most commonly involves the ethmoid
sinuses .

42-NK/T-cell lymphoma is the most common type of lymphoma


affecting the sinonasal tract.

43-In orbital-sparing maxillectomy the Integrity of the orbital floor


support.

44-Most patients with fibrous dysplasia can be observed. When lesions


are disfiguring or create functional impairment, recontouring is
appropriate.

45-Most patients with fibrous dysplasia can be observed. When lesions


are disfiguring or create functional impairment, recontouring is
appropriate.

.
55

46-orbital floor fracture:should be treated urgently to prevent


irreversible ischemic damage to the inferior rectus.

47-Complete removal of sinus mucosa is essential to reduce the


incidence of delayed mucocele formation.

48-Facial and nasal fractures are less common in children than adults .

49-Imaging is not indicated in uncomplicated acute sinusitis.

50-Basal cells:are responsible for the regenerative capacity of the


olfactory neuroepithelium.

51-The thinnest part of the anterior skull base is the lateral lamella of
the cribriform plate,

52-The degree of sphenoid pneumatization is classified into three


types: sellar( 8 6 % ) , presellar ( 1 1 %) , and conchal ( 3 % ) .

53-Large Onodi cells may cause the surgeon to become disoriented


and put anatomic structures such at the optic nerve and carotid at risk.

54-patients with suspected CSF rhinorrhea, the skull base should be


evaluated first with a high-resolution CT scan.

55-carotid artery and optic nerve are dehiscent in the lateral aspect of
the sphenoid sinus in:25 %_6 % respectivelly.

56-The natural os of the maxillary sinus lies in a somewhat oblique


parasagittal plane.

57-The use of prophylactic antibiotics in traumatic CSF leaks is


controversial.

58-amoxicillin is still considered to be first-line therapy for acute


bacterial sinusitis.
.
56

59-Most chronic rhinosinusitis with nasal polyposis is characterized by a


pronounced eosinophilic inflammation.

60-The thinnest portion of the anterior skull base is at the medial


aspect of the fovea ethmoidalis (the lateral lamella of the cribriform
plate) .

61-medications may be considered "allowed " at various stages of


pregnancy-for example, Loratadine.

62-Intrathecal fluorescein may be introduced via a lumbar puncture or


spinal drain for better identification of a CSF fistula.

63-Radionuclide study: a highly sensitive test to detect a CSF leak.

64-B2-transferrin. When present, a CSF leak is confirmed; however, a


negative test result does not exclude a diagnosis of CSF leak.

65-Decongestant nose drops,beta-blockers, (ACE) inhibitors—and oral


contraceptive use,causes Rhinitis Medicamentosa.

66-Le Fort III: craniofacial dysjunction.

67-Le Fort II: pyramidal fracture with detachment of the maxilla.

68-Le Fort I: isolated detachment of the alveolar process.

69-Churg-Strauss SyndromeTriad of (1) asthma, (2) allergic rhinitis,


and (3) eosinophilia, p-ANCA positive in 70 %.

70-Glioma:60 % Extranasal, 30 % intranasal, 10 % combined.

71-Angiography and embolization Complications:Blindness, necrosis,


stroke, facial pain, paresthesia.

72-Deficiency of von Willebrand protein leads to defective platelet


adhesion and decreased activity of clotting factor VIII;
57

73-Acoustic rhinometry = uses sound waves to measure the cross-


sectional area at points along the nasal airway.

74-Foster Kennedy Syndrome = unilateral anosmia, optic atrophy, and


papilledema due to frontal lobe masses.

75-Types of frontal cells:I: Single cell above the agger nasi, II: Two or
more cells above the agger nasi, III: Single cell extending from the
agger nasi superiorly into frontal sinus, IV: Cell isolated within frontal
sinus.

76-Keros classification:I: Cribriform plate 1–3 mm inferior to fovea


ethmoidalis, II: 4–7 mm inferior, III: 8–16 mm inferior.

77-Rhinoscleroma is caused by Klebsiella rhinoscleromatis with


histopathology showing Mikulicz cells (macrophages containing
pathogen) and Russell bodies(plasma cells).

78-On sinus MRI, dried secretions show as hyperintense on T1,


hypointense on T2, and polyps show hypointense on T1, hyperintense
on T2.

79-Nasal polyps in a pediatric patient suggest CF until proven


otherwise.

80-Encephalocele: always has a dural connection, transilluminates,


positive Furstenberg test, associated with meningitis, histologically an
ependymal lined sac that communicates with the CSF spaces.

81-Surgical approaches for dermoid nasal component: vertical midline


dorsal excision, external rhinoplasty, bicoronal.

82-NPC is treated by radiation. In general, patients with stage I and II


NPC are treated by radiation only, whereas stage III and stage IV
patients are treated by concurrent chemotherapy and radiation.

83-About 10% of NPC patients the lesion is submucosal.

.
58

84-Frank epistaxis, however, is not typical of NPC.

85-The nodal metastasis is usually located in the superior aspect of the


neck, corresponding to high level V and level II. The first echelon of
nodal metastasis is in the retropharyngeal nodes. NPC.

86-The IgM and IgG class of antibodies to antigens on the EBV are
raised in acute and convalescent phases of infection. NPC.

87-EBV is present for the remaining life of the person within specific
circulating B lymphocytes or shed through saliva.in NPC.

88-nasolabial angle:ideal is 90°to 95° in men and 95° to 110° in


women.

89-McCune–Albright syndrome affects mostlyfemales and is


characterized by polyostotic fibrous dysplasia with cutaneous
hyperpigmentation, and endocrinopathy,

90-Fractures of the orbital floor can cause diplopia on upward gaze due
to entrapment of the inferior rectus muscle.

91- Flutamide(nonsteroial androgen) druge used 3 wk before op.in


NPAF specially for intracranial ext. fro shrincage of tumore.by 30%.

92-proptosis: abnormal protrusion of the eyeball.

93-abducens nerve, is most commonly cranial nerve involved in


nasophar ca. Genetic associations with nasopharyngeal carcinoma
include HLA-BW46 and HLAB17.

94-The most common benign lesion of the PNS is the inverted


papilloma.

95-The foramina of Breschet are venous drainage channels located


in the posterior wall of the frontal sinus.

.
59

96-complications of frontal sinus fractures. include mucoceles, severe


persistent pain, and infectious intracranial complications. Wound
infections, CSF leaks, numbness over the forehead area,

97-The anterior wall of the frontal sinus is significantly thicker than the
posterior wall.

98-Kennedy coined the term “functional endoscopic sinus surgery”.

99-CRS:broad spectrum antibiotics, a nasal steroid spray, saline


irrigation. Strong consideration should be given to a tapered course of
oral steroids.

100-Caldwell (to visualize the frontal and ethmoid sinuses), Waters (for
the maxillary sinuses), lateral (for the anterior and superior walls of the
frontal,
maxillary, and sphenoid sinuses), and submental vertex views (for the
ethmoid and sphenoid sinuses).

101-The most common biofilm formers in CRS are Pseudomonas


aeruginosa , S. aureus, and Haemophilus influenzae .

102-Churg–Strauss syndrome is a rare granulomatous vasculitis


involving small to medium sized vessels and is characterized by asthma,
hypereosinophilia, and extravascular eosinophilic granulomas.

103-Nasal gliomas appear hypo- or isointense on T1-weighted images


and hyperintense onT2-weighted images.

104-Intranasal gliomas can present as pale mass in the nasal cavity


causing nasal congestion and obstruction.

105-Nasal encephaloceles occur as a result of herniation of intracranial


tissues through skull base defect.

106-Encephaloceles are compressible, trasilluminate and enlarge with


crying.
60

107-Nasal dermoid:MRI is more sensitive and specific than CT. appear


isointense on T1-weighted images and hyperintense on T2-weighted
images.

108-Up to 45% of nasal dermoid cysts have an intracranial connection.

109-Zinc sulfate therapy may enhance improvement in olfaction


following trauma.

110-congenital anosmia is Kallmann syndrome , an X-linked


disorder. Caused by mutation in the KAL gene,

111-Kallmann syndrome is characterized by hypogonadotropic


hypogonadism,

112-5–10% of adult patients with head trauma report olfactory loss.

113-Arhinia is defined as the congenital absence of a nose.

114-Vit.A used in treatment of anosmia.

115-mixed membranous–bony atresias being most common and


occurring in 70% of cases of choanal atresia.

116-The articulation between the dorsal septum and the nasal bones in
the midline is referred to as the "keystone" area.

117-continuous positive airway pressure is the first-line therapy in the


care of severe OSA.

118-Most patients with fibrous dysplasia can be observed.

119-Advanced NP carcinoma is technically inoperable. Multiple studies


have shown concomitant CRT has the best outcomes.

120-The cornerstones for the medical management of chronic sinusitis


continue to be prolonged broad-spectrum antibiotic therapy, irrigation
with normal saline, and topical nasal steroid spray.
.
61

121-The presence o f a bifid crista galli and enlarged foramen cecum i


s highly suggestive of intracranial involvement.

122-CT scan has become the gold standard for the evaluation of
chronic sinusitis in children.

123-In addition to nasal obstruction, pituitary disorders along with


dental and facial anomalies are seen with congenital nasal pyriform
aperture
stenosis.

124-Orbital floor fractures known as blowout fractures are


commonly encountered as isolated fractures. The ideal time for the
repair is often 7–14 days after the injury;

125-Nasal intubation may be necessary if mandible has to be


dislocated.

126-pterygopalatine fossa communicates laterally with the masticator


space via the pterygomaxillary fissure.

127-pterygopalatine fossa connects to the orbit via the inferior orbital


fissure.

128-The nasal cavity connects with the pterygopalatine fossa via the
sphenopalatine foramen.

129-Sinonasal imaging is performed for two main


indications:inflammatory disease and mass.

130-JNPA.The most common feeding vessels are the distal branches of


the internal maxillary artery, ascending pharyngeal artery, vidian artery,
accessory meningeal artery, mandibular artery, and facial artery.

131-Embolization should be performed within 24 hoursof surgery. A


prolonged wait after embolization can allow the development of
significant collateral blood supply and subsequent revascularization of
the tumor .
62

132-The KTP laser is associated with a high incidence of postoperative


nasopharyngeal stenosis and is not recommended for adenoidectomy.

133- Common causes of failure of endoscopic sinus surgery are


lateralized middle turbinate, missed middle meatal antrostomy,
maxillary ostium stenosis, frontal recess scarring, residual ethmoidal air
cells, and adhesions..

134-The Müller maneuver is performed in an awake patient who


generates negative pressure by inhaling against a closed glottis with
the nose and mouth closed, which triggers airway collapse.

135-The lateral wall of the frontal recess is defined by the lamina


papyracea of the orbit.

136- FESS is a minimally invasive technique used to restore sinus


ventilation and normal function.

137-CSF:Clear watery discharge from ear or nose: appears during


straining or leaning forward (Dandy maneuver); salty taste.

138-Prenatal use of antithyroid medicat. (carbimazole).cause choanal


atresia.

139- Sites with low Lymphatic and blood vess. (Lip, Para nasal sinus,
Glottis).

140-Goal of all FESS procedures is the restoration of mucociliary


clearsnce,not the removal of so-called diseased tissue.

141-Craniopharyngioma arises from Rathke’s pouch.

142-Chordoma arises from remants of notochord.

143-Beta-2 transferrin is specific for CSF.not found in tears,nasal


secretions and serum.is also seen in aqueous humour& serum of
patients with ch.liver dis. Due to alcohol.
63

144-Horgan’s operation is transantral ethmoidectomy.

145-Sluder’s neuralgia:pain in the lower half of face in addition to


congestion of nose,watering of eyes and rhinorrhoea.(V2).

146-Trotter’s triad in NPC:conductive HL,ipsilateral immobility of palate


and neuralgic pain (V3).

147-Most common cranial n.involved NPC is CN VI causing diplopia.

148-Aspergillus hyphae are septate and acute branching.

149-Rhizopus & mucor, hyphae in these fungi are broad,non-septate


and irregularly branching at 90’ .

150- ACP can be best treated by ESS and removal of its stalk from
maxillary S.
151-7 cm from ant.nasal spine to ant. Of sphenoid, 9cm from ant.nasal
spine to inside sphenoid sinus. 7cm from ant.nasal spine to sphenoid
ostia.

152-Concha bullosa :38% with RS &20% bilateral.

153- nasal congestion :caused by beta-


blokers,antidepressant,contrceptive pills and diuretics.

154-the most common cause of oroantral fistula is:dental extraction of


upper second premolar tooth .
.
155-a congenital choanal atresia is:commoner in female .

156-The “gold standard” for treatment of advanced Wegener’s


granulomatosis is glucocorticoids and cyclophosphamide. Methotrexate
and corticosteroids have been used in milder forms.

157-Well-organized fibrous stroma is a prominent histologic feature of


juvenile nasopharyngeal angiofibroma.
.
64

158-Cranialization provides the best means of managing the through-


and-through fracture of the frontal sinus.

159-Incisional components of the midfacial degloving exposure Include


intercartilaginous (between the upper and lower lateral cartilages)
incisions.

160-Ohngren’s line divides the facial skeleton into two parts as it


passes from the medial canthus of the eye to the angle of the
mandible.

161-Interventional and diagnostic angiography are indicated for


embolization of vascular lesions.

162-Cluster headaches have severe lancinating pain that often


awakens patients. They are associated with unilateral nasal congestion,
rhinorrhea, and lacrimation.

163-Cytokines responsible for the migration of eosinophils in chronic


sinusitis include interleukin-4 and interleukin-5.

164-In primary ciliary dyskinesia the mucociliary clearance time is


lengthened.

165-Biopsy in Wegener’s granulomatosis is not pathognomonic,

166-Rhinoviruses are the etiologic agents in more than 50%


of cases of the common cold.

167-Bowman’s glands:glands contribute to mucus production


in the olfactory neuroepithelium.

168-Boyette’s solution is( 4% lidocaine, phenylephrine, and


sodium chloride, with sterile water added to make 200 mL.).

169-The maxillary line is an important landmark for


endoscopic DCR.
65

170-main focus of FESS.. ostiomeatal unit.

171-goal of FESS is to restore physiologic drainage patterns.

172-Caldwell–Luc procedure:the opening into the maxillary sinus must


be made high and lateral to the roots of the canine teeth.

173-An air-fluid level is more predictive of acute rhinosinusitis but may


also be seen in chronic rhinosinusitis.in CT.and plain XR.

174-Approximately 80% of all orbital complications from ethmoid


sinusitis are in children.

175-Zinc has been found to stimulate the growth of Aspergillus


fumigatus. presence of zinc in aspergillosis maxillary sinusitis is related
to the overfilling of dental paste.

176-US may have a role in the diagnosis of acute sinusitis during


pregnancy.

177-The most common presentations of sinus disease in children are


cough and rhinorrhea.

178-Pseudomonas aeruginosa play an important role in cystic fibrosis


and in the immuno- compromised adult or in nosocomial or intensive
care unit (ICU) acquired sinusitis.

179-nitrate testing has been proposed both for the identification of


Kartagener syndrome KS as well as a general screening tool for
rhinosinusitis.

180-The cilia beat normally at a frequency of approximately 1000


cycles per minute to propel mucus to the ostia.

181- Samter’s triad of nasal polyposis, aspirin sensitivity, and asthma


may initially manifest with sinonasal symptoms.
66

182- About 80% of children undergoing sinus surgery have a positive


skin test response to inhalant allergens.

183- About 50% of children with otitis media will ultimately be


diagnosed with allergic or infectious sinonasal disease.

184- Secondhand smoke exposure may thus lead to diminished


mucociliary clearance in children and subsequently to rhinosinusitis.

185- GERD be evaluated and treated in all children with refractory


rhinosinusitis in which FESS is being considered.

186- Primary ciliary dyskinesia is a rare disorder involving structural or


functional abnormalities of cilia that may be isolated or occur as a
component of Kartagener’s syndrome.

187- Cystic fibrosis (CF) is the most common life-limiting recessive


disorder in the Caucasian population, on the 7q31 chromosome.

188- Healthy paranasal sinus epithelium generates nitric oxide in


relatively large quantities under normal inflammatory conditions,
altered production of nitric conditions (11). Under ioxide may inhibit the
antiviral and Bacteriostatic protection mechanisms afforded by this
substance in the sinuses .

189- The blanket of mucous contains mast cells, neutrophils,


eosinophils, lysozymes, and immunoglobulin A (IgA) and is renewed
every 10 to 15 minutes.

190- Smooth mucociliary flow is considered to be about 0.8 cm/min,


while mucostasis is less than 0.3 cm/min.

191-Small frontal sinuses. are associated with Down syndrome, Apert


syndrome, Treacher Collins syndrome, and pituitary dwarfism.

192-Extreme frontal sinus development is associated with osteogenesis


imperfecta tarda, Turner syndrome, Klinefelter syndrome, and
acromegaly.
67

193-The frontal sinus is hypoplastic in 30% of skulls examined.

194-The relationship between the ethmoid infundibulum and the skull


base, especially the frontal recess, depends on the uncinates process.

195-The indications for surgical treatment of NPC are currently for local
and regional recurrences.

196-The most common cranial nerve to be affected postradiation, other


than the cochlear-vestibular nerve, is the hypoglossal nerve. In NPC.

197-two thirds of NPC patients are classified as stage III or IV at the


time of diagnosis.

198- 10% of NPC patients the lesion is submucosal. more aggressive


disease.

199- The most common nerves affected in descending order of


frequency are the sixth, fifth, twelfth, and ninth/tenth nerves.in NPC.

200- NPC :60% of patients presented with a neck lump. blood-stained


saliva or sputum (41.3%). Deafness was more commonly reported
(30.2%). This is due to otitis media with effusion, unilateral tinnitus.
Persistent headaches. Cranial nerve palsies are encountered in about
10% of the patients.

201- most common environmental factor associated with NPC is diet.(


salted fish, eggs, and vegetables).

202-One exception to the latter was malignant minor salivary gland


tumors arising in the nasal cavity and paranasal sinuses, in which case
adenocarcinoma was the most common type.

203-A more common metastasis to the PPS is from nasopharyngeal


carcinoma.

204-inferior turbinate functions as the thermostat of the nose.


.
68

205-middle turbinate is the principal landmark for endoscopic


microsurgery of the paranasal sinuses.

206-indications of Caldwell -Luc's operation :1.Recurrent antrochoanal


polyp 2. Foreign body in antrum 3. Dentigerous cyst 4. Oroantral fistula
5. Biopsy for tumors of maxilla 6. As an approach to sphenoid,
pterygopalatine fossa, orbital decompression .

207-Bernhard Fraenkel triad include: 1. Fetor 2. Crusting of nasal


mucosa 3. Atrophy of nasal mucosa.

208-Lefort I type of fracture It is also known as Guerin's fracture .

209-inverted papilloma:MRI:exhibits a cerebriform-columnar pattern.

210-Inverted papilloma is suspected to have a viral etiology. with a


prevalence of serotypes 6, 11, 16, and 18. The last two serotypes have
been specifically found to be associated with inverted papillomas.

211-The association of inverted papilloma with squamous cell


carcinoma. 3.4%and 9.7%.

212-Inverted papilloma most frequently arises from the lateral nasal


wall in the fontanelle area. The maxillary sinus is the second most
commonly affected site,

213-inverted papilloma is the most common surgical indication for


benign tumors of the sinonasal tract.
.
214-Osteoma and inverted papilloma are the first and second most
frequent benign tumors of the sinonasal tract.
215-The Nasopharynx is the most common unknown primary with
glands in the posterior triangle.
216-Narrow/crowded nasopharynx may be part of Down's syndrome,
Crouzon's syndrome, Apert syndrome or Treacher Collin syndrome.
217-Recurrence of juvenile angiofibroma after surgical removal is
common (36–39 percent. reducing recurrence by meticulously drilling
out the pterygoid base, the pterygoid canal and the basisphenoid.
Recurrence is common and can be predicted by CT features
69

218-JNA:With the expansion of the tumour, anterior bowing of the


posterior antral wall may be seen on plain radiographs and CT. This is
the ‘antral sign’ described by Holman and Miller in 1965 .
219-CT diagnosis of JNA is based on two constant features which are:
1. mass in the nose and pterygopalatine fossa;
2. erosion of bone behind the sphenopalatine foramen at the root of
the pterygoid plate and clinical findings.
220-JNA:The diagnosis is made clinically and radiologically.
221-JNA :Intracranial invasion appears in 10–20 percent of all patients

222-Nasal encephaloceles occur as a result of herniation of meninges,


with or without brain.

223-In general, MRI is more sensitive and specific than CT; it is


superior for visualizing soft tissues and diagnosing intracranial
extension and is thus the preferred imaging study. Nasal dermoid cysts
appear very hyperintense on T1-weighted MRI images.

224-Nasal dermoid cysts have an intracranial connection in up to 20 -


45% of cases.

225-nasal dermoid cysts are the most common, accounting for 61% of
all midline nasal lesions .

226-To date no randomized controlled trials of antifungal therapy in


patients with classic AFRS have been performed.

227-Pathophysiology The development of AFRS requires that an


individual genetically predisposed to fungal allergy be exposed by
inhalation of the mold spore.

228-Itraconazole 100 mg 1-2 po bid with meals Active against many


dematiaceous fungi variably active against Aspergillus; species; not
active against Zygomycetes Best absorbed in acidic stomach co-
administer with cola.

229-AFRS;The diagnosis is made histopathologically from the finding of


an allergic mucin characterized by necrotic inflammatory cells,
eosinophils, and Charcot-Leyden crystals. Culture of the allergic mucin
reveals a variety of fungal species.
70

230-Charcot-Leyden crystals (a by-product of eosinophil


degranulation).

231-CT findings of metallic or calcified densities within an opacified


sinus cavity have a positive predictive power of around 60% for
afungus ball and a high negative predictive power. Thus mycologic and
histologic studies are essential to confirm the diagnosis.

232-Chandler et al described a classification scheme for orbital


complications of sinusitis in 1970. first stage, preseptal periorbital
cellulitis. orbital cellulitis. Subperiosteal abscess, orbital abscess.
cavernous sinus thrombosis.

233-Visible pulsations on endoscopy and enlargement of the mass with


ipsilateral internal jugular vein compression (Furstenberg’s sign) help to
confirm thediagnosis. (encephalocel).

234-Unilateral polyps should raise concern for allergic fungal sinusitis


or inverted papilloma.

235-Thornwald's cyst: Derived from pharyngeal notochord remnant


and found in the nasopharynx, in the midline superior to adenoidal pad.

236-The gold standard in the diagnosis of sleep apnea is


polysomnography.

237-Central apnea is caused by a lack of drive in the central nervous


system.

238-Sleep apnea is defined as more than five episodes of apnea per


hour

239-Surgery for unilateral choanal atresia can be postponed until


school age.

240-definitive surgical repair of bilateral choanal atresia is performed


during the first weeks or months of life.
71

241-Choanal atresia is bony in 90% of cases and membranous in only


10%.

242-The main indications for CT scanning of the nose and paranasal


sinuses are chronic sinusitis, trauma (especially frontobasal fractures),
and tumors.

243-Odorants with a trigeminal component(menthol, acetic acid,


formalin),

244-Active anterior rhinomanometry the test cannot be performed


when one nasal cavity is completely obstructed.

245-Rathke's pouch tumor: Cyst lined with squamous cells, filled with
kerating, and located in the nasopharynx.

246-Nasal Glioma:Herniation of brain tissue through the floor of the


anterior cranial fossa.

247-Hysteric anosmiacs will deny smelling ammonia which is sensed by


the trigeminal nerve, not the olfactory nerve.

248-Rhinoscleroma. caused by Klebsiella:Mickulicz cell" characteristic.


Treated by Tetracycline.

249-Bilateral choanal atresia is treated initially with McGovern's nipple.

250-If vessel ligation fails to control epistaxis, angiography with


Gelfoam embolization of the offending vessel may be of some aid.

251-Posterior inferior bleeding usually comes from the sphenopalatine


vessels. Superior bleeding usually comes from the anterior and
posterior ethmoidal arteries.

,
72

252-Lupus vulgaris:Form of tuberculosis involving external nose and


adjacent facial areas. Produces brownish, gelatinous nodules which may
coalesce.

253-production of nasal secretions; over 1000 cc of secretions


produced per day; acts to clear contaminants of inspired air&
humidified to 75-80%,

254-Olfactory areas: Nonciliated pseudostratified columnar epithelium


with serous glands of Bowman and bipolar olfactory cells.

255-Anterior part of nasal cavity to submandibular nodes. Posterior


part of cavity to upper deep cervical glands.

256-Sphenopalatine A: Most of the posterior part of the nasal septum


and most of the lateral wall of the nose, especially posteriorly.

257-The anterior ethmoid, maxillary, and frontal sinuses drain into the
ethmoidal infundibulum.

258-NLD:the duct is located on the lateral wall from 3-3.5 cm behind


the posterior margin of the nostril .

259- The Aspergillus species demonstrate more narrow hyphae with


regular septations and 45-degree branching on histopathology.

260- Aspergillus species most commonly responsible for invasive


disease in the United States is A. fumigatus.

261-Nasal endoscopy may show necrosis of the nasal mucosa, which is


indicative of
mucormycosis,

262- Rapid diagnosis of invasive fungal rhinosinusitis in the


immunocompromised patient, with the use of immediate fungal
staining, culture, and biopsy, if possible,
is critical to rapid implementation of therapy..
73

263- earliest symptom of mucormycosis is often anesthesia of the


nose.

264-Chronic Granulomatous Invasive Fungal Sinusitis: Primarily found


in Africa.

265-perennial rhinitis are almost invariably allergic to house-dust mite


and typically have
more than one allergy.

266-Malignant tumors such as rhabdomyosarcoma and


olfactoryneuroblastoma can mimic JA;

267-Histologic confirmation is not usually necessary for the diagnosis


of JA.

268-JNA:residual lesions tend to grow submucosally, contrast-enhanced


CT or MRI plays a key role in their early detection. most residual lesions
are diagnosed within 1 year after surgery.commonly monitor residual
lesions with MRI to assess their pattern of growth..

269- sino nasal tumor Metastases Rare [ even in advanced ] LN < 10%.

270-Mucoceles The primary goal is marsupialization and not


enucleation.

271-A 6- to12-week course of itraconazole can be given before


undertaking surgery. This will localize the disease and not only make
surgery easier but allow more mucosa to be preserved. A 6-week
postoperative course also reduces recurrence rates.

272-Topical and syst.steroids are the mainstay of medical treatment in


patients with chronic rhinosinusitis of all causes because the underlying
mucosal pathology has a large inflammatory component and steroids
reduce this.
.
74

273-In unilateral nasal purulent disease, atypical infections such as


aspergillosis should be borne in mind.

274-Antibiotics that cover the normal range of pathogens in chronic


rhinosinusitis include amoxicillin with clavulanic acid, clindamycin, or
metronidazole and a second- or third generation cephalosporin.

275-Be aware that only 6% of patients with allergic rhinitis have


hypertrophic or polypoidal mucosa, and that the majority of patients
with nasal polyps do not have atopic disease.

276-If the patient with purulent rhinosinusitis is not responding to the


appropriate antibiotics, then the possibility of an immune deficiency,
ciliary dysmotility, or an atypical infection, e.g., aspergillosis, should be
considered.

277-As many as 20% of patients with allergic rhinitis have asthma.

278-Most people sneeze, but more than three sneezes together are
almost pathognomonic of allergic rhinitis.

279-Be careful about operating on someone who has a sensation of


“blockage” but whose airflow is normal,

280-If the patient has persistent unilateral nasal obstruction, the most
common cause is septal deviation.

281-The nasal cycle happens in about 80% of patients every 3−8


hours, with one side being congested while the other is clear.

282-In periorbital cellulitis, an assessment of vision, parenteral


antibiotics, and topical decongestants should be given.

283-Any unilateral nasal polyp should be treated with suspicion, even if


it looks harmless..
75

284-It is vital to send the whole specimen for histological examination,


as 8−15% of inverted papillomas are associated with malignancy or
atypia.

285-inverted papilloma is typically whiter and less translucent than


inflammatory polyps.

286-Anterio rrhinorrhea is usually secondary to viral or allergicrhinitis.


The reason for anterior rhinorrhea in a viral rhinitis is not only an
increase in mucus production but also paralysis of the cilia.

287-patient with anosmia who has had previous surgery is unlikely to


regain any sense of smell if systemic steroids have not helped.

288-Aufricht . said: "Where the septum goes, there goes the nose“ .

289-The vomeronasal organ (Jacobson's organ) is essential for the


social and mating behavior of all mammals, tiny, paired, blindly
terminating canals located in the anterior nasal septum.

290-The nasal mucosa is the "front line" of the human immune system.

291-Air entering the nose is warmed to a relatively constant 31-34°C.

292-Histologic confirmation is not usually necessary for the diagnosis


of JAF. A definitive diagnosis is usually made with clinical and imaging
findings

293-Embolization should be considered as part of the preoperative


preparation and should be performed shortly before the surgical
procedure (<24 hours), because JAF is known to achieve rapid
revascularization.

294-Creating an antrostomy does not solve all maxillary sinus


problems.

295-A retrograde approach to the uncinate process decreases the risk


of orbital injury.
76

296- WHO classificaiton of NPC:


Type I: Keratinizing SCC.Not associated with EBV
Type II: Nonkeratinizig SCC associated with EBV.
Type III: Undifferentiated Carcinoma associated with EBV.

297- NPC:The major histcom.complex is on the short arm


ofchromosome 6

298-HLA is the only genetic system to show strong association with


NPC.

299-Caldwell view for post. Ethmoid cells.

300-Incisive foramen : palatine n. & nasopalatine artery.

301-Nasoturbinal : Agger nasi cell .

302-Amount of the air the nose breath every 24 hr : 500 cubic feet

303-Crepitus noted on palpation confirms the suspicion of a nasal


fracture.
304-Saddle nose is corrected by filling the dorsal concavity with a
cartilage graft taken from the septum, auricle, or rib.

305-After five years of age, adenoid size remains constant while the
nasopharynx increases in size.

306- More common conditions such as postnasal drip and


laryngopharyngeal reflux can cause an irritative pharyngitis.

307-Combination of bony sclerosis and mucosal thickening is often


seen in chronic sinusitis.

308-To quickly identify a gadolinium-enhanced T1WI: nasal mucosa is


white, fat is white, and CSF and vitreous are black.
.
77

309-The Waters view can show the maxillary sinuses, anterior ethmoid
air cells, and orbital floors.

310-The Caldwell view displays the frontal sinuses and posterior


ethmoid air cells.

311-Awake fiberoptic nasotracheal intubation is now a common


technique employed in the management of the dificult airway in both
the operating room and intensive care unit settings.

312-Awake fiberoptic intubation should be considered in the obese


patient with obstructive sleep apnea (OSA), in conditions resulting in
supraglottic obstruction (e.g., edema, supraglottic tumor).

313-Dermoid cysts consist of epithelium-lined cavities filled with


skin appendages (e.g., hair, hair follicles, sebaceous glands).

314-In 8% to 14% of DCR surgeries, dacryoliths composed of


precipitated organic material will be found.

315-Over-reduction of the turbinates in some patients may lead to loss


of sensation of nasal airflow. This is termed empty nose syndrome.

316-Lateral traction on the cheek opens the nasal valve. Relief with
this maneuver pinpoints the obstruction to the nasal valve area. This is
called eponymously a Cottle maneuver.

317-Narrowest point of the nose is at the internal nasal valve.

318-frontal sinus with a single agger nasi cell (Kuhn-Bolger type1).

319-An antrochoanal polyp enters the nose via an accessory ostium in


the posterior fontanelle of the maxillary sinus, but it does not usually
have to be connected to the natural ostium.
.
78

320- Induction of elective general anesthesia for ESS in the poorly


controlled asthmatic patient may cause severe bronchospasm and even
death.

321- If the monopolar cautery is used adjacent to the cribriform plate


at the entry point of the anterior ethmoid artery, it is likely to cause a
CSF leak.

322- A posterior ethmoid cell that pneumatizes over the sphenoid sinus
(an Onodi cell) can be adjacent to the optic nerve or cavernous/carotid
complex.

323-Aggressive postoperative irrigation by the patient with 20 mL of


saline once or twice a day reduces the amount of postoperative office
endoscopic débridement required.

324-blood in the endoscopic field correlates with heart rate, it does not
correlate with mean arterial blood pressure.

325-Oxymetazoline is the decongestant of choice because of its lack of


cardiac side effects.

326-Profuse rhinorrhea that accompanies the common cold can also be


managed with ipratropium bromide inhalations.

327- usual capillary hemangioma in the infant subglottis.

328-Adult hemangiomas are usually found at or above the level of the


vocal folds. more often the cavernous form.

329-The soft tissue radiograph is the single most important view in


children.

330-The CO2 laser delivers energy that is absorbed by intracellular


water, effectively vaporizing the cells.
Part III

THROAT
Part 3

Throat
79

1-Plunging ranulas are mucoceles or retention cysts of the floor of


mouth that usually present as slow-growing, painless, submental
masses. They arise from the sublingual gland and are defined as
plunging when they extend through the mylohyoid muscle into the
neck.

2-Patients with laryngoceles present with hoarseness, cough, dyspnea,


dysphagia, a foreign body sensation, or anycombination of these
symptoms.

3-laryngocele is defined as an abnormal dilation or herniation of the


saccule of the larynx.

4- In mandibular recostruction:four most commonly used osseous free


flaps are the radial forearm, the scapula, the iliac crest, and the fibula.

5- Nasopharyngeal Ca.risk factores:genetic predisposition, viral


infection with the Epstein–Barr virus (EBV); dietary factors, including
salt-cured fish; and environmental factors such as sawdust and smoke
inhalation.

6-Squamous cell cancers of the oral cavity are primarily treated


surgically, while those of the oropharynx are primarily treated with
definitive radiotherapy (RT).

7-Leukoplakia is usually a benign condition that is unlikely to progress


into
cancer (5%). Erythroplakia is more likely to be malignant at the time
of the initial biopsy (51%).

8-Cobblestoning of the mucosa in the posterior orophar- ynx indicates


the presence of submucosal lymphoid hypertro- phy and is often seen
in the setting of infection, allergic rhinitis, and reflux.

9-Tonsillar asymmetry is most often benign, but when the enlarged


tonsil has an atypical appearance, lym- phoma must be considered .
80

10-Aphthous ulcers, or the common canker sore, are painful white


ulcers that can be on any part of the mucosa but are commonly present
on the buccal membrane.

11-White plaques that may represent oral thrush, which easily scrapes
off with a tongue blade, or leukoplakia, which cannot be removed.

12-HPV :proliferate on sq.epithelium mainly so seeding of papilloma


lower down suggest metaplsia.

13-The only absolute contraindication to primary voice restoration is


separation of the party wall at the puncture site.

14-High-resolution CT scanning is the imaging modality of choice to


stage the cNO neck in oral cavity cancer and is superior to MRl or PET-
CT in evaluation of the neck.

15-most commonly encountered cause for squamous cancer of the


tongue Base , (HPV) type 16 infection.

16-The three main indications for tracheotomy in the pediatric


population are respiratory failure and anticipated need for prolonged
ventilation, upper airway obstruction, and providing access for
pulmonary toilet.

17-Permanent vocal fold augmentation materials include:Silastic.

18-Temporary vocal fold augmentation materials include:Hyaluronic


acid.

19-A recent large review showed that iatrogenic injury is the most
common cause of UVFP.( unil.vocal fold paralysis).

20-The retropharyngeal space is medial to the carotid sheath, anterior


to thedanger space , and posterior to the buccopharyngeal fascia.

21-Parastomal recurrence is usually unresectable .


81

22-Laser excision of T3 supraglottic tumors is possible, and reports


have demonstrated less dysphagia and aspiration with this procedure
than with open techniques.

23-Severe dysplasia of the vocal fold may becomes invasive carcinoma


in up to 25% of patients.

24-The most common secondary primary cancer in patients with


laryngeal carcinoma is bronchogenic carcinoma.

25-Botulinum toxin inj. of the thyroarytenoid muscle is currently


considered the treatment of choice for patients with adductor
spasmodic dysphonia.

26-Laryngeal tuberculosis is one of the most common granulomatous


diseases of the larynx. may be mistaken for laryngeal carcinoma.

27-True sulcus (sulcus vergeture) that is the adherence of the vocal


fold epithelium to the vocal ligament secondary to the absence of the
superficial layer of lamina propria.

28-Vocal process granulomas may take 6 months or more to heal on


antireflux therapy.

29-presbylaryngis may result in secondary muscle tension dysphonia.

30-Isolated laryngeal candidiasis is usually secondary to inhaled


corticosteroid.

31-child with suspected epiglottitis should be taken to the operating


room immediately to establish the diagnosis and secure an airway.

32-Closure of all mucosal lacerations is a major tenet of managing


laryngeal fractures. Fractures should be reduced within 72 hours of
injury.

33-CT scan should be obtained in all patients with suspected


laryngealtrauma.
82

34-Symptoms of Thornwaldt’s cysts include intermittent or persistent


postnasal discharge of tenacious mucus associated with odynophagia,
halitosis, unpleasant taste, and, occasionally, a dull occipital headache.

35-Saccular cysts are best diagnosed with direct laryngoscopy.

36-Ludwig angina is commonly caused by Alpha-hemolytic streptococci.

37-The tonsil is the most common location for HPV-related squamous


cell carcinoma.

38-Normally, the recurrent laryngeal nerve is encountered posterior to


the inferior thyroid artery.

39-The external branch of the superior laryngeal nerve runs


posteromedial to the superior thyroid artery.

40-The glossopharyngeal nerve courses between the internal and


external carotid arteries with the styloid muscles.

41-Tumor classification is a more significant prognostic indicator than


nodal classification in HPV-related oropharyngeal cancer.

42-The Superficial lamina propria(SLP) is responsible for the elasticity


that allows TVC vibration and phonation.

43-Superior laryngeal nerve. Innervates the cricopharyngeus.

44-Inflammation in the masticator space, most commonly odontogenic


in origin.

45-Trismus is especially worrisome in a patient with an oral cavity or


oropharyngealcancer and may imply invasion of the pterygoid
musculature.

46-A tracheotomy is generally described as a procedure that involves


opening the trachea. A tracheostomy is a procedure that exteriorizes
the trachea to the cervical skin,
83

47-The most common congenital CNS abnormality resulting in vocal


cord palsy is the Arnold–Chiari malformation.

48-In severe cases of laryngomalcia, polysomnography can be


performed to detect episodes of hypoxia or hypercapnia. The results of
this study can influence the decision to undertake surgical management
of the condition.

49-Hemilaryngectomy is the removal of one vertical half of the larynx.

50-Multiple warty lesions of “true” and “false” vocal cords. Laryngeal


papilloma.

51-The incidence of an unknown primary tumor is between 2% and


8% of all patients with head and neck squamous cell carcinoma.

52-The diagnosis of metastatic squamous cell carcinoma to the neck


can be diagnosed accurately by FNA biopsy.

53-Vocal cord fixation is usually a sign of paraglottic space invasion or


arytenoid involvement.

54-The TVC is made of stratified squamous epithelium,

55-The Broyle tendon is the anterior attachment of the vocal ligament


to the inner perichondrium of the thyroid. an anterior barrier to tumor
spreading.

56-oropharyn. tumor involving the medial pterygoid muscles is stage


T4a.

57-oropharyngeal tumor involving the lateral pterygoid muscle is stage


T4b.

58-Both postcricoid and posterior pharyngeal wall tumors have a


potential for bilateral metastasis.

59-Lymphatic drainage of the hypopharynx includes levels II and III,


retropharyngeal lymph nodes, and the nodes of Rouvière.
84

60-As many as 60% of all hypopharyngeal SCCs arise from the piriform
sinuses.

61-The nerve supply to the nasopharyngeal mucosa is derived from the


maxillary division of the trigeminal nerve, which arises in the
pterygopalatine fossa.

62-Treatment options for stage I and II oropharynx cancer include


definitive radiation therapy (RT) or primary surgical resection with or
without neck dissection.

63-The most common first-echelon nodal region for metastatic disease


from the base of the tongue and tonsils is level II,

64-Oral cancer subsites has the worst prognosis is Buccal mucosa.

65-Resection of cancers of more than one third of the oral tongue are
best managed by Free radial forearm flap.

66-The two most important factors predictive of local recurrence in


cancer of the oral tongue are Close/positive margins and perineural
invasion.

67-oral cancer:highest Frequency :Oral tongue,, lowest


Frequency:Hard palate.

68-The incidence of occult metastases from cancers of the oral cavity is


20% to 30%.

69-The most important predictor of neck metastases in early oral


tongue cancer is the tumor thickness.

70-The diagnosis of OSA is best made with a polysomnogram (PSG) .


Neck circumference > 1 7 inches (or 1 4 . 5 inches in women) is the
best prediction in this question.

71-Multilevel obstruction, including retropalatal and retrolinguaL is


present in 70% to 80% of patients with OSA.
85

72-Clinical outcomes in the assessment of (OSA) treatment its Daytime


sleepiness.

73-Accurate diagnosis of obstructive sleep apnea {OSA) can best be


made by:A home sleep study or an in-lab polysomnogram.

74-only absolute contraindication to primary tracheoesophageal


puncture, Separation of trachea and esophageal party wall.

75-Sentinel node biopsy has been shown to be useful in oral tongue


cancer.

76-Children with neurologic impairment, has the lowest chance for


successful decannulation.

77-A staccato cough is most commonly associated with Chlamydia in


infants.

78-H. influenzae:etiology for tonsillar hypertrophy,

79-Laryngeal electromyography:distinguish arytenoid dislocation from


recurrent laryngeal nerve injury in the setting of an immobile vocal fold.

80-Killian dehiscence is located posteriorly between the


cricopharyngeus muscle and the inferior constrictor muscle.

81-permanent treatment for vocal fold paralysis be undertaken, When


laryngeal electromyography (LEMG) demonstrates poor prognosis for
recovery.

82-Fundoplication is the only treatment presently available that


consistently reduces reflux of gastric contents.

83-Sulcus vocalis implies loss of lamina propria,

84-PPis do not prevent reflux; they simply raise the pH of gastric


secretions.
86

85-The intermediate layer has numerous elastin fibers and is also the
thickest layer of the lamina propria.

86-The purpose of CPAP is to stent open the airway, which typically


collapses during end-expiration in OSA patients .

87-Klebsiella pneumoniae is not a common pathogen in the larynx.

88-Interarytenoid activity maintains vocal fold approximation,

89-Stroboscopy is performed during vocal fold adduction and therefore


does not assess vocal fold movement.

90-Vocal process granulomas are highly associated with LPR.

91-Oral manifestation of Crohn disease: cobblestoning of the buccal


mucosal, angular cheilitis, or deep linear ulcerations in the
gingivalbuccal sulcus.

92-Retropharyngeal space infections are usually seen in children


younger than 5 years and result from upper respiratory tract infections
spreading to nodes of Rouviere.

93-laryngeal form of the TB is particularly infectious .

94-A common side effect associated with phenylephrine includes:Reflex


bradycardia.

95-Respiratory papillomas are most common at the transition between


columnar and squamous epithelium.

96-Apneic event: cessation of ventilation for 10 s or longer leading to


an arousal.

97-PIERRE ROBIN SEQUENCE Most commonly association with Stickler,


velocardiofacial syndromes.
87

98-adenotonsillectomy may be less successful in treating syndromic


children with sleep-disordered breathing/obstructive sleep apnea.

99-levels II, III, and IV neck dissection for N0 patients and


comprehensive levels I to V neck dissection for node-positive patients.in
hypopharyn.ca.

100-Chemotherapy is not used to treat hypopharyngeal and cervical


esophageal carcinoma as single-modality treatment except in palliative
treat.

101-PARTIAL PHARYNGECTOMY. This option is considered when


tumors are staged as T1 or T2 and are limited to the posterior or the
lateral wall of the pyriform sinus.

102-Histologic subtypes , hypopharyngeal SCC:basaloid,


lymphoepitheliomas and adenosquamous carcinoma.

103-Causes for surgical failure include submucosal extension,


involvement of the thyroid gland, and metastasis to paratracheal and
upper mediastinal lymph nodes.in hypopharyngeal ca.

104-Vocal fold paralysis is the second most common cause of stridor in


neonates after laryngomalacia. The most common etiology is secondary
to hydrocephalus from a malformation such as Arnold-Chiari.

105-The treatment of choice for aerodigestive tract foreign bodies is


rigid endoscopic removal under general anesthesia.

106-Objects such as button batteries can cause mucosal erosion in as


little 6 hours from the time of ingestion.

107-Posteroanterior and lateral plain films of the neck and chest are
the imaging studies of choice.in foreign body is aspirated or ingested,
88

108-Elective neck dissection is not indicated in most cases For patients


with hard palate malignancies, 10% to 25% present with cervical
metastases, most frequently to the submandibular and upper jugular
cervical nodes.

109-Squamous carcinoma of the hard palate is treated surgically. Small


lesions may be treated with transoral wide local excision. Partial or
subtotal maxillectomy is required for larger lesions or those involving
the maxillary antrum.

110-the oral cavity, tumor thickness has a direct correlation to


prognosis. In T1/T2 N0 tumors, depth of invasion greater than 5.17
mm at the primary site has been correlated with a risk of ipsilateral
regional lymphatic spread.

111-Cervical metastases are noted in approximately 50% of patients


with floor of mouth carcinoma. The submandibular lymph nodes
(levelIB) are most frequently involved. Bilateral metastases are
common with lesions of the anterior floor of mouth.

112-Carcinomas of the floor of mouth typically present in males in their


sixth decade of life.

113-Majority of patients with retromolar trigone carcinomas tend to


present with advanced disease. At the time of presentation, up to
50%of patients present with regional metastatic disease.

114-Oral tongue carcinomas(75%) are T2 or smaller. The surgical


treatment for limited, small (T1to T2) primary tumors is transoral wide
local excision.

115-The most common presentation is that of a painful ulcerated or


exophytic mass, in oral tongue.

116-With mandibular invasion, the inferior alveolar nerve may become


secondarily involved by perineural invasion.
.
89

117-Elective neck dissection is not typically advocated for lip carcinoma


because the incidence of occult metastases is low.

118-For tumors requiring resection of more than two thirds of the lip,
the reconstructive options are the Gilles fan flap, bilateral advancement
flaps, Karapandzic, or a free radial forearm with palmaris longus
tendon.

119-When lesions require resection of up to one third to two thirds of a


lip’s length, reconstructive options include a lip-switch (Abbe-Estlander).
or a Johansson stepladder flap.

120-Adverse prognostic features of lip primaries include perineural


invasion, bony involvement, cancer arising on the upper lip or
commissure, regional lymphatic metastasis, and age younger than 40
years at onset.

121-For patients with oral cavity and oropharyngeal malignancies, the


site of a second malignancy is most frequently in the cervical
esophagus.

122-Lower lip paresthesia may be an indication of perineural invasion


at the level of the mandibular foramen with these lesions.

123-The base of the tongue drains to the upper cervical lymphatics.

124-lymphatic drainage of the oral tongue varies by the region within


the tongue. The tip drains preferentially to submental nodes, whereas
the lateral tongue drains primarily to the levels I and II.

125-Adenocarcinoma presented more commonly in women than men


and was most frequently diagnosed on the hard palate.

126-One of the most important instruments in diagnosing laryngeal


trauma is the fiberoptic nasopharyngoscope.

127-Laryngeal electromyography may be helpful in distinguishing


between denervation of the intrinsic muscles and vocal cord fixation.
.
90

128-distinguishing between recurrent laryngeal nerve paralysis and


vocal cord paralysis secondary to disruption of the vagus nerve can be
difficult.

129-In patients with carcinoma of the tonsillar region, ipsilateral


elective neck dissection is commonly performed even for small lesions
and clinically negative necks.

130-Laryngeal clefts:failure of fusion and/or incomplete development


of the
tracheoesophageal septum.

131-dysplasia(atypia):its hyperplasia with distortion of cellular


organization + some malignant changes in the nucleoli.these changes
do not affect the whole thickness of epitheliun.

132-Ca.insitu:entire epithelium replaced by hyperplastic malignant cells


but basement memberan is intact.its my be reversible.

133-Most common site in H&N for SCC giving distent metastasis its
orophar.ca.

134-ca.tonsil,post.triang.l.n.may involve,safer to doe RND not a MRND.

135-25% of metastic cer.L.N,with unknown pri.from SCC of tonsil.

136-Primary tracheal tumors are very rare and account for less than
0.2% of all respiratory malignancies.

137-In evaluating a patient with dysphagia, a barium swallow should


be the first test performed.

138-5% of ca.larynx my develop second primary.

139-High-resolution fine-cut CT of the larynx is the best radiographic


tool available to evaluate laryngeal trauma.

140-most common cause of deep neck infections in


children(tonsil),adult(dental).
.
91

141-Croup:is the most common infectious cause of airway obstruction


in children, usually occurring between the ages of 6 months and 3
years.

142-The major causes of inflammatory stridor in children are (croup),


epiglottitis, and bacterial tracheitis.

143-Risk of injection of cidofovir in RP inducing malignant


transformation.

144-The primary treatment modality for respiratory papillomatosis is


surgery.

145-Hoarseness, abnormal cry, or both are the most common


presenting symptoms of respiratory papillomatosis.(RP).

146-Pulmonary papillomatosis is rare but carries high morbidity and


mortality.

147-For the treatment of subglottic hemangiomas, steroids can be


administered either systemically or by intralesional injection.

148-subglottic hemangioma:confirmation of the diagnosis requires


laryngo -tracheobronchoscopy under general anesthesia.

149-subglottic hemangioma:do not start to proliferate until after birth,


they rarely present in the first weeks of life. But 80–90% will have
presented by the age of 6 months.

150-Congenital subglottic stenosis is considered to be the third most


common congenital abnormality of the larynx.

151-Vocal cord paralysis second most common congenital abnormality


of the larynx,

152-laryngoceles, which usually present in adults,

153-Saccular cysts do not communicate with the laryngeal lumen.


usually congenital in infants

.
92

154-Complications of supraglottoplasty include bleeding, aspiration,


and supraglottic scarring.

155-Infants with laryngomalacia usually have no sign of respiratory


abnormality at birth. Inspiratory stridor typically develops after a few
days or weeks and is initially mild, In most cases, symptoms are mild
and self-limiting.

156-Laryngomalacia is the most common cause of stridor in infants,


and is also the most common congenital laryngeal abnormality,
accounting for approximately 60% of cases.

157-Stertor :airway noise originating in the nose, nasopharynx, and


oropharynx.

158-Stridor is a harsh noise produced by turbulent airflow through a


partially obstructed airway.

159-Injection laryngoplasty is often unsuccessful in cases of complete


vagal nerve paralysis because the relatively abducted position of the
vocal cord leads to failure of injected material to adequately displace
the cord medially.

160-medialization thyroplasty:placement of a Silastic implant or Gore-


tex lateral to the vocal fold via a window cut in the thyroid cartilage.

161-Most unilateral cord palsies compensate within 6–18 months.

162-supraglottic laryngectomy Patients must learn a double-swallow


technique called the supraglottic swallow to minimize aspiration with
oral intake.

163-Hemilaryngectomy is the removal of one vertical half of the larynx.

164-Contraindications for endoscopic laser resection include the entire


tumor cannot be visualized, large tumors. and cartilage invasion.

165-Laser cordectomy has been shown to provide excellent local


control and laryngeal preservation of early-stage glottic cancer;
.
93

166-A neck without clinically apparent nodal metastases should be


treated in larynx cancer if the risk of nodal metastasis exceeds 15%.

167-Radiation is generally viewed as ineffective in treating laryngeal


chondrosarcoma.

168-Verrucous carcinoma:warty, exophytic tumor that is highly


differentiated low metastatic, is typically treated surgically, as being
radiation-resistant.

169-Variants of SCC include verrucous carcinoma, spindle cell


carcinoma, basaloid SCC, and adenosquamous carcinoma.

170-Radiologic imaging of the larynx and neck is not necessary for an


early-stage glottic cancer with a clinically N0 neck.

171-Moure’s sign:clicking movement of larynx from side to side across


the pharynx and prevertebral fascia.loss can reveal postcricoid or even
retropharyngeal invasion.

172-A breathy voice may indicate a vocal cord paralysis and a muffled
voice, a supraglottic lesion.

173-A reversal of laryngeal leukoplakia after treatment with retinyl-


palmitate has been demonstrated.

174-larynx is the most common site in the respiratory tract for amyloid
deposition. presence of a submucosal mass. “apple green”
birefringence.

175-saccular cysts :the absence of air within the lesion distinguishes it


from a laryngocele.

176-The laryngeal saccule arises as a diverticulum from the anterior


end of the laryngeal ventricle.

177-Intracordal cysts may be simple mucous retention cysts or


epidermoid cysts containing keratin. usually of the middle third of the
vocal cord.

.
94

178-Recurrence of VP granuloma after surgical excision is common;


the incidence may be reduced by the concomitant use of botulinum
toxin to paralyze the affected hemilarynx and hence prevent further
vocal process trauma.

179-Vocal process granulomas are usually unilateral and are related to


the vocal processes of arytenoid cartilage with an underlying
perichondritis.

180-Vocal cord polyps:most commonly found in men with a history of


voice abuse and heavy smoking. are pedunculated, unilateral,at middle
of the true junction of the anterior and middle thirds of the vocal fold.

181-Speech therapy is the mainstay of treatment of VC nodules in both


children and adults.

182-Vocal cord nodules are the most common cause of persistent


dysphonia in children.

183-Vocal cord nodules:Bilateral, pale lesions at the junction of the


anterior one third and posterior two thirds of the vocal cords.

184-Bernoulli effect:aerodynamic properties of phonation include the


subglottic air pressure (P s ),the airflow(AF),the supraglottic pressure (P
s), the intraoral pressure (P io ), and the glottal resistance.

185-length of the vibratory portions of the vocal cords at rest is


approximately 13 mm for women and 16 mm for men.

186-The maximum width of the posterior commissure occurs during


inspiration or cough and measures approximately 9–12 mm,

187-A laryngocele is defined as an abnormal dilation or herniation of


the saccule of the larynx.

188-Radiation therapy fields include bilateral neck and supraclavicular


nodes, as well as retropharyngeal nodes, in NPC.

,
95

189-Recent reports of oropharyngeal cancer treated with IMRT and


concurrent chemotherapy have noted 3-year locoregional control rates
ranging from 87% to 93%.

190-Survival is higher in patients with HPV-positive oropharyngeal


tumors.

191-Definitive RT with or without chemotherapy is the treatment of


choice for tumors of the tonsil and soft palate.

192-Squamous cell cancers of the oral cavity are primarily treated


surgically, while those of the oropharynx are primarily treated with
definitive radiotherapy.

193-CT scans with intravenous contrast have become the cornerstone


of diagnosis of PPS abscess.

194-Lymphoma and squamous cell carcinoma are the most common


primary tonsillar neoplasms,

195-severe cases of OSAS can lead to pulmonary hypertension, cor


pulmonale, and alveolar hypoventilation resulting in chronic CO2
retention.

196-tonsillectomy is indicated in patients with recurrent acutetonsillitis


involving 6–7 episodes of acute tonsillitis in 1 year, 5 episodes per year
for 2 consecutive years, or 3 episodes per year for 3 consecutive years.

197-The definitive tests to determine GABHS infection is measuring


serum titers of antistreptolysin O (ASO).

198-Most tumors found in the parapharyngeal space are benign. 80%


are pleomorphic adenomas that arise from the deep lobe of the parotid
gland.

199-Sublingual immunotherapy (SLIT) is a new, safe, efficacious and


more convenient method for delivering immunotherapy.
96

200-Studies have reported that residual retropalatal obstruction was


observed in over 80% following traditional UPPP.

201-The treatment o f choice for most children with obstructive sleep


apnea (OSA) is adenotonsillectomy. CPAP is used adjunctively with
persistent OSA.

202-continuous positive airway pressure is the first-line therapy in the


care of severe OSA.

203-only absolute contraindication to primary tracheoesophageal


puncture are Separation of trachea and esophageal party wall.

204-The oral tongue is the most common subsite of the oral cavity to
develop cancer, occurring in 3 2 % of cases, followed by the floor of
mouth.

205-Selective neck dissection for oral cavity cancer of levels I to III is


appropriate for oral cavity tumors, with the exception of oral tongue
cancer which can spread to level IV and requires dissection of levels I
to IV.

206-About 50 % of all Waldeyer ring non-Hodgkin lymphomas arise in


the palatine tonsil, 20% of which are bilateral .

207-The most common presenting symptom in SCCA of the trachea is


hemoptysis.

208-SCCA typically affects the lower lip and is more common in men.
inadequate transoral access to the entire tumor.

209-Transoral laser microsurgery (TLM) utilizes transtumoral cuts to


assess the depth of the tumor and multibloc resection.

210-oral HPV infection is strongly associated with squamous cell


carcinoma (SCCA) of the oropharynx in those patients with and without
risk factors of alcohol and tobacco use.
97

211-More than 20% of early stage oral tongue cancers harbor occult
nodal metastasis.

212-The major contraindication to TLM for early supraglottic cancers is


inadequate transoral access to the entire tumor.

213-Most common cause for failed tracheoesophageal (TE) voice:


Hypopharyngeal bar.

214-Sentinel node biopsy has been shown to be useful in oral tongue


cancer.

215-Chondroradionecrosis occurs in approximately 5 % of patients with


laryngeal cancer, and the most common cartilage affected is the
arytenoid cartilage.

216-Vocal fold nodules are the most common cause of hoarseness in


preschool and school-aged children,

217-In patients with cleft palate the levator veli palatini has three
abnormal attachments : the superior pharyngeal constrictor, the tensor
aponeurosis, and the posterior edge of the hard palate.

218-Acquired subglottic stenosis most common type of laryngeal


stenosis in children.

219-Laryngeal clefts are graded as Grade I involving only the


interarytenoid muscle, Grade II involving part of the cricoid, Grade III
involving the entire cricoid, and Grade IV extending into the trachea.

220-Main indications for tracheotomy in the pediatric population are


respiratory failure and anticipated need for prolonged ventilation, upper
airway obstruction, and providing access for pulmonary toilet.

221-Laryngeal abnormalities are typically found only in children with


the velocardiofacial syndrome.
.
98

222-The most common syndrome with CL/P is Van der Woude


syndrome, anautosomal dominant syndrome characterized by blind
lower lip pits in addition to CL/P.

223-Children with foreign bodies lodged at the upper esophageal


sphincter frequently present with early symptoms of dysphagia and
drooling.

224-Hemorrhage can be primary (within 24 hours of surgery) and


secondary (more than 24 hours after surgery) . The incidence of
primary hemorrhage ranges from 0 . 2 % to 2 . 2%, while secondary
hemorrhages range from 0 . 1 % to 3 % (adeno-tonsillectomy).

225-The narrowest part of an infant's airway is the subglottis and it


normally measures 4 to 7 mm. Less than 4 mm in size in a newborn or
3 . 5 mm in a premature infant is diagnostic of subglottic stenosis.

226-The findings of micrognathia and glossoptosis in the neonate with


airwayobstruction are pathognomonic for Robin sequence. Cleft palate
is seen in approximately 50 % of these patients .

227-More than 90 % of both third and fourth branchial cleft anomalies


occur on the left side.

228-submucosal tumor of the larynx most commonly represents a


chondrosarcoma or a metastatic lesion.

229-Lesions of the hypopharynx have a propensity to spread to


retropharyngeal nodes,

230-lesion of the pyriform apex approaches the upper margin of the


cricoid cartilage and may erode cartilage even when relatively small.

231-abscess has a well-defined enhancing rim and a nonenhancing pus


filled center, and it exerts mass effect on local tissues rather than
infiltrating along and obscuring fascial planes ,in imaging.

232-Clindamycin is one of the first-line drugs for the treatment of


parapharyngeal space infections (including Ludwig angina),

.
99

233-RRP is both the most common benign neoplasm of the larynx


among children and the second most frequent cause of childhood
hoarseness.

234-Supraglottitis..The diagnosis is confirmed only after the airway has


been secured by endotracheal intubation or tracheostomy.

235-The KTP laser is associated with a high incidence of postoperative


nasopharyngeal stenosis and is not recommended for adenoidectomy..

236-Thyroid alar cartilage is excellent for use in the anterior subglottic


region but is limited to grade II and few grade III lesions.

237-Acquired subglottic stenosis is more common than congenital


stenosis because of the use of prolonged endotracheal intubation for
respiratory support.

238-The normal subglottic lumen diameter in the full-term neonate is


4.5to 5.5 mm and in premature babies is about 3.5 mm.

239-Submandibular duct transposition with excision of the sublingual


glands is the current procedure of choice for excessive drooling.

240-Tracheostomy tubesToo large will predispose to suprastomal


collapse and may increase the risk of granulations and stenosis. Ideally,
the tube should be at least 2 cm, inside the stoma and 1–2 cm, clear of
the carina.

241-A classical cause of congenital bilateral vocal cord palsy is


hydrocephalus with the Arnold-Chiari malformation.

242-Vocal cord paralysis is the second most common congenital


anomaly of the larynx after laryngomalacia.

243-Mucosal melanoma has a lower incidence of regional lymph node


metastasis than cutaneous melanoma, both at presentation and later in
the course of the disease.
100

244-The Müller maneuver is performed in an awake patient who


generates negative pressure by inhaling against a closed glottis with
the nose and mouth closed, which triggers airway collapse.

245-The most common complication after palatoplasty is


velopharyngeal insufficiency.

246-most common cause for benign ulcerations of the oral cavity and
the oropharynx is recurrent aphthous stomatitis.

247-Half of the trachea can be safely resected and anastomosed


primarily. The incidence of tracheal squamous cell cancers and adenoid
cystic tumors is equivalent, and together they account for 75% of all
primary tracheal tumors.

248-Ninety percent of primary tracheal tumors in adults are malignant.


Signs of submucous cleft palate:Bifid uvula, Zona pellucida, Notched
hard palate.

249-Pharyngeal branch of the internal maxillary (major supply) of


Adenoid.

250-Torticollis or decreased neck range of motion post-tonsillectomy


should be suspicious for Grisel’s syndrome.

251- The most common selective neck dissection performed for the
management of the regional lymphatics in patients with oral cavity
cancers is the supraomohyoid neck dissection.

252-The recommendations, as for the adult tracheostomy is to have


the outer diameter of the tube no larger than two-thirds the diameter
of the trachea.

253-Approximately 50% of the trachea (5-7 cm) can be safely resected


and anastomosed primarily with appropriate mobilization techniques.

254-Nodes larger than 2 to 4 cm without central necrosis often indicate


lymphoma or sarcoidosis.
101

255-Branchial cleft cysts are relatively common; in one series, they


comprise one third of congenital neck masses.

256-Difficulty swallowing only saliva suggests globus, especially when


associated with anxiety.

257-Significant sleep apnea disorder is present when there are > 5


episodes of apnea or hypopnea per hour of sleep.

258-(OSAS) is defined as cessation of airflow due to collapse of the


upper airway for at least ten seconds.

259-Elective neck dissection is not typically advocated for lip carcinoma


because the incidence of occult metastases is low.

260-The most common symptoms of achalasia are dysphagia for solids


and liquid, regurgitation, and chest pain.

261-Continuous pH monitoring studies are believed to be the “gold


standard study” for diagnosing GERD and EER.

262-most common selective neck dissection performed for the


management of the regional lymphatics in patients with oral cavity
cancers is the supraomohyoid neck dissection (SOHND),

263-The larynx is the second most common site in the head and neck
for VC. (verrucous carcinoma).

264- Laryngeal trauma :a stridulous patient with respiratory distress at


presentation should immediately undergo tracheotomy with local
anesthesia.

265-CT imaging is best for evaluation of occult fractures and


dislocations during laryngeal trauma.

266-Medications cause CL/P —phenytoin, sodium valproate,


methotrexate.

267-most common technique for cleft lip repair is the Millard


technique.
102

268-muscles that contribute to the velopharyngeal sphincter include


the palatopharyngeus, superior constrictor, and musculus uvulae.

269-Cleft lip occurs more commonly in boys than in girls, whereas cleft
palate occurs more commonly in girls than in boys.

270-The most common syndrome associated with cleft lip is van der
Woude. Lower lip pits and CLP are the characteristics features of this
syndrome,

271-Approximately 70% of CLP patients are thought to be


nonsyndromic, whereas 50% of CP patients are nonsyndromic.
Syndromic clefts may be the result of a single gene transmission as
occurs in autosomal dominant, autosomal recessive, or X-linked
heredity.

272-An isolated cleft palate usually involves the secondary palate only,

273-Simonart’s band is the bridge or bar of lip tissue, of variable size,


that bridges the cleft lip gap.

274-Cleft lip and cleft palate is also known as the Van der Wounde
Syndrome.

275-Torus-palatinus usually no treatment is reguired.

276-Mc Naught Keel is used to prevent formtion of laryngeal web.

277-Anti-reflux treatment can be very useful in improving the stridor of


laryngomalacia.

278-A recent study has suggested the possibility of a gene on


chromosome 13q14 predisposing infants to severe reflux,

279-Alteration in p53 is the most common gene mutation in squamous


cell carcinoma of the head and neck and in malignant tumors
throughout the body.

280-in skin flaps.skin should be raised with platysma.


103

281-Glottic Tumor remains within its subsite whether involving one


cord,both cords,ant.or post.commissure its still T1..

282-Androphonia is male character of voice.

283-Perichondritis is contraindication to any type of radiotherapy.

284-tracheostomy and thyroplasty will not completely prevent


aspiration.

285-full term neonate the diameter of subglottis is 4.5-5.5 mm. in


premature neonate 3.5 mm.

286-Submucous cleft palate its contraindication for adenoidectomy as it


results in velopharyngeal insufficiency and speech defects.

287-Severe vomiting can cause tear of oesophageal mucosa only


(Mallory-Weiss syndrme) or all layers of oesoph.wall (Boerhaave
syndrome).

288-Fordyce’s spots are yellowish-white granules in the buccal mucosa


and represent enlarged ectopic sebaceous glands.

289-Structures pass between base of skull and superior constrictor m.


are: levator v.palatini,tensor v.palatini,Eustachian tube,ascending
palatine artery (facial A.) and ascending pharyngeal A.(ECA).

290-Dysphagia in plummer-vinson syndrme is due to post-cricoid


web.easily dilated to relieve dysphagia.its precancerous condition.

291-HIV infection causes :recurrent oral ulcers,gingivitis,angular


stomatitis,hairy leukoplakia and Kaposi’s sarcoma.

292-Wickham’s striae are interlacing white lines in buccal mucosa seen


in Lichen planus. (precancerous but rarely).

293-Ebstein-barr virus causes:infectious


mononucleosis,NPC(nonkeratinising type),Burkitt’s lymphoma,non-
Hodgkin’s lymphoma and hairy leukoplakia.

.
104

294-Grisel’s disease:atlantoaxial dislocation.in severe extension of neck


during adeno- tonsillectomy.

295-Chordoma arises from remants of notochord.

296-Infection of pharyngeal bursa give rise to Thornwaldt’s cyst which


causes postnasal discharge,hlitosis and occipital headache.

297-moure's sign is absence of laryngeal click .

298-Peutz-jegher’s syndrom:dark oral lesion and intestinal


polyposis,.

299-The most common source of infection of the lateral pharyngeal


space is from the tonsils.

300-Bacterial tracheitis is an acute, life-threatening disease in children


and is most commonly caused by Staphylococcus aureus.

301-Most tracheal injuries from extrinsic chest trauma are 2.5 cm


above the carina.

302-C1-INH concentrate is the most reliable in the acute management


of airway obstruction owing to hereditary angioedema.

303-Acute onset of edema of the tongue is most likely to be associated


With angiotensin converting enzyme inhibitors.

304-The treatment of choice for verrucous carcinoma is surgical


resection.

305- 24-hour multichannel ambulatory esophageal pHmonitoring is the


most sensitive study for extraesophageal reflux;

306-cancers of the tonsil are the most radiosensitive.

307-most tumors of the oral cavity and oropharynx, surgical resection


is recommended for stage Iand II tumors, with combination therapy
using surgery and postoperative radiotherapy for stage III and IV
tumors.
.
105

308-Parastomal recurrence is usually unresectable.

309-Severe dysplasia of the vocal fold becomes invasive carcinoma in


up to 25% of patients.

310-The most common synchronous secondary primary tumor in


patients with laryngeal carcinoma is bronchogenic carcinoma.

311-Teflon injection owing to the late complication of granuloma


formation.

312-Staccato bursts in speech and harsh voice quality may occur in


patients with multiple sclerosis.

313-Wet hoarseness --amyotrophic lateral sclerosis. -speech in


Parkinson’s disease -- trailing loss of volume.

314-Isolated laryngeal candidiasis is usually secondary to inhaled


corticosteroid use.

315-The child with suspected epiglottitis should be taken to the


operating room immediately to establish the diagnosis and secure an
airway.

316- (CT) scan should be obtained in all patients with suspected


laryngeal trauma.

317-Subcutaneous emphysema after a tracheostomy results from


closing a tracheostomy incision too tightly.

318-Nonsyndromic cleft lip and cleft palate is secondary to


multifactoral inheritance.

319-Pierre Robin sequence (PRS) includes the triad of glossoptosis,


micrognathia, and cleft palate.

320-Rathke’s cyst most commonly presents with headache followed by


galactorrhea, visual field loss, and hypopituitarism.
106

321-Chordomas arise from primitive notochordal remnants.

322-The most commonly encountered congenital anomaly of the head


and neck is thyroglossal duct cyst.

323-Saccular cysts are best diagnosed with direct laryngoscopy.

324-Polysomnography in a sleep laboratory is the definitive diagnostic


study for sleep apnea.

325-Approximately how many taste buds are there on the human


tongue :4600.

326-herpangina, caused by coxsackie virus.

327-Ebner’s glands secrete mucus around the circumvallate papillae.

328-Lamina propria:Loose connective tissue underneath the


epithelium.

329-oropharyngeal cancer:Early-stage lesions can be treated surgically


with or without postoperative radiation. Advanced-stage lesions can be
treated surgically with or without reconstructive surgery and
postoperative chemoradiation therapy or with definitive chemoradiation
therapy in case of inoperability.

330-differential diagnosis of parapharyngeal masses includes deep lobe


parotid tumors, minor salivary gland tumors, and neurogenic and
vascular tumors.

331-masticator space :attaches foramen ovale in its roof, allowing


potential tumour spread.

332-More accurately, the poststyloid compartment is separated from


the prestyloid compartment by the tensor palati fascial layer; in PPS.

333-Cleft lip by itself is rare.


107

334-AchlasiaFailure of relaxation of LES during swallowing due to


degeneration of myenteric(Aurbach) plexus.

335-Barium study is contraindicated in patients with suspected


perforation of oesophagus.

336-Referred otalgia with dysphagia is a sinister symptom and poor


prognostic sign.
--Stevens-Johnson syndrome is a more serious complications of
antibiotic therapy.

337-Tumors in the prestyloid parapharyngeal space are usually of


salivary gland origin. Tumors in the poststyloid parapharyngeal space
are usually neurogenic in origin.

338-The body of the hyoid and preferably a wedge of tongue base


should therefore be included in the excision of theTGD cyst (Sistrunk’s
Procedure).

339-The external carotid artery arises from the carotid sinus at the
level of the fourth cervical vertebra.

340-Internal carotid artery. It has no branches in the neck, and


ascends medial and posterior to the internal jugular vein toward the
skull base.

341-lingual artery is the second branch of the external carotid artery,


arising at the level of the greater horn of the hyoid bone,

342-More than half of patients with cancer of the oropharynx are


initially found to have stage III or IV disease.

343-Mandibulotomy is rarely necessary, even with removal of very


large tumors in the PSPS.prestyloidparapharyngeal space.

344-Transoral incisional biopsy of parapharyngeal space masses is to


be discouraged.

345-The treatment of laryngospasm includes providing gentle positive-


pressure ventilation with 100% oxygen.
108

346-maximal phonation time (MPT). This is done by simply instructing


the patient to take a deep breath and phonate an “ee” vowel for as
long as possible. Normal MPT for a healthy adult is approximately 25
seconds-

347-Medications such as the vinca alkaloids (vincristine and


vinblastine), and cisplatinum, are known to cause neurotoxicity of the
RLN (unilateral or bilateral).V C paralysis.

348-Reinke’s edema is an accumulation of a gelatinous type of fluid


throughout the superficial aspect of the lamina propria.

349-causes of Positive Jackson's sign :Malignant growth involving the


deep portion of the pyriform fossa , Foreign body being lodged in the
pyriform fossa. Growth involving the crico pharynx or upper
oesophagus .

350-Jackson's sign :Pooling of saliva seen in the pyriform fossa.

351-gustatory agnosia:inability to recognize a taste sensation.

352-ageusia:inability to detect qualitative gustatory sensations, i.e.


absence of taste;

353-Approximately 4600 goblet-shaped taste buds are found within the


oral cavity,

354-KOPLIK'S SPOTS. White specks may be seen in the buccal mucosa


in early measles.

355-Burning mouth syndrome:may be seen in diabetes, and drugs


(e.g. ACE inhibitors, protease inhibitors, cytotoxic agents, clonazepam).

356-Fordyce spots. sebaceous glands, seen beneath the buccal or


labial mucosa.

357-The genioglossus muscle lies medial to the sublingual gland,


separated from it by the lingual nerve and submandibular duct.
109

358-paratonsillar vein:large vein descends from the soft palate lateral


to the tonsillar hemicapsule before piercing the pharyngeal wall.

359-dexamethasone in pediatric tonsillectomy Its use significantly


reduces postoperative emesis.

360-Hairy Tongue:characterized by the hypertrophy and growth of


the filiform papillae on the dorsal surface of the tongue,

361-Geographic Tongue:results from a loss of filiform papillae.

362-Lichen Planus:may be isolated or may coalesce to form


interlacing white lines (Wickham’s striae).

363-Hand-foot-and mouth disease is a viral illness of the oral cavity


mucosa caused by a coxsackie A virus.

364-Both vitamin B12 and folic acid deficiencies may cause a


megaloblastic anemia and present with recurrent oral ulcers and painful
atrophy of the oral mucosa and tongue.

365-Blue-black lines along the gingival margins and spots on the


buccal mucosa, palate, and tongue may occur with lead, mercury,
arsenic, and bismuth poisoning. Fluoride may cause staining of the
teeth,

366-The adenoid (also known as the pharyngeal orLuschka’s tonsil) is a


lobulated mass of lymphoid tissue found on the superior and posterior
walls of the nasopharynx. the adenoid has no capsule.

367-A stroboscope is defined as “an instrument for determining the


speed of cyclic motion (as rotation or vibration).

368- Congenital lower lip palsy Affects 1% of newborn Unilateral


Resolves spontaneously in 6 months.

369-Onsils, Soft pallet, and posterior pharyngeal wall → level II and III
sometimes bilateral.
110

370-TGTC;The tract has various relations to the hyoid bone, usually


behind, sometimes infront & rarely through the body of the hyoid bone.

371-phonosurgery, which is defined as ‘any surgery designed primarily


for the improvement or restoration of the voice’.-

372-GABHS is the pathogenic organism responsible for most cases of


bacterial pharyngitis
in adults.

373-Pharyngitis in adults is caused by:(5% to 10%bacterial)/(30% to


60%viral) and rest are Occupational and environmental exposures.

374-Influenza A rather than type B is responsible for most of the


significant morbidity and mortality.

375-internal branch of superior laryngeal nerve supply the valleculae,

376-Foliate papillae, filiform ,fungiform and circumvallate papillae, all


except the filiform papillae bear taste buds.

377-Tonsil starts to atrophy at puberty so that, by old age, only a little


tonsillar lymphoid tissue remains.

378-Fordyce's spots:representing ectopic sebaceous glands may be


evident on the internal surface of the cheek.

379-The first signs of acute leukemia may be oral lesions.

380-anterior commissure tendon is a band of fibrous tissue 1 mm wide


and 10 mm in length.

381-invasion of the laryngeal framework was seen in over half of


transglottic tumors over 2 cm. in primary tumors greater than 4 cm in
dimension, 55% of tumors had nodal metastases.

382-transglottic: tumor that crosses the ventricle in a vertical direction.


111

383-Supraglottic tumors with PES involvement are staged as T3


lesions.

384-The lymphatics of the PES drain through the thyrohyoid


membrane, spreading to lymph nodes on both sides of the neck,
primarily in zones II and III.

385-Hyoepiglottic ligament provides a barrier to spread of tumor to the


tongue base.

386-pre-epiglottic space (PES) and the paraglottic space (PGS) provide


pathways for spread of laryngeal tumors.

387-glottic cancers must invade deeply before getting access to


lymphatic channels.and propensity for unilateral metastases.

388-Glottic & subglottic drain into prelaryngeal and pretracheal nodes


(Level VI), before reaching the deep cervical chain nodes in level IV.

389-The stage of disease is the most important factor predictive of


prognosis, with N (nodal) stage more significant than T (tumor) stage.

390-Stridore :Inspiratory : larynx and 1st two rings of trachea


,Biphasic: mid trachea ,Expiratory : lower trachea and bronchi.

391- Dysphagia: Due to pain (odinophagia ) ,Due to mechanical


narrowing of the lumen ,Due to neurological disorders .

392-Preoperative ultrasonography provides valuable information


regarding whether the thyroid gland is present when a TGDC is found.

393- Virchow first coined the term “laryngocele” in 1863.

394-EBV produces a protein called LMP-2. ( Latent Membrane Protein).

395-EBV cause not only NPC but also Infectious Mononucleosis,


Burkitt’s Lymphoma .

396-Stroboscopy is a lighting technique that helps clinicians examine


vibration patterns and the relationship between the vocal fold body and
cover.
112

397- patients with an extensive tongue base tumour and bilateral


cervical lymphadenopathy will usually be inoperable at presentation.

398-It is important to be aware of the existence of retropharyngeal


nodes and to evaluate them properly when assessing tumours at risk
and magnetic resonance imaging (MRI) is probably more accurate than
computed tomography (CT).

399-presence of retropharyngeal nodes with a very poor prognosis.

400-Trotter's sign (loss of laryngeal crepitus) may be present with large


(usually postcricoid) tumours.also named as Moure’s sign.

401-Fixation of the ipsilateral hemilarynx will be seen in half the cases


of piriform fossa cancer.

402-Most posterior pharyngeal wall tumours are exophytic and spread


widely, and will often be visualized at the superior extension.

403-Symptoms of hypopharyngeal cance.The cardinal symptoms and


signs are dysphagia, hoarseness of voice, sore throat, otalgia and
weight loss with or without a lump in the neck.

404-Up to 80 percent of patients with carcinoma of the posterior


pharyngeal wall will have neck node metastases at presentation.

405-Also important to note that 11 percent of patients with piriform


fossa cancer with node metastases have disease in the supraclavicular
fossa or posterior triangle.

406-Less than 10percent of piriform fossa tumours have bilateral


metastases at the time of presentation.

407-The incidence of lymph node metastases in hypopharyngeal


cancer is very high: between 70 and 80 percent of patients with
piriform fossa cancer will have lymph node metastases at the time of
diagnosis.

408-The piriform fossa or sinuses represent paired channels for


swallowing.
113

409-Nearly all tumours of the hypopharynx and cervical oesophagus


are malignant and most are squamous cell carcinomas.

410- The most important risk factor for hypopharyngeal cancer was the
excessive consumption of alcohol.

411-Total laryngectomy:This remains the mainstay of treatment for


advanced laryngeal cancer and has 150 years of proven oncological
safety. It is largely the same operation as that first performed by
Billroth in the 1870s.

412-Subtotal laryngectomy:three-quarter laryngectomy, combining


supraglottic laryngectomy with vertical hemilaryngectomy on the side of
the tumour.

413-Ca.larynx,Referred otalgia (via the vagal complex) is a sinister sign


suggesting deep invasion.

414-Verrucous carcinoma is a form of well differentiated SCC. Relative


nonaggressive with low metastatic rate (no positive neck nodes
reported). Hoarseness is the primary presenting.

415-The second most common malignancy of the larynx, lymphoma, is


treated differently depending on stage and histology.

416- Invasion through Broyle's ligament directly into cartilagedue to


the lack of an inner perichondrium in that area.

417-Papilloma (recurrent respiratory papillomatosis (RRP)) is the most


common benign neoplasm of the larynx.

418-Aphonia is almost always associated with very severe injury.

419-Up to 5 cm of the trachea can be excised to rejoin the trachea


back onto the cricoid or the first tracheal ring.

420-The average length of the adult trachea is 11 cm (range 10–13


cm) with between 14 and 20 rings ,Hyperextension of the neck results
in approximately 50 percent of the trachea being brought into the neck.
114

421-MITOMYCIN C antineoplastic and antiproliferative propertiesinhibit


fibroblast proliferation both in vivo and invetro.

422-The predominant cause of chronic laryngeal stenosis is trauma,


occurring either subsequent to the failed treatment or nonrecognition
of acute trauma, secondary to prolonged intubation or as a
complication of tracheostomy.

423-Fractures of the hyoid tend to heal without complication.

424-An external/combined laryngocoele presents as an intermittent


neck swelling that can be inflated by the patient with raised expiratory
pressure during a Valsalva manoeuvre Compression may empty air and
fluid into the larynx (Bryce's sign).

425-Pertussis should be considered in patients of any age presenting


with a prolonged acute spasmodiccough of three or more weeks
duration. Once recognized treatment with a, 7–14-day course of
erythromycin should be commenced .

426-Croup affects mainly young children, aged six months to three


years,

427- Prior to vaccination programs, Haemophilus influenzae type b was


the commonest pathogen especially in children. A wide range of
pathogens has been described in adults and these include Group A
Streptococci, Streptococccus pneumoniae Staphylococcus aureus and
Klebsiella pneumoniae.

428-Epiglottitis affects all age groups.

429-There is no convincing clinical or scientific evidence that


reinnervation techniques give better orequal results to injection or
laryngeal framework surgery.

430-Medialization laryngoplasty using Gortex is a relatively simple


technique.
431-The recurrent laryngeal nerve is a mixed nerve containing afferent
, sympathetic, parasympatheticand 500–1000 motor axons in an
adductor to abductor ratio of 5:1.
115

432- Medialization laryngoplasty can be performed in any patients with


a unilateral vocal cord paralysis.

433-Injectible materials include Teflon, fat, glycerine, collagen and


silicon . All provide immediate results. All can ideally be carried out
under local anaesthetic in the normal anatomical position with the
patient sitting up by injecting through the cricothyroid membrane to
obtain the most reliable voice outcomes. These products demonstrate a
range of reversibility with glycerine being totally reversible, followed by
fat and Teflon being totally irreversibleCost is not a major issue in
deciding which to use.

434-Papillomas are excised using either CO2 laser or a


microdebriderSingle papillomas are excised at the base.

435-Reinke's oedema:making a cordotomy incision on the superior


aspect of the vocal fold (preserving the medianvibrating edge)..
elevation of the mucosa and aspiration or removal of
themyxoedematous contents.

436-Polyps-too little resulting in reformation of the polyps, too much in


scarring of the vocal folds.

437-The common sites giving unknown primary are:Nasopharynx, Base


of the tongue, Tonsil, pyriform fossa .

438- In unknown primary the first step is to do an FNA to:exclude a


primary Lymphoma, It may give you a clue to the possible primary site
as in papillary CA of Thyroid or Adenoidcystic CA of the salivary gland
andThis diminishes the indications of pan endoscopy.

439-Barking couph : subglottis / Brassy couph : trachea .

440-adenoids have three types of surface cells.

441-size of laryngeal saccule :75% < 8mm , 17% (10-15 mm) ,


8%(>15mm) .

442-Broyles tendon : vocalis m. tendon insert into thyroid cartilage .


.
116

443- Beahr's triangule : between inf.thyroid A. &common carotid A.&


recurrent L .N.

444-Joll's triangule : external br. Of sup. Laryngeal n .

445- Cuniform cartilage : car. Of wrisberg ,// corniculate cartilage :


car.of santorini .

446- Rt.bronchus : 25 degree,// Lt.bronchus : 75 degree .

447- tonsil no afferent lymph vessels.

448-The average length of the adult trachea is 11 cm (range 10–13


cm) with between 14 and 20 rings ,Hyperextension of the neck results
in approximately 50 percent of the trachea being brought into the neck.

449-The predominant cause of chronic laryngeal stenosis is trauma,


occurring either subsequent to the failed treatment or nonrecognition
of acute trauma, secondary to prolonged intubation or as a
complication of tracheostomy.

450-Fractures of the hyoid tend to heal without complication.

451-An external/combined laryngocoele presents as an intermittent


neck swelling that can be inflated by the patient with raised expiratory
pressure during a Valsalva manoeuvre, Compression may empty air and
fluid into the larynx (Bryce's sign).

452-Pertussis should be considered in patients of any age presenting


with a prolonged acute spasmodic cough of three or more weeks
duration. Once recognized treatment with a, 7–14-day course of
erythromycin should be commenced .

453- There is no convincing clinical or scientific evidence that


reinnervation techniques give better or equal results to injection or
laryngeal framework surgery.

454-Medialization laryngoplasty using Gortex is a relatively simple


technique.
117

455-The recurrent laryngeal nerve is a mixed nerve containing afferent


, sympathetic, parasympatheticand 500–1000 motor axons in an
adductor to abductor ratio of 5:1 .

456-Medialization laryngoplasty can be performed in any patients with


a unilateral vocal cord paralysis.

457-Papillomas are excised using either CO2 laser or a microdebrider


Single papillomas are excised at the base.

458-Reinke's oedema:making a cordotomy incision on the superior


aspect of the vocal fold (preserving the medianvibrating edge).
elevation of the mucosa and aspiration or removal of the
myxoedematous contents.

459- Polyps-too little resulting in reformation of the polyps, too much


in scarring of the vocal folds.

460-Reinke's oedema It occurs almost exclusively in moderate to heavy


smokers.

461-In the UK, the mainstay of treatment for persistent vocal nodules
is voice therapy. With the aim of modification of lifestyle and vocal
behaviour.

462-V.C polyp:>3mm,unilateral,smoker,male>female& 30-50 yr. V.C


nodule:<3mm,bilat., singer,teacher,female>male in adult ,boys>girls .

463-Nodules disappear spontaneously in boys with the relatively large


growth of the larynx in puberty. In girls, they may persist into early
adulthood.

464-Very rarely, large polyps can cause difficulty in breathing and


episodes of choking.

465-High-pressure jet (Venturi) ventilation of anaesthetic gases


without an ET tube can be administered via the supraglottis, glottis,
subglottis or via the ranstracheal/tcricothyroidotomy route.
.
118

466-Most of the larynx is lined by pseudostratified ciliated columnar


‘respiratory’-type epithelium. The upper half of the posterior surface of
the epiglottis, the upper part of the aryepiglottic fold, the posterior
glottis and the vocal folds are covered with nonkeratinizing stratified
squamous epithelium.

467- Blood supply to the adenoids is from the ascending phary. artery,
the ascending pala. Art, the phary branch of the maxil. artery, the
artery of the pterygoid canal and branches from the tonsillar branch of
the facial artery.

468-After five years of age, adenoid size remains constant while the
nasopharynx increases in size.

469-Galen's anastomosis and is purely sensory.between SLN & RLN.

470-Adenoids are not radiologically visible on plain radiographs in


infants under one month of age, becoming radiologically demonstrable
in all infants at six months.

471-adenovirus is well known as a causative agent of pharyngitis with


conjunctivitis (pharyngoconjunctival fever) in children. cause outbreaks
of febrile respiratory illness in military recruits.

472- Benzathine penicillin can be alternatively used as a single


intramuscular injection that provides bactericidal levels for 21 to
28 days.in GABHS.

473-A laryngocele is an abnormal dilation or herniation of the


saccule of the larynx.

474-Most thyroglossal duct cysts are seen in the midline near the level
of the hyoid bone because the tract passes just anterior to the
hyoid Bone.

475- Tumor deposits in paratracheal nodes are thought to be


responsible for the high incidence of stomal recurrence in patients with
tumors that exhibit subglottic extension.
119

476- Bilateral neck dissection is required for all supraglottic cancers, all
hypopharyngeal cancers that involve the medial wall, and cancer of the
anterior commissure with cartilage invasion.

477- routine elective neck dissection is not required forT1 or T2 glottic


cancer.

478-Squamous cell carcinoma is the most commonly encountered


neoplasm in the soft palate.

479-Eagle's syndrome occurs more frequently in women than in men


and is usually found in patients older than30 years. Palpation of the
tonsillar fossa will reveal a hard mass and reproduce the patient's
symptoms.

480-Relative indications for tonsillectomy included tonsillar hyperplasia


with upper airway obstruction, dysphagia, speech impairment, halitosis,
recurrent or chronic pharyngotonsillitis, peritonsillar abscess, and
streptococcal carriers.

481- Absolute indications for tonsillectomy included adenotonsillar


hyperplasia with obstructive sleep apnea, failure to thrive, abnormal
dental and facial growth, suspicion of malignant disease, and
hemorrhagic tonsillitis.

482-internal carotid artery is located approximately 2.5 cm


posterolateral to the tonsils.

483-Vocal fold injection is an excellent tool for the treatment of glottal


insufficiency secondary to vocal fold paralysis, paresis, or atrophy.

484-Indications for medialization laryngoplasty and/or arytenoid


adduction include symptomatic glottic insufficiency (dysphonia,
aspiration).

485-arytenoid adduction should be strongly considered because it is


the only operation that consistently corrects glottic insufficiency in the
posterior aspect of the glottis.
120

486-Medialization laryngoplasty (type I thyroplasty) and arytenoid


adduction are excellent surgical options for rehabilitation of the voice in
patients suffering from glottic insufficiency.

487-Injections for vocal fold medialization should target the paraglottal


space lateral to the vocalis muscle.

488-Acidic refluxate left in the esophagus can be neutralized by


gastric glandular secretions and buffering agents in Saliva.

489-Continuous pH monitoring studies are believed to be the “gold


standard study” for diagnosing GERD and EER.

490-Specifically, 10% to20% of patients with Wegener’s


granulomatosis present with subglottic involvement and subsequent
stenosis.

491-white lesions on the surface of the vocal fold epithelium,the


differential diagnosis includes hyperkeratosis/leukoplakia, malignancy,
thick mucus, and candidal infection.

492- Contact granuloma or ulceration is seen primarily in men .

493-Capillary ectasia is diagnosed most often in female singers.

494- Vocal fold stripping has no place in the surgery of nodules.

495-Vocal nodules occur most commonly in boys and women.

496-Acid ingestion leads to coagulative necrosis; alkali ingestion leads


to liquefactive necrosis.

497-GER is found in more than 90% of cases of laryngomalacia.

498-Children with Down syndrome have a higher rate of posterior


glottic stenosis than the general population.
.
121

499-Premature infants tolerate more prolonged intubation (weeks


rather than days).

500- Approximately 90% of cases of acquired chronic subglottic


stenosis in infants and children occur secondary to endotracheal
intubation.

501- congenital subglottic stenosis is the third most common


congenital disorder of the larynx after laryngomalacia and recurrent
laryngeal nerve paralysis.

502-The normal subglottic lumen diameter in the full-term neonate is


4.5 to 5.5 mm and in premature babies is about 3.5 mm.

503-The most common causes of failed decannulation in children are


significant suprastomal collapse and granulation.

504-Stertor describes the snoring-like noise, which typically originates


from nasopharyngeal or oropharyngeal obstruction.

505-Stridor is an audible respiratory noise derived from turbulent


airlow due to narrowing or obstruction of the upper airway.

506-CO2 laser is the most commonly used laser for RRP in the larynx,
the potassium titanium phosphate KTP), 585-nm lash dye, and argon
laser could also be used.

507-Cidofovir First member of group of antiviral agents known as


acyclic phosphonate nucleotide analogs;

508-there is no “cure” for RRP, and no single modality has


consistently been shown to be effective in eradication of RRP.

509-Recurrent respiratory papillomatosis (RRP) is the most


common benign neoplasm of the larynx in children and the second
most frequent cause of childhood hoarseness..
122

510- traditional surgical option to relieve the airway obstruction


associated with subglottic hemangioma is a tracheotomy.

511- In infants with feeding problems, recurrent aspiration, and stridor,


the otolaryngologist must maintain a high index of suspicion that a
laryngeal cleft may be presen.

512- Patients with subglottic hemangiomas often present with a history


of “recurrent croup” because the lesion mimics the subglottic swelling
of croup, and with steroid treatment.

513- All children with anterior laryngeal webs should undergo genetic
testing for abnormalities of chromosome 22q11.

514- Supraglottoplasty is a highly successful treatment for severe


laryngomalacia with a low complication rate.

515- Destroyed epiglottis resulting in an irreversibly compromised


swallowing function with a high probability of aspiration.

516-Poor presenting quality of voice, which is unlikely to reverse with a


primary nonsurgical therapeutic approach.

517-Large, bulky tumors in which concurrent chemoradiation is unlikely


to achieve local control of disease.

518-Preservation of at least one functional cricoarytenoid unit


makes it possible to consider an organ-preservation procedure. Primery
surgecal approach in:cricoarytenoid unit consists of an arytenoid
cartilage, the cricoid cartilage, the associated musculature, and the
superior and recurrent laryngeal nerves for that unit.

519-preservation of at least one arytenoid and an intact


circumferential ring at the level of the cricoid is suficient for
speech and swallowing without a tracheostomy.

520-arytenoid fixation, which implies cricoarytenoid joint invasion and


is a contraindication to conservation laryngeal surgery.
123

521-stripping: one-piece subepithelial cordectomy .

522-TLM is not totally transoral , not entirely laser ,not exclusively


surgical.

523-Transoral laser microsurgery (TLM) The limits of resectability


based on access and functional consequences and not on extent or T
stage.

524- most common cause of death in patients with early laryngeal


cancer is second primary tumorThe most common second primary
tumors are lung followed by other head and neck primaries.

525- EGFR overexpression is also associated with radioresistance and


poor local control of laryngeal cancer, and therefore a tumor’s EGFR
status may be used to predict radiosensitivity and chemosensitivity.

526-The most commonly mutated gene in human cancer including


laryngeal cancer isp53.

527-Verrucous carcinoma is less aggressive and has a better


prognosis than conventional SCC.

528-The standard treatment for recurrent laryngeal SCC is total


Laryngectomy.

529-impaired vocal cord mobility is usually secondary to either


tumor bulk or deep invasion. radiotherapy is less effective in
controlling these lesions, and this is probably often due to tumor
volume

530- In comparison with open surgical techniques, TLM avoids


atracheostomy, the hospital stay is shorter, the cost is reduced, and
there is a lower incidence of dysphagia postoperatively.

531-Cordectomy and vertical hemilaryngectomy (VHL) are the two


classic open procedures for the treatment of early glottic carcinoma.
.

.
124

532- T2 tumors with normal vocal cord mobility, RT achieves local


control in 64% to 87%, with laryngeal preservation rates of 75% to
87%.

533-Primary RT for T1 glottic SCC provides 5-year local control


rates of 81% to 90% and laryngeal preservation in 90% to 98% of
patients.

534-mirror may provide an excellent three-dimensional view of the


larynx.

535-The recurrence rate for CIS treated with surgery is greater than
those treated with RT and is reported to be approximately 20% after
initial excision.

536- 30% of patients with dysplastic lesions that progressed to


invasive cancer eventually underwent total laryngectomy.

537-The supraglottic larynx is derived from the buccopharyngeal


primordium, which develops from the third and fourth branchial arches.
The glottis and subglottis are derived from the tracheobronchial
primordium from the sixth branchial arch.

538-Surgical salvage is less likely if the neoplasm totally encases or


invades vascular structures.

539-Neoplasms of the pyriform sinus generally are more


aggressive than lesions of the endolarynx and have a high incidence
of thyroid cartilage invasion.

540-Nonmalignant causes of thyroid cartilage sclerosis include


rheumatoid arthritis and polychondritis;.

541-Ossified portions of cartilage are at higher risk for invasion


possibly because of vascular channel penetration, whereas intact
perichondrium surrounding avascular unossified cartilage resists tumor
encroachment.
125

542-If the extent of tumor caudal from the inferior margin of true
vocal cords is greater than 1 cm anteriorly and 6 mm posteriorly, total
laryngectomy may be indicated.

543-Tumor that has reached the anterior commissure can grow


into the thyroid cartilage pre-epiglottic space, opposite vocal cord,
or subglottic space.

544-carcinomas confined to a normally mobile true vocal cord


may be treated with radiotherapy, laser, cordectomy, Or partial
laryngectomy.

545-Zenker’s diverticulum is classically defined as a pulsion


diverticulum between the cricopharyngeal muscle and inferior
constrictor muscle in an area of weakness called Killian’s
dehiscence.

546-30% of patients with caustic esophageal injury do not show any


evidence of oropharyngeal damage.

547-The subglottis is the narrowest part of the pediatric airway.


endotracheal intubation remains by far the most common cause of
. laryngeal trauma.

548-Endotracheal intubation is not a preferred method of airway


control in laryngeal trauma, because it can exacerbate a
laryngeal injury and precipitate total airway obstruction.

549-The three major functions of the larynx—respiration, phonation,


and airway protection.

550-The RLN contains 1000 to 4000 motor axons, efferent axons,


and sympathetic and parasympathetic secretomotor Fibers.

551-Several procedures to manage glottal insuficiency, including vocal


fold injection for medialization, medialization thyroplasty, arytenoid
adduction, adduction arytenoidopexy, and a variety of reinnervation
procedures.

.
126

552-Electromyography is the only test available to evaluate the


integrity of the laryngeal motor unit.

553-Direct stimulation of sensory receptors in the larynx by aspirated


or refluxed material can result in reflexive vocal fold adduction
or laryngospasm. This laryngeal chemoreflex is associated with
bradycardia, central apnea, and hypotension.

554-“hey” lowers the tongue base, facilitating placementThe


patient is then instructed to sustain “ee,” which moves the epiglottis
anteriorly.

555-most important pharyngeal dilator is the genioglossus.

556-Oral breathing requires activation of the levator veli palatine to


elevate the soft palate as well as activation of the musculus uvula.

557-Because of these physiologic differences, the phrenic nerve is not


an ideal choice for reinnervation of the PCA in patients with laryngeal
paralysis.

558-The posterior cricoarytenoid muscle (PCA), the only active dilator


of the larynx.
.
Part 1V

HEAD & NECK


Part 4

Part 4
HEAD & NECK
127

1-Malignant lymph nodes appear more rounded as compared to benign


nodes, which are bean-shaped.

2-A well-defined cystic-appearing parotid tumor in an elderly male, who


is a smoker, is almost always a Warthin’s tumor.

3-Hemangiomas enhance intensely with contrast and contain flow voids


representing feeding vessels.

4-Schwannomas and paragangliomas are the most common benign


tumors of the carotid space.

5-The thyroid gland develops from the third and fourth pharyngeal
pouches anddescends during fetal development from the foramen
cecum at the midline base of the tongue to the thyroid bed along the
course of the thyroglossal duct.

6-Subacute thyroiditis also commonly referred to as de Quervain’s


Thyroiditis, is aself-limiting viral disease that is more common in
females. Clinical symptoms include a painful, tender gland after an
upper respiratory infection.

7-Goiter refers to abnormal thyroid growth with enlargement of the


gland and can be associated with normal, decreased, or increased
thyroid hormone production; both size and function impact clinical
management.

8-Almost all hot nodules represent benign hyperfunctioning adenomas.

9-incidence of cancer in a cold nodule is 15–20 %;

10-Ultrasonography fi ndings suspicious for thyroid cancer include a


taller-than- wide shape, spiculated or microlobulated margins, marked
hypoechogenicity, and microand macrocalcifi cations.

11-“raccoon” sign (periorbital ecchymosis) is associated with basilar


skull fractures that involve the middle or anterior cranialfossa.
128

12-Thyroglobulin is a very useful marker for thyroid cancer, both to


assess treatment efficacy and to monitor for recurrence after total
thyroidectomy and radioiodine 131 I therapy.

13-The goiter in Hashimoto’s thyroiditis is usually moderate in size and


firm in consistency.

14-vonRecklinghausen disease is an autosomal dominant disease with


clinical findings of cafe-au-lait spots and neurofibromas.

15-The diagnosis of metastatic squamous cell carcinoma to the neck


can be diagnosed accurately by FNA biopsy.

16-(PET-CT), is increasingly used to assess the extent of primary tumor


invasion, evaluate for regional and distant metastases, and detect
synchronous second primary tumors.

17-FNA is best avoided when a paraganglioma is suspected because of


the potential for bleeding .

18-Necrotizing sialometaplasia is a benign, self-healing inflammatory


process mainly involving the minor salivary glands. painless ulceration
or swelling usually over the hard palate,

19-Adenoid cystic carcinomas have a low incidence of regional


metastases but greater distant metastases.

20-Treatment of xeroderma pigmentosum is total avoidance of the


sun,.

21-venous malformations are bluish-purple in color, raised, and easily


compressible.

22-The treatment of choice for a capillary malformation is laser


photocoagulation.

23-Treatment of heamangioma: steroid,propronalol,interferon alph


2a,venicristin ,laser,surgical excision.
129

24-Protamine and IV contrast agents can potentially provoke


hypersensitivity responses in patients with known shellfish or other fish
allergies.

25-Similarly, patients with allergic or adverse reactions to soy- beans or


eggs may react to propofol,

26-28% to 67% of children with spina bifida demonstrate positive skin


tests to latex proteins.

27-In adults,lymph nodes greater than 1 cm in size are more likely to


be pathologic; in children, nodes greater than 1.5 cm in greatest
diameter are concerning.

28-The Virchow node is not in the VB region but is located in level IV.

29-More worrisome for a precancerous condition is erythroplakia;


there- fore all red lesions and most white lesions should be biopsied.

30-A Chvostek sign, facial nerve hyperactivity elicited by tapping over


the common trunk of the nerve as it passes through the parotid gland,
and Trousseau sign, finger and wrist spasm after inflation of a blood
pressure cuff for several minutes, are clinically important indicators of
latent hypocalcemia.

31-First branchialcleft anomalies represent around 10% of all branchial


cleft anomalies, with second branchialcleft anomalies being by far the
most common, comprising around 85% of cases.

32-Treatment options of hemangioma include oral corticosteroid


therapy, intralesional corticos teroids, β-blockers, interferon, vincristine,
and surgical removal.

33-Hemangiomas are benign vascular tumors of the endothelial cells


that appear as a red discolored area on the skin. They are usually not
present at birth, appear in the first weeks of life, and grow in early
infancy followed by spontaneous resolution later in childhood.
130

34-The (A train )has been shown to be the only intraoperative EMG


pattern associated with deterioration of facial nerve function.

35-BCC rarely metastasizes to cervical lymph nodes . In the absence of


clinical nodal disease, elective neck dissection is not indicated.

36-A sentinel lymph node biopsy is not offered to patients with lesions
measuring 0.75 mm or smaller and to those with clinical nodal or
distant metastasis.

37-the absence of necrosis in large nodes is suggestive of lymphoma.

38-propofol produces sedation and amnesia with rapid onset and rapid
clearance.

39-PET/CT has very limited utility in the evaluation of thyroid disease


because ultrasound provides high-quality neck and thyroid imaging.

40-PET/CT is more accurate than conventional imaging in detecting


recurrent or residual neoplasm and is useful to monitor treatment
response to therapy.

41-8 5 % of Subarachnoid hemorrhage are caused by aneurysms.

42-Paragangliomas are the only laryngeal neoplasm with a female


predominance.

43-Streptococcus viridans is a common infectious agent in chronic and


. recurrent sialadenitis.

44-Staphlococcus aureus is the most common microorganism


associated with acute bacterial sialadenitis.

45-Ideal neck dissection performed for elective treatment of a patient


with SCC arising in the oral cavity with N0 neck is supraomohyoid neck
dissection.

46-Carcinoma in situ without an invasive component is relatively rare.


131

47-Radiologic studies is the most valuable for evaluating deep neck


infections is contrast-enhanced computed tomography.

48-Pneumocystis carinii infections of the middle ear respond well to


trimethoprim-sulfamethoxazole.

49-ENoG done within 3 days of paralysis may not accurately represent


the degree of injury because wallerian degeneration occurs over 3 to 5
days following injury or insult.

50-ENoG can differentiate between minor conduction blocks


(neurapraxia) versus wallerian degeneration (axonotmesis and
neurotmesis),

51-Surgical decompression can be considered if degeneration is greater


than 90% within 14 days of onset and there is no voluntary
electomyographic activity.

52-Fibrillation potentials found on facial EMG indicate intact motor end


plates but no evidence of reinnervation.

53-Aided speech recognition scores are the primary audiologic


determinant of cochlear implant candidacy.

54-Aminoglycoside antibiotics are poorly absorbed through the


gastrointestinal tract or through intact skin.

55-Advantage of a jejunostomy tube over a gastrostomy tube is


Decreased aspiration.

56-Recommended daily energy intake and protein intake for HNSCC


patients undergoing surgery or chemoradiation therapy 30 kcal/kg/day
and 1.2 g protein/kg/day.

57-Dysphagia after chemoradiation therapy typically results from


Formation of esophageal strictures.

58-Structures pass through the cavernous sinus:Medially the ICA with


its sympathetic plexus and laterally CN VI.
.
132

59-The parathyroid glands are both exclusively supplied by the inferior


thyroid artery of the thyrocervical trunk.

60-Targeted treatment has been most difficult to design For Tumor


suppressor genes.

61-Balanced translocation events in sarcomas can be used for


Differentiating sarcomas of the central part of the neck from anaplastic
thyroid carcinomas.

62-The most common tumor suppressor gene mutation identified in


HNSCC is p53.

63-Most common site of p53 mutations is Exons 5-8 .

64-A direct inhibitory effect of HPV has been corroborated for p53.

65-Salivary duct carcinoma is Usually positive for androgen receptors


by immunohistochemical analysis.

66-About 40% of Hodgkin lymphoma cases are associated with EBV.

67-The average weight of a normal parathyroid gland Is 90 mg to 100


mg.

68-The distinction between follicular adenoma and follicular carcinoma


cannot be made on cytological or fine-needle aspiration (FNA) material
in most cases.

69-Adenoid cystic carcinoma is composed histologically of epithelial and


myoepithelial cells.

70-Tall cell variant is a more aggressive histological variant than


classical type papillary carcinoma.

71-The presence of amyloid is a characteristic feature of medullary ca.


133

72-Clark levels (microscopic extent of tumor invasion of the skin


layers).

73-Basal cell carcinomas rarely metastasize.

74-BCC can be mistaken histologically for adenoid cystic carcinoma.

75-MRI is more helpful than CT for imaging cartilage invasion in the


adult larynx.

76-Ultrasonography is more sensitive than CT, MRI, PET for identifying


early lymph node metastasis in well-differentiated papillary thyroid
cancer.

77-MRI is less susceptible to artifacts from dental amalgam compared


with CT.

78-CT is able to identify early extracapsular spread from nodal


metastasis better than MRI.

79-Toxic effects are most typically associated with cetuximab are


Hypomagnesemia and acneiform rash.

80-Mechanism of action of cetuximab is binds to the extracellular


domain of EGFR.

81-5-FU:is commonly administered as a continuous intravenous


infusion.

82-5-FU causes profound diarrhea and mucositis during his first cycle.

83-Main mechanism of action of platinum chemotherapy Formation of


DNA cross-links.

84-Cisplatin is excreted almost entirely in the urine.

85-Carboplatin exerts its cytotoxic effect by binding to DNA.

86-CT-based landmark differentiates level III from level IV :Cricoid


cartilage.
134

87-CT-based landmark differentiates level II from level III: Hyoid bone.

88-TSH :routinely checked in follow-up visits after a patient receives


radiation therapy.

89-The typical daily fraction size in definitive treatment for head and
neck cancer is 2 Gy.

90-Scleroderma can be a contraindication to delivering radiation


therapy.

91-postoperative setting, radiation therapy should typically begin within


6 weeks.

92-Antioxidants may interfere with the effectiveness of radiation


therapy

93-The primary mechanism by which radiation kills cancer cells is


Damage to DNA.

94-Adenoid cystic cancers respond well to neutron beam radiation


therapy.

95-Human papillomavirus (HPV)-positive tumors are more sensitive to


radiation than HPV-negative tumors.

96-Accelerated fractionated radiation therapy results in higher long-


term toxic effects than conventionally fractionated radiation therapy.

97-Hypothyroidism is common after radiation therapy for squamous cell


carcinoma (SCC) of the head and neck.

98-Radiology report “sunburst appearance of the anterior mandible,”:


Osteosarcoma and intraosseous hemangioma.

99-If the radiology report “cotton-wool appearance of the right


mandible” :Cementoosseous dysplasia, Paget disease, and Gardner
syndrome.
135

100-initial treatment of osteoradionecrosis begins With Curettage of


the area, chlorhexidine gluconate rinses, 30 dives of hyperbaric oxygen,
and systemic antibiotics.

101-hyperbaric oxygen 100% oxygen at two to three times


atmospheric pressure for 90 minutes.

102-iliac crest flap based on the deep circumflex iliac artery and vein,

103-fibula free flap is based on the peroneal artery and vein.

104-jejunal free flap is based on superior mesenteric artery.

105-40% of the radius can be harvested safely with the flap.

106-Blood supply to the temporalis muscle is from the deep temporal


artery,

107-Rotation flaps are good options for small- to medium-sized scalp


defects, large cheek defects, and dorsal nasal defects.

108-Melolabial flaps are a very good option for repair of alar defects.

109-hemiglossectomy defect is better reconstructed with a free flap.

110-Tonsil and lateral pharyngeal wall defects in salvage situations are


better repaired with free tissue transfer.

111-Rectus abdominis flap can be used in a contaminated field.

112-Blood supply to the superior trapezius flap Is from the paraspinal


perforators.

113-pectoralis major muscle blood supply is from the pectoral branch


of the thoracoacromial artery.
136

114-Advancement flaps Are good options for repair of forehead


defects.

115-Blood supply to the temporoparietal fascial Flap from Superficial


temporal artery.

116-Rotation flap It is better to design a superiorly based flap for face


reconstruction.

117-Both latissimus dorsi and serratus anterior muscle flaps can be


raised based on the thoracodorsal artery.

118-Fibula free flap blood supply is from the peroneal artery.

119-parascapular flap It is based on the circumflex scapular artery.

120-lateral arm flap The blood supply is from the profunda brachii.

121-lower trapezius island musculocutaneous flap The blood supply is


from the transverse cervical and dorsal scapular arteries.

122-Most oropharyngeal defects after transoral resection of the tonsil


and tumors of the base of the tongue can be left to granulate without
the need for reconstruction.

123-The scapular tip free flap is based on the angular branch of the
thoracodorsal artery.

124-The deltopectoral flap receives its blood supply from the internal
mammary artery.

125-most common odontogenic tumor Odontoma.

126-most common odontogenic cysts Dentigerous cyst, radicular cyst,


residual cyst.

127-lesions have a“soap bubble” appearance on a CT scan:


Ameloblastoma , Odontogenic myxoma and Aneurysmal bone cyst.
137

128-Aneurysmal bone cysts may be treated primarily with radiation


therapy.

129-Aneurysmal bone cysts have a radiographic “soap bubble”


appearance.

130-Fibrous dysplasia may develop into osteosarcoma.

131-Fibrous dysplasia has three radiographic patterns:cystic, sclerotic,


and mixed radiolucent and radiopaque.

132-Odontomas are the most common odontogenic tumors.

133-Malignant ameloblastomas most commonly metastasize to the


lungs.

134-Ameloblastoma is an uncommon benign, locally aggressive


odontogenic neoplasm.

135-The most common site of presentation of osteosarcomas of the


head and neck is the Mandible.

136-Alveolar RMS is pathologically and clinically distinct from


embryonal RMS.

137-Hemangiopericytomas arise from Zimmermann pericytes.

138-Capillary hemangioma Is also known as a strawberry nevus and


regresses with time.

139-Cavernous hemangioma Fails to regress with time.

140-The most common subtype of liposarcoma is Atypical


lipomatous/well-differentiated
138

141-Liposarcoma is the most common of the soft tissue sarcomas.

142-Liposarcoma round cell tumors have the worst survival outcome.

143-Chondrosarcomas are predominantly low-grade sarcomas.

144-Osteosarcomas do not metastasize to the cervical lymph nodes.

145-Paragangliomas arise from extraadrenal paraganglionic cells


derived from the neural crest.

146-Schwannomas are isointense on T1 (MRI), hyperintense onT2


MRI,

147-Glomus vagale tumor:displaces the internal carotid artery


anteromedially.

148-Botulinum toxin injection into the parotid gland is a standard of


care for the management of the first bite syndrome.

149-The diagnosis of paraganglioma is generally established By A


combination of CT and MRI.

150-Dissection of carotid body tumors should be performed in a


subadventitial plane.

151-Rapid growth, cervical lymph node metastasis, or radiographic


features of local invasion of paragangliomas should raise the suspicion
of malignancy.

152-Radiation therapy is an effective modality in the management of


paragangliomas and schwannomas and has 10-year local progression-
free rates ranging from 92% to 100%.

153-the average rate of growth of paragangliomas is 1 mm/year.


139

154-An octreotide scan can be used to detect additional


paragangliomas and to screen patients thought to be at risk of
SDH(succinate dehydrogenase) mutations.

155-Paragangliomas express somatostatin receptor.

156-Multiple low-signal flow voids on T1-weighted (MRI) are typically


characteristic of paragangliomas.

157-Plexiform neurofibromas are typically associated with familial


neurofibromatosis type 1(NF1).

158-The paraganglioma syndromes are associated with mutations in


the SDH gene.

159-Approximately 10% of paragangliomas are reported to be familial.

160-Carotid body tumors are the most common paragangliomas in the


head and neck.

161- Pleomorphic adenomas are thought to arise from the intercalated


duct and have epithelial and myoepithelial components.

162-The most consistent intraoperative landmark for facial nerve is the


superior aspect of the posterior belly of the digastric.

163-The best imaging modality for diagnosis of Sjoِgren : MRI with MR


.

164-Collagen vascular disease is not part of primary Sjoِgren.


Sialography.

165-Buccal branch of the facial nerve Extends along the path of the
Stensen duct.

166-Sialadenitis is the most frequent complication of 131I therapy for


thyroid cancer.
.
140

167-Persistent reduction in salivary flow from the parotid gland occurs


at a total dose of 50 to 60 Gy.

168-Staging of minor salivary gland tumors is based on the site of


origin.

169-There is no distinct staging system for tumors arising in minor


salivary glands; hence there is no uniformity for the staging of salivary
glandtumors.

170-Overall, 65% of salivary gland cancers arise inthe parotid gland,


27% arise in the minor salivaryglands, and 8% arise in the
submandibulargland.

171-Minor salivary glands secrete Mucous saliva.

172-The potential cells of origin for pleomorphic adenomas include


Myoepithelial cells .

173-Characteristic MRI findings for pleomorphic adenoma include


Hyperintensity on T2.and enhancement with gadolinium

174-The most reliable landmark for the facial nerve is the


Tympanomastoid suture line.

175- submandibular gland contributes approximately 75% of saliva


secretion.

176-Saliva stimulated parasympathetically is more watery.

177-Saliva stimulated by sympathetic innervation is thicker.

178-Metastases to the lungs are usually asymptomatic or “silent” in


adenoid cystic carcinoma..

179-The solid growth pattern of adenoid cystic carcinoma is associated


with a higher incidence of metastases.
141

180-Combined patterns of growth (tubular and solid) are common in


adenoid cystic carcinoma.

181-Adenoid cystic carcinoma is considered a slowgrowing highly


malignant tumor.

182-Adenoid cystic carcinoma was formerly known as a cylindroma.

183-Pediatric parotid neoplasms are more likely to be malignant.

184-Parathyroid cells are extremely liable to implantation; therefore


when such surgery is performed, the capsule of the gland must remain
intact.

185-The finding of black thyroid has been associated with


administration of minocycline, commonly prescribed for acne vulgaris.

186-Anaplastic thyroid ca. is rapidly progressive and almost universally


fatal.

187- 25% of medullary thyroid cancer will have a heritable genetic


defect.

188-Medullary thyroid cancer is more aggressive and difficult to cure.

189-Risk assessment strategies exist for predicting survival (grade,


age, metastases, extension, size [GAMES]) and recurrence.

190-The most important differential diagnosis in patients with


suspected anaplastic thyroid cancer is Lymphoma.

191-Level IV posterior to the great vessels.a common site for nodal


metastases from thyroid cancers ,

192-The ideal incision for a thyroidectomy and neck dissection should


run in a skin crease at the level of the cricoid cartilage.
142

193-Rate of incidental malignancy in multinodular goiters 10% to 30%.

194-side effects is associated with the administration of radioactive


iodineDysphagia ,Xerostomia and Secondary malignancy

195-The prognosis of patients with unresectable medullary cancer is


best assessed with the use of Calcitonin doubling time.

196-Children at risk of medullary thyroid cancer on genetic screening


Should undergo risk assessment depending on the genetic mutation
detected.

197-Lateral nodal disease in papillary thyroid cancer Predicts worse


survival rates for older patients.

198-extended MRND:Sacrifice of Accessory nerve, hypoglossal nerve,


internal jugular vein.

199-Papillary thyroid carcinoma is most likely to give rise to isolated


metastasis at level IV.

200-The accessory nerve Supplies the lower fibers of the trapezius,


which is responsible for rotation of the scapula.

201-The treatment of choice for lymphadenopathy caused by atypical


mycobacteria in children is Complete surgical excision without
antimicrobials

202-Anti-VCA EBV IgA antibodies most useful at pinpointing a primary


Site In metastatic SCC of the neck from an unknown primary,

203-The radiological demarcation between level I and level II in the


neck is:A vertical line through the posterior border of the
submandibular gland.

204-The surgical demarcation between level III and level IV in the


neck is A horizontal line along the inferior border of the cricoid
cartilage.
143

205-phrenic nerve runs from lateral to medial, over the


scalenusanterior muscle, deep to the transverse cervical vessels.

206-The hypoglossal nerve is always located deep to the facial vein.

207-Most level II unknown primary SCCs have an occult primary site


within the tonsil.

208-Adenoid cystic carcinoma of the parotid gland has a low risk of


cervical metastases; thus elective neck dissection is not considered
necessary.

209-Castleman disease is characterized by progressive lymph node


enlargement, iron-deficiency anemia, and risk of progression to non-
Hodgkin lymphoma.

210-postoperative radiotherapy of neck dissection improves regional


control but not survival rates.

211-In the head and neck, more than 90% of HPV-associated tumors
appear to be caused by type 16.

212-The current treatment recommendation for T1 nasopharyngeal


tumors includes Definitive RT and elective neck irradiation.

213-In nasophar. cancer the most common site of metastases is the


Bone.

214-The World Health Organization (WHO) has classified NPC as


keratinizing SCC (type 1), nonkeratinizing(differentiated) carcinoma
(type 2), and undifferentiated carcinoma (type 3).

215-Identification of the sentinel node requires the use of preoperative


lymphoscintigraphy using radioactive technetium and then blue dye
injection (toluidine blue) at the time of surgery.

216-all erythroplakia lesions should be treated aggressively by wide


surgical excision.

.
144

217-Mean survival for a patient with adenoid cystic carcinoma lung


metastases is 3 years reflecting the relatively indolent activity of the
tumor.

218-Adenoid cystic carcinomas present with cervical nodal metastases


in <10% of cases.

219-Cervical nodal metastases are present in 22% of minor salivary


gland cancers predominantly by small cell carcinomas,

220-Postoperative chemoradiation therapy is indicated in patients with


Extracapsular lymph node spreading.

221-An absolute indication for postoperative radiation therapy is


Positive margins.

222-The occurrence of a second primary head and neck cancer exists


at a constant rate of 4% to 7% per year.

223-Aneuploidy decreases the risk of malignant transformation of oral


dysplasia.

224-13-cis-Retinoic acid can reverse epithelial dysplasia and reduce the


risk of a second primary cancer.

225-Sentinel node biopsy is defined as the first-echelon lymph node to


which cancer spreads.

226-Fibrous dysplasia:Progressive replacement of normal cancellous


bone by fibrous tissue and immature woven bone.

227-The classic imaging finding in clival meningioma is “Dural tails.”

228-The MRI T1 and T2 salt and pepper appearance is seen In


Paragangliomas greater than 2 cm.

229-indication for orbital exenteration:invasion of orbital fat, orbital


musculature, or involvement of the orbital apex.

.
145

230-Mucosal melanoma is often best confirmed with


immunohistochemical analysis.

231-All primary intestinal-type adenocarcinomas are reactive to CK20,

232-The infraorbital nerve can be followed through a transfacial


approach.

233-For assessment of the skull base and intracranial extension of


cancer , CT and MRI is most informative.

234-Perineural spreading to the skull base is a hallmark of adenoid


cystic carcinoma.

235-The most common malignant neoplasm of the lacrimal gland is


adenoid cystic carcinoma.

236-The most common benign neoplasm of the lacrimal gland is


pleomorphic adenoma.

237-The most common pediatric intraorbital malignancy is


retinoblastoma.

238-Approximately 80% of orbital lesions are benign.

239-Vincristine in treat rhabdomyosarcoma Inhibits microtubule


formation during M and S phases.

240-Tungsten eye shields are preferred to protect the cornea during


electron-based radiation therapy.

241-lowest tolerance for radiation:Lens.

242-best describes chemosis:Conjunctival edema.

243-von Hippel-Lindau associated with Retinal capillary hemangioma.

244-Merkel cell polyomavirus (MCV) is detected in 50% to 100% of


patient samples.
146

245-3% to 5% of melanomas in situ evolve into invasive melanoma.

246-Tc- 99 m scanning gives excellent information about bone function


but poor information about bone structure.

247-Evidence of clinically suspicious lymph node metastases includes


size > 1 cm, central necrosis, round rather than ovoid shape, and
poorly defined borders suspicious for extracapsular spread.

248-Most patients with fibrous dysplasia can be observed. When


lesions are disfiguring or create functional impairment, recontouring is
appropriate.

249-most common odontogenic cyst, Radicular.

250-More than 9 0 % of both third and fourth branchial cleft anomalies


occur on the left side.

251-Sturge-Weber syndrome typically presents with a facial port-wine


stain in the ophthalmic distribution of the trigeminal nerve, glaucoma,
and vascular eye abnormalities and in ipsilateral intracranial vascular
malformations.

252-orbital floor fracture:should be treated urgently to prevent


irreversible ischemic damage to the inferior rectus .

253- The condyle is the most commonly involved site of pediatric


mandible fractures.

254-Immediate facial paralysis after penetrating trauma should be


considered a nerve transection and the patient should be surgically
explored.

255-Thyroid surgery carries a higher risk of bilateral vocal fold


paralysis,

256-Palifermin:has efficacy in preventing chemoradiation-related


mucositis..
.
147

257-The most likely complication is Lemierre syndrome (lung abscesses


from septic emboli from jugular thrombophlebitis).

258-Topical decongestants act via a-adrenergic receptors .

259-Pseudoallescheria boydii , this fungus is susceptible to


voriconazole.

260-medications may be considered "allowed " at various stages of


pregnancy-for example, Loratadine.

261-propofol produces sedation and amnesia with rapid onset and


rapid clearance.

262-Thermal effects of the laser may be reduced by the pulse-wave


mode.

263-Sestamibi scanning is the most commonly used modality for


localization of a parathyroid adenoma.

264-Aztreonam inhibits mucopeptide synthesis in the bacterial cell wall.

265-four phases of wound healing include:Hemostasis and coagulation,


inflammation, proliferation, remodeling.

266-p 16 protein immunohistochemistry is the preferred method for


identifying a human papillomavirus (HPV) -related carcinoma.

267-Nortriptyline is a well-tolerated tricyclic antidepressant with


efficiency in treating migraine-associated vertigo .

268-Intrathecal fluorescein may be introduced via a lumbar puncture


or spinal drain for
better identification of a CSF fistula.

269-Radionuclide study: a highly sensitive test to detect a CSF leak.

270-Le Fort I: isolated detachment of the alveolar process.


148

271-Le Fort II: pyramidal fracture with detachment of the maxilla.

272-Le Fort III: craniofacial dysjunction.

273-High-dose interferon-a 2b is the only FDA-approved adjuvant


therapy for stage III melanoma.

274-p16& p 21: tumor suppressor proteins; suppress cyclin and cyclin-


dependent kinase pathways.

275-Gardner’s syndrome = skull osteomas, colonic polyps, and soft


tissue tumors.

276-Hemophilia– Deficiency in clotting factor VIII (type A) or IX (type


B).

277-Deficiency of von Willebrand protein leads to defective platelet


adhesion and decreased activity of clotting factor VIII;

278-Rhabdomyosarcoma:Embryonal (75 %):Alveolar (20 %):


Pleomorphic.

279-Torticollis or decreased neck range of motion post-tonsillectomy


should be suspicious for Grisel’s syndrome

280-Microtia repair before atresiaplasty to preserve vascularity of tissue


flaps.

281-Syndromes associated with microtia/atresia: CHARGE, Crouzon’s,


Goldenhar’s, hemifacialmicrosomia, Pierre Robin sequence, Treacher
Collins, VATER.

282-Ossicles: adult-sized at birth.

283-a child presenting with atresia; the initial management step is to


amplify.

284-Cafe-au-lait spots, which are the hallmark of NF1, are also


frequently found in patients with NF2.
149

285-Skin tumors are present in nearly two-thirds of patients with NF2.

286-Gardner and Wishart NF2 subtypes .

287-Inheritance of NF2 is autosomal dominant and gene penetrance is


above 95%.

288-the gene responsible for NF1 was located near the proximal long
arm of chromosome 17, whereas the gene responsible for NF2 was
located on chromosome 22.

289-B symptoms including fevers, drenching night sweats, and weight


loss are usual HL.

290-B symptoms including fevers, drenching night sweats, and weight


loss are unusual.NHL

291-Classical HL(lymphoma) occurs most commonly in young adults


and usually presents as painless lymphadenopathy in the cervical
and/or supraclavicularregions. rarely involves the preauricular or
occipital nodes or extranodal sites of the head and neck such as the
tonsil or sinus.

292-parathyroid chief cells are responsible for the production and


storage of PTH.

293-Primary hyperparathyroidism, the most common cause of


hypercalcemia in nonhospitalized patients, is treated surgically in most
cases.

294-A normal thyroid gland weighs approximately 10–20 g, depending


on dietary iodine intake, age, and weight.

295-increased T2 and decreased T1 signal intensity on MRI, which


indicates cartilage involvement.

296-CT scan was more sensitive and specific than the bone scan and
Panorex in predicting mandibular invasion.

.
150

297-Syphilitic leukoplakias seen in tertiary syphilis have a higher


reported rate of malignant transformation.

298-Paradoxically, an increased risk of malignant transf. of leukoplakic


lesions is seen more commonly in nonsmokers compared with smokers.

299-chemotherapy in the treatment of salivary gland malignancies is


currently limited to palliative treatment of locally advanced
unresectable, recurrent, and metastatic disease,

300-if the preoperative facial nerve functionis fully intact, the nerve is
most likely not invaded, and all attempts should be made to preserve it.

301-total parotidectomy, removing all of the lateral and deep tissue, is


indicated for (1) high-grade malig. tumors with a high risk for
metastasis, (2) any parotid malignancy with an indication of having
metastasized to intraglandular or cervical lymph nodes, and (3) any
primary malignancy originating within the deep lobe itself.

302-superficial parotidectomy is generally adequate treatment for a


small low-grade tumor that is well encapsulated, such as an acinic cell
carcinoma.

303-no known benign acinic cell neoplasm.

304-histologic patterns are solid/lobular, microcystic, papillary-cystic,


and follicular of Acinic C C.

305-Acinic cell carcinoma:More than 90% occur in the parotid. second


most common childhood salivary gland malignancy.

306-Polymorphous low-grade adenocarcinoma(PLGAs) arise almost


exclusively from the minor salivary glands and, inmost series, are the
second most common minor salivary gland carcinoma.

307-PLGAs:The most common site for these neoplasms is the palate,


particularly at the junction of the hard and soft palates. most cases
have perineural invasion
.
151

308-ACC:Microscopically, there are three growthpatterns: tubular,


cribriform, and solid.

309-Enlarged foramina at the skull base and the presence of


hyperintense enhanced tumor tissue are suggestive of perineural
spread.

310-In general, benign epithelial neoplasms (such as


pleomorphicadenomas) and low-grade malignancies have low T1-
weighted and high T2-weighted signal intensities. High-
gradecarcinomas tend to have low-to-intermediate signal intensities on
bothT1-weighted and T2-weighted imagin.

311-(MRI) is the radiologic modality most often recommended to


assess salivary gland neoplasms if there are no contraindications.

312-Compared with the major salivary glands, minor salivary glands


have minimal capsular tissue, making local invasion of tumors into
surrounding tissue common.

313-the site most frequently involved with minor salivary gland


malignancies is the hard palate. 314-Carcinoma in situ is described as
malignant transformation without invasion,

315-recurrent local disease may be treated by brachytherapy.

316-to confirm the efficacy of using intensity-modulated radiation


therapy (IMRT) to treat NPC patients. This has resulted in control rates
similar to conventional radiation. It has also resulted in better treatment
outcomes, especially xerostomia and skin burns.

317. The most common cranial nerve to be affected postradiation,


other than the cochlear-vestibular nerve, is the hypoglossal nerve.

318-In particular, a negative IgA Ea likely will indicate that the patient
has no NPC.

319-EBV serology titers are performed. These include IgA VCA


(viralcapsid antigen) and IgA Ea. The former is highly sensitive and the
latter specific.
152

320-An audiogram and tympanogram are performed to obtain a base


line level. In NPC. certain chemotherapy agents like cisplatin cause
ototoxicity.

321-skeleton being the most common distant site of metastasis in NPC.

322-Patients with dermatomyositis are at high risk for developing this


cancerand should be screened regularly because approximately 10% of
these patients can be at risk for developing the cancer.

323-Cranial nerve palsies are encountered in about 10% of the


patients. The most common nerves affected in descending order of
frequency are the sixth, fifth, twelfth, and ninth/tenth nerves.

324-EBV is present for the remaining life of the person within specific
circulating B lymphocytes or shed through saliva.in NPC.

325-The protein p16 has been reported to be inactivated in NPC


patients.

326-Postoperative RT is recommended when multiple nodes,


extracapsular spread, extralaryngeal invasion, and perineural or
lymphovascular invasion is present.

327-Lesions in close proximity to the mandible should not be treated


with brachytherapy owing to the risk of osteonecrosis.

328-Elective neck dissection for adenoid cystic carcinoma generally is


not recommended because the risk of occult nodal metastasis is low.

329-prognosis for the cribriform subtype is intermediate. The


tubularsubtype (35%) carries the best prognosis and is characterizedby
cords and nests of malignant cells The solid subtype (21%) has the
worst prognosis.in ACC.

330-McCune–Albright syndrome affects mostlyfemales and is


characterized by polyostotic fibrous dysplasia with cutaneous
hyperpigmentation, and endocrinopathy.

.
153

331-T1-weighted enhanced images can show an enhancing dural tail


(meningeal sign).

332-MRI is the study of choice for the diagnosis of meningiomas. Are


hypo- to isointense on MRI T1-weighted images, Areas of calcification
appear dark on both T1- and T2-weighted images

333-Molecular studies have shown deletions in chromosome 22 in


nearly 75%of meningiomas.

334-Meningiomas arise from arachnoid villi cap cells and are located
along dura, venous sinuses, and neurovascular foramina.

335-acetazolamide (to decrease CSF production).

336-computed tomography (CT) scanning of the head is usually the


first study performed on patients with head trauma.

337-Fractures of the orbital floor can cause diplopia on upward gaze


due to entrapment of the inferior rectus muscle.

338-Mohs surgical technique— Refers to complete micrographic


excision of the tumor using intraoperative histopathology to assess for
positive margins.

339-DiGeorge syndrome, can be responsible for the


hypoparathyroidism.

340-Myxedema coma is characterized by hypothermia, bradycardia,


alveolar hypoventilation with CO 2 retention, hyponatremia,
hypoglycemia, and either stupor or coma.

341-Hashimoto’s thyroiditis is the most common cause of


hypothyroidism.
342-2% of patients on amiodarone(37.3% iodine) will develop
hyperthyroidism.

343-Thyroid crisis is an acute exacerbation of all symptoms of


thyrotoxicosis..
154

344-thyroid stimulating immunoglobulin(TSI) is specific for Graves’


disease but the lack of a TSI elevation does not exclude the diagnosis.

345-Glycosaminoglycans can accumulate in the dermis layer, causing


thickening of the skin, especially over the anterior tibia (pretibial
myxedema). In Graves disease.

346-Graves’ disease is an autoimmune disorder characterized by the


production of immunoglobulins that bind and activate the TSH receptor,
which stimulates thyroid growth and hormone secretion.

347-Lymphoma can present rapidly in patients with long- standing


Hashimoto’s thyroiditis,

348-Anaplastic thyroid carcinoma is resistant to all treatment


modalities. Treatment is palliative and includes isthmectomy to prevent
tracheal compression,

349-MEN 2 patients who present with palpable MTC have a low rate of
surgical cure.

350-There is no role of 131 I or chemotherapy in the treatment of


MTC.

351-Patients with MTC with no evidence of LN involve. or distant


metastasis total thyroidectomy with prophylactic (level VI) neck
dissection. with local metastatic in the regional LN should also have
lateral neck dissection.

352-The pheochromocytoma should be surgically resected prior to


surgery for MTC.

353-MEN 2b have the most aggressive form, whereas the cancers


found in patients with MEN 2a are the least aggressive

354-MEN 2b consists of medullary carcinoma, pheochromocytoma, and


multiple mucosal neuromas.
155

355-MEN 2a is composed of medullary carcinoma, pheochromocytoma,


and hyperparathyroidism.

356-medullary carcinoma can secrete histaminase, prostaglandins,


serotonin, and other peptides. In addition to secreting calcitonin.

357-Papillary and follicular carcinomas involve thyroid epithelial cells;


medullary carcinoma is a disorder of the parafollicular or C cells.

358-It is often difficult to distinguish a follicular carcinoma from a


follicular adenoma on an FNA biopsy; frozen section at the time of
surgery is necessary.

359-After undergoing a total thyroidectomy, patients should receive


radioiodine to ablate the normal thyroid remnant or residual
microscopic disease.

360-Papillary carcinoma:25% to 50% of patients have involvement of


cervical lymph nodes at presentation.

361-Psammoma bodies are laminated calcified spheres and are usually


diagnostic of papillary carcinoma.

362-Cowden syndrome (eg, multiple hamartomas of the skin and


mucous membranes),

363-toxic multinodular goiter, control of the hyperthyroid state with


antithyroid drugs followed by 131 I treatment is treatment of choice.
Subtotal thyroidectomy is an alternate option.

364-Thyroid hormone suppressive therapy may be effective in


suppressing the growth of the multinodular goiter and preventing the
development of new nodules.

365-For many pregnant patients found to have differentiated thyroid


cancer on FNA, surgery can be deferred until after delivery.

366-Treat.of a malignant thyroid nodule, as indicated by FNA, requires


total thyroidectomy, with careful attention paid to local, palpable lymph
nodes that may require neck dissection at the time of surgery.
156

367-malignant growth is less likely to occur in a purely cystic lesion,


the fluid should still be sent for cytologic examination.

368-When more than two thyroid nodules are >1 cm, those with
suspicious sonographic appearance should be biopsied.

369-the diagnostic procedure of choice is a fine-needle aspiration


(FNA) biopsy of the thyroid nodule.

370-Nonpalpable nodules have the same risk of malignancy as palpable


nodules of the same size…thyroid nodule.

371-123 I can be used to assess both radioactive uptake and scan, but
99m Tc can only be used for scanning.both are contraindicatedin
pregnancy.

372-Thyroglobulin is a very useful marker for thyroid cancer, both to


assess treatment efficacy and to monitor for recurrence after total
thyroidectomy and radioiodine 131 I therapy.

373-thyroid stimulating immunoglobulin (TSI), is an indirect test that


confirms the diagnosis of Graves’ disease. Its +v in 90%.

374-Almost all patients with Hashimoto’s thyroiditis have antithyroid


peroxidase (anti-TPO) antibodies present.

375-Befor any Rx for malig.melanoma do total body scan to exclude


dist.m.

376-Most common type of malig.melanoma: superficial spreading type.

377-Primary tracheal tumors are very rare and account for less than
0.2% of all respiratory malignancies.

378-the adult male tracheal diameter is 1.8–2.3 cm compared to the


female airway, which is 1.4–2.0 cm… two rings per centimeter of
trachea.
.
157

379-Barrett esophagus (ie, metaplastic changes from squamous to


columnar epithelium), may lead to the development of
adenocarcinoma.

380-laparoscopic Heller myotomy and partial fundoplication is


considered today to be the primary treatment modality for esophageal
achalasia.

381-Traumatic FN paralysis second most common after Bell’s palsy.

382-proptosis: abnormal protrusion of the eyeball.

383-The most common congenital CNS abnormality resulting in vocal


cord palsy is the Arnold–Chiari malformation.

384-amifostine, are being used to mitigate side effects and preserve


salivary function in the setting of radiation.

385-Uncommon side effects of radiation include hypothyroidism,


chondroradio -necrosis, and osteoradionecrosis. Rare complication is
radiation-induced sarcoma.

386-Common long-term side effects of radiation include varying


degrees of xerostomia, fibrosis, and edema.

387-Short-term side effects of radiation, lasting up to 6 weeks after the


conclusion of therapy, include mucositis, odynophagia, dysphagia, skin
erythema, altered taste, and edema.

388-carious teeth are extracted before starting radiation owing to the


radiation-induced dental decay and increased risk of osteoradionecrosis.

389-An elective neck dissection has the advantage over elective


irradiation of the ability to stage the neck pathologically, which provides
prognostic
information.

390- (PET) scanning uses fluorescence-tagged glucose and the


increased metabolic rate of malignant tissues to identify cancers.
.
158

391-patients with head and neck cancer should have a chest X-ray as
part of a routine metastatic evaluation.

392-chylous fistula; if the drainage persists, is >600 cc/d, or is noted


immediately postoperatively, surgical exploration with ligation of the
stump may be necessary.

393-The incidence of an unknown primary tumor is between 2% and


8%of all patients with head and neck squamous cell carcinoma.

394-vonRecklinghausen disease is an autosomal dominant disease with


clinical findings of cafe-au-lait spots and neurofibromas.

395-in a patient with an FNA highly suggestive of lymphoma, an open


biopsy may be necessary to obtain sufficient tissue for histopathologic
classification.

396-An elevated angiotensin-converting enzyme (ACE) level is seen in


60–90% of patients with sarcoidosis.

397-Actinomycosis is a gram-positive bacillus it is confirmed


histologically by the presence of granulomas with sulfur granules.
Penicillin is the treatment of choice.

398-Radiation therapy fields include bilateral neck and supraclavicular


nodes, as well as retropharyngeal nodes, in NPC.

399-Because of the difficulty in obtaining adequate surgical margins,


the primary treatment for nasopharyngeal carcinoma is with definitive
radiation therapy, even in early-stage lesions.

400-EBV titers are elevated in undifferentiated NPC and should be


checked in the case of an unknown primary tumor of the head and
neck.

401-abducens nerve, is most commonly cranial nerve involved in


nasophar ca.
.
159

402-Genetic associations with nasopharyngeal carcinoma include HLA-


BW46 and HLAB17.

403-late complications of radiation include osteoradionecrosis and


carotid artery rupture.

404-FNA is best avoided when a paraganglioma is suspected because


of the potential for bleeding.
. 405-Elective neck dissection for adenoid cystic carcinoma generally is
not recommended because the risk of occult nodal metastasis is low.

406-In children, the capillary hemangioma is the most common


salivary gland tumor, more than 90%of parotid gland tumors in less
than 1 year of age.

407-Warthin tumor:12% of benign tumors of the parotid gland. This


tumor is more commonly seen in males in the fifth to seventh decades
of life and there is an associated risk with smokers.

408-Necrotizing sialometaplasia is a benign, self-healing inflammatory


process mainly involving the minor salivary glands.

409-Benign lymphoepithelial lesions are also known as Godwin


tumor, Mikulicz syndrome, or punctate parotitis.

410-Sialography is the most accurate imaging method to detect calculi.

411-The submandibular gland is more susceptible to calculi formation


than the parotid gland because of the longer course of its duct, higher
salivary mucin and alkaline content, and higher concentrations of
calcium and phosphate.

412-Penicillin is the drug of choice for treatment of actinomycosis.

413-Dx of TB sialadenitis may be made with acid-fast staining for


organisms, a culture of the saliva, and placement of a purified protein
derivative skin test..
160

414-Uveitis, facial palsy, and parotid enlargem. are suggestive of


sarcoidosis.

415-Lymphoepithelial cysts associated with (HIV) occur almost


exclusively in the parotid gland; CT scan or ultrasound may reveal
bilateral multiple cystic masses in the parotid
gland.

416-Predisposing factors for acute suppurative sialadenitis include


dehydration, immunosuppression, trauma, and debilitation.

417-minor sublingual ducts, referred to as the ducts of Rivinus,


Some of these ducts unite to form the major ducts of Bartholin

418-Biofilms are initiated when free-floating planktonic bacteria anchor


to various biological or inert surfaces.

419-Churg–Strauss syndrome is a rare granulomatous vasculitis


involving small to medium sized vessels and is characterized by asthma,
hypereosinophilia, and extravascular eosinophilic granulomas.

420-Pleomorphic adenomas are the most common benign tumors of


the lacrimal gland.

421-blepharochalasis, which is a rare inflammatory disorder of the


eyelids characterized by recurrent edema.
ْ
422-during drilling can cause osteoblasts to die. -Heating bone to more
than 5oC

423-pogonion is the most anterior or prominent point on the chin,

424-Hematoma formation is the most common complication during the


facelift.

425-The most important element in the extracellular matrix for cell


adhesion and proliferation is glycosaminoglycans.

.
161

426-Great auricular nerve is most commonly injured during face-lifting.

427-Laser light is monochromatic, collimated, and coherent.

428-Hemangiomas typically appear in the few weeks after birth.

429-fat pads of the upper eyelid are deep to the orbital septum.

430-temple and medial canthal area are examples of cutaneous defects


that heal well by secondary intention.

431-The small bright dots in the middle of the mass are


microcalcifications, which are specific for papillary thyroid carcinoma. In
US.

432-BCC rarely metastasizes to cervical lymph nodes . In the absence


of clinical nodal disease, elective neck dissection is not indicated.

433-Warthin tumors can be multifocal in up to 50 % of cases, and


bilateral Warthin tumors are seen in over 10% of patients.

434-Rituximab is an anti-CD20 monoclonal antibody.

435-High-resolution CT scanning is the imaging modality of choice to


stage the cNO neck in oral cavity cancer and is superior to MRl or PET-
CT in evaluation of the neck.

436-optimal treatment of recurrent cervical lymph node metastasis in


nasopharyngeal carcinoma is:Radical neck dissection.

437-The most common type of BCC is nodular.

438-The most commonly used particle for radiation therapy is:Photon.

439-ulcerated, circumscribed lesion with elevated or rolled margins and


a keratinized central region is characteristic of keratoacanthoma,

440-keratoacanthoma, which is a benign self-limiting lesion that often


regresses without intervention.
162

441-Most patients with fibrous dysplasia can be observed.

442-the most frequently mutated gene in head and neck cancer:p53.

443-Tobacco smoke preferentially mutates p53.

444-absence of necrosis in large nodes is suggestive of lymphoma.

445-ultrasound features of a thyroid nodule are associated with high


probability of malignancy are Hypoechogenicity, Increased intranodular
vascularity and Microcalcification.

446-Double adenomas are the most common cause of false-positive


intraoperative PTH levels.

447-Salivary duct cancer is a highly aggressive malignancy with a high


rate of nodal metastases,

448-The ophthalmic artery is inferolateral to the optic nerve.

449-Lentigo maligna, also known as melanoma in situ,

450-Localization and mapping of sentinel lymph nodes typically


involves preoperative and intraoperative use of technetium- 9 9 m-
labeled sulfur colloid,( lymphoscintigraphy)

451-The thoracic duct is located at the base of the neck, medial and
deep to the carotid artery and vagus nerve

452-Different tissues have different radiation toxicity levels. The lens of


the eye is affected by 10 Gy and the optic chiasm at 55 Gy.. parotid 25
Gy.

453-Patients with Sjogren disease have more than 40 times greater


relative risk of developing non-Hodgkin lymphoma.

454-The gold standard for diagnosis of lymphoma in the head and neck
is open excisional biopsy because it provides adequate tissue for both
diagnosis and definitive subclassification.
163

455-The use of IMRT employs multiple beams to allow effective tumor


dose while reducing dose to uninvolved tissues .

456-Hypercalcemia is not associated with secondary


hyperparathyroidism,

457-A solitary parathyroid adenoma is the most common cause of


primary

458-A drop in intraoperative PTH levels to normal levels is predictive of


surgical success.

459-Chronic exposure to arsenic has been associated with the


development of multiple squamous and basal cell carcinomas.
hyperparathyroidism.

460-The most common type of MENII is MEN IIA, which is


characterized by the triad of MTC, pheochromocytoma, and primary
hyperparathyroidism.

461-Advanced NP carcinoma is technically inoperable. Multiple studies


haveshown concomitant CRT has the best outcomes.

462-Sturge-Weber syndrome is associated with Capillary


malformations.

463-A staccato cough is most commonly associated with Chlamydia in


infants.

464-endoscopy should be performed between 12 and 48 hours


following ingestion to achieve the highest degree of patient safety while
providing the most information.

465-A recent study showed that for pediatric salivary gland malignancy
the overall 5 -year survival was 93 %, and 26 % developed a
recurrence.
164

466-Cafe-au-lait spots and cutaneous neurofibromas are not consistent


findings of NF2 .

467-disadvantages of Mohs technique are its high cost, lack of easy


availability, and long procedure time.

468-Mohs micrographic surgery is the treatment of choice for


sclerosing or recurrent BCC, large or poorly differentiated SCC, and
most cases of fibrohistiocytic and adnexal cancer.

469-Excisional biopsy with a 2-mm margin is preferred for pigmented


lesions thought to present a high risk for cutaneous melanoma.

470-Treatment of choice for a capillary malform. is laser


photocoagulation.

471-Capillary malformations are the most common of the vascular


malformations and occur in 0.3% of newborns. These lesions are also
known as nevus flammeus or port wine stains .

472-Vincristine is gaining popularity as another efficacious treatment


for complicated or refractory hemangiomas.

473-MRI:with contrast is the most useful of all radiologic evaluations of


hemangiomas. MRI can differentiate a hemangioma from a vascular
malformation .

474-Congenital hemangiomas, pathologically distinguishable from


the hemangioma of infancy, in that they are glucose transporter 1
protein (glut-1) negative.

475-The argon, KTP-532, and Nd:YAG lasers can be transmitted


through the laryngoscope to the tissue via a fiberoptic cable.

476-The erbium:YAG laser emits a 29–40 W, which is highly absorbed


by water (12–18 times more efficiently thanCO 2 laser). disadvantage
of poor hemostatic qualities and limited collagen tightening compared
with the CO 2 laser.

477-KTP-532, mainly used in the treatment of vascular lesions..


.
165

478-Nd:YAG laser is used for tracheobronchial lesions, particularly for


its excellent hemostatic qualities; nonablative skin resurfacing; and hair
removal in ethnic patient populations. depth is 3–5 mm

479-argon laser is typically used for the coagulation of hemangiomas.


depth of 1 mm, almost completely absorb. by hemoglobin, melanin,
and myoglobin

480-CO 2 laser is used for excision of laryngeal lesions and deep skin
resurfacing for rhytids and acne scarring. 0.2-mm depth.

481-Nasal intubation may be necessary if mandible has to be


dislocated.

482-Muscle relaxants should be avoided if nerve monitoring is used.

483-Fractionation refers to the division of total dose into a number of


separate fractions

484-Brachytherapy is a form of radiotherapy in which a radioactive


source is placed inside or adjacent to the area requiring treatment.

485-In the head and neck, primary radiotherapy is most frequently


delivered via a linear accelerator with 6-megavolt(MV) photons.

486-The excretory duct cells can give rise to either mucoepidermoid or,
perhaps, squamous cell carcinoma.

487-Palifermin is a recombinant human keratinocyte growth factor 1


that is thought to offer mucosal protection by inducing epithelial
hyperplasia.

488-40% of patients with Crohn disease have oral symptoms at


presentation, includingcobblestoning of the buccal mucosal, angular
cheilitis, or deep linear ulcerations in the gingivalbuccal sulcus..
166

489-Parathyroid adenoma is typically high signal on T2-weighted MRI,

490-The cervical sympathetic chain lays posteromedial to the internal


and common carotid arteries.

491-The pleomorphic adenoma typically presents as a round or ovoid,


well-circumscribed soft tissue mass. typically high signal intensity onT2-
weighted MRI.

492-Benign lymphoepithelial lesions are seen most commonly in


association with HIV,

493-strept. viridans is the most common cause of orbital subperiosteal


abscess. These organisms are also the most common cause of
intracranial complications.

494-CD3 is known as the pan-T cell marker.

495-On noncontrast CT scans, normal lymph nodes are ovoid,


homogeneous soft tissue masses with a short-axis diameter of 5–10
mm

496-MRI is the study of choice to assess for a lesion of the skull base
or carotid sheath.

497-MRI is less sensitive to dental artifact than CT scanning and


provides superior soft tissue contrast for most processes

498-certain absolute contraindications to MRI, including ferromagnetic


intracranial aneurysm clips, cardiac pacemakers, and many cochlear
implants.

499-The most common adverse effects associated with voriconazole


were reversible visual disturbances and liver toxicity.

500-Metronidazole can also decrease the metabolism of warfarin and


increase the prothrombin time.
167

501-Like tetracyclines, tigecycline may also cause photosensitivity and


pseudotumor cerebri.

502-prolongation of the QT interval has been observed in


fluoroquinolones as a class.

503-Patients allergic to penicillins may be allergic to imipenem and


meropenem.

504-Warthin’s tumor is the second most common tumor of the salivary


glands and constitutes approximately 14% to 21% of salivarygland
neoplasms. The tumor is almost exclusively found in the parotid

505-Warthin’s tumor, classified by the WHO as adenolymphoma, is


most common in cigarette smokers older than the age of 40 and
appears most commonly as a partially cystic lesion arising in the lower
portion of the parotid gland at the angle of the mandible.

506-Elective neck dissection is NOT indicated if the neck is not clinically


involved Overall, only 8-10% of sarcomas metastasize to nodes.

507-Ewings Sarcoma2nd most common malignant bone tumor in


children.

508-In adult patients, cerebrovascular accident with subsequent lower


cranial nerve palsies is the most common cause of intractable
aspiration; pediatric patients include cerebral palsy.

509-acinic carcinoma almost certainly has the best survival of any


salivary cancers it has along natural history with a 15-year survival rate
of only 55 percent.

510-Bilateral salivary malignancy is very rare but acinic cell carcinoma


is the most common after Warthin's tumour (benign) .

511-The important features distinguishing malignant melanoma from


other lesions are

.
168

512-In evaluating neoplasms of the salivary glands, the findings of the


99mTc-pertechnetate scan are almost pathognomonic of Warthin’s
tumor and oncocytoma.

513-lateral medullary syndrome (Wallenberg’ssyndrome)The syndrome


usually results from occlusion of the ipsilateral vertebral artery and
rarely from occlusion of the posterior inferior cerebellar artery.

514-Bilateral central compartment neck dissection should be performed


in patients with medullary thyroid carcinoma.

515-Neck dissection, mostly of the left side, carries a risk of chylous


fistulae, occurring in 1-3 per cent of cases.

516-Nodes larger than 2 to 4 cm without central necrosis often indicate


lymphoma or sarcoidosis.

517-A neck dissection is not indicated because paragangliomas do not


metastasize to the cervical nodes.

518-Follicular ca. less likely present with nodal metastasis than


papillary ca. but higher rate of distant metastasis at presentation.

519-Medullary thyroid ca.:calcitonin immunohistochemistry can confirm


the FNA diagnosis.

520-Thyroid malignancy is usually treated surgically.

521-Cervical metastatic disease very common in papillary thyroid


cancer, does not usually affect prognosis.

522-Indication of surgery in Graves:failure of medical R,significant risk


of medical R ,need for rapid return to euthyroid,massive goiter.to avoid
radioactive iodine.

523-Cri-Du-Chat syndrome:abnormal cry.caused by partial deletionof


no.5 group B chromosome .associated with mental ret.,facial
abnormalites, hypotonia and strabismus.
.
169

524-hemangiomas. Steroid therapy alone has been advocated as the


primary method of management.

525-The primary echelon of nodes at risk for metastatic spread is the


submandibular and submental region.

526-A family history of Bell’s palsy can be elicited in 8% of patients..

527-Bilateral facial paralysis suggests the possibility of Lyme disease.

528-Osteogenic sarcomas and chondrosarc. of the facial skeleton the


most successful therapy is based on wide en bloc resection . Irradiation
seems to be an effective adjuvant for both tumors.

529-The presence of extracapsular spread is an important prognostic


factor with regard to recurrence in the neck after neck dissection.

530-Postoperative radiation is routinely administered to treat the necks


if there is extracapsular spread or if more than one lymph node is
involved..

531-Ultrasound, combined with fine needle aspiration, may be a very


effective tool in evaluating metastatic lymph nodes in head and neck
cancer.

532-ACC of the submandibular gland has a much more aggressive


clinical course than ACC of other sites. ACC is a relatively radioresistant
tumor;

533-Adenoid cystic car(ACC) is the second most common salivary gland


mal. overall and is the most frequent malignant tumor inthe SMG and
sublingual gl. 22% of minor sal gland tumors are ACC. Three pathologic
subtypes exist: cribriform, tubular, and solid. The solid lesions have
higher cellularity and are correlated with a worse prognosis.

534-Measurement of secreted calcitonin is useful for the diagnosis of


MTC and for postsurgical surveillance for residual and recurrent
disease.
.
170

535-Papillary carcinomas are the predominant thyroid malignancy in


children 75%. Only 6% of papillary carcinomas are associated with
familialdisease.

536-Most patients who present with a thyroid nodule are euthyroid.


The finding of hypothyroidism or hyperthyroidism tends to shift the
workup away from thyroid carcinoma to a functional disorder of the
thyroid gland,

537-most patients with thyroid cancer are euthyroid at presentation,

538-irradiated VC may undergo anaplastic transformation.

539-Tumor suppressor genes are the counterparts to oncogenes. In


general, these genes have a role in growth suppression activities such
as apoptosis, cell cycle regulation, DNA repair, and cell adhesion.

540-most common cause of brain abscess in patients with


AIDS:Toxoplasma gondii.

541-Psammoma bodies are laminated calcified spheres and are usually


diagnostic of papillary carcinoma.

542-small fraction of unbound T 4 and T 3 hormones are responsible


for biologic activity.

543-Elective neck dissection is not typically advocated for lip carcinoma


because the incidence of occult metastases is low.

544-CSF:Clear watery discharge from ear or nose: appears during


straining or leaning forward (Dandy maneuver); salty taste.

545-To prevent blindness as a complication of temp.arteritis with high


dose And prolonged prednisolone .
.
171

546-Microbiology of primary Atro. Rh. is almost uniformly Klebsiella


ozenae.

547-embryologically the parotid gland is not truly a bilobed structure.

548-Von Hippel–Lindau disease: neurofibromat. and vestibular


schwannoma

549-Vagal paragangliomas, like carotid body tumors, may present as


apalpable neck mass that is more mobile in a lateral direction than a
craniocaudal orientation.

550-Rashes occur among 5% of persons who take penicillin.

551-Protooncogenes and tumor suppressor genes regulate normal cell


proliferation, differentiation, and apoptosis (programmed cell death).

552-Cigarette smoking is known to cause p53 mutations .

553-The p53 tumor suppressor gene is responsible for arrest in the cell
cycle after genetic injury. It allows the cell to repair the DNA defect
before the next cell division. The gene also induces apoptosis

554-Ocillopsia :eye movements on sound exposure.

555- Signs of malignancy in CT scan:Multiple LN, LN > 3cm, Central


breakdown + peripheral rim of enhancement, Rupture of the capsule &
extracapsular spread.

556- extra-capsular spread reduces the 5 year survival rate by 75%.

557- Shuller:A single unilateral L.N. reduces the expected 5 year


survival rate by 50%. A single contralateral L.N. reduces the 5 year
survival rate by 60%.

558-Once there is a single positive node in the neck the case becomes
stage III or VI regardless of the size of the primary tumor (even T1)
.
172

559-The marginal mandibular branch of the facial nerve running


between the upper pole of the gland and the mandible is in danger
during operations on the submandibular gland.

560-The submandibular duct ( Wharton`s duct) is about 5cm long and


Opens close to the frenulum in the floor of mouth. The duct of SMG
crosses laterodorsally over the lingual nerve.

561- The parotid duct( stensen`s duct) is approximately 6cm


long.It crosses the the masseter, perforates the buccinator.
The orifice lies opposite the second upper molar tooth.

562-Hypovolaemic shock sets in when a person has lost 10-


20 % or more of total blood volume.

563-Rigid and flexible endoscopy are best sterilized by


immersing in 1% glutaraldehyde solution for at least 30
minutes.

564-Bull’s lamp has a 70 or 100 watt bulb and a planoconvex


lens with focal length of about 12 inches .focal length of head
mirror 9 inches.

565-Sjogren’s syndrome are 44 times more at risk of


developing B-cell lymphoma of salivary glands.

566-Largest cranial n. is trigeminal n.

567-CN VI its largest intrcranial course.

568-Medullary thyroid ca. arises from parafollicular C-cells of


thyroid.secrete calcitonin and unlike rest of thyroid cells they
do not take up iodine.
173

569-Ionising radiations not used in MRI,Ultrasound and Doppler.

570-Structures removed in RND : SCM, IJV, XI n.,all LN in levels


I,II,II,IV and V ,submaxillary salivary gl., tail of parotid.

571-Structures preserved in RND: vagus n.,XII n.,brachial


plexus,phrenic n., lingual n., marginal mandibular n.of facial n.

572-Neck dissection done for N0(no neck nodes palpable) is called


elective ND.
573-Anerior compartmentND(removal of level VI nodes).

574-Posterolateral ND(removal of II,III,IV,V and retroauric. &


suboccipital).

575-Lateral ND(removal of levels II,III and IV nodes).

576-Supraomohyoid ND(removal of only levels I,II and III nodes) .

577-Vincristine is neurotoxic and causes sensory or motor


periph.neuropath.

578-Cisplatin is nephroyoxic&ototoxic_affects(4kH-8kH)may be
permanent.

579-5-Fluorouracil causes myelosuppression,stomatitis and diarrhoea.

580-Bleomycin causes pulmonary toxicity in form of pneumonitis


&pul.fibrosis.

581-Concomitant chemotherapy used at same time of Radiotherapy.

582-Adjuvant chemoth. refers to its used after operation or


Radiotherapy .

583-Neoadjuvant chemoth. refer to its used before surgery or


Radiotherapy in hope to
shrink the size of tumor and eradicate micrometastases.
174

584-Verapamil is acalcium chanal bloker useful in prophylaxis of


migraine and cluster headaches.

585-HIV infects CD4(helper)T-lymphocytes,macrophages and


monocytes and leads to cell mediated immune deficiency which
predisposes to opportunistic infectios and malignancies.

586-Types of thyroplasties: Type I_medialisation of cord, Type


II_lateral. of cord, Type III_shortening of cord, Type IV_ lengthening of
cord.

587-Heerfordt’s disease or Uveoparotid fever:parotid swelling,uveitis


and facial paralysis with fever.
588-Haemangioma is the most common benign neoplasm of the
parotid seen at birth. Involute spontaneously by 5 yr of age &
Treatment is only observation.

589-Sjogren’s syndrme:autoimmune process often associated with


collagen disorders such as RA,myositis,secleroderma or disseminating
lupus.

590-Ptosis is due to denervation of Muller’s muscle which supplied by


sympathetic fibres.

591-Beta-2 transferrin is specific for CSF.not found in tears,nasal


secretions and serum.is also seen in aqueous humour& serum of
patients with ch.liver dis. Due to alcohol.

592-Wallenberg’s syndrome due to thrombosis of posterior-inferior


cerebellar artery leading to ischaemia of lat.part of medulla.causes:
vertigo, anaesthesia of ipsilat.side of face,paralysis of palate,pharynx
&larynx.

593-Most common cranial n.involved NPC is CN VI causing diplopia.

594-Aspergillus hyphae are septate and acute branching.

595-Rhizopus & mucor, hyphae in these fungi are broad,non-septate


and irregularly branching at 90’.
175

596-Russell bodies :eosinophilic structures seen in the cytop. of plasma


cells.

597-Mikulicz’s cells are foamy histiocytes teaming with gram negative


coccobaclli in the cytoplasm.

598- Neural pathways for hearing are: cochlea_CNVIII_cochlear


nuclei_olivary comlpex_lat.lemniscus_inf.colliculus_superior temporal
gyrus.

599-Features of middle ear carcinoma include:sever nocturnal


pain,bleeding friable granulation or polyp,facial paralysis and refractory
otitis externa.

600-stellate ganglion is situated on neck of first rib and is related to


symp. chain.its formed by union of inf.cervical symp.gang.and first
thoracic gangl.

601-spiral ganglion situated on cochlear n.

602-Scarpa’s gang. in the lat.part of internal acoustic meatus on


vestibular n.

603-mumps is the most common viral infection causing unilateral


SNHL.

604-aspirin should not be used in children due to the well documented


Reye’s syndrome.

605-ligation of common carotid artery may leed to mortality in: 30-50


%.

606-parotid gland is separated from submandibular gland by


stylomandibular ligament.

607-facial nerve is not fully developed untill a childe is:4 year.

608-Bogorad syndrom :facial palsy with profuse lacrimation during


eating.
176

609-Leiomyomas account for more than half of all benign nonepithelial


tumors of the esophagus.

610-The first step in management of an airway fire in laser surgery is


to remove the existing tube and replace it with a smaller diameter tube.

611-Mycetomas are most commonly caused by Aspergillus species.

612-Paragangliomas are the only laryngeal neop. with a female


predominance.

613-Crystal clear aspirate suggests a parathyroid cyst,in FNA.

614-the most critical postoperative period for monitoring serum


calcium is 24 to 96 hours.

615-Sestamibi scan has sensitivity for solitary parathyroid adenomas of


85 to 90%, with a specificity of nearly 100%.

616-Medullary carcinoma does not take up radioiodide,

617-The most common malignant tumor of the submandibular glands


is the adenoid cystic carcinoma.

618-Streptococcus viridans is a common infectious agent in chronic


and recurrent sialadenitis.

619-Staphlococcus aureus is the most common microorganism


associated with acute bacterial sialadenitis.

620-multiple intraparotid lymphoepithelial cysts in the parotid glands


that is considered almost pathognomonic of (HIV) infection.

621-risk of developing lymphoma by an individual with Sjögren’s


syndrome is 44 times.

622-Classic mumps is caused by the Rubulavirus, a paramyxovirus.


177

623-The most common malignant neoplasm of the parotid glands is


mucoepidermoid carcinoma.

624-The fascia covering the parotid and submandibular glands


originates from the superficial layer of the deep cervical fascia.

625-Regional metastases to cervical lymph nodes decrease 5-year


survival rates by 50%.

626-cisplatin and external beam radiation combination of treatments


has been most successful for treating stage III nasopharyngeal
carcinoma.

627-PET is more sensitive and specific than CTor MRI in detecting


occult nodal disease but when used alone does not provide detailed
anat-information.

628-Wallenberg’s syndrome results in vocal fold paralysis, ipsilateral


reduction of pain/temperature sensation of the face, nausea, vertigo,
diplopia, and intractable hiccupping.

629-Impairment of speech movement programming is characteristic of


apraxia.

630-Interventional and diagnostic angiography are indicated for


embolization of vascular lesions.

631-Diagnosis of giant cell arteritis, currently requires an arterial


biopsy.

632-Cluster headaches have severe lancinating pain that often


awakens patients. They are associated with unilateral nasal congestion,
rhinorrhea, and lacrimation.

633-Tension headache features include greater prevalence in women,


bilaterality, tightening or dull pain, exacerbation by stress, and
association with depression.
178

634-cluster headaches, which affect men more often than women.

635-Cytokines responsible for the migration of eosinophils in chronic


sinusitis include interleukin-4 and interleukin-5.

636-Sulfur granules” are white-yellow granules of bacterial filaments


seen in the exudates or infected tissues of patients with actinomycosis.

637-Pneumocystis carinii infections of the middle ear respond well to


trimethoprim-sulfamethoxazole.

638-Approximately 10% of HIV-infected persons progress to AIDS


within the first 2 to 3 years following infection.

639-The main target of HIV is the:CD4+ T lymphocyte.

640-Cogan’s syndrome has a unique antigen target (55 kD).

641-Boyette’s solution is( 4% lidocaine, phenylephrine, and sodium


chloride, with sterile water added to make 200 mL.) .

642- MRI is superior to CT in assessing the status of the anterior skull


base and orbit in neoplastic disease.

743- diagnosis of IgA deficiency cannot be made before the age of two
years.

644- The treatment of immunodeficiency may include long-term


antibiotics
and intravenous gammaglobulin therapy (IVIG). Many of these
patients
may improve with time,

645- Pleomorphic Adenomas: there is a 2–4% malignant change.


-
646- Mikulicz’s disease” was subsequently applied to a variety of cases
of bilateral salivary or lacrimal gland enlargement,
.
179

647- Pleomorphic most common prestyloid--Neurogenic tumors are


retrostyloid.

648-Paraganliomas grow 1.0-1.5 mm per year.

649-Paraganglioma in pps on MRI: Salt and pepper” appearance.

650-Schwannoma in pps:High signal intensity on T2.

651-Pleomorphic adenoma:Low signal intensity on T1, High signal


intensity on T2.

652-Characteristic appearances of pps tumor types on MRI allows


preoperative Dx in 90-95% of patients.

653- If paraganglioma is suspected need to check 24 hour urine for


catecholamines: VMA, metanephrines.

654-Classically, paragangliomas mobile anterior-posterior but not up


and down.

655-The protein p16 has been reported to be inactivated in NPC


patients.

565-salivary gland malignancie:comprehensive modified radical neck


dissection in N+ necks.

657-Modified Blair incision for parotidectomy operations.

658-For the most part, salivary gland malignancies are primarily


treated surgically when resectable.

659-Salivary Duct Carcinoma:one of the most aggressive primary


salivary gland tumors.

660-Acinic Cell Ca:More than 90% occur in the parotid ,the second
most common childhood sal. G. malignancy.D.D, includes normal
parotid gland,FNA biopsy, difficulty because of the resemblance to
normal tissue.

.
180

661-ACCs have basaloid cells with dark chromatin and little cytoplasm,

662-Polymorphous low-grade adenocarcinoma:arise almost exclusively


from the minor salivary glands.

663-Microscopically, there are three growth patterns: tubular,


cribriform, and solid.of Adenoid cystic ca.

664-The parotid glands normally contain, on average, 10 to 20


intraglandular lymph nodes.

665-fluid secreted by the parotid gland is almost exclusively serous,


while that from the sublingual gland is almost exclusively mucous and
that from the submandibular gland is a mixture of serous and mucous.

666-mucoepidermoidcarcinoma was the most frequent malignancy of


the parotid, but adenoid cystic carcinoma was the most frequent of the
submandibular and minor salivary glands.

667-most common primary malignancy presenting in the salivary


glands to be mucoepidermoid carcinoma.

668-salivary gland tumors in children are much more likely to be


malignant than those of adults.

669-Salivary gland malignancies :Indications for adjuvant postoperative


radiation include advanced stage, positive margins, high-grade
histologic types, and evidence of local tissue invasion.

670-Frey’s syndrome experience flushing and sweating of the


ipsilateral facial skin during mastication (gustatory sweating). involves
aberrant cross-reinnervation between the postga. secretomotor
parasymfibers to the Parotid G.and the postgangl. Sympath.fibers
supplying the sweat G.of the skin.
181

671-facial nerve is usually 6 to 8 mm deep to the tympano-mastoid


suture line.

672-most common method of identifying the main trunk of the facial


nerve is in its course in the region located between the tragal pointer
and the attachment of the posterior belly of the digastric muscle to the
mastoid bone.

673-The standard incision for parotidectomy is a modified Blair’s


incision.

674-Warthin’s tumor accounts for 10% of all parotid tumors and is


found almost exclusively in the parotid gland. associated with cigarette
smoking.

675-Basal cell adenomas may be difficult to distinguish from the solid


variety of adenoid cystic carcinomas.

676-Histologically, an intact basement membrane helps differentiate


basal cell adenomas from pleomorphic adenomas.

677-The risk of malignant transformation in pleomorphic adenoma is


1.5% within the first 5 years of diagnosis but increases to 10% if
observed for more than 15 years.

678-pleomorphicAdenoma :surgical excision with a surrounding cuff of


normal tissue to prevent recurrence.

679-Pleomorphic adenomas are the most common benign tumors of


the lacrimal gland.

680-FDG PET is unreliable both in tumor detection and in


distinguishing benign from malignant salivary tumors.

681-glomus tumors have characteristic serpiginous flow voids (salt and


pepper appearance) on MRI.

682-An intermediate to low T2 signal mass, with or without invasion of


surrounding tissue planes, is more likely tobe a malignant mass such as
adenoid cystic or mucoepidermoid carcinoma.
.
182

683-If the mass is unilateral, shows postcontrast enhancement, has a


high T2 signal, and doesnot invade surrounding tissue planes, it is more
likely to be a pleomorphic adenoma.

684-parotid gland mass is bilateral, it is more likely to be Warthin’s


tumor. unilateral, nonenhancing mass with a high T2 signal is more
likely to be a Warthin’s tumor.

685-two standard approaches to the PPS:transparotid


approach=prestyloid transcervical approach =poststyloid.

686-Typical café au lait spots associated with multiple neurofibromas


are pathagnomic of vonRecklinghausen's disease.

687-Neurofibroma:may be multiple and are often associated with von


Reckling-hausen's disease.

688-MEN-2 gene is located in chromosome 10.

689-combination of medullary thyroid carcinoma, phaeochromocytoma


and hyperpara –thyroidism was termed MEN-2A.

690-The association of phaeochromocytoma with paraganglioma raises


the possibility of a relationship with the multiple endocrine neoplasia
type 2 syndromes.

691-multiple paragangliomas occur synchronously in up to 50 percent


of patients in familial cases, 10 percent for nonfamilial cases.
Commonest synchronous tumours are carotid body and jugulotympanic
tumours.

692-paraganglioma:serum catecholamine estimation should be made


and a 24-hour urinary vanillylmandelic acid (VMA) collection.

693-Paragangliomas:a typical appearance of epithelial cell clusters in


an extremely vascular and fibrous stroma. The clusters have been given
the name zellballen. Up to 10 percent of paragangliomas are malignant.

.
183

694-The paraganglionic cell is sensitive to changes in the blood gas


partial pressure, particularly of carbon dioxide but also of oxygen, and
is also sensitive to pH.
695-nearly all tumours of the prestyloid compartment are deep lobe
parotid neoplasms, where as most tumours of the poststyloid
compartment are either paragangliomas or schwannomas.

696-benign salivary tumours developed only in the prestyloid


compartment of the PPS.

697-ground-glass appearance typical of the late stages of WG.

698-Acoustic neuromas originate at the junction of the glial and


Schwann cells, that are bilateral (5%) are seen in(NF2). This is an
autosomal dominant disease due to an aberration on the long arm of
chromosome 22.

699-Tumors arising in the deep lobe of the parotid gland may pass
through the stylomandibular tunnel to reach the parapharyngeal space.
These tumors have a characteristic dumbbell shape.

700-Wound healing begins at the time of tissue injury with the


coagulation phase, polymorphonuclear leukocyte (neutrophil) is the
predominant cell type for the first 48 hours .

701-treatment of MH includes hyperventilation and the use of


bicarbonate for the acidosis. needs to be cooled, mainstay therapy iv
dantrolene.

702-Malignant hyperthermia:The earliest signs reported during


anesthesia are masseter muscle rigidity, tachycardia, and hypercarbia
as a result of increased carbon dioxide production.

703-Nocturnal polysomnography is the gold standard for the diagnosis


of OSA.
.
184

704-Prolonged general anesthesia that lasts more than 3 hours.

705- Lazy Man S incision :superficial / total parotidectomy .

706-Glomus jugulare - associated with pulsatile tinnitus, conductive


deafness, positive Brown's sign. CT scan shows eorsion of jugular
foramen.

707-familial glomus tumors may occur in 20% of patients with glomus


tumors.

708-Glomus tumors of the temporal bone are neoplasms arising from


normally occurring paraganglionic bodies or formations that are located
on the dome of the jugular bulb and along the course of Jacobson's
nerve (ninthcranial nerve) onto the cochlear promontory.

709-The largest concentration of paraganglionic cells is with the


adrenal medulla. The other predominant locations of paraganglionic
tissue are found in the branchiomeric chemoreceptor system of the
aortic arch, carotid bifurcation, and temporal bone .

710-Miculicz syndrome simply reflects other disorders characterized by


lacrimal and salivary gland disease that mimics Sjögren syndrome, but
is associated with disorders such as sarcoidosis, lymphoma and other
similar systemic disorders.

711-Miculicz's disease is a disorder characterized by multiple


lymphoepithelial lesions of the lacrimal and salivary glands..

712-sublingual gland:weighs 3–4 g. major sublingual duct (Bartholin's


duct.

713- foramen caecum, that indicates the site of the embryonic thyroid
diverticulum that gives origin to the thyroid gland.

714-bupivacaine :an amide anesthetic with high lipid solubility .

.
185

715- The unit of measure for radiation is the gray (Gy), which
corresponds to 1 joule being deposited per kilogram of material. two
main energy sources in use are electrons and photons.

716- Warning signs for melanoma are lesion asymmetry, border


irregularity, color variation, and diameter more than 6 mm. The T
classification is defined by tumor thickness.(ABCD).

717-Sjِgren’s syndrome predisposes patients to the development of


malignant lymphopro -liferative disorders.

718-The biopsy of labial accessory salivary glands demonstrating


salivary gland disease is one of the most consistent features of primary
Sjِgren’s syndrome.

719- Acrophobia (fear of heights).

720-Agrophobia ( fear of crowds).

721- Facial Nerve Neurinoma Commonest site is geniculate region,


20% of cases of recurrent Bell’s.

722-Traumatic FNP in newborns, 60% of FN paralysis at birth.

723- Mobeius syndrome partial or complete agenesis of FN nucleus &


nerve roots usually bilateral associated with other cranial nerves
affection

724- Lacrimation test (Shirmer’s test): 25% reduction from


normal side.

725- Stapedial reflex text: absent in 1% of normal individuals .

726- T1:< 2cm. T2: 2-4cm T3: 4-6cm T4: >6cm __salivary T staging. .
186

727- Pain. Facial nerve paralysis. Lymph nodes. Skin


involvement=malignancy.

728- contrast is contrandicated during acute infections of salivary gl.

729- production of saliva: IOOO to 1500 ml / day, 99.5% water, Parotid


--Serous, Sudmandibular– Mixed Sublingual – Mucous.

730-SMG The autonomic supply is from lingual nerve via chorda


tympani.Sympathtic from s.c.ganglia.

731- The duct of SMG crosses laterodorsally over the lingual nerve.

732- The submandibular duct ( Wharton`s duct) is about 5cm long.

733- The parotid duct( stensen`s duct) is approximately 6cm long.

734-parotid gl. no true superficial and deep lobes in relation to the


facial nerve.( Lobes are product of surgical dissection and are not a true
separate units.).

735- Late N1, N2, & N3 should be treated surgically by neck dissection.

736- Neck recurrence even if there is primary control is more than


70%.

737-The rate of RT success in N2 & N3 is less than 20%.

738-Bronchogenic CA and cancer stomach are the most common


causes of supra-clavicular LN affection.

739- Neck CLNM with unknown Primary: The first step is to do an FNA
to exclude a primary Lymphoma and it may give you a clue to the
possible primary site .

740- Advantages of Elective RT in the Control of N0 Neck:Less


morbidity and mortality than modified elective neck dissection ,Survival
rate is the same,Only 5% of the cases when the primary tumor is
controlled develop LN failure later on, and RT dose for elective N0
control (5000-6000 cGY).
187

741- If the primary tumor is to be treated surgically, the N0 neck is


controlled by a modified elective neck dissection unilaterally or
bilaterally.

742-If the primary tumor is to be managed by RT the N0 Neck should


be managed by RT.

743- When the Probability of CLNM is greater than 15-20% and so we


must control the N0 neck.

744- The N0 Neck: When there is a primary tumor in the ADS and
the neck is negative for enlarged LN after all clinical exami and
investigation.

745-Presence ECS decrease the survival rate by 50%.

746-Signs of malignancy in CT: Multiple LN,LN > 3cm. Central


breakdown + peripheral rim of enhancement, Rupture of the capsule &
extracapsular spread (ECS).

747-Nasopharynx → level II and level V, Base of the tongue → level II


and III usually bilateral, Larynx → level II and III usually unilateral,
Pyriformfossa → level II,III and IV commonly bilateral, Postcricoid
region → level II,III and IV usually bilateral.

748- Once there is a single positive node in the neck the case becomes
stage III or IV regardless of the size of the primary tumor (even T1).

749- The most important single factor that determines. The prognosis,
Cure rate and Survival rate is the presence or absences of CLN M.

750- Non-Hodgkin's Lymphoma: Nodal and extra-Nodal involvement is


common, the most common extra-Nodal site is involvement of
Waldeyer ring, particularly tonsil's adenoids in the Naso-pharynx and
the lingual tonsil. The second most common extra-nodal site is the
Orbit. treatment of the low grade type is by RT only and of the
intermediate and high grade malignancy is RT and Chemo.
188

751- Hodgkin’s Lymphoma :It affects mainly the lymph nodes, extra
nodal disease is rare.Age from (15-30), (level IV and III) and the
supra-clavicular LN. symptoms as irregular fever, malise and weight loss
is characteristic, The treatment is by both Chemo and RT.
752- Malignant degeneration can occur in 3% to 7% of pleomorphic
adenomas.
753- Acinic Cell Carcinoma:best survival rate of any salivary
malignancy.
754-Adenoid cyc. Ca.. Local recurrences 50%. Facial palsy 20%.
755-Adenoid cyc. Ca. Four histological patterns: -Cribriform-Tubular-
solid cellular- Cylindromatous . Cribiform: best prognosis, solid: poorer
prognosis.
756- Adenoid cystic carcinoma is the most common malignancy of the
submandibular gland. Characterized by slow growth, Propensity of
perineural invasion and spread along the haversian system and neural
canal of bones.

757- Mucoepidermoid Carcinoma: Most common salivary malignancy


accounting for 29% to 43% of tumors. > 90% in the parotid, 8% in the
SMG, 1% in the sublingual gland. Most common salivary gland tumor in
children.

758- salivary duct carcinoma is high grade.

759- Oncocytoma: Arises from the striated duct cell.

760- Warthins tumor: Never malignant.

761-there are two or three straight blood vessels clearly visible along
this line of nerv-The facial recess is, therefore, bounded medially by the
facial nerve and laterally by the tympanic annulus, with the chorda
tympani nerve running obliquely through the wall between the two.e.
These are the only straight blood vessels in the middle ear and indicate
quite clearly that the facial nerve is very close by.
189

762-The facial recess is, therefore, bounded medially by the facial


nerve and laterally by the tympanic annulus, with the chorda tympani
nerve running obliquely through the wall between the two.

763-EMG -Primary used in acute phase of facial paralysis (Ist 2 wk) is


confirmatory for other tests.

764- Pseudo. = ciprofloxacine, amikacine… Staph. = coamoxiclav

765-antibiotic treatment does not appear to affect the incidence of


acute poststreptococcal glomerulonephritis.

766-The parotid duct drains in the region of a small parotid papilla


opposite the maxillary second molar tooth.

767-Superficial parotidectomy:‘lazy S’ incision.

768-Warthin's tumour is the most common monomorphic adenoma.

769-pleomorphic adenomas are derived from intercalated duct and


myoepithelial cells.

770-presence of Calcifications or bone formation in a salivary gland or


a parapharyngeal mass strongly suggests that the tumour is a
pleomorphic adenoma.

771-cartilage of pleomorphic adenomas, though often termed ‘pseudo-


cartilage.

772-myxoid stroma of pleomorphic adenomas is one of its most


characteristic features

773-capsule of pleomorphic adenomas may be thick and fibrous or


absent in some part.

774-CT scanning will show calcifications in a salivary gland tumour and


this strongly suggests that it is a pleomorphic adenoma.

775-Chordoma: Arises from remnant of notochord in nasopharynx.

.
190

776-Actinomycosis. presence of sulfur granules.

777-G.Jug:only 1% to 3% of these tumors actively secrete


norepinephrine. Catecholamines are much more likely to be secreted by
glomus jugulare tumors than by glomus tympanicum tumors.

778-CT better than MR for demonstrating L.N necrosis. CT better than


MR for detecting extracapsular spread.

779-glomus jugulare tumors may present with dysfunction of jugular


foramen contents (Vernet’s syndrome, involving cranial nerves IX, X,
and XI). These tumors may blanch with positive pressure on
pneumotoscopy (Brown’s sign) or have cessation of tumor pulsation
with ipsilateral carotid artery compression (Aquino’s sign). salt-and-
pepper appearance on MRI .

780-Glomus tumors may secrete vasoactive substances or be


associated with other tumors, such as pheochromocytomas.

781-high jugular bulb is present in about 25% of cases.

782- Patients receiving renal dialysis and deferoxamine also are at


higher risk for mucormycosis.

783-Loss of the corneal reflex is an early sign of trigeminal nerve


impairment. In acoustic neuroma.

784-The total inorganic content of saliva is about 2.5 g/L.

785-A substance having hormonal activity has been isolated as a


protein compound of the parotid saliva termed parotin

786-The only blood group antigens which have been described in


saliva are those of the Lewis group.

787-Phospholipase, Cholinesterase and Ribonuclease:These are


present in similar concentrations in plasma and saliva.

.
191

788- parotid saliva it comprises about 10% of the total protein. Its
concentration is high in submaxillary gland saliva.

789-Amylase:splitting cooked starch down to maltose. Lysozyme:splits


the carbohydrate.

790-No correlation has been shown between susceptibility to dental


caries and gamma globulin concentrations in saliva.

791-Sublingual Glands weighs approximately 3-4 g.

792-The parotid duct passes forward across the masseter muscle, turns
around its anterior border to pierce the buccinator. opposite the second
upper molar tooth open.

793-Each parotid gland is pyramidal in shape and weighs


approximately 25 g. serous in nature.

794- parathyroid weighs approximately 35 mg, may be oval, round,


irregular, or flattened, and has a characteristic tan brown color.

795-Sarcoidosis-Hypercalcemia.

796-radioactive iodine is the preferred method of treating recurrent or


persistent hyperthyroidism.

797-Salicylates, because of their ability to facilitate conversion of T4 to


the more potent T3,

798-Sodium iodide, the first drug used effectively in the management


of thyroid storm,

799-Node dissection is indicated in medullary carcinoma because of the


high incidence of lymphatic metastases.

800-Anaplastic carcinoma, because of its extent of local invasion, is


generally unresectable.

801-Medullary carcinoma is the only thyroid carcinoma which is


familial. It occurs, as a component of type II multiple endocrine
adenopathy.
.
192

802-Unlike papillary carcinoma, follicular carcinoma is typically solitary


and encapsulated. It preferentially metastasizes by the hematogenous
route to involve the bones and lung.

803-Papillary carcinoma, the most frequent of the thyroid cancers, is


unencapsulated and multifocal in origin.

804-Papillary and follicular carcinoma are well differentiated with a


relatively good prognosis compared with the poorly differentiated
anaplastic carcinoma.

805-Malignant transformation of a hot nodule is rare.

806-Pregnancy is an absolute contraindication for radioactive iodine


usage.

807- sonography cannot differentiate between benign and malignant


lesions.

808-Normal adults on an average require 80 mg of iodine per day.

809-thyroid gland, which in normal adults weighs 15-25 g.

810-Wharton, who gave the name thyroid to the gland because of its
resemblance to an oblong shield.

811-Microtia repair should be performed prior to undertaking atresia


repair. 5-6 years of age.

812- Excision of a misdiagnosed ectopic thyroid gland will result in


postoperative hypothyroidism.

813- The initial switch on of the device(CI) usually occurs after about 4
weeks, when the p.op scalp swelling has settled and the wound is fully
healed.

814-Electrophysiological testing may be carried out at the end of the


procedure to check the function of the device (CI) cochlear implant.

..
193

815- Once the device is in place only the bipolar diathermy can be
used,(CI).

816-The normal rate of migration is about 0.1 mm/day.EAC.

817-The posterior auricular appears to be the dominant artery and


supplies the medial surface (except the lobule) of the auricle.

818-Palpation of the Neck – superficial group of CLN ½ cm in diameter


can be palpated and 1cm in deep group.

819-The younger the age, the more possibility of CLNM.

820- Small LN (<1cm) may show ECS .

821-Once there is a single positive node in the neck the case becomes
stage III or IV regardless of the size of the primary tumor (even T1).

822-Non-Hodgkin's Lymphoma:Nodal and extra-Nodal involvement is


common, the most common extra-Nodal site is involvement of
Waldeyer ring, particularly tonsil's adenoids in the Naso-pharynx and
the lingual tonsil. The second most common extra-nodal site is the
Orbit,The histological picture is infiltration with abnormal Lympho sites
B and T cells and plasma cells.

823-Hodgkin’s Lymphoma:It affects mainly the lymph nodes, extra


nodal disease is rare. Age from (15-30). Most common nodes affected
are the lower Jugular (level IV and III) and the supra-clavicular LN.
abscess formation and the discharging sinuses never occur.
characteristic histological picture of the LN is the presence of the
Dorothy-Read cell.

824-Parotid –Serous, Sudmandibular –Mixed, Sublingual –Mucous.

825- Physical, chemical and mental factors stimulate the production of


saliva. IOOO to 1500 ml / day. 99.5% water.

.
194

826- The autonomic supply is from lingual nerve via chorda


tympani.Sympathtic from s.c.ganglia.

827-The lingual and hypoglossal nerves are in danger during removal


of the gland ie sialolithiasis &tumours.

828-The marginal mandibular branch of the facial nerve running


between the upper pole of the gland and the mandible is in danger
during operations on the submandibular gland.

829- The duct of SMG crosses laterodorsally over the lingual nerve.

830-The submandibular duct ( Wharton`s duct) is about 5cm long and


Opens close to the frenulum in the floor of mouth.

831- Inf.salivary nucleus to glossopharyngeal to the inferior ganglia,


join the tympanic nerve & finaly to lesser superficial petrossal nerve .

832- The parotid duct( stensen`s duct) is approximately 6cm long.It


crosses the the masseter, perforates the buccinator. The orifice lies
opposite the second upper molar tooth .

833- impaired T-cells functions are found in > ½ of newly diagnosed


and untreated pts with NPC.

834- other suppressor genes p16,p21,p27.

835- H&N tumours develop because of mutation in tumour suppressor


genes i.e:p53 – Located at 17p13.

836-Marrow infiltration better detected with MRI.

837- Perineural invasion better detected by MRI.

838- Cervical lymphadenopathy is depicted better by CT than MRI.

839- MRI is recommended for use in staging tu. recurrences.


195

840- MRI superior to CT in identifying submucosal infiltration .

841- MRI is superior in determining IC spread.

842- Contraindication for superfa. Parotidectomy: Any large malignant


neoplasm > 4 cm , recurrent pleomorphic adenoma, recurrent
malignant neoplasm, Congenital first arch abnormalities.

843- Facial nerve lies 6-8 mm anteroinferior to the tympanomastoid


suture line .

844- Tragal pointer( Conley's pointer) advantage is :direct access to


pes anserenius ,1 cm inferior & deep to the tragal pointer .

845-Facial n. markers:Tympanomastoid fissure,tragal pointer,posterior


belly of digastric m.

846-Avoid unnecessary trauma to the facial nerve by :repeated


stimulation , electrocautery, suctioning.

847- ophthalmic division of facial n. is most important because of


blinking of the eyes.

848-IndicationsTotal conservative parotidectomy: benign neoplasm


affecting the deep lobe , recurrent pleomorphic adenoma, malignant
tumors > 4cm, sever parotid enlargement due to sialectasis, Chronic
suppurative parotitis 2ry to calculi or ductal stenosis. 1st arch
branchial abnormalities.

849- Total radical parotidectomy Contraindicated: Lymphoma , T.B ,


HIV , cat scratch disease , Benign lymphoepithelial lesion .

850-Indications for Superficial parotidectomy . pleomorphic


adenomaWarthin 's tumor. Small malignant neoplasm 4 cm or
less,Cysts,intraparotid LN. Vascular malformation. Others: Lipoma

851-radiosensitive tumours such as a lymphoma or undifferentiated


nasopharyngeal carcinoma which can usually be controlled with
radiotherapy...
196

852-Radical neck dissectionThis operation removes the lymph node


containing levels in the neck (I–V), and all three nonlymphatic
structures (spinal accessory nerve, sternomastoid muscle and the
internal jugular vein)..
853- The contraindications for radical and modified radical neck
resection include those patients whose primary tumours are
untreatable, those who are unfit for major surgery and those with
distant metastases.,
854-A neck dissection is never carried out with a hemilaryngectomy
but for a supraglottic laryngectomy, it is pedicled on the thyrohyoid
membrane..
855-Modified radical neck dissectionThis operation consists of
removal of all lymph nodes groups (levels I–V) with preservation of one
or more of non L S..
856-Early reactions typically occur within the first few weeks of
fractionated radiotherapy and are manifest by mucositis, taste loss, skin
erythema and skin desquamation resulting from mitotic arrest of rapidly
dividing cells.
857-HYPERFRACTIONATED RADIOTHERAPY This approach employs a
larger number of fractions compared to a conventional schedule given
over the same period of time and is also achieved by giving multiple
fractions per day.

858-Branchogenic carcinoma the carcinoma should be demonstrated as


arising in the wall of a branchial cyst.

859-Patients with bilateral nodes when one side is fixed are usually
incurable.
860-Only 5 percent of all patients will present with N3 swellings,
861-Any patient with a head and neck primary tumour requires
assessment of the neck. This begins with a history and full clinical
examination which may be supplemented by an examination under
anaesthetic.

862- Further assessment with radiological imaging may be indicated


and fine needle aspiration cytology (FNAC) can help confirm or refute
the diagnosis. Occasionally, an open biopsy may be required.

863-Not every lymph node in the neck requires FNAC investigation.


.
197

864-Sentinel node biopsyThis is the first node that is involved in the


drainage of a tumour within the primary lymphatic basin of an N0 neck.
the technique is considered standard for melanoma in non-head and neck
sites.

865-PET a very sensitive tool for detecting primary malignant lesions, as


well as metastatic spread, and is known to be superior to CT or MRI in
terms of sensitivity..

866-(lateral neck dissection levels II–IV.

867-Selective (levels II–III) dissection.

868-damage to the accessory nerve (10 percent with either selective or


modified Radical dissections).

869- Modified radical neck dissection ‘type III’,This procedure removes


lymph nodal groups I–V, but preserves the sternocleidomastoid muscle,
accessory nerve, internal jugular vein and the submandibular gland.

870-Papilloma (recurrent respiratory papillomatosis (RRP)) is the most


common benign neoplasm of the larynx. –

871-Heliox is a mixture of 80 percent helium and 20 percent oxygen. -


send for home .

872-Restlessness may be the result of anxiety or may be indicative of


hypoxia.

873-The disadvantage of CO2 laser use (over cold steel) is the potential
for thermaldamage.

874-Extended RND: In addition to the usual structures removed in the


RND other structures are removed.:Internal or common carotid arteries.
Hypoglossal nerve. Scalene muscles.Mediastinal LN.

875- Types of SND Supra omohyoid(level Ia&b, level IIa&b, level III ),
Lateral SND(level IIa&b, level III, level IV) ,postero-lateral (level
II,III,IV,V) Central (anterior(Only level VI is removed ).
.
198

876- Selective ND .The aim is removal of selective levels of fascia and


LN leaving the other levels where the possibility of metastasis from this
particular tumour site is very rare.

877--Indications MRND:N0 neck, N1, N2a & some cases of N2b .

878-Contra-indications:MRND, N2c & N3. Fixed LN to any of XI nerve,


SCM or IJV.

879- Indications RND:All N3 lesions,Late N2 lesions, Infiltration of the


IJV or SCM, Involvement of the XI nerve and LN, Macroscopic evidence
of extracapsular spread.

880-Therapeutic ND in the clinically positive neck. Elective


(prophylactic) ND in the N0 neck with a primary with a possibility of
metastasis more than 15-20%.

881-If the primary tumor is to be treated surgically, the N0 neck is


controlled by a modified elective neck dissection unilaterally or
bilaterally, Post operative RT when indicated if there is extracapsular
spread or involvement of multiple levels.

882-If the primary tumor is to be managed by RT the N0 Neck should


be managed by RT.

883- the clinically negative N0 Neck proves in general to be


histologically positive in 20% of the cases.

884-The N0 neck must be controlled in Any T4 stage ,Most T3 Supra-


glottis T2 .

885-When the Probability of CLNM is greater than 15-20% and so we


must control the N0 neck?:

886-The incidence of unknown primary is about 10% of cases


presenting with MCLN.

887-N1, N2, & N3 should be treated surgically by neck dissection.


.
199

888-The rate of RT success in N2 & N3 is less than 20% .

889-Advantages of CT:Can detect impalpable retro-pharyngeal


LN, Can demonstrate LN 1cm in diameter, Can demonstrate the
primary tumor , Can demonstrate extra-capsular spread, Can
demonstrate involvement of surrounding structures.

890-Extra Capsular Spread It is a definite indication of post-op -


RT..

891-Usually MCLN are not responsive to RT except if they are


micro-metastasis (N0 neck) or early N1 (single less than 3cm.).
892-Signs of malignancy in CT:Multiple LN,LN >
3cm,Central breakdown + peripheral rim of
enhancement,Rupture of the capsule & extracapsular
spread.

893– Superficial group of CLN ½ cm in diameter can


be palpated and 1cm in deep group + palaption under
general anesthesia.

894-Reactive nodes indicate good immunity.

895-The cellular immunity of the patient represented


mainly in the T-cells and natural killer cells is an
important factor to destroy malignant cells reaching the
LN.

896-The younger the age, the more possibility of CLNM.

897-High freguency audiometry : sensitive to ototoxicity .


200

898-Virchow node : supraclavicular node of level V.

899-Pretrematic n. for I arch : corda tympani ./ for II arch Jacobson n

900-Mean PH of saliva : 6.7 .

901-PNST : to masure total IgE / RAST : for allergen-specific IgE

902-Antibiotic secreted by saliva : clindamycin

903-Aminoglygocid potentiate the effect of botulinum toxin .

904-Gunidin & aminopyridin limit the effect of botulinum toxin .

905-Urine out put : 0.5 – 1.5 ml/kg/hr for adult .

906-Samter triad : nasal polyps in association with asthma & aspirin


sensitivity .

907- Leukotrienes : slow-reacting substances of anaphylaxis .

908-Arachidonic acid is cleaved from cellular membranes by


phospholipase A2 .

909-Prostaglandin E2,a product of the cyclooxygenase pathway


,inhibits 5-lipoxygenas inFeedback loop

910-Tumor suppressor gen ; p53 ,p16 .

911-Function of IL2 : stimulate of T&NK cells .

912- Warthin-finkesdey giant cell : measles .

913-In esophageal wall : submucosal plexus--- meissner's plexus ,//


myentric plexus---- - auerbach's plexus .

914- Nasolacrimal duct : 12-24 mm length .

915-length of fallopian canal : 30 mm

916- Mike's dot : macula cribrosa superioris vestibule .


201

917-Loop of Galen : Vagus n.

918- cozzollino's zone : otosclerosis .

919- Broca's aphasia : area 44 ///// Wernick's aphasia : area 22 .

920-vertebral level of larynx in newborn(C1-C2),,infant(C2-C3)


,adult(C3-C6)

921- Emperipolesis is pathognomonic of : Rosai-Dorfman disease .

922-Cat scrach : Rochalimea henselae .

923- Total number of skull bones : 22

924- Carotid plexus : sup. Cervicle ganglion .

925- Zimmerman cell : Hemangiopericytoma .

926- foramin monro connect ; lat. To 3rd ventricle // foramin Luschka :


4th vent.& sub.arch.s .

927- Auditary n. ---sup.olivery complex ---- inf.colliculus --- auditary


cortex

928- COX-2 : involved in prostaglandin synthesis .

929- Surface area of T.M : 70-80 sq.mm , and vibrating area : 55


sq.mm / surface area of stapes : 3.2 sq.mm /wieght of ossiculat
mg(incus 27 mg,malleus 25 mg and stapes 3 mg)/TM in chain 55
obliqe position 55`

930- Perilymph ; scala vestibule & scala tympani / endolymph : scala


media
.
202

931-Sylvius aqueduct connect : 3rd to 4th ventricle .

932-COWS : cold : opposite ,,,,, warm : same .

933-physaliferous cells : chordoma .

934-Lyme disease : borrelia burgdorferi .

935-paracusis willisi ; (otosclerosis) hear better in noisy surrounding .

936-hypercusis : pain on exposure to loud noise .

937-diplacusis : difference of perception of sound by two ears .

938-lehrmittes syndrome due to radiotherapy .

939- Epulis : mucoperiostium of gum .

940--- Reed sterinberg's cell : (kidney shape) ,Hodgkin disease .

941-Erb's point ; XI n. inter sternoclidomastoid m .

942-Mikulicz's syndrome : symmetrical enlargement of lacrimal &


salivary glands

943-verocay's body : schwanoma (acustic neuroma) in antoni –A .

944-Birbek's granules : langerhans cells .

945-phelp's sign : Glomus jugulare .

946-Hennebert sign : congenital syphilis & menier's disease .

947-Most pathogenesis for Bell's palsy : Herpis simplex v.

948-- lyre sign : carotid body tumor / Riolan's bouquet : styloid


apparatus

949-Parotid : serous /submand . gland :mixed /sublingual gland :


mucous /minor sal. Gland : seromucous .
.
203

950-Glottis in new born : 24 sq.mm .


length of vocal cord at birth : 7 mm .
= = = = at puberty : 14 mm .
= = = = of adult female :15-16 mm
= = = = of adult male: 17-21mm.

951-Limitation of motion without fixation attributed to:Bulky


tumour,Moderate thyrohyoid muscle invasion, Extension of growth along the
superior surface .

952-The so-called ‘Delphian’ node (also known as the midline anterior


metastatic node orPoirier's prelaryngeal ganglia node) is very rare.

953-Invasion through Broyle's ligament directly into cartilage was


thought inevitable. This was perceived to be due to the lack of an inner
perichondrium in that area.

954-Heliox is a mixture of 80 percent helium and 20 percent oxygen.

955-Restlessness may be the result of anxiety or may be indicative of


hypoxia.

956-Aphonia is almost always associated with very severe injury.

957-Up to 5 cm of the trachea can be excised to rejoin the trachea


back onto the cricoid or the first tracheal ring.

958-MITOMYCIN C antineoplastic and antiproliferative properties


,inhibit fibroblast proliferation both in vivo and in vitro..

959-The disadvantage of CO2 laser use (over cold steel) is the


potential for thermal damage.

960-Croup affects mainly young children, aged six months to three


years,

961-Prior to vaccination programs, Haemophilus influenzae type b was


the commonest pathogen especially in children. A wide range of
pathogens has been described in adults and these include Group A
Streptococci, Strept pneumoniae ,Staph aureus and Klebsiella
pneumoniae.
204

962-Epiglottitis affects all age groups.

963-Injectible materials include Teflon, fat, glycerine, collagen and


silicon . All provide immediate results. All can ideally be carried out
under local anaesthetic in the normal anatomical position with the
patient sitting up by injecting through the cricothyroid membrane to
obtain the most reliable voice outcomes. These products demonstrate a
range of reversibility with glycerine being totally reversible, followed by
fat and Teflon being totally.

964-The left brachiocephalic vein may project upwards into the neck to
form an anterior relation of the cervical trachea and is a potential
surgical hazard during tracheostomy.

965-Adenoids are not radiologically visible on plain radiographs in


infants under one month of age, becoming radiologically demonstrable
in all infants at six months.

966-Down syndrome is genetically defined by a nondisjunction


mutation that results in trisomy 21. The incidence is one in 700, which
is more common than all other chromosomal anomalies.

967-A neck incision with dependent drainage of the bilateral floor


of the mouth through the mylohyoid muscle is mandatory in cases of
Ludwig’s angina to reduce the risk of airway obstruction.

968-Presence of an air-fluid level or greater than 5 mm of thickening


in a child or greater than 7 mm of thickening in an adult of the
prevertebral tissue at C2 indicate retropharyngeal infection until proven
otherwise.

969- poststyloid compartment contains neurovascular structures:


carotid artery, Internal jugular vein, sympathetic chain, and cranial
nerves IX, X, XI, and XII .

970-prestyloid compartment contains fat, muscle (styloglossus and


stylopharyngeus), lymph nodes, deep lobe of the parotid, internal
maxillary artery, inferior alveolar, lingual, and auriculotemporal nerves.
205

971-Methicillin-resistant Staphylococcus aureus is an increasingly


common cause of deep neck space infections.

972-EBV is the causative agent of infectious mononucleosis.

973-Injected and systemic steroids have shown success for treating


these AIDS-related ulcers.

974-(HIV) type 1 infection causes a mononucleosis-like syndrome in


40% to 90% of patients.

975- Chlamydia trachomatis single oral dose of azithromycin as first


choice therapy.
976-Adults with GABHS pharyngitis. Penicillin remains the treatment of
choice due to its proven efficacy, narrow spectrum, and low cost.

977- PET, because of limited spatial resolution and lack of anatomic


detail, is often insuficient for the assessment of submucosal extent of
disease and involvement of adjacent structures.

978-T2WIs show the vitreous and CSF as high signal intensity


(bright) relative to the low to intermediate signal intensity of head
and neck fat and muscle

979-To quickly identify a T1WI: fat is white, CSF and vitreous are
black, and nasal mucosa is low signal.

980- One Gy is the amount of radiation needed to deposit the energy


of 1 Joule (J) in 1 kg of tissue (1 Gy = 1 J/kg).

981-transorbital projection is a frontal projection of the mastoids


and petrous bones.

982-Stenvers projection is an oblique projection of the petrous


bone obtained withthe patient’s head slightly flexed and rotated 45
degrees toward the side opposite the one under study.
.
206

983- The Schüller projection, a lateral view of the mastoid


obtained with 30 degrees of cephalocaudad angulation.

984- The SMV view can show the sphenoid sinuses and the
anterior and posterior walls of the frontal sinuses.

985-Radiation exposure from CT examination is substantial and should


be taken into account particularly in young patients who require repeat
examinations .

986-Aspirin, an irreversible inhibitor of platelet function, leads to


prolonged bleeding time.

987-Congenital deficiencies of hemostasis affect up to 1% of the


population.

988-The term thyroid storm refers to a life-threatening exacerbation of


hyperthyroidism that results in severe tachycardia and hypertension.

989-Patients with allergic to eggs may react to propofol.

990- 28% to 67% of children with spina bifida demonstrate positive


skin tests to latex proteins.

991-Anaphylactic reactions to cephalosporins in true penicillin-allergic


patients are probably less than 2%.

992-A normal adult swallows unconsciously 600 times in a 24-hour


period.

993- Tympanoplasty alone, without mastoidectomy, is indicated in


patients with normal-appearing mucosa in the middle ear .

994-Biopsy of the minor salivary glands of the lip has been considered
the gold standard for the diagnosis of Sjögren's syndrome.

995-Approximately 50% to 70% of tumors arising in the minor salivary


glands are malignant.
.

.
207

996-Enlarged foramina at the skull base and the presence of


hyperintense enhanced tumor tissue are suggestive of perineural
spread.

997- Patients with malignancies of the minor salivary glands most


often present with a painless submucosal swelling.

998- The site most frequently involved with minor salivary gland
malignancies is the hard palate.

999-Cranial nerve neuropathies, manifesting as a decreased gag reflex


(IX, X), aspiration (IX, X), asymmetric palate elevation (X) hoarseness
(X), dysphagia (X), weakness of the trapezius muscle XI
atrophy/paresis tongue (XII).

1000- Mucoepidermoid carcinoma was the most frequent


malignancy of the parotid, but adenoid cystic carcinoma was the most
frequent of the submandibular and minor salivary .

1001-Most common primary malignancy presenting in the salivary


glands to be mucoepidermoid carcinoma. The next most common type
was adenoid cystic carcinoma (22%).

1002-The pectoralis major flap is the most commonly used muscle or


myocutaneous flap worldwide in head and neck surgery and certainly
one of the most reliable of all the pedicle flaps.

1003-Titanium is a safe metal in a magnetic field. Prostheses made of


nitinol are also safe during MRI.

1004-Digital subtraction angiography (DSA) is the best method to


delineate the vascular supply of a specific anatomical area or lesion.

1005-Biofilms are described as a collection of bacteria coated by an


extracellular matrix, which may effectively protect the bacteria from
penetration by the antibiotic.
.
208

1006-Cholesterol granuloma is a comprehensive term describing a


loculated cystic or solid foreign body reactive process directed against
cholesterol crystals resulting from the by-products of degraded blood.

1007-Routine hematologic tests of coagulation and platelet function


are not necessary in patients without a personal or family history
suggestive of a coagulopathy.

1008-Patients should stop taking all aspirin for 10 days before surgery
and all other nonsteroidal anti-inflammatories, vitamin E, and ginkgo
biloba, which can
all prolong bleeding, 7 days preoperatively.

1009- DNA is the most critical target for the biologic effects of
radiation.

1010-The unit of measure for radiation is the gray (Gy). 100 rad = 1
Gy.

1011- Autologous fat and Cymetra are the materials most


commonly used today. Cymetra has been shown to provide excellent
phonatory results lasting 6 to 12months with little or no
inflammatory response.

1012- Like bovine collagen, Cymetra may last 3 to 9 months and can
be used for temporary medialization.

1013-Pemphigus typically has acantholytic lesions that are


subepithelial with basement membrane involvement .

1014-Dapsone have been tried in refractory cases of relapsing


polychondritis.-

1015-The main proteolytic agent produced by the stomach, pepsin is


active principally at acidic pH.

1016-The disease is diagnosed from the identification of the


causative organism, a gram-negative coccobacillus, within macrophages
obtained from mucosal biopsy specimens. These are the characteristic
Mikulicz cells of rhinoscleroma.

.
209

1017- HPV has been estimated to be present in the genital tract of as


many as 25% of all women of child-bearing age worldwide.

1018-Propranolol, a beta-blocking agent, has been used success-fully


to treat infantile hemangiomas.

1019- Lugol’s solution, as well as toluidine blue, have been used to -


stain laryngeal tissue in an effort to discriminate normal tissue
from tumor .

1020- PET image False positives may also result from infection
radionecrosis, or accumulation of saliva in the vallecula.

1021-Two entities in particular can be dificult to distinguish from SCC.


Pseudoepitheliomatous hyperplasia (PEH )& Necrotizing
sialometaplasia .

1022- Optical coherence tomography (OCT) is a new diagnostic


modality under investigation to examine the epithelial and subepithelial
architecture that uses near-infrared light waves to produce cross-
sectional images of tissue in vivo, with a resolution approaching
that of histology.

1023- Pet is a functional imaging technique that relies on the


increased metabolic uptake of glucose in tumors to identify
unknown primary tumors, stage malignancies, search for metastatic
disease, and evaluate recurrences.

1024-In case of changes in architecture and in the presence of atypia


in cells, we speak of dysplasia. Dysplasia is considered a range and it
is somewhat challenging to divide the dysplastic spectrum into mild,
moderate, and severe .

1025-Hyperplasia describes increased cell counts in the spinous


layer or in the basal/parabasal layers. The cells are free of atypia and
tissue architecture is regular.

1026-Carcinoma in situ is described as malignant transformation


without invasion.
.
210

1027- Most esophageal diverticula are acquired lesions found in


Adults.

1028-Topical mitomycin C may be useful for the treatment and


prevention of subsequent re-stenosis and scar formation in the larynx
and trachea. Mitomycin C is an antineoplastic antibiotic that acts as an
alkylating agent by inhibiting DNA and protein synthesis. It can inhibit
cell division, protein synthesis, and fibroblast proliferation.

1029-broad-spectrum antibiotics, and peripheral nutrition, is the


cornerstone of managing esophageal injuries.

1030-Bile acid is known to be associated with tumor formation in the


esophagus through the overexpression of cyclooxygenase2.

1031-Continuous pH monitoring studies are believed to be the “gold


standard study” for diagnosing GERD and EER.

1032-GERD have reduced laryngopharyngeal sensitivity compared


with healthy controls which could result in a higher risk of aspiration.

1033-Mechanisms of Symptoms due to Extraesophageal Relux Head


and neck disorders associated with EER are postulated to occur via the
following mechanisms: direct mucosal damage and direct effect on
mucociliary clearance from exposure to gastric contents related
distal esophageal damage that results in vagally mediated,
referred symptomatology; and laryngeal relexes mediated by the
stimulation of distal esophageal afferents

1034-In falsetto mode, only the superior edges of the vocal folds
contact.

1035-The posterior cricoarytenoid muscle (PCA), the only active dilator


of the larynx.
Chapter two

SLIDES
211

Slide 1
1-Name the surgery ?
2-Name structure A?
3-Name structure B ?
4-Name structure C ?

A------
C----
B -----

Answer

1-Modified radical neck dissection type 1..

2-Tenden of posterior bely of digastric m.

3-Sternohyoid muscle

4-Accessory nerve
212

Slide 2

Write the name of the structure in this graph.

A:

B:

C:

D:

E:

Answer

A: Lingual nerve .

B: Submental gland .

C: Submandibular gland and duct .

D: Mylohyoid M.

E: Genioglosus M :
213

Slide 3

Write the name of the structure in this graph?

A:

B:

C:

D:

Answer

A: Submental triangule .

B: Submandibular triangule.

C: Muscular triangule.

D: Carotid triangule .
214

Slide 4

Write the name of the structure in this graph?

A:

B:

C:

Answer

A: Posterior pelly of digastric M.

B: Sternoclidomastoid M.

C: Anterior pelly of Omohyoid M.


215

Slide 5

Name the structures in the mastoid antrum?

1.
2.
3.
4. 3 2

Answer
1. Horizontal semicircular canal;
2. Osseous wall that forms the lateral wall of the attic.
3. External auditory canal.;
4. Sinodural angle

Notes:
The most important landmarks for the facial nerve are the HSCC, the
short process of the incus, and the posterior bony external auditory
canal; the digastric ridge is also a landmark,
216

Slide 6

A-Name the Structures of the mastoid antrum and attic.


B-Why you should be identifying the short process of the incus?
C-Whats the most important landmarks for the facial nerve?
D-Surgical identification of the endolymphatic sac?

2
4
3
1 6

A:
1- External auditory canal;
2. Tympanic membrane;
3. Body of the incus;
4. Malleus head;
5. Horizontal semicircular canal;
6. Dural plate of the middle cranial fossa

B:-To provides an important landmark for facial nerve dissection.

C:-The HSCC,
-the short process of the incus,
- the posterior bony external auditory canal;
-the digastric ridge is also a landmark,
D:-Posterior and inferior to the posterior semicircular canal.
217

Slide 7

Write the name of this Facial landmarks From ( A—J ) ?

B
C ?

D
E
F
G

H
I
J
218

Slide 7
Answer
219

Slide 8

Write the name of the structures in the cavernous sinus? From (1 – 12)

6
3 4 5
2
1

12
1110 9 8 7
220

Slide 8

Answer
221

Slide 9

This is an otoscopic finding of a patient who presents with fullness of


ear and mild hearing loss.
A-Diagnosis?
B- Name the enumerated structures?
C-What can we do further to treat this patient?

Answer
A--Adhasive otitis media
B- Name the enumerated structures?
1-Handel of malleus .
2-Lateral process of malleus.
3-umbo.
4-Promontery.
5-Round window.
6-Long process of incus.
7-lantecular process of incus
8-Stapedial tenden.
9-Facial nerve.
C-Hearing aids.
-Myringotomy and grommet if there is
aspace for insertion
222

Slide 10

1-Name of this test?

2-Age reguierd for this test?

3-Principle of this test ?

Answer

1-Play audiometry.

2- > 24 month .

3-The child is conditioned to put the wooden sailor in place in the boat
on hearing a pure tone delivered by headphones
223

Slide 11

1-Name of this test ?

2-What is its use?

3-Age requierd for this test ?

Answer

1-Visual reinforcement audiometry.

2-Its used to screen infants for hearing disabilities.

3- 6- 24 months
224 A

Slide 12
B
1-Name of the instrument A and uses ?

2-Name of the instrument B and uses ?

Answer
A-Sinus ostium ballpoint probe.
-Uses :-To locate maxillary and sphenoid sinuses ostia
-To delinate and resect UP
-To identify the consistency of mass in the nasal cavity.

B-Blakesley Weil 45 degree upturned forceps.


Uses:-Use in FESS to remove tissue and bone fragments from the
lateral wall and frontal recess ,Used for biopsy.
-remove the contents of the maxillary sinus through the ostium.
-reach the structures that are higher up in the nasal cavity and the
lateral wall of the nose
225

Slide 13

1-What is the name of this surgical technique?

2-Name 3 indications?.

Answer

1-Mohs micrographic surgical technique.

2-Name 3 indications?
-Recurrent basal cell carcinoma.
-High risk squemous cell carcinoma.
-Malignant melanoma.
226

Slide 14

A-What is this investigation?


B-What is the abnormality? (blue arrows and starix)
C-Name the structures neumarated ?

Answer

A-CT scan axial view with contrast of suprahyoid of the neck at level of
tongue base.

B-Tumour base of tongue crossing the midline, bilateral necrotic cervical


lymphadenopathy, level II.

C-Name the structures neumarated ?


1. Sternomastoid muscle.
2. Mandible (ramus).
3. Spinal cord.
4. Internal jugular vein.
227

Slide 15
1-Name of structures labelled from A-I?

2-Name of the view of this section?


G
A

H
I
Answer
1-Name of structures labelled from A-I?
A:Frontal sinus.
B:Ethmoid cells.
C:Ethmoid bulla.
D:Middle turbinate.
E:Uncinate process.
F:Maxillary sinus.
G:Osteomeatal complex.
H:Inferior turbinate.
I:Nasal septum.
2-Coronal section of nose and PNS shown multiple structures.
228

Slide 16

1- Name of this classification ?


2-What is the A ,B,C,?

Answer

1-Fisch classificartlon of Infratemporal fossa approaches.

2-Type A: exposure to the level of the anterior eustachian tube, middle


meningeal artery/foramen spinosum

-Type B: exposure to the foramen ovali, V3.

-Type C: exposure to the foramen rotundum, V2.


229

Slide 17

1. Name this instrument?


2. Give 3 indications?

Answer

1-Rigid oseophagoscope.

2-a- foreign body removal.


b-taking biopsy.
c- stricture dilatation.
230

Slide 18

1. What is this structure?


2. Give 2 uses.

Answer

1-Montogmery T-tube.

2-Stenting in subglottic stenosis repair.


-stenting in tracheal repair
.
231

Slide 19

1- Name the structures labelled A–F


2- What is the modiolus?

Answer

1-A: scala vestibuli


B: scala media
C: scala tympani
D: Reissner’s membrane
E: stria vascularis
F: tectorial membrane

2--The conical- shaped central axis of the cochlea


232

Slide 20

1- Describe pathology in this picture:


2- Symptoms:
3- Life style modifications:
4-Drugs for treatment:
5- Surgical operations:

Answer

1- Endolymphatic hydrops
2- Vertigo , Tinnitus , Aural fullness , fluctuating Low freq. SNHL
3- Salt restriction , decrease stress , alcohol , tea and coffee
4-During attacks: Anti emetics, Diazepam
-in between attacks: Betahistine , Diuretics
5-Endolymphatic sac decompression , labrynthectomy , vestibular
neurectomy , intratympanic gentamycin injection.
233

Slide 21

Write the names of incisions below?


. (A)
. (B)
. (C)
. (D)

A B

C
D

Answer

. (A) Gluck incision for unilateral and bilateral neck dissection


. (B) Double-Y incision of Martin
. (C) Single-Y incision
. (D) Schobinger incision.
234

Slide 22

1-What is this image?


2- Name the parts that are numbered.?

Answer

1-Axial CT scan of skull at temporal bone.


2- Name the parts that are numbered.?
1-Mastoid air cells.
2-Jugular foramen.
3-Zygomatic air cells.
4-Ossicles .
5-Cochlea .
6-Sphenoid sinus.
7-Internal carotid artery.
8-Sgmoid sinus,
9-External auditory canal.
235

Slide 23

1-Write the name of each flaps ?


2-Name one disadvantage of flap B?
3-What the blood supply of flap C?

A B

Answer D
1-A- Bernard burrow flap.
B- karpandizic flap.
C-Abbe-Estlander flap.
D-Gilles fan flap.

2-Microstomia.

3-Axial labial from facial artyery.


236

Slide 24

1-Name of this device ?


2-Indications ?
3-Disadvantages ?

Answer

1-Body worn Bone Conduction Hearing Aid.

2- moderate-to severe hearing loss.


- operative risks associated with surgically implanted hearing aids.
-speech discrimination scores >60%.
-As a trial period before cochlear implant.

3-No localization of sounds


-Big size ,
-patient can hear sounds of clothes.
237

Slide 25

1-Name of this flap ?


2-Blood supply ?
3-Disadvantages ?

Answer

1-Trapezius pedicled flap,

2-Ascending cervical artery.


-Transverse cervical artery.
-Suprascpular artery .

3-Bulky flap
-Donor site my required skin graft.
-Base of the flap is too thick.
-Sacrifice of the accessory nerve.
238

Slide 26

1-Name of procedure in this graph?


2- 3 complications of this procedure?
3-Write the name of each number?

7
5
6

4
5
9
2 8
3
1

Answer
1-Submandibular gland excision.
2-Injuery to the marginal mandibular nerve.
-Injuery to the lingual nerve.
-Injuery ot hypoglossal nerve.
3-1-Submandibular gland.
2-Anterior belly of digastric muscle.
3-Stylohyoid muscle.
4-Mandibule.
5-Facial artery and vein ligated.
6-Marginal mandibular nerve.
7-Lingual nerve.
8-Hypoglossal nerve.
9-Submandibular duct ligated.
239

Slide 27

A-Name of this procedure?


B-Write the name of each number in this
graph?
C-Purpase of use of No:3 in this graph?

6 3 2 6
1

7
4
5

Answer

A-Stapedotomy.
B-Write the name of each number in this graph?
1-Piston.
2-Footplate.
3-Fat.
4-Saccule.
5-Utricle.
6-Otoseclerotic focus.
7-Vestibule.
8-Long process of incus.
C- Loose connective and fat tissue is placed around the piston and the
footplate to close of the perforation.
240

Slide 28

1-Define the site of pathology?


2-What’s view of this graph?
3-Write the name of each number in this graph?

2 1
6
3
4

Answer

1-Rt CerebelloPontin Angle and Internal Auditory Meatus.


2-Axial view.
3-Write the name of each number in this graph?
1-Trigeminal gangilion.
2-Facial nerve.
3-Vestibular nerve.
4-Acoustic neuroma.
5-Cerebellum.
6-EAC
241

Slide 29

1-What is the patholoy?


2-What’s direction of arrows mark?
3-Write the name for each number?

1
2
5
3 6
7

Answer

1-Glomus jugulare.
2-Extension of the tumor.
3-Write the name for each number?
1-Tympanic memberan.
2-Facialnerve.
3-Sigmoid sinus.
4-Internal jugular vein.
5-Internal carotid artery.
6-Glomus jugulare tumor.
7-Parotid gland.
242

Slide 30

Write the names of structures labelled below ?

A
1
A

3 2

4
A 9
6 5

A
7A
8

9
aa
a
aaa
aaa
Aaa
243

Slide 30

Answer
244

Slide 31

A-Write the name of structures from (1-8) labelled in this graph?

B- Absolute Contraindications for rhinoplasty

C-Indications for open rhinoplasty?

Answer

A:1. Nasal bone


2. Nasomaxillary suture line
3. Ascending process of maxilla
4. Osseocartilaginous junction (rhinion)
5. Upper lateral cartilage
6. Anterior septal angle
7. Caudal free edge of upper lateral cartilage
8. Sesamoid cartilage
B:• Bleeding and coagulation disorders which are uncorrectable
• Pregnancy
• Any systemic illness which might be worsened by rhinoplasty
surgery
• Significant psychiatric disorder
C:-Revision rhinoplasty
-Severe deformity.
-Congenital deformity.
245

Slide 32

A-Write the name of structures labelled below ?


B-Name the area of vascular anastomosis?
C-Name of blood supply of this area?

1
2 9
3
4 8
7
5 6

Answer
A-1-Anterior and posterior ramus of attachment of middle turbinate
2-Frontal sinus opening
3-Hiatus semilunaris.
4-Uncinate process
5-Nasolacrimal duct opening.
6-Sphenopalatine foramen
7-turbinate process of ethmoid
8-Bulla ethoidalis
9-Posterior ethmoid sinus
B-Little’s area.
C-Sphenoplatine artery.
-Anterior ethmoid arterty.
-superior labial artery.
-Grater palatine artery
246

Slide 33

1-Write the name of the parts of the inner ear?


2-Common cataion of part G?
3-Space between bony and membranous labyrinth is filled with?

G
F
A
E

B
D
C

Answer

1-
A:Reissner’s membrane.
B:Osseous spiral lamina.
C:Basilar membrane.
D:Scala tympani.
E:Stria vascularis.
F:Scala media.
G:Scala vestibuli.

2--Sodium.
3--Perilymph.
247

Slide 34

1-Whats the test?


2-Diagnosis ?
3-Write the structures labelled below ?

Answer

1-Barium swallow.

2-Pharyngeal pouch.

3-D: Cricopharyngeus.
-E:Zenker diferticulum.
-F:Esophagus.
-G:Trachea
248

Slide 35

1-Whats the photograph shown ?


2-Whats the view ?
3-Write the structures labelled below? from(-A –I ):

I
A

B H
G
F

C E
D

Answer

1-External nose structure.


2-Basal view.
3-Write the structures labelled below?
-A: Lobule
-B: External soft triangle
-C: Ala
-D: Maxillary crest
-E: Lower caudal septum
-F: Lateral crus
-G: Medial crus
-H: Dome (intermediate crus)
-I: Interdomal space
249

Slide 36

1-Write the structures labelled below ?


-A:
-B:
-C:
-D:
-E:
-F:
-G:

E D
A G

B
F C

Answer

1-Write the structures labelled below ?


-A: Tongue.
-B: Sublingual gland.
-C: Submadibular gland.
-D: Parotid gland.
-E: Buccinator muscle.
-F: Mylohyoid muscle.
-G: Lingual nerve.
250 stractures
Slide 37

1-Name of this ring ?


2-Write the name of this structures ? from (A – D ):

A e
s
B

Ces

r
a
Answer
e
1-Name of this ring ? s
-Waldeyer’s Ring.
2-Write the name of this stractures ?
A: Adenoid.
B: Tubal tonsil.
C: Palatine tonsil.
D: Lingual tonsil.
251

Slide 38
Dr.Sundus Al-Sedra

Name the structures labelled as `a-h` in the figure. Indicate the sides
as well as the orientation ( anterior & posterior aspect) of the drawing.

Answer

a-Epiglottis.
b-Aryepiglottic fold.
c-Anterior commisure.
d-True vocal cord.
e-False vocal cord.
f-Arytenoid cartilage.
g-Piriform fossa.
h-Rima glottis
.
252

Slide 39
Dr.Sundus Al-Sedra

A-Name the following instrument.


B-Name parts labelled `a-g`.
c-List five risks related to this procedure.

Answer

A. Ventilating bronchoscope.
B. a-proximal attachment(e.g. for glass window, rubber guide,
bridge & a telescope).
b-bridge (inserted between the bronchoscope & telescope).
c-fibreoptic connection (light source).
d-prismatic light deflector.
e-slide attachment (for injection cannula or guide for suction
catheter).
f-guide for suction catheter .
g-ventilation connection.
C- pain, bleeding, infection, dental damage, voice change /loss,
temporary worsening of the airway, pneumothorax/pneumo -
mediastinum
.
253

Slide 40
Dr.Sundus Al-Sedra

A. Name structures labelled `a-d`


B. What surgery did the patient undergo?
C. What dose T.O.P stand for?
D. What is the purpose of T.O.P?

Answer
A. a-pharynx
b-oesophagus
c-laryngectomy stoma
d-T.O.P=tracheo-oesophageal puncture
e-trachea
B. Total laryngectomy.
C. Tracheooesophageal puncture –an artificial communication between
the posterior tracheal wall and esophagus (anterior wall) . It can be
performed at time of total laryngectomy (1ry puncture) or later (2ndary
puncture).
D. The purpose of puncture is voice restoration following total
laryngectomy. The puncture can initially house a feeding tube to later
accommodate an artificial speaking valve.
254

Slide 41
Dr.Sundus Al-Sedra

A. Label parts `a-d`.


B. Name 6 nerves passing through A.
C. What structures pass through B ?
D. What passing through C ?
E. Which nerve pass through D ?

Answer
A. a-superior orbital fissure
b-internal auditory meatus.
c-foramen rotundum.
d-jugular foramen.
B. a-frontal (branch of V1)
b-nasocilliary (branch of V1)
c-lacrimal (branch of V1)
d-oculomotor (CN III)
e-trochlear (CN IV)
f-abducent (CN VI)
C. a-facial nerve (VII) with nervus intermedius.
b-vestibulocochlear nerve (VIII).
c-labyrithine artery.
D. Maxillary division of trigeminal nerve.
E. a-glossopharyngeal (CN IX).
b-vagus (CN X).
c-accessory (CN XI).
255

Slide 42
Dr.Sundus Al-Sedra

A. Label parts `A-D`.


B. What structure(s) passes through `A`?
C. Name three structures attached to `B`.
D. Label part `C`.
E. Name structure passing through `C`.
F. Name three muscles attached to `D`.

Answer.

A- a-foramen ovale.
b-styloid process.
c-stylomastoid foramen.
d-mastoid process.
B. MALE- a-mandibular branch of trigeminal nerve.
b-accessory middle meningeal artery.
c-lesser petrosal nerve (parasympathetic to parotid gland).
d-emissary vein.
C. a- stylohoid muscle & ligament. b-styloglossus.,
c-stylopharyngeus. ,d-stylomandibular ligament.
D. Stylomastoid foramen.
E. Facial nerve (VII).
F. SCMM, splenius capitis, posterior belly of digastric,longissimus
capitis.
256

Slide 43
Dr.Sundus Al-Sedra

A. Name the structures labelled `a-e`.


B. Which nerve pass through the foramen labelled `a-c` & why do
they origin from?
C. Which muscle originate & inserts in the area dotted red?
D. Which group of muscles dose it belong to?
E. What other muscles belong to this group?
F. Nerve supply?

Answer
A. a-supraorbital fissure.
b-infraorbital foramen.
c-mental foramen.
d-coronoid process of mandible.
e-condylar process of mandible.
B. a-supraorbital nerve , from frontal nerve from (V1).
b-infraorbital nerve , from( V2).
c-mental nerve from inferior alveolar nerve from (V3).
C. Temporalis muscle.
D. Muscles of mastication.
E. Masseter, medial & lateral pterygoid muscles.
F. Mandibular division of trigeminal nerve.
257

Slide 44
Dr.Sundus Al-Sedra

A. What is structure shown in figure?


B. Label items from `a-g`.
C. Which structure(s) detect angular acceleration?
D. Which of the structures play a role in the pathophysiology of
BPPV?
E. Which structures are tested during the caloric test?
F. What is the temperature of water in caloric testing?

G
A
C

F
E B
D
Answer
A. Membranous labyrinth.
B. a-saccule.
b-utricle.
c-cochlear duct.
d-Endolymphatic duct.
e-lateral scc.
f-posterior scc.
g- superior scc.
C. Lateral, posterior & superior semicircular canal.
D. Utricle & posterior semicircular canal.
E. Lateral semicircular canal.
F. Cold-30 degree C, warm -44 C, Ice – 10 degree C.
258

Slide 45

6 week embryo .name the structures labelled A-G ?

A
B E

C
F
D

G
A

Answer

A:Frontal process.
B:Lateral nasal process
C:EYE
D:Maxillary process.
E:Medial nasal process
F:Naso-optic groove.
G:Mandibular process.
259

Slide 46

Write the name of structures labelled below?

A:
B:
C:
D:
E:
A

C D
E
Answer

A:Auditory ossicles(incus)
B:Auditory tube.
C:EAC
D:Meatal plug
E:Primitive tympanic cavity.
260

Slide 47

1-Whats the picture show?


2-Write the structures labelled A-G ?
3-Whats the Structures derived from this?

B A

D
C
E

G
Answer
1-First and second branchial arches
2-Write the structures labelled A-G ?
A:Incus.
B:Malleus
C:Meckels cartilage.
D:Stapes.
E:Styloid process.
F:Stylohyoid ligament.
G:Hyoid bone
3-1st arch :malleus head&neck ;incus body&short process,tensor
tympani, masticator m.,ant.belly of digastric m.,tensor palate m,
-2nd arch :manubrium of malleus Long process of incus,lenticular
process of stapes,styloid process, platysma ,stapedius m.and tendon
,facial m. auricul m.,posterior belly of digastric m.,stylohyoid m ,
261

Slide 48

1-Whats the length of Eustachain tube?


2-Whats the defferance between adults and childran?
3-whats the parts?

Answer

1-Whats the length of Eustachain tube?


-36 mm in adult

2-Whats the defferance between adults and childran?


-Length/36mm in adult 18mm in infant
-Angle with horizontal/45 in adult and 10 in infant.
-Lumen/narrower in adult. 0

3-whats the parts?


a-Lateral 1/3 is bony.
b-Medial 2/3 is fibro-cartilaginous.
262

Slide 49

1-Write the structures of inner ear labelled A-J ?


2-Whats the defferance between IHC and OHC ?

E D
A
B
F

H I
C G
A

Answer
1-Write the structures of inner ear labelled A-J ?
A-scala vestibuli /B-scala media /C-scala tympani
D-stria vascularis /E-Reissner’s membrane /F-tectorial membrane
G-basilar membrane /H-outer hair cells /I-Deiter’s cells /
J-inner hair cells
2-Whats the defferance between IHC and OHC ?
-OHC:12000,3 or 4 rows, cylindrical and efferent.
-IHC:3500,single row,flask-shape and afferent
263

Slide 50

Spot diagnosis .

35 years old wemon present with this feature after rapid thyroid gland
enlargement.

1-Name of this clinical feature?


2-Diagnosis ?

Answer

Spot diagnosis .

1-Right Horner ’ s syndrome.

2-Thyroid cancer
264

Slide 51

Spot diagnosis .

1- Name of this device ?


2-Principle of this device ?

Answer

Spot diagnosis .

1-Frenzel lenses .

2-To provide magnification of the eyes and remove visual fixation.


265

Slide 52

Spot diagnosis.

A:

B:

C:

A
B

C
Answer

Spot diagnosis.

A :Hemorrhagic polyp

B :Mucus retention cyst

C :Epidermal cyst
266

Slide 53

Spot diagnosis .

A:

B:

C:

B
A

Answer C
Spot diagnosis .

A :Intubation granulomas

B : Laryngeal Papillomata

C :Lateral saccular cysts


267

Slide 54

Spot diagnosis.

A:

B:

C:

A B

C
Answer

Spot diagnosis.

A :Lateral rhinotomy incision.

B :Lip-splitting incision for mandibulotomy

C :Incision for transcervical approach to the parapharyngeal space


268

Slide 55

Spot diagnosis.

A:

B:

C:

A B

C
Answer

Spot diagnosis.

A :Rt.EAC Exostoses

B :Lt.EAC Osteoma.

C :Lt.Aberrant carotid artery


269

Slide 56

Spot diagnosis .

A:

B:

C:

A B

Answer
C
Spot diagnosis .

A :Attic cholesteatoma

B : Congenital cholesteatoma

C : Advanced tympanosclerosis of the tympanic membrane and middle


ear
270

Slide 57

Name of instruments?

A:
A
B:

C:

D:
B

D
Answer

Name of instruments?

A:Luc’s forceps.

B:Mollison pillar retractor and tonsil dissector

C: Crurotomy knife tip.

D: Cottle nasal elevator


271

Slide 58
A A

Spot diagnosis .

A:

B:

C:

Answer

Spot diagnosis .

C
A:Torus mandibularis

B:Torus palatinus

C:Rhinophyma
272

Slide 59

Spot diagnosis

A: A
B:

C:

Answer

Spot diagnosis C
A:Bullous myringitis

B:Serous Otitis media with middle ear bubbles

C:Chronic adhesive OM with tympanosclerosis


273

Slide 60

Spot diagnosis

A:

B: A
C:

Answer

Spot diagnosis
C
A: Concha bullosa.

B: Paradoxical middle turbinate.

C: Nasal septal spur


274

Slide 61

Spot diagnosis

A:
A
B:

C:

Answer
B
Spot diagnosis

A:Microtia with atresia of the External Auditory Canal

B:Cauliflower ear

C:Chondroma of the Auricle

C
275

Slide 62
Spot diagnosis

A:

B:
A
C:

Answer C
Spot diagnosis

A:Acute invasive fungal sinusitis by Mucormycosis.

B:Sarcoidosis /Lupus Pernio.

C:Dry centeral perforation.


276

Slide 63
Spot diagnosis

A:
A
B:

C:

C
Answer

A:Atypical craniofacial clefts.

B:Pus expressed from Stenson’s duct that reflects an acute bacterial


parotitis.

C:The sialoendoscope placed into Stenson’s duct.


277

Slide 64

Spot diagnosis.

A: A
B:

C:

Answer

Spot diagnosis.
C
A:Nasal papilloma.

B:Aural polyp.

C:Transverse incision across the columella/Open rhinoplasty


278

Slide 65

Spot diagnosis A
A:

B:

C:

Answer
C
Spot diagnosis

A:Sialolith of Stenson’s duct.

B:Recurrent respiretory papilloma.

C:Artificial /electric larynx


279

Slide 66

Spot diagnosis

A:

B:
A
C:

Answer

Spot diagnosis
C
A:Dynamic splints for trismus.

B: Minor aphthous ulcer.

C:Erythroplakia.
280

Slide 67

Spot diagnosis

A:

B:
A
C:

C
Answer

Spot diagnosis

A:Herpes Zoster virus Infection

B:Lichen planus.

C:Cerebrospinal fluid otorrhoae


281

Slide 68

Spot diagnosis

1-Name of this signs? A


A:
B:
C:
2-commonlly find in ?

Answer

Spot diagnosis C
1-Name of this signs?
A:Battle Sign(Postauricular ecchymosis)
B: Raccoon sign(Periorbital ecchymosis)
C:Hemotympanum.

2-Commonlly find in? C


-Temporal bone fracture
282

Slide 69

1-Name of this triangle?

2-Contains?

3-Distance from antrum ?

Answer

1-Name of this triangle?


-MacEwen’s Triangle.

2-Contains?
- spine of Henle.

3-Distance from antrum?


- lies 12-15 mm deep to triangle
283

Slide 70

1-Name of this structure?

2-Name of label A-G.

AA B C

F A
G

Answer

1-Name of this structure?


-Internal auditory meatus.

2-Name of label A-G.


A-Facial n.
B-Bill’s bar
C-Superior vestibular n.
D-Transverse crest.
E-Inferior vestibular n.
F-Singular foramen.
G-Cochlear n.
284

Slide 71

1-Name of this device?

2-How this device work ?

Answer

1-Name of this device?


-Politzer Bag.

2-How this device work ?


-Rubber tube attached to a Politzer bag put into one nostril & both
nostrils pinched.
-Patient asked to swallow or repeat “k”.
-Politzer bag is squeezed simultaneously.
285

Slide 72

1-Name of this device ?

2-How this device work ?

Answer

1-Name of this device ?


-Eustachian tube catheter.

2-How this device work ?


-E.T. catheter passed along nasal floor till it touches posterior wall of
naso-pharynx.
-Catheter rotated 90° medially & pulled forward till it impinges on
posterior nasal septum.
-Catheter rotated 180° laterally, & its tip inserted into opening of E.T.
-Politzer bag attached to outer end of catheter.
286

Slide 73

1-Name of this device?


2-Uses of this device?

Answer

1-Name of this device?


-Pure tone audiometer.

2-Uses of this device?


- To find type of hearing loss.
- To find degree of hearing loss.
- For prescription of hearing aid.
-Predict hearing improvement after ear surgery.
- To predict speech reception threshold.
- A record for future medico-legal reference
287

Slide 74

1-Name of this incisions?

2-Commonlly used in which disease?

3-Most common complications of type A incision?

C
A B

Answer

1-Name of this incisions?


A-Trans-labyrinthine
B-Retro-sigmoid
C-Middle cranial fossa

2-Commonlly used in which disease?


-Acoustic neuroma.

3-Most common complications of type A incision?


-Complete loss of hearing,and short term vertigo.
288

Slide 75

1-Name of this device?


2- Components of this device?
3-Write the name of label A-D?

AElectrode arraye array


C

B
D
Z

e
array

Answer

1-Name of this device?


-External part of Cochlear Implant.

2- Components of this device?


-Microphone .
-External speech processor.
- Transmitter.
-Receiver.
-Electrode array.

3-Write the name of label A-D?


A:Transmitting coil
B:Magnet
C:Microphone
D:Speech processor
CHAPTER 3

OSCE
OSCE
Of EAR,NOSE,ThROAT,
HEAD & NECK SURGERY
290

Station 1

1-Name of this device:


3-Main componants:
2-Indications

Answer

1-Middle ear implant.

2-Candidates for the Envoy device implantation are


A-adults (≥18 years)
B-with a moderate-to-severe sensorineural hearing loss ,
C-mixed HL and
D-speech discrimination scores ≥60%.

3- A-internal surgically implanted part, the vibrating ossicular prosthesis


(VORP),
The VORP is made up of a receiving coil, conductor link and transducer.
The transducer has a small electromagnetic coil and magnet to
produce vibrations
B- an external audio processor.
291

Station 2
1.Describe the images

2. Provisional diagnosis.

3. Give 2 differntial diagnosis ?

Answer :

1-Coronal views of CT scan and MRI showing hyperdense soft tissue


mass

-on CT scan of ethmoid sinus and olfactery grove with low signal
intensity on MRI of ethmoid sinus.

2- Olfactery esthesioneuroblastoma.

3- Allergic fungall rhinosinusitis , sinonasal adenocarcinoma .


292

Station 3
1-Names of this instruments ?
2-Uses:
3-Complications from uses:

Answer

1- Weerda diverticoloscope , forcepes ,dilators.

2- Endoscopic stapling or diathermia for pharyngeal pouch ,foreign


body removal ,and biopsy .

3- Esophageal perforation ,bleeding ,infection ,injury to the


lips,teeth,palae and larynx
293

Station 4

40 years old patient presented with otitis externa 7 days back.presently


his pinna is extremely painful and appears as below.

A:What is your most probable diagnosis ?


B:How will you treat it?.
C: What is the complication likely to occur if left untreated.

Answer

A:Acute auricular perichondritis.

B: Treating of underling otitis externa and intravenous


antibiotics,drainage of pus collection with mastoid dressing .

C: Cartilage necrosis and coliflower ear.


294

Station 5

1-Define this picture ?.


2-most likely diagnosis ?
3-Surgical approach?
4-Incidence of malignent changes?

Answer

1-Axial T1 wieghted MRI with contrast demonstrate high signal


enhancement of a dumbbell mass involving Lf deep loop of parotid
gland with displacement of adjacent tissues and well demarkated
margins.

2-Pleomorphic adenoma.

3-Transcervical /trans-parotid approach.

4- 3-7 % from 5 to 15 years


295

Station 6

1-Desrcibe this picture?


2- Most likely diagnosis?
3-Signs confirm the diagnosis?
4-Test diffrentiat it form Otoseclerosis?

Answer

1-Axial CT scan bone widow of Rt temporal bone showing bony


dehiscence of Rt SSCC.

2-SSCD dehiscence
3-Signs confirm the diagnosis?
A-Low frequency CHL.
B- Stapedial reflex present.
C-Tullio phenomena .
D-Hennibert sign.
4- Acoustic reflex present in SSCD.
296

Station 7

A 60 yrs old patient with left nasal obstruction and bloody nasal
discharge with maxillary swelling and Lf orbital swelling and proptosis
of 6 month duration. From history and CT:

A-Most probable Diagnosis ?


B-Investigations to confirm the Diagnosis?
C-Management ?

Answer:

A-Most probable diagnosis : Left Maxillary Ca.


B-Investigations : CT,MRI,Biopsy
C-Treatment :T1 & T2 : Resection and post-op radiotherpy
T3 &T4 :surgery &post op chemoradiotherpy.
Others options
- Resection with maxillectomy either inferior , medial or total .
- if the orbit involved orbital exenterationdone.
- cranio-facial resection my needed.
297

Station 8

65 years old diabetic patient with history of 3 weeks of otalgia and ear
discharge,refractory to treatment .

1-Propable diagnosis?
2-Investigations?
3-Treatment?
4-Time to stop antibiotics?

Answer:

1- Malignant otitis externa.

2- CT scan of the temporal bone


Te 99/Gallium67/ESR.

3-Treatment.
Strong iv Antibiotics, depridment of granulation tissue,topical antibiotic
(Ciprofloxacillin aural drop)+/- steroid. For 6 weeks.treat facial palsy if
present

4: When pain stop , decrease ESR, normal Gallium scan.


`
298

Station 9

1-What is the study ?


2-Indications?
3-Contraindications?
4-Types of this study?
5-Most propaple diagnosis
from the this picture ?
6-Aetiology.

Answer

1- Barium swallow

2-To visualise oesophageal lumen,to diagnose pharyngeal


pouch,achalasia, stricture, Hiatus hernia and oesophageal tumour

3- Suspected perforation , corrosive injury, impacted FB ,aspiration


,absolute dysphagia.allergy to contrast.

4- Standard and modified Barium swallow

5-Pharyngeal pouch.

6- Mucosal herniation of posterior pharyngeal wall between


cricopharyngeus m. and thyropharyngeus m in weak area (Killian
dehecscent) due to incoordination of swallowing.
299

Station 10

Infant with feeding problems, recurrent aspiration, and stridor,since


birth.

1-Define this picture?


2-Most propaple diagnosis?
3-Grading system for classification?
4- What grade is this?
5- Clinical features?
6-Aetiology?
7-Treatment?

Answer

1-laryngoscopic photograph showing complet cricoid cartelage cleft


2- Laryngeal cleft
3-Benjamin-Inglis Classification
4-Grade III
5- Hoarse voice,cough, choking,, recurrent chest infection, feeding
problems, recurrent aspiration, and stridor,
6- Incomplete development or Failure of fusion of the
tracheoesophageal septum .
7. Surgical: according to the grading system
300

Station 11

12 years old boy presented with sore throat, fever,odenophagia with


upper cervical lymph
nodes enlargment .

1-Deffrential diagnosis?
2-What types of investigations are reguared?.
3-Most common Complications ?
4-Treatment:

Answer
1- Acute folicular tonsilitis,
-glandular fever.
-Vencent’s disease ,
-dephtheria

2 -FBC,monospot test, throat swab

3-Peritonsillar abscess,retropharyngeal abscess,


toxiemia,otitis media ,rheumatic fever, glomeriolonephritis.

4-Rest , hydration, Antibiotics, analgesic antipyretic and avoid sport


contact specially in infectious mononucleosis.
301

Station 12
4 year old child presents with history of hoarseness of 3 months
duration and stridor of 5 days duration. Laryngoscopy reveals the
following picture.

1- What is the most probable diagnosis ?


2-What is its cause?
3-How will you confirm the diagnosis?
4-What will you advice the mother after surgically treating the child?

Answer

1-Juvenil respiretory papillomatosis .

2-Human papilloma virus type 6, 11.

3- Microlaryngoscope and biopsy.

4-Its highly recurrent lesion,my needs Multiple operations


302

Station 13
A 35 year old malay woman presents with right sided tinnitus and
blocked feelig in that ear since 4 months. Of late she has developed
diplopia as shown below.

1-What is the most propable diagnosis?


2-Name 2 relevant investigations that will confirm the diagnosis?
3-What is the treatment of choice?

Answer

1-Nasopharyngeal carcinoma.

2- CT scan or MRI and biopsy.

3-Definitive Radiotherapy or CRT with neck control.


303

Station 14

1- Describe the photograph?


2- What abnormality show?
3- Causes of this deformity?
4- What type of surgery don for this patient?

Answer

1 - Old female with cosmetic deformity of the neck associated with


dropped Rt shoulder.

2 - Dropped Rt shoulder.

3- Excision of spinal accessory nerve.

4- Radical neck dissection


304

Station 15

1-Type of operation don in picture A?


2-Type of neck disection don in picture B?
3-Name of incision in A?
4-Name of incision in B?

A B

Answer

1 - Total laryngectomy
2 - Radical ND.
3- Gluck sorenson incision.
4-MacFee incision.
305

station 16

1-Most likely Diagnosis ?


2-Causes ?
3-Managements:

Answer

1-Fungal infection of soft palate. (oral candidiasis)

2-Diabetics ,Aids.Renal Failur.any immune defeciency ,drugs like


steroid,antibiotic, chemo-radiotherapy

3 -Local antifungal,treat the causes.oral hygein.


306

Station 17

1-Mention different types of hearing aids?


2-Name their components?
3-What is the meaning of acoustic gain?

Answer

1- BWHA,BTE,ITE,ITC,CITC .

2-Microphone ,Amplifier ,Receiver , Battery .

3-Result from relation between surface area of TM and footplate of


stapes its 14:1 and lever arm of malleus and incus its 1.3 :1
this give gain for middle ear about 20-35 dB
:
307

Station 18

1-Describe the image?


2-What is the most probable diagnosis?
3-What is the underlying condition?
4-What other diagnostic features?
5-What is the inheritance?
6-What is your line of management?

Answer

1- Axial T1 W MRI with gadollinum enhance- ment of both CPA &


IAM.showing enhancing lesion of both CPA & IAM

2- Bilateral Acoustic neuroma


3- NF 2
4- Posterior subcapsular cataract, first degree relative family member.
5-Autosomal dominant.
6-What is your line of management?
-1-conservative with serial MRI.
2-stereotactic radiosurgery.
3- surgical excision and brainstem implant
308

Station 19

1-Define.
2-Etiology :
3-Differential diagnosis?
4-Treatment :

Answer

1-Axial CT scan of temporal bone and picture of Rt EAC showing


multiple swelling with narowing of the ear canal.

2- Swimming in cold water,surfers

3-Exostosis ,ostioma

4-Treatment :
- if asymptomatic no needs treatment.
-if symptomatic -canalplasty
309

Station 20

1-Identify this instrument?


2-Name 2 indications?
3-Name 2 complications of the above surgery?

Answer

1- Breckmann adenoid curette.


2-Name 2 indications?
A- used to curette the adenoids
B-also used to remove tubal tonsils

3-Name 2 complications of the above surgery?


-Bleeding , velopharyngeal insufaciency
and injury the atlanto-occipital joint.
310

Station 21

1-Describe this CT scan?


2- Most likely diagnosis ?
3-What is the cause ?
4-What presenting symptoms ?(ENUMERATE 5).
5-Treatment.

Answer

1-Coronal CT scan of nose and paranasal sinusis demonsterate


,heterogenous opacity in Lt maxillary sinus and bony defect in floor of
the sinus at aleviolar side connected with
oral cavity
2-Oro-antral fistula.
3-What is the cause ?
-upper second premolar or first molar extraction,
-maxillary sinus tumor,
-sinus fungal infection,
-maxillary trauma and Caldwell luc operation.
4-Purulant discharge,bad oder,recurrent sinus infection ,facial pain,
tenderness
5-Caldwell luc operation,composite graft(mucosa and bone graft in
large one)

.
311

Station 22

1-Define this picture shon?


2-Indications:
3-Names of parts ?

Answer

1- Bone anchord hearing aide


2-Indications:
- Bilateral aural atresia ,
-Bilateral chronic discharging ear,
- Single sided deafness,
-Down’s syndrome,craniofacial anomalies
3-Names of parts ?
-Titanum scrow,
-Abutment,
-Sound processor
312

Station 23

1-Define this photograph ?


2-Indications :
3-Complications :

Answer
1 -Photograph showing Lt Caldwell luc operation
2-Indications :
-Recurrent antrocoanal polyp,
-Fungal ball (mycetoma) ,
-Foreign body,
-Biopsy ,
-Approachs : for maxillary artery ligation
, orbital decomprssion,dentigerous, cyst,
sphenoid sinus, and pterigopalatine fossa
3- Bleeding , infection , oroantral fistula ,
infrorbial vessles and nerve injury, tooth
injury.
313

Station 24

58 years old man presents with hoarse voice since 6 months duration
and long history of cigarate smoking.

1-What is the probable diagnosis?


2-Defferntial diagonsis :
3-Investigations :
4-Staging system :
5-If you confirm that this aggressive disease what your line of
management?

Answer
1-Rt vocal fold carcinoma.
2-:-Sguemous cell carcinoma of Rt VC ,
-Recurrent respiretory papilloma,
-laryngeal granuloma,
-TB laryngitis
3-CT scan & microlaryngoscopic biopsy
4-Tumore Node Metasteses .
5- -T1,T2 singel modality RT or Surgery.
-T3,T4:either surgery and p.o.RT or CRT
314

Station 25
9 years old boy presents with Rt eye swelling 7 days after upper
respiratory tract infection.
1-Most likely diagnosis from CT and picture shown below ?
2-Defferintial diagnosis:
3-What are the key features you would look for on examination?
4-Do you know any staging systems used for this condition?
5-How would you manage the patient?

Answer
1-Preseptal cellulitis due to acute ethmoid sinusitis
2-Skin &soft tissue infection,local trauma, Insect pit, dacrocystitis
3-Evidence of acute inflammation,purulant Secretion,colour vision,visual
acuity,eye Movement,proptosis,chemosis.pupillary reflex
4 -Chandler staging system
5-Admition ,IV ATB,decongestant,if abscess not present
Surgical drainage endoscopicly or modified Lynch Howarth
If abscess present.
315

Station 26

1-From this CT scan what is the most likely diagnosis ?


2-Investigations :
3-Management :

Answer

1-Lt submandibular duct stone.


2-Plain X-ray ,sialogram, ulrasound
, diagnostic sialoendoscopy,CTscan
3-Conservative:hydration , analgesic, sialogogues
-lithotripsy.( >7mm)
-Therapeutic sialoendoscopy.
-Surgical removal by widening of duct or excision of SMG.
316

Station 27

14 yrs old boy with unilateral nasal obstruction & sever bleeding.

1-Diagnosis :
2-Investigations:
3-Staging systems:
4-line of management :

Answer

1-Jovenil nasaopharyngeal angiofiberoma.

2- Imging(CT, MRI,Carotid angiography), endoscpy,hormonal

3- Anderw,Radkowski ,Fisch

4- Preoperative embolization and surgery


-approachs: endoscopic ,transpalatal
,external (lat.rhinotomy , mid-facial degloving)
317

Station 28

1-Describe ?
2-What is the diagnosis from this MRI :
3-Defferntial diagnosis:
4- What percentage of orbital hemangiomas are intraconal?

Answer
1-Coronal and axial T1 wieghted MRI with contrast shown the high
signal intensity of intraconal and optic nerve of the left orbit.

2- Orbital hemangioma

3-Schwannoma/neurogenic tumor,
-meningioma,
- leiomyoma,
- metastasis,
-optic nerve glioma,
- lymphoma,
- hemangioma .

4-80%
318

Station 29
45 yr old woman presented with Rt parotid swelling ,send for CT scan
shown this picture ,no facial weakness , no neck lymph node
enlargment.
1- Diagnosis:
2- Defferntial diagnosis :
3- What parotid cyst is associated with AIDS?
4- What other disease entity is associated with the same type of cyst?
5- Name the two most common cystic neoplasms of the parotid gland.

Answer

1-Parotid Cyst

2--Mucus retention cysts,


-First branchial cleft cysts,
-Sialocele,
-Lymphoepithelial cyst.

3-Lymphoepithelial cyst

4-Sjogren syndrome

5-Pleomorphic adenoma and Warthin tumors.


319

Station 30

10 yrs old girl present with mixed hearing loss and preauricular pit and
tag with cervical sinus and auricular malformation and recurrent renal
problems.

1- Most probable diagnosis:


2-Other ENT features :
3-Gentic type :

Answer

1-Branchio-oto-renal syndrome.

2- Dysplastic pinnae , CHL,SNHL or mixed ,mondini dysplasia,dilated


vestibular aqueducts , branchial fistula or cyst

3-Autosomal dominant disorder


320

Station 31

1- What is the term used for complete agenesis of the inner ear
structures?

2- Name the common congenital condition associated with inadequate


spiralization of the cochlea.

3- Is the hearing loss in a Mondini malformation usually gradual or


sudden?

Answer

1--Michel deformity.

2-- Enlarged vestibular aqueduct syndrome.

3-- Sudden.
321

Station 32

1- What is the differential diagnosis for this lesion?


2- Identify the mechanism of development?

Answer

1-Cholesterol granuloma, mucocele, aneurysm with clot, hemorrhagic


metastasis, and epidermoid

2-Hemorrhagic foreign body reaction.


322

Station 33

1-Which sign is show in this photograph.


2- Name of this syndrome.
3-Charactastic features .
4- What is character of SNHL .
5-How is inherited .

Answer

1--Hetrochromia irides.

2-Waardenburg syndrome.

3--White forelock ,dystopia canthorum ,SNHL ,skin pigmentary changes


,Hirschsprung disease

4--Non-progressive SNHL.
-Unilateral 70%.
-Bilateral 30%.

5-Autosomal dominant.
323

Station 34

1-Define this photograph ?


2-Type of hearing loss?
3-Indications :
4-Name of parts of device :
5-Factors that are known to affect outcome.

Answer

1-- Rt ear showing cochlear implant device.


2-- Sever to profound SNHL
3--Sever to profound bilateral SNHL.
-Congenital hearing loss in child less than 4 years.
- postlingual deafness in adults or older children

4--External:microphon, speech processor, transmiter


-Internal: receivor,electrod array

5--Age , Duration of deafness , Age of onset of severe-to-profound


acquired deafness , Parental expectations , Available appropriate
education options , Aetiology of deafness,
324

Station 35

1-Decribe?
2-Diagnosis?
3-Clinical features?
4-Treatment :

Answer

1--Multible small punctate telangiectases of lip and tongue


2--Hereditary hemorrhagic telangiectasia (Osler-Weber-Rendu disease
3-- Epistaxis ,GIT bleeding,hemoptysis,CNS
complication,cardiac&hepatic failure.
4--Conservative ,medical(oesrogen or progesteron) tranexamic acid
avastin,oestrogen gel
- surgical (laser,septodermoplasty,modified young’s procedure).
325

Station 36

1-Define :
2-Defferntial diagnosis :
3-Complications :
4-Management :

Answer

1--Lateral plain X-ray of soft tissue of neck ,showing extended


prevertibral soft tissue and loss of loardoses of cevical vertibre,with well
defined anterior margin, and anteriorly displace airway.

2--Retropharyngeal abscess ,postcricoid ca. TB ,lymphoma ,

3--Airway obstruction, parapharyngeal abscess septecemia,


mediastinitis, aspitation peumonea,

4--Admission.
-Secure airway.
-I.V. AB. Fluid. Analgesia. Antipyretic.
-Surgical drainage ( trans oral or external cervical approach).
326

Station 37

1- What is the differential diagnosis?


2- What percentage of polyps are antrochoanal?
3- What is the characteristic shape?
4- What would solid nodular enhancement imply?

Answer

1--Antrochoanal polyp , Mucocele, inverted papilloma, and,fungall


sinusitis.

2-- 3% to 6% of all polyps

3-Dumbbell , African contenant

4-More likely a neoplasm.


327

Station 38

1-Describe :
2-Diagnosis:
3-Causative agent:
4-Treatment:

Answer

1-- Skin papule and left cervical lymphadenopathy

2--Cat scratch

3--Bartonella henselae

4-- Resolve within 2-6 months.


328

Station 39

1-Define this photograph:


2- Type of hearing loss :
3- Grading systems:
4-Associated conditions :
5- Line of management :

Answer

1--Rt auricular hypoplasia with tag

2--Conductive hearing loss.

3--Marx,Weerda,Jaharsdorfer grading systems.

4--Goldenhar,Treacher-colin ,CHARGE ,Apert,Branchio-oto- renal


synderomes.

5-- If bilateral treat hearing first(soft band BCHA then BAHA).


reconstruction in 6 yr. unilateral later on.
329

Station 40

1-Differential diagnosis of this lesion shown below ?.


2-Most common presenting symptoms ?
3-Best diagnostic image ?

Answer

1--High jugular bulb


-Glomus jugulare.
- AOM
-EAC trauma.
-Vascular malformation.
2-Pulsitil tinnitus.
-Conductive deafness.
-Aural fullness.
3-High resolution CT scan of temporal bone.
330

Station 41

Patient with moderate SNHL after minor head trauma rapidly progress
to profound SNHL.do CT scan and showing this pathology.

1- Diagnosis :
2- What is the upper limit of size allowable
3- How commonly is this finding encountered bilaterally?

Answer

1-Enlarged vestibular aqueduct .

2-1.5 mm.

3-90 %

Notes:
Large vestibular equduct:It is seen in isolation, as part of the Mondini
malformation, and in patients with branchio-otorenal syndrome and
Pendred syndrome.
It is well shown on high resolution CT imaging of the temporal bone.
331

Station 42

1-Diagnosis :
2-Clinical features :
3-Genetic type :
4-Name of device behind the ear :
5-Type of ear reconstruction :

Answer

1--Treacher colin syndrome

2--Antimongaloid palpepral fissure coloboma,maxillary and mandibular


hypoplasia,microsia ,atresia and conductive hearing loss,

3--Autosomal domenant

4--Bone anchord hearing aids(BAHA).

5--Bone anchured auricular prosthesis.


332

Station 43

28 year old school teacher presents with hoarseness of voice of 7


months duration and laryngeal examination shown this picture .

1-What is the most probable diagnosis ?


2-Name the surgical procedure of choice in treating this patient ?
3-What will you advice the patient to prevent its recurrence ?

Answer

1-Vocal nodule.

2-Microlaryngoscopy with precise excision under GA.

3--Speech therapy , vocal hygiene , voice rest. ?


333

Station 44
Prof.Dr.Ismail Zohdi

A 42 -year-old anaemic female complains of intermittent


dysphagia limited to solids & felt in the throat

a. Determine the site of the lesion in the radiograph


b.Describe the endoscopic finding.
c. What is your provisional diagnosis
d. Name three manifestations of this disease
e.What options do you have for the treatment of such a case?

Answer

a. Postcricoid , upper oesophagus


b. Thin web is detected
C-Plummer Vinson syndrome
d. -Angular cheilitis
- Glossitis
- Koilonychia, spleenomegaly

e. - Iron replacement
- Endoscopic dilatation
- ND:YAG laser therapy?
334

Station 45

A)Describe above image?


B)What is diagnosis?
C)Give 2 possible microorganism?
D)What type of management and for how long?

Answer

A) - Otoendoscopic picture showing white cotton like lesion with black


spots in EAC.

B) - Otomycosis.

C) - Candida albicans,
-aspergilus nigor.

D) -1-Aural toilet .
2- Topical antifungal e.g clotrimazol.
3- Analgesic .
4-Avoid water exposure.
- For two weeks
335

Station 46

1-Describe?
2- Eitiology?
3-Name of this sign?

Answer

1-The crease over the bridge of the nose created by repeated rubbing.

2-An external clinical sign in allergic rhinitis.

3- A nasal salute in allergic rhinitis.


336

Station 47

1-Most likely diagnosis?


2-Defferntial diagnosis?
3-Treatment?

Answer
1-Antrochoanal polyp.
2 --Angiofiberoma.
-Thornwldat cyst.
-Anurysem of internal carotid artery.
-Dermoid cyst.
-Pharyngeal abscess.
3-Surgery is always indicated for this type of polyp because it does not
respond to medical treatment.
If you cannot reach the root of the polyp in the maxillary antrum
endoscopically using the 45 or 30o endoscope the treatment is surgical
excision by caldwel luc operation
337

Station 48

A 50 yr old male presented with a growth of lower alveolus ,as shown.


Neck exam.was normal.rest of ENT exam. was normal .

1-What 2 immediate important investigations will you do?


2- what is the treatment of choice?

Answer

1-
a-CT scan for oral cavity,neck and chest.
b-Biopsy

2-
Excision with partial mandibulectomy, neck dissection and
reconstruction of defect.
338

Station 49

1-Describe finding in this picture?


2-What is pathophysiology of diagnosis ?
3- Line of management ?
4-Types of approachs ?

Answer

1- Frontal view of photograph showing broad nasal dorsum with midline


punctum and most likely due to infected dermoid cyst.

2-Frontal nasal inclusion cysts of tracts related to adefect of the


anterior embryological neuropore..

3-Surgical excision.

4--A-external rhinoplasty.
B-lateral rhinotomy.
C-transglapellar subcranial approach.
D-craniotomy in case of intrcranial connection?
339

Station 50

55 yr old laborer is refered to you with throat pain.your laryngeal exam.


Is presented in this clinicl photograph.

1-Describe the lesion?


2- Typically how do patient with this type of lesion presented?
3-List 2 risk factors?
4- Name the staging system currently in use?
5- List 2 investigation you would perform?
6-List 2 management strategies?

Answer

1-Proliferative growth involving right true vocal cord.


2-Hoarse voice.
3-Smoking , alcohol consumption.
4-Tumor,node,metastesis(TNM).
5- -CT scan of neck.
-microlaryngoscopy and biopsy.
6-Radiotherapy and surgery
340

Station 51
Prof.Dr.Ismail Zohdi

The picture show fiberoptic laryngoscopy view of 36 yr old female , she


had presened with hoarseness for 6 months perform in sitting postion
.vocal cord found mobile.

1- Describe the lesion?


2-What more information you want from the patient?
3-Whats your Dx?
4-Whats your management ?

Answer

1-Proliferativ lesion with irregular surface involving Lf posterior vocal


cord and Lf arytenoid

2-Any history of intubation or freign body.

3-Laryngeal granuloma of Lf arytenoid and vocal cord

4--Medical therapy (Cough prevention and treatment , antireflux tt.)


Speech therapy
Botox injection
Microlaryngeal surgery (with/without laser)?
341

Station 52
Prof.Dr.Ismail Zohdi
A 38 year old lady complains of slowly progressive dysphagia with
retrosternal discomfort

a. What abnormality is revealed on this radiograph?


b.What is your provisional diagnosis?
c. What is the causative pathology?
d. What complications may arise?
e. Mention options to treat such cases?

Answer

a. Dilatation of the lower part of the oesophagus


b. Achalasia
c. Lack of nonadrenergic, noncholinergic, inhibitory ganglion cells,
causing an imbalance in excitatory and inhibitory neurotransmission
d. Weight loss,
Aspiration,
Oesophagitis,malignant changes
e. Graded pneumatic dilatation
Botulinum toxin therapy
Laparoscopic Heller myotomy
342

Station 53
Prof.Dr.Ismail Zohdi

Male patient had undergone a right parotidectomy for a pleomorphic


adenoma
a. Describe his parotid region.after the patient was asked to chew a
lemon slice for 5 minutes
b. What is the most probable diagnosis of this case?
c. What is it’s pathophysiology ?
d. What test can confirm your diagnosis?
e. Management include ?

Answer

a. Scar of parotidectomy incision & flushed parotid region


b. Frey’s syndrome
c. Post-ganglionic secreto-motor para-sympathetic fibres(auriculo-
temporal nerve) which normally innervate the parotid gland, become
connected to sympathetic receptors which innervate sweat glands

d. Minor starch-iodine test.

e. Management include:
Anticholinergics ,topical anti-perspirant
Botox injection Temporal fascia grafting
, tympanic neurectomy
343

Station 54
-This patient its 14 yr old boy presented with Rt side nasal block and
recurrent epistaxis
.patient underwent imaging studies.films are displayed.

1-Please comment on images displayed ?


2-What is your diagnosis?
3-What is the stage and which classification?
4-What is your further management? What approach is recommended?
5- State 2 complications associated with your approach ?

Answer

1-Axial CT with contrast enhancing lesion in post.nasal cavity, adjacent


nasopharynx and sphenoid sinus minimal extension to PPF…. Rt carotid
angiogram showing region of increase vascularity With tumor
blush.bl.supply from internal maxillary A.\SP artery.
2--Juvenil nasopharyngeal angiofibroma
3-Stage 1A Anderw classification
4-Embolization with surgical excision\endoscopic \transpalatal\external
5-Endoscopic: excessive bleeding,orbital complication.
transplatal: excessive bleeding,platal fistula.
external :excessiv bleeding, poor scar,affect facial growth.
344

Station 55

1-Name of this flap?


2-Commonly use for?
3-What is the object of this approach?

Answer

1-Langen beck's osteoplastic flap.

2--In angiofibroma.

3--To hinge the maxilla on the lateral aspect of the nose without
interfering with the alveolar and palatine tissues or the floor of the
orbit.
345

Station 56

1-Define this intraoperative photograph?

2-What is the pathology shown?

3-Others indications for this approach ?

Answer

1-Midfacial degloving approach.

2-Intraosseous venous malformation affecting Lf maxilla

3-Angiofibroma ,inverted papilloma ,ca.maxilla, fracturs maxilla,


sinonasal tumors.
346

Sation 57
Prof.Dr.Ismail Zohdi

1.Name this shown procedure


2. The gauge needle used is Nr.
3.Wet smears are placed in 95% ethyl alcohol and treated with which
stain?
4. Diagnostic sensitivity of the test is defined as:
5. The specificity of a test is defined by :

Answer

1-Fine needle aspiration

2-21-gauge needle

3-Papanicolaou technique and stain

4-The sensitivity of a test is defined by:


true positive/(true positive + false negative)

5-Diagnostic specificity of the test is defined as:Probability of a negative


diagnostic test in the absence of disease
347

Station 58

1-Name of this flap?


2-Type of the flap?
3-Blood supply :
4-Diadvandages :
5-Advantiges:
6-Give 2 Complications?

Answer
1-Pectoralis major flap .
2- Myocutaneous flap
3-Pectoral branch of the thoracoacromial artery
4-Diadvandages :
-bulky flap,
-Relatively immobile
-Violates breast in females
-Pedicle rests on the clavicle which may cause decrease in
vascularity
5-Advantiges:
-easy positioning,
-doner site primery closure,
-high vascularity.,
-large skin territry..,
-transferred with out delay..,
-harvestid in supine position.
6-Give 2 Complications?
-Hematoma and infections
348

Station 59

This 15 yr old patient presented with painless Lf neck swelling since 5


yr CT scan was done as shown?

1- What are the possible Dx ?


2-What is treatment of choice ?

Answer

1-a- Brancheal cyst.


b-Upper cervical LAP.

2-Treatment of infection and surgical excision.


349

Station 60

This child was seen in emergency department in the evening and you
were called to see him ,the mother give history of fever,URTI,followed
by pain in ear,decrease hearing for few days but today got worse .

1-What is your Dx?


2-What are your next line of management?
3- Name 6 complication if left untreated?

Answer

1-Acute mastoiditis.

2-
1- Admition and IV antibiotics.
2-CT of the mastoid.
3-If fluctuant then incision and drainage under GA.
4- Cortical mastoidectomy if complications occure or failure to resolve.

3---Meningitis
- Facial palsy.
- Brain absaese..
- Bezold,citelli,luc’s absaeses.
- CSOM.
-Post aural fistula
350

Station 61

A 42 -year-old male complains of hoarseness and a low-pitched pressed


voice quality with frequent throat clearing.

1. Determine the site of the laryngeal lesion shown by flexible


laryngoscopy
2. What is your provisional diagnosis
3. What are the primary causes of such a condition?
4. How would you manage this case?

Answer

1-Left vocal process.

2-Vocal process (contact) granuloma

3-Intubation
Voice abuse
Laryngopharyngeal reflux disease

4-Medical therapy (Cough prevention and treatment , antireflux tt.)


Speech therapy
Botox injection
Microlaryngeal surgery (with/without laser)
351

Station 62
Prof.Dr.Ismail Zohdi

A 51-year-old male complains of persistant sore throat and ear ache


since 1 year Both vocal cords are freely mobile. No palpable neck
lymph nodes were Detected ,Biopsy report confirmed the diagnosis to
be squamous cell carcinoma

1-Describe the site of the lesion?


2.Determine it’s TNM classification?
3. What options do you have for treating this case?

Answer

1-Supraglottic, epiglottis and ventricular band

2-T2 N0 Mx

3--Radiotherapy (larynx & neck)


- Supraglottic laryngectomy & bilateral
selective (lateral) neck dissection
- Laser supraglottic laryngectomy & bilateral
selective (lateral) neck dissection
352

Station 63
A 40 yr old male presents with hearing impairment,tinnitus in the Rt
ear,with dizziness you perform the tests below :

1- What is the type and configuration of the hearing loss seen in


figure 1?
2- What is the name of the test and findings seen in the figure 2 ?
3- What is the diagnosis?

2
1

Answer

1- Sensorineural hearing loss\low frequency\up rising

2-Electrocochleography\ECochG.
SP\AP ratio > 0.5

3-Hydrops \ Meniere’s disaese


353

Station 64
Prof.Dr.Ismail Zohdi

A 14 year-old boy complains of unilateral nasal obstruction & unilateral


recurrent; usually severe epistaxis that necessitates medical attention

1. What is your provisional diagnosis?


2. What is the characteristic CT picture?
3.Name a specific investigation you would ask for?

Answer

1-Juvenile nasopharyngeal angiofibroma

2-Erosion of roof of medial ptyregoid plate ,Widening of pterygopalatine


fissure ,Invasion of pterygopalatine fossa, anterior Bowing of
posterior maxillary wall.

3-Carotid angiography
354

Station 65
Prof.Dr.Ismail Zohdi

A 46 year-old patient complains of sudden dramatic onset of


progressive proptosis after FESS

1. What is your provisional diagnosis?


2. What is the cause of such a complication?
3.How would you manage this case?

Answer

1. Post-operative orbital haematoma

2-Anterior or posterior ethmoid artery severed

3-Canthotomy and cantholysis promptly performed Orbital


decompression (external or endoscopic)
Nasal packing is removed
Ophthalmologic consultation
355

Station 66

1.Type of image?
2.Give 3 symptoms?
3.Types of treatment?

Answer

1-PET CT scan.

2. -Persistent sor throat


-Odenophagia.
-Refered otalgia

3. -Treatment depend on stage of disease:


stage I&II: single modality (radiotherapy or surgery)
stage III&IV: multimodality (surgery & postoperative RT or CRT).
356

Station 67

1-Most likely diagnosis?


2-Most common cause?
3-Name of stain used in this picture?
4-How you can confirm the diagnosis?
5-A negative test result exclude a diagnosis of CSF leak?

Answer

1-Cerebrospinal fluid rhinorrhae.

2-Trauma/surgical or accidental

3-Fluorescein-stained.

4-B2-transferrin. When present, a CSF leak is confirmed

5-A negative test result exclude a diagnosis of CSF leak.


- no
357

Station 68

1-Define?
2-Clinical features :
3-Management:

Answer

1-Photograph showing perforation and slight bleeding of TM my be due


to traumatic perforation.

2-Pain , deafness, tinnitus,dizziness , slight bleeding

3- Observation,avoid water contamination, (80%) mostly healed


spontenously within 4-6 wks.
-My needs analgesic
358

Station 69

1-Define this picture ?


2-Name of this sign ?
3-Name of the sign when the mass blanch during pneumatic otoscope
examination
4-Origin of this pathology?
5-Most common symptom ?
6-Treatment:

Answer

1-Red mass with fluid level behind intact TM.


2-Rising sun sign.
3-Brown’s sign.
4- From paraganglia on the promontory located along the course of
Jacobson’s and Arnold’s nerves
5- Pulsatile tinnitus.
-Conductive hearing loss.
-Some time asymptomatic and incidentally discovered on physical
examination
6-Surgery, stereotactic radiation therapy, combination therapy,
observation
359

Station 70

1.Provisional diagnosis?
2.Name of the operation ?
3- Types of reconstrction ?

Answer

1-Rt pyriform sinus tumor.

2-Partial or total pharyngolaryngectomy & bilateral lateral neck


dissection.

3-Pectoralis major myocutaneos flap,


- Radial forearm free flap,
- Jejunal free flap,
- Gastric pull up,
360

Station 71

1.Describe the image?


2. Type of intervention ?
3-Presenting symptoms ?

Answer

1-Attic erosion and cholesteatoma.

2-Tympanomastoid exploration with eradication of disease &


reconstruction of the defect.

3-Conductive deafness.
-Scanty offensive purulant discharge.
-Tinnitus.
-Vertigo.
361

Station 72

1-Describe ?
2-Commonlly seen in which cases?

Answer

1-Flat type B tympanograms.

2-Otitis media with effusion.


-Space occupying lesions of tympanic cavity.
-Tympanic membrane perforations.
362

Station 73

1-Most probable diagnosis?


2-Clinical features?
3-Treatment ?
4-Name of the device in second picture ?

Answer

1-Otitis media with effusion.

2-Hearing loss ,aural fullness,tinnitus.

3--Watchfull watting for 3 monthes.


-Hearing aids.
-Myringotomy and Grommate insersion if
bilateral and persist.

4- Grommet ventilation tube


363

Station 74
20 years old patient presents with otalgia of 4 days duration and facial
weakness of 24 hours.

1-Most likely diagnosis?


2-Eitiology?
3-Grading system used?
4-Treatment?

Answer

1-Ramsay Hant syndrome.

2-Herpis zoster virus.

3-House-Brackmann grading system.

4-Eye care:tape closed at naight,artificial teer and tarsorraphy


-Analgesia.
-Systemic Steroid tapering dose.
-Systemic Antiviral/aciclovir.
364

Station 75

25 yrs old female with prevous history of tinnitus and hearing loss do
ear surgery after 9 months presents with this picture.

1-Most likely diagnosis?


2-Name of the operation ?
3-Name of the device shown?
4-Common complications of this surgery?

Answer

1-Otoseclerosis
2-Stapedectomy.
3-Stapes piston prosthesis.
4- Extruded metal piston in the external auditory canal.
-Facial nerve injury.
-Failure of air-bone gab narowing.
-Tinnitus.
-Vertigo
-Reparative granuloma.
365

Station 76

4 yrs old child presents with history of unilateral purulent nasal


discharge with bad odor and nasal obstruction.

1-Most probable diagnosis?


2-Common age of presentation?
3-Management?

Answer

1-Foreign body in the Lt nasal cavity.

2--2-4 years

3-Remove with out aensthesia if anterior and cooperative child or under


GA and airway secure,to avoid risk of inhalation in incooperative child
,small ,soft surface and posterior.
366

Station 77

1-Most likely diagnosis?


2-Name of this test?
3-Treatment?

Answer

1-Congenital dacryostenosis.

2-Lengthened fluorescein drainag time on the right side.

3-Dacryocystorhinostomy (DCR(
367

Station 78
65 yr old woman presents with neck swelling of 3 yr duration its
Firm,painless noncompressible mass .

1-Site of swelling ?
2-Most probable diagnosis ?
3-Describe image below ?
4-How can you confirm the diagnosis?
5-Treatment?

Answer

1-Lf parotid area .

2-Pleomorphic adenoma .

3-Axial T1 wieghted MRI with contrast shows tumor in the left parotid
area.

4-FNA-Biopsy.

5-Surgical (parotidectomy) with facial nerve preservation.


368

Station 79
55 yr old female with history of prograssive Lf side hearing loss with
tinnitus and vertigo of 7 yrs duration send for imaging and shown like
this picture.

1-Most likely diagnosis ?


2-Defferntial diagnosis ?
3-How you can confirm the diagnosis?
4-Nonsurgical treatment is preferred in which situations :

Answer

1-Acoustic neuroma.

2-Meningioma ,epidermoid cyst,archnoid cyst,cholesterol


granuloma,lipoma.

3-PTA ,T1 W MRI with contrast ,ABR.

4-Small tumors in elderly patients.


-Tumor in the only hearing ear.
-Tumor in the patient with poor general condition.
369

Station 80
45 years old woman presented with bilateral
salivary glands swelling with dryness of mouth
and eyes.CT scan shown this picturs.

1-Define this CT scan ?


2-Most probable diagnosis?
3-What is the best imaging modality for diagnosis?
4-How you can confirm the diagnosis?
5-What is the classical charactrastic features?

Answer

1-Multiple tiny bilateral parotid cysts are seen on postcontrast


axial CT imaging of the neck.
2-Sjogren syndrome.
3- MRI with MR sialography.
4- With minor salivary gland biopsy
-Detection of autoantibodies SS-A and SS-B
5- Parotid enlargement, xerostomia , keratoconjunctivitis sicca,
370

Station 81

1-Name of this device?


2-The Aims of this device?

Answer

1-Intraoperative facial nerve monitoring.

2-Localization of the facial nerve (mapping).

-Identification of the facial nerve.

-Minimization of neural trauma.-

-Confirmation of the functional integrity of the nerve.

-Assessment of neural status and prediction of post-operative


facial nerve function.
371

Station 82
Endoscopic photograph and axial CT scan for nose and PNS for
neoborn baby with respiretory destress shown this picture.

1-Most likely diagnosis ?


2-What is the incidence of this ?
3-What is the initial treatment?
4-How you can confirm the diagnosis?
5-What is the time for definitive surgical repair?

Answer

1-Bilateral choanal atresia.

2-Incidence 1:5000-8000 births.

3- Treated initially with McGovern's nipple.

4-Endoscopy and CT confirm the diagnosis.

5- Performed during the first weeks or months of life.


372

Station 83

Nasoendoscopic picture shown this lesion involving Lt nasal cavity


causing complete obstruction, rhinorrhea, and unilateral epistaxis .for 2
years duration.

1-Most likely diagnosis?


2-Other names for this lesion?
3-Common associated eitiology?
4-Treatment?

Answer

1-Inverted papilloma.

2- Schneiderian papilloma and Ringertz tumor.

3-There is an association with human papilloma virus (HPV).

4- Total excision of the tumor. approach :lateral rhinotomy or


midfacial degloving with medial maxillectomy for total tumor
removal. or endoscopic modified Lothrop
373

Station 84

12 years old girl presented with this finding and multiple fracturs now
complaining of hearing impairment.

1-What could be the diagnosis?


2-What investigations?
3-What management?

Answer

1-Osteogenesis imperfecta \van dur hoave synderom

2- CT ,PTA,tympanometry,

3-Medical therapy with calcitonin, sodium fluoride, and vitamin D is


unclear.
-Surgical intervention with stapedectomy to improve conductive hearing
loss
374

Station 85

1-Describe?
2- Diagnosis ?
3-Causes of bilateral vocal cord paralysis?
4-Most diagnostic image?
5-Management?

Answer

1-Sagital T1 wieghted MRI showning herniation of the cerebellar tonsils


through the foramen Magnum of the skull
2- Arnold–Chiari malformation.
3-Pressure injury on the vagus nerves may result in bilateral vocal cord
paralysis
4-Diagnosed by MRI brain
5 - Early evaluation and neurosurgical decompression are most often
recommended

Notes:
Classical cause of congenital bilateral vocal cord palsy is hydrocephalus
with the Arnold-Chiari malformation.
375

Station 86

1-Define this CT scan of the neck ?


2-Diagnosis from CT scan?
3-How the patient with this pathology present?
4-How you can investigate this patient?
5-Treatment?

Answer

1-Axial CT scan showing air-filled dilasion of the saccule extending


through the thyrohyoid membrane into the neck..
2-Laryngocel.
3- Asympyomatic ,or hoarseness,, dysphonia, ,stridor,,sor throat.
- my present as acute airway obstruction,,or infected neck mass
or neck swelling.
4- Plain x ray during valsalva manoeuver,
US ,CT scan,laryngoscopy.
5- Observation,, endoscopic resection,,
endoscopic marsupialization or external cervical resection.
376

Station 87

1-Define:
2-Diagnosis?
3-Pathogenesis?
4-Common clinical presents?
5-Treatment?

Answer
1- Barium swallow (nagetive picture)showing a herniation of pharyngeal
mucosa through a defect in posterior pharyngeal wall.

2-Pharyngeal pouch.
3-Herniation of pharyngeal mucosa through a weak area in posterior
pharyngeal wall (Killian
dehiscence) between cricopharyngeal fiber and thyropharyngeal fiber of
inferior constructor muscle. due to in coordination of swallowing.

4-Dysphagia ,regurgitation of undigestid food ,halitosis , weight loss


,aspiration.
5- Watchful waiting .
- Endoscopic stapling.
- Endoscopic laser.
- External:Cricopharyngeal myotomy,,
diverticulectomy ,,diverticulopexy,
inversion.
377

Station 88

1-Define this picture for 15 yrs old young male. presented with
bilateral nasal obstruction and epistaxis with sever headache since
2 yrs.
2-Diagnosis ?
3-How the pterygopalatine fossa communicates laterally with the
masticator space ?

Answer

1-Axial Ct scan of nose and PNS shown extensive expansion of the


tumour,with anterior bowing of the posterior antral wall may be seen
and widdening of pterygomaxillary fissure ,erosion of medial pterygoid
plate with extension to Rt nasal cavity and sphenoid sinus.

2-Juvenile nasopharyngeal angiofibroma.

3-via the pterygomaxillary fissure.


378

Station 89

1-Describe ?
2-What is the best image for evaluation?
3-What is the immediate indication for tracheostomy?

Answer

1-Axial CT scan at level of the larynx demons- terate a Moderately


displaced laryngeal fracture with emphesema and endotracheal tube in
place.

2-CT scan.

3- A stridulous patient with respiratory distress at presentation.


379

Station 90

1-Describe?
2-Diagnosis ?
3-What is the endoscopic and clinical diagnostic features ?
4-What is pathophysiology of diagnosis ?
5-What is histological diagnosis?

Answer
1-Coronal CT scan demonstrates an extensive
cribriform plate/ethmoid roof defect and soft tissue herniation.

2- Meningoencephalocele.

3- Visible pulsations on endoscopy and enlargement of the mass with


ipsilateral internal jugular vein compression (Furstenberg’s sign) help to
confirm the diagnosis.

4- Result from failure to close embryologic spaces enclosing the


anterior neuropore.

5- Histologically an ependymal lined sac that communicates with the


CSF spaces.
380

Station 91

1-Name of the sign in picture A?


2-Causes of A ?
3-Name of the sign in picture B ?
4-Mention possible audiological findings in pictures A and B ?

B
A

Answer

1-Battle sign

2- Which is a postauricular ecchymosis resulting from extravasated


blood from the postauricular artery or mastoid emissary vein.

3-Haemotympanum.

4-Conductive in longtudinal fracture ,and SNHL if transverse fracture or


mixed
381

Station 92

1- Name of this graph ?


2-What is the type in early stapes fixation?
3-What is the mechanism of action?
4-What is the type in the presence of a severe or profound hearing
loss.

Answer

1-Acoustic Reflexes .

2- On-off effect or diphasic.

3- Acoustic reflex occurs when the stapedius muscle contracts in


reaction to a loud sound.

4-Absent
382

Station 93

3 months old infant complaining of stridor and poor feeding.

1-Most likely diagnosis ?


2-What is the sign present at birth?
3-What is the indications of intervention in severe cases?
4-most common cause of stridor in infancy?

Answer

1- Laryngomalacia .

2- Usually have no sign of respiratory abnormality at birth.

3--A-Airway obstruction resulting in apnea


and/or cyanosis .
B- weight loss from poor feeding.
C-Pulmonary hypertension and cor pulmonale
D-Severe chest deformity.

4-Laryngomalacia
383

Station 94

1-Site of lesion?
2- Differntial diagnosis ?
3-Name of usual pathogen ?
4-Management ?

Answer

1- Peritonsillar space

2-Quinzy,parapharyngeal abscess, Ca.tonsil ,

3- GABHS

4-Incision and drainage under cover of antibiotics and local


anaesthesia
384

Station 95
Femal patient 32 yr old complaining of bilateral nasal
obstruction, excessive crustation with past history of previous
nasal operation 2 yr ago, endoscopic examination shown:

1-Most likely diagnosis?


2- Line of treatment ?
3-Mension one operation used?

Answer

1-Atrophic rhinitis

2-Nasal saline douche,25% glucose in glycerin drops, nutritional


support, treatment of underlying local or systemic disease.

3- Modified young operation.


385

Station 96
A 65-year-old woman presents with an ulcerated lesion of the left upper
gum measuring 3 ×2 cm. A staging CT scan shows erosion of the bone
of the upper alveolus with extension into the maxillary sinus. No
enlarged cervical lymph nodes were present.

1-Site of lesion ?
2-Most common histopathology ?
3-What 2 immediate important investigations will you do?
4-What is the TNM staging?

Answer

1-Oral cavity /Lt upper gum.

2-SCC of upper alveolus.

3-A-CT scan of oral cavity,neck and chest.


B-Biopsy.

4- T4a No Mx
386

Station 97

6 years old girl present with midline painless swelling.

1-Most likely diagnosis ?


2-Most common cause?
3-What is the tests used before surgery?
4-Name of operation don for this disease?
5-How to prevent recurrence?

Answer

1-Thyroglossal duct cysts.


2-failure of oblitration of thyroglossal duct in 7-10 weeks of gestation
during emberiological development of decened thyroid gland to the
neck.
3-Radio-iodin uptake and U\S for neck to be sure its not the only
functioning thyroid tissue.
4-Sistrunk operation.
5-Removal of the cyst with cuff of normal tissue and follow the track
till the hyoid and remove its middle part then continue till base of
tongue.

-avoid aspiration and incision and drainage of the cyct.


387

Station 98

infant present with neck mass that are characterized histologically by


dense fibrous tissue and the absence of normal striated muscle.

1-Most probable diagnosis?


2-Typical presentation ?
3-What is the course of this disorder?
4-Treatment ?

Answer

1-Sternocleidomastoid tumors.
2-Firm, painless, discrete masses within the sternocleidomastoid
muscle.
3-Slowly increase in size for 2–3 months and then regress for 4–8
months.
4-Treatment ?
-Eighty percent of cases resolve spontaneously
-physical therapy to prevent restrictive torticollis.
-Surgical resection is reserved for persistent cases
388

Station 99
1-Diagnosis ?
2-What should be assessed in examination of the eyes?
3-What imaging may be raquired:
4-How this condition graded ?
5-Treatment options?

Answer

1-Orbital abscess.

2-Colour vision,visual acuity,eye movement,pupillary reflex.

3-CT scan for nose ,PNS and brain,axial sections through orbit to check
for an abscess

4-Chandler’s grading system.

5-Treatment options?
-medical :IV antibiotics if no abscess present.
-Surgical if abscess present by drainage with open approach or
endoscopic
389

Station 100

1-Describe?
2-Differntial diagnosis ?
3-What the diagnosis if the patient woodworker?
4-Most common malignant neoplasm of the nose and paranasalsinuses?
5-Treatment ?

Answer

1- Coronal T2 wieghted MRI of nose and paranasal sinuses,showing


high signalintensity lesion of heterogenous opacity of Lt maxillary sinus
and isosignal intensity of Lt nasal cavity.

2-Inverted papilloma or sinonasal malignancy, allergic fungal


rhinosinusitis

3-Adenocarcinomas.
4-Sqeumous cell carcinoma accounting for 60–80%.

5-Treatment ?
-is surgical resection followed by radiation therapy in T1,T2 and CRT
inadvance.
390

Station 101

1-Site of lesion?
2-Define ?
3-Diagnosis ?
4-Name of this sign shown in this picture?
5-Primary treatment modality ?

Answer

1-Lower esophageal sphenctor and gastroesophageal junction.

2-Dilated esophageal body and poorly relaxing LES.

3-Achalasia.

4-Fluoroscopic esophagram showing the '"bird'sbeak" appearance.

5-Laparoscopic Heller myotomy and partial fundoplication


391

Station 102

1-Name of this device ?


2-Gives two indications ?
3-Gives two approaches ?

Answer

1-Auditory brainstem implants.

2-Neurofibromatosis II (NF2)
-Cannot perform a cochlear implant (Michel deformity, cochlear nerve
agenesis, and ossified cochlea).

3- Translabyrinthine (especially when removing a vestibular


schwannoma)
-Retrosigmoid
392

Station 103

1-Site of the lesion ?


2-Most common presenting symptoms ?
3-What is the staging system ?
4-Indications for postoperative radiation ?

Answer

1-Glottic /Lesion spreading on both vocal cords and anterior


commissure.

2-Hoarsness of voice .

3-Tumor Node Metasteses /TNM staging system .

4- Include T4 primary, bone/cartilage invasion, extension into soft


tissue of the neck, perineural invasion, vascular invasion, multiple
positive nodes, nodal extracapsular extension, margins less than 5 mm,
positive margins, carcinoma in situ at margins, and subglottic extension
of primary tumor.
393

Station 104

1-Type of the approach ?


2-Name of incision ?
3-Indications ?

Answer

1-Right maxillary swing approach.

2-Weber fergueson incision.

3- Recurrent Nasopharyngeal carcinoma.


- Angiofiberoma (JNA).
-ca.maxilla
394

Station 105

Patien presented after he hit the dash board of the car

1-What is your line of management?


2-What is the treatment if presented with sever stridore or respiretory
distress?
3-How you can evaluate this patient ?
4-What classification system used in this case ?

Answer

1-- 24-Hour admission to an acute care unit


-- Regular observations
-- Serial flexible nasolaryngoscopy
-- Humidified oxygen
-- PPI therapy\anti-reflex
-- Systemic corticosteroids
-- Prophylactic antibiotic
2- Immediate tracheostomy under local anaesthesia.

3-Fiberoptic laryngoscpe and CT scan to evaluate the laryngeal trauma

4-Schaefer classification of laryngeal trauma


395

Station 106

1-Describe ?
2-What is the clinical features ?
3-Common type of fracture in this area?
4-Direction of trauma in this type of fracture?

Answer

1-Axial CT scan bone window of Lt temporal bone shown transverse


fracture.

2-Headache,SNHL,tinnitus,dizziness,facial palsy,CSF otorrhoea.


Pneumolabyrinyh,

3-Longitudinal 80%.

4-Either occipital or frontal trauma.


396

Station 107

65 yr old female presents with Rt parotid swelling since 3 yrs,painless


with Slowlly growning mass?

1-Most probable diagnosis ?


2-Whats investigations you do to confirm the diagnosis?
3-Treatment ?
4-Complications of surgery ?

Answer

1-Pleomorphic adenoma

2-FNA cytology ,Ultrasound and MRI

3- Parotidectomy with facial n.presevation

4- Bleeding ,infection ,salivary fistula ,hematoma ,seroma, Frey’s


syndorme ,facial palsy,cosmatic defect and numbness of Lobule
(great auriclar n. injury)
397

Station 108

1-What is the CT findings ?


2-Most probable diagnosis ?
3-How you confirm the diagnosis ?
4-Treatment ?

Answer

1-Coronal CT scan of nose and PNS demonsterate a metallic or calcified


densities within an opacified sinus cavity .

2-Fungus ball or mycetoma.

3-Mycologic and histologic studies are essential to confirm the


diagnosis.

4-Treatment ?
-Surgery
398

Station 109

1-Name of this device ?


2-Common indication ?
3-Commonlly used with ?
4-Commonlly used under which guidance ?

Answer

1-Stone-extractor wire basket.

2-Submandibular duct stone.

3-Sialoendoscopy.

4- Wire basket extraction under radiologic guidance


399

Station 110

1- Define ?
2-Whats sensitivity and specificity of this scan ?
3-Incidence of accuracy of this scan ?

Answer

1-CT and sestamibi fusion image in coronal and sagittal planes shows
ectopic superior parathyroid adenoma located superior to the thyroid
lobe.

2-Sensitivity 85-90 % and specificity nearly 100 %.

3- Sestamibi scans are wrong at least 50% of the time even at the best
places in the world.
400

Station 111

1-Describe ?
2-Most probable diagnosis ?
3-Charactrastic features of this lesion ?

Answer

1-Axial CT scan bone window shown multple broad base bony swelling
in Rt EAC .

2- Exostoses of EAC .

3- Usually multiple and bilateral and are often found near the annulus
of the tympanic membrane.
401

Station 112

Laryngoscopic picture of patient with long history of stay in intensive


care unit with mechanical ventilation.

1-Define?
2-Most common causes ?
3-Name of classification system ?
4-What is the normal subglottic lumen diameter in the full-term
neonate?

Answer

1- Laryngoscopic picture showing subglottic stenosis.

2--Acquired and congenital.


-acquired:prolong intubation ,cuffed trachiostomy tube, FB reaction..
-acquired its common.

3-Cotton-Myer classification.

4-- 4.5 to 5.5 mm


402

Station 113

1-Describe ?
2- Differential diagnosis ?
3-Most likely diagnosis ?
4- What syndrome is this entity linked to?

Answer

1-Axial CT scan bone window of Rt temporal bone shown bony erosion


of posterior petrous bone and growth involve the endolymphatic sac.

2-Cholesterol granuloma, metastasis, glomus tumor, and epidermoid


.endolymphatic sac tumore,congenital cholesteatoma.

3- Papillary adenocarcinoma of the endolymphatic sac

4-Von Hippel–Lindau disease.


403

Station 114

35 years old female presents with chronic cough and hoarse voice for 8
months duration ,no neck lymph node enlargement and other wise
healthy weman .laryngoscopic photograph shown this picture.

1-Most probable diagnosis ?


2- Cell of origin ?
3-Classic location ?
4- What is the treatment ?

Answer

1-Laryngeal Granular cell tumor

2-Schwann cell.

3- Posterior true vocal folds, but also can be found in supraglottis and
subglottis.

4-Complete surgical excision


404

Station 115

30 Yr old male asthmatic presented with throat pain and hoarse voice .

1-Differntial diagnosis ?
2-Most probable Diagnosis ?
3-Treatment:

Answer

1- Fungal laryngitis,TB,GERD,chronic laryngitis

2-Fungal laryngitis

3-Treatment: antifungal and stoop steroid for asthma


405

Station 116

1-Name of this flap ?


2-Blood supply ?
3-Advantages ?
4-Disadvantages ?

Answer

1-Jejunal free flap.

2- Is based on superior mesenteric artery.

3- Large segment that can be used for anastomosis as high as the


nasopharynx
-Intrinsic mucous production may assist in swallowing
-Tolerates postoperative RT.
Lower mortality/morbidity rates compared with gastric pull-up -
. -
4- Short ischemia time.
-Dysphagia from jejunum peristalsis
-Poor vocal rehabilitation from TEP prosthesis
- Morbidity from second incision in the abdomen
- Need for two microvascular anastomotic sites
406

Station 117

10 years boy complaining gradual deterioration in vision and hearing


loss .

1-From history and pictuers what is the diagnosis ?


2-What is the type of inheritance ?
3-How you confirm the diagnosis ?
4-What factors distinct three subtypes clinically?

Answer

1-Usher syndrome .

2-Autosomal recessive.

3-Diagnosed with electroretinography; requires ophthalmology


consultation.

4- Which differ with severity or progression of hearing loss, onset and


severity of retinal degeneration, and manifestation of vestibular
dysfunction
407

Station 118
Bronchoscopic photograph for neoborn baby after birth with Cyanosis
during nursing and respiretory distress shown this picture .

1-Most likely diagnosis ?


2-Most common type?
3- Management?

Answer

1-Tracheo-oesophageal fistula(TEF)

2- Esophageal atresia (EA) and distal TEF (85%).

3- Management?
• Immediate gastrostomy tube placement
• Surgical correction at 3 months of age
• May require dilation of esophageal strictures
• Risk of recurrent laryngeal nerve injury
408

Station 119

19 years old male with long history of sickle cell anaemia presented
with righ check swelling:

1-Describe image ?
2-Give 2 differential diagnosis ?

Answer

1-Axial CT scan of nose and PNS showing hyperdense lesion involving


the Rt maxillary bone, aleviolar bone with classic ground glass
apperance

2-Fibrous displasia.
-Ossified fibroma.
409

Station 120

1-Mention five systemic causes of the below deformity?


2-Operation that result in this deformity?
3-Other causes related to trauma?
4-How to treat such case?

Answer

1-Syphilis, T.B , Wagner’s granuloma, Sarcoidosis , Relapsing


polychondritis

2-SMR or Rhinoplasty (excessive dehumbing)

3-Untreted septal hematoma

4- Rhinoplasty
410

Station 121
65 years old patient with carcinoma of the left anterior buccal mucosa.

1-Define ?
2-What type of therapy used?
3-What is indications for this therapy?
4-Contraindication for this therapy?

Answer

1-Plastic tube technique for iridium-192 implant to a carcinoma of the


anterior buccal mucosa

2-Brachytherapy.

3-Ca.of buccal mucosa.


-Recurrent nasopharyngeal Ca.
-Recurrent tongue base Ca.
-lip Ca.

4- Lesions in close proximity to the mandible .


411

Station 122

A 30- year- old woman presents with gradual- onset bilateral hearing
loss with normal tympanic membranes.

1- Describe the findings on this audiogram?


2- What is the most likely diagnosis?
3- List four treatment options?

Answer

1-Right conductive hearing loss with Carhart’s notch (narrowing of air-


bone gap at 2000Hz)

2-Otosclerosis.

3- List four treatment options


-Observation
-Fluoride
-Hearing aid
-Stapedectomy
412

Station 123

Photograph of 20 years old Patient 3 days post tonsillectomy afebrile


with good oral intake.

1-What is this white colour in the tonsillar fossa?


2- What makes you think of this?
3-If patient present with bleeding what is the probable causes?

Answer

1- Post tonsillectomy healing membrane.

2- No fever or signs of infection and good oral intake

3-According to amount either sloughing of membrane or secondary


post tonsillectomy bleeding mostly due to infection.
413

Station 124

1- What is the diagnosis?


2- List two presenting symptoms?
3-List two medical treatments?
4-If this is a child list 3 causes?
5- List three main complications of surgical removal

Answer

1-Nasal polyposis.

2-Nasal obstruction , Anosmia

3-Intranasal steroids
-Systemic steroids

4- Cystic Fibrosis, Kartagner syndrom, Imunosuppresion

5-Bleeding
-Risk to vision
-CSF leak
414

Station 125

1-Diagnosis ?
2-Complications ?
3-How you can manage the airwayobstruction ?

Answer

1-Ludwig angina.

2-Parapharyngeal abscess,
-airway compromise,
-septicemia, carotid blu out,
-mediastinitis,death.

3-Nasotracheal intubation.
-Tracheostomy.
415

Station 126

1-Mention the diagnosis ?


2-What the defects you may find in this case?
3-Name the operation needed to repair that ear?
4-Two complications apart from pain and infection:

Answer

1-Bat ears.

2-Loss of antihelical fold and increased distance between the


helix and mastoid

3- Otoplasty (e.g. Mustarde suture)

4-Assymetry, Auricular hematoma , perichondritis


416

Station 127

1. Provisional Diagnosis ?
2. Indication of surgical intervention?
3.Instruction given to patient post operativelly.
4.Principle of surgical.

Answer

1-Reinks oedema.

2-Failure to responde to conservative medical treatment


, or symptomatic patient.

3-Give up smoking, voice rest, speech therapy.

4-Microlaryngoscopic excision UGA via laterally placed incision over


superior surface of VC, removal of gelatinous material , trimming of
redundant mucosa & replacement of mucosal flap
417

Station 128

1-What does this three pictures show?


2- What would be your immediate concern?
3- Do you know of any grading systems for microtia?
4- Do you know of any associated conditions?

Answer

1-Microtia.

2- Likely association with significant hearing loss

3- Several different authors have described grading systems including


Marx, Weerda and Aguilar, and Jahrsdoerfer. They are broadly similar.

4- Treacher Collins syndrome


-Goldenhar syndrome/hemifacial microsomia
-Trisomy 18 (Edwards syndrome)
418

Station 129

1-Please describe the abnormalities shown in this scan?


2-What is the likely underlying diagnosis?
3-What are the other hallmark features of this and what is the usual
age of onset?

Answer

1-T1 MRI (probably post-gadolinium contrast) showing bilateral


enhancing masses in CPA with extension into IAM (more so on the
right) and meningioma in Rt side. Likely bilateral vestibular
schwannomas.

2-Neurofibromatosis Type 2.

3-Bilateral vestibular schwannomas (diagnostic for NF2), other cranial


nerve schwannomas and meningiomas, posterior subcapsular lenticular
opacities (cataracts), spinal tumours, skin tumours/lesions.
419

Station 130

1- Identify the primary disease ?


2- What its secondary complication?
3- Where is MacEwen’s triangle, and what is its importance?

Answer

1-Otomastoiditis

2-Sigmoid sinus thrombosis.

3-A small triangular depression at the junction of the posterior and


superior borders of the externalauditory canal, posterior to the
suprameatal spine; this is the site of surgical entry to the mastoid
antrum,
420

Station 131

1-What are the finding ?


2-Most common presenting symptoms ?
3-What is the surgical treatment of this condition ?

Answer

1-Deviated nasal septum.

2-Nasal obstruction,
-anosmia or hyposmia ,
- postnasal drip
- epistaxis.

3-Septoplasty
421

Station 132

1- Most probable diagnosis ?


2-List 3 type?
3-List 4 features consistent with Dx ?
4-Name 2 histological staging systems :
5- Best modality for treatment?

Answer

1-Malignant melanoma.
2-Nodular.
-Lentigo maligna
-superficial spreading
3-ABCD :
-Asymmetry ,
-Border irregular ,
-Colour variasion ,
-Diameter grater than 6 mm.
4-Clark and Breslow.
5- Best modality for treatment?
Surgery
422

Station 133

Indian man presented with progressive unilateral nasal obstruction with


bloody discharge.

1-What is diagnosis ?
2-Causative agent ?

Answer

1-Rhinosporidosis

2-Rhinosporidium seebiri
423

Station 134

1-What’s the sign of malignant nodes in CT scan ?


2-What is the signs of extra capsular spread ?

Answer
1---Rounded.
-Greater Than 1.5 Centimeter in juglo
digastric area.
-Greater than 1 Centimeter elsewhere.
-Central necrosis: Regardless of size.
-Peripheral enhancement
-Extracapsular spread: regardless of size.

2---Spiculated margins.
-Fatty invasion.
-Encasement of vessels.
424

Station 135

1- Name of this test?


2-Indications ?
3-Limitations?
4- Can be used in patients with tympanostomy tubes?
5-Aim of this test?

Answer

1-Otoacoustic emissions test.

2- Neonatal hearing screen ,


- Validation of normal hearing ,
-early, rapid test after meningitis.
-Ototoxicity ,noise induce HL,non organic HL.

3- Identifies normal hearing only , Relatively low specificity , May be


technically difficult in a crying infant , can fail with wax.

4- Yes.

5- Are objective, noninvasive, and rapid measures (typically less than 2


minutes) used to determine cochlear outer hair cell function
425

Station 136

1- Name of this test ?


2-Its required active patient cooperation?
3-Indications ?
4-Aim of this test?
5-Advantages?

Answer

1-Auditory brainstem response (ABR).

2-No.

3- Screening for an acoustic neuroma.


- Children who failed auditory screening.
- Suspected malingerers.
- Coma. Intraoperative monitoring.
-Asymmetric or unilateral auditory process
such as hearing loss, vestibularweakness, or tinnitus .

4- Evaluates the function of the auditory nerve pathway

5- Noninvasive, reliability on repetitive testing,


and not affected by sedatives or drugs
426

Station 137

Infant presented with bilateral nasal swelling and


obstruction,compressible and enlarged with crying.

1-Most probable diagnosis?


2- 2 Differential diagnosis?
3-What this in the mouth and why ?
4-How you can differentiate clinically?
5-Treatment ?

Answer

1- Encephaloceles .
2-Dermoid.
-Glioma.
3- McGoverun nipple in the mouth,duto bila.nasal obstruction
.
4-Pulsatile, bluish compressible lesions that transilluminate and
Furstenberg sign positive

5-Treatment ?
- Is surgical excision :Small cranial bone defect and smaller lesions
may be treated endoscopically.

-Larger lesions often require a combined craniotomy approach


427

Station 138

1-Describe image (c) ?


2-Give 2 differential diagnosis ?
3-Define picture (B) ?
4-Treatment options?

Answer

1-Coronal CT scan bone window of Rt temporal bone showing


soft tissue density lesion involving the Rt middle ear cleft.

2--Congenital cholesteatoma.
-Acquierd cholesteatoma .

3-Intraoperative photograph of middle ear with elevated eardrum


demonstrating ossicles and cholesteatoma.

4-Treatment options?
-surgical.
- transcanal if small and limited to the middle ear.
- tympano-mastoidectomy .
428

Station 139

55 years old female with Rt neck dissection was performed in a


metastatic cervical lymphadenopathy from carcinoma of larynx,drain
put and its continue discharge of white material

1-What is likely diagnosis ?


2- What is your line of management ?
3-What is the effective method for controlling the leak?
4-What is incidence of this complication in neck dissection ?

Answer

1-Chylous leak .
2- Conservative (medium-chain fatty acid diet,parantral feeding
,pressure dressing)
- Surgical exploration with ligation of the stump may be necessary.
- Accelerate radiotherapy.
3- Thoracoscopic ligation of the thoracic duct.
4-- 1 - 2%.
429

Station 140

1-What modalities of imaging used?


2-What is your finding ?
3-What is the diagnosis ?
4-What the most accepted theory of etiology?

Answer

1-T2 wieghted MRI .

2- High signal intensity lesion involving sella /mostly CSF .

3-Empty sella syndrome.

4-Weak diaphramatic sella leading to CSF herniation.


- Benign intracranial hypertension.
430

Station 141

You are consulted from neurology department for this patient.

1-What is the relevant pathology ?


2-What could be the cause ?
3-Is their any prevention method ?
4-Whats your treatment ?

Answer

1-Labrynthitis ossificance.

2-Postmeningitis .

3-Streptoccocal vaccine.

4-Cochlear implant as soon as possible.


431

Station 142

1- What is this investigation?


2- What abnormalities are demonstrated?
3- Name 3 intracranial complications ?

Answer

1-Axial CT scan soft tissue at level of the orbit.

2-Proptosis .
-Subperiosteal abcess.
-Preseptal cellulitis.
-lateral displacement of medial rectus muscle.
-Rt ethmoid sinusitis.

3-Meningitis.
-Cavernous sinus thrombosis.
-Brain abcess.
432

Station 143

1-Name of this operation?


2- Name of instruments in the photographs?
3-Indications?

Answer

1-Treacheotomy.

2- Molleson hemosteatic self retuning mastoid retractor


-tracheal dilator(trousseau).
- 2 Langenbeck retractors.

3- Acute airway obsrtuction,


-Sever laryngeal trauma,
-Bilateral VC paralysis,
-long term ventilation support,
-lung toilet.
433

Station 144

1-Name of this flap ?


2-Blood supply ?
3-Advantages?
4-Disadvantages ?

Answer

1-Latissimus dorsi myocutaneous flap.


2-Thoracodorsal artery.
3-Advantages?
-Large flap with long pedicle.
-Second largest skin paddle.
-possibility of axillary megaflap.
-Can reach upper face and scalp.
-multiple skin paddle.

4-Disadvantages ?
-Difficult positioning.
-two team harvest.
-Bulky flap.
-Donor site wound dehiscence.
-Reduction in upper limb power.
434

Station 145

1-Describe the lesion in this photograph?


2-Most probable diagnosis ?
3- 2 Differntial diagnosis?
4-Treatment ?

Answer

1- Ulcerated, circumscribed lesion with elevated or rolled margins


and a keratinized central region.

2-keratoacanthoma.

3-Basal cell ca.


-Sqeumous cell ca.

4-Treatment ?
- which is a benign self-limiting lesion that often regresses without
intervention.
435

Station 146

1-Site of the lesion ?


2-Most likely diagnosis ?
3-Staging system ?
4-Management ?

Answer

1-Unilateral (Rt side)arise from the olfactory epithelium superiorto the


middle turbinate.extended to the Rt medial orbital wall
.
2- Olfactory esthesioneuroblastomas .

3-No TNM staging system,its staged by Kadish to 4 groups.

4-Management ?
-combined modality therapy:surgery and postop.RT.
436

Station 147

1. Describe the findings?


2- Provisional diagnosis ?

Answer

1-Normal wave shape & latency.

2- Normal auditory brain responses, traced to 20 dB hearing level


437

Station 148

Young male presented with Rt facial palsy 10 days after head trauma.

1-describe image above ?


2-What’s the most important question in the
history taken?
3-What’s important one investigation used ?
4-How can above investigation help you in
management ?

Answer

1-Axial CT scan bone window of temporal bone demonsterate


transverse fracture lineThrough the bony labyrinth of Rt temporal bone

2- Duration , onset of facial palsy and its partial or complete at time of


trauma.

3-Electroneurography .

4- If the degeneration more than 90% it candidate for exploration, but


if its less than 90% its chance more than 10% for recovery and treat as
bells palsy.
438

Station 149

4 yrs old boy presents with otalgia,hearing loss,irritability, poor feeding


and fever for 5 days after URTI .

1-Most likely diagnosis ?


2-How you can confirm the diagnosis?
3-Treatment?

Answer

1-Acute otitis media .

2-Clinical diagnosis by otoscopic finding,cultur and sensitivity if


perforation present.

3-Treatment?
-Analgesic ,antipyretic,antibiotic if less than 2 years.
-myringotomy with or without Grommet insertion if persist and
sever pain or complication
439

Station 150

1-Describe this photograph for child with 3 years?


2-Diagnosis?
3-Investigations?
4-What is surgical approach?
5-How the child presents ?

Answer

1 - Photograph of child with sinus/fistula in the neck above the hyoid


and below mandibule.

2- Type II 1st branchial cleft sinus.

3-Sinugram/MRI/CT.

4-Parotidectomy appraoch for facial N protection and tract excision.

5 -Presentation:periauricular sinus or abscess ,EAC swelling or


discharge.abscess or sinus opening below the mandibule,above the
hyoid bone
440

Station 151

25 years old patient complaining of Rt painful/tender neck mass,


odynophagia, dysphagia ,trismus, dyspnea. and fever.

1-Describe the image?


2-Most likely diagnosis ?
3-Most common cause ?
4-What is the arrows indicate ?
5-Ultrasound its effective in this case ?

Answer
1-Axial CT scan soft tissue window shown Features of ring
enhancement around a hypodense center in Rt parapharyngeal space
with displace lateral pharyngeal wall medially and normal Lf PPS
(arows).

2-Parapharyngeal abscess.

3-Pharyngitis and dental infections.

4-Indicate normal left parapharyngeal space.

5-May be a more effective means of distinguishing an abscess from


cellulitis.
441

Station 152

Photograph of infant came with this lesion in Rt upper lip and nostrile
its absent at birth but appears during infancy.

1-Diagnosis ?
2-Other 2 differntial diagnosis ?
3-What is the most appropriate imaging modality for evaluation?
4-Management ?

Answer

1-Heamangioma of infancy.

2-Congenital heamangioma .
-Vascular malformation.

3-MRI .

4-Management ?
- Observation.
- Propranolol.
-Oral or intralesional steroids.
-Surgical excision: cold knife or laser; ideally best to wait until after
the proliferation stage
442

Station 153
Adult male presents with Rt neck swelling during valsalva
manoeuvre,other wise normal person.

1-Most likely diagnosis ?


2-Origon of this pathology?
3-What type ist?
4- Compression may empty air and fluid into the larynx what name of
this sign ?

Answer

1-Laryngocele.

2-Abnormal herniation or dilation of laryngeal saccule.

3-External type.

4- Bryce's sign
443

Station 154

65 years old female presents with Lt neck swelling below Lt auricle


since 5 years ago with slowlly growning painless nontender mass.

1-Differential diagnosis ?
2-Signs of malignancy in this area?
3-If its benign whats treatment?

Answer

1-Salivary :
-Pleomorphic adenoma,Warthine tumor, mucoepidermoid ca.adenoid
cystic ca.
-Nonsalivary :maseter hypertrophy or heamangioma , upper cervical
LAP,brancheal cyst,carotid body tumor, lymphoma.

2-Painfull swelling.
-facial palsy.
-cervical LAP.
-skin involve.

3-Complete surgical excision of the tumor with uninvolved margins is


the recommended treatment ,with facial nerve preservation.
444

Station 155

1-Name of this procedure ?


2- Advantages ?
3- Disadvantages ?
4- Indications ?

Answer

1-Type I Thyroplasty.

2-Performed with local anesthesia with minimal or no discomfort to the


patient.
- Long lasting
- Reversible.

3-Open procedure
-Technically more difficult
-Closure of the posterior glottis may be limited

4-a. Permanent vocal fold paralysis (>12 months from insult,


nervesacrifice, and EMG findings)

b. Vocal fold bowing because of aging or cricothyroid joint fixation


c. Sulcus vocalis
d. Soft tissue defects from excision of pathologic tissue
445

Station 156

1-Whats the arrow indicated ?


2-What’s indications ?
3-Complicaions ?

Answer

1-Anteriorly positioned tympanostomy tube.

2- Persistence OME.
-Recurrent AOM.
-Severe retracted tympanic dram.
-Instelation of medications.

3-Recurrent infections .
-Persistent perforation.
-Extrosion.
-Granuloma formation.
-Dislogement.
-Blocked
446

Station 157

Endoscopic view of a left nasal cavity of 30 yrs old young male with
previous history of endoscopic nasal surgery since 9 yrs ago due to
recurrent unilateral nasal bleeding with obstruction.

1-Most likely diagnosis ?


2-Most important image for evaluation ?
3-Incidence of recurrence ?
4-Surgical approach for this case?

Answer

1-Recurrent NP angiofibroma.

2-MRI

3-6-36 %.

4-Endoscopic approach with preoperative embolization


447

Station 158

1-Describe the finding in this audiogram ?


2-Most likely diagnosis ?
3-Genetic type?
4-In earlier stages of disease whats stapedial reflex show?
5-Management options?

Answer

1-Moderate conductive hearing loss with Carhart notch of Rt ear.

2-Otoseclerosis.

3-Autosomal dominant disease of incomplete penetrance

4-Overcompliance, followed by under compliance (“on–off ” effect).

5-Observation
-Sodium Fluoride
-Hearing aid
-Stapedotomy
448

Station 159

40 yrs old woman Presents with Hoarse voice, Weak cough 7 days after
cold.

1-Define this laryngoscopic picture?


2-Most likely diagnosis?
3-Other clinical features?
4- 2 important investigations?

Answer

1-Failure of medialization of Lt vocal cord and anterior diaplacement of


Lt arytenoid with short and atrophy of Lt vocal fold.

2-Left vocal fold palsy.

3-Breathy voice , Difficulty in breathing, Pain, Choking with liquids.

4-Chest x-ray.
-CT scan skull base to mediastinum.
449

Station 160
45 years old woman with 2 years history of slowlly growning mass in Rt
upper neck ,compressable and pulsitil ,MRI and angiography don.

1-Most likely diagnosis ?


2-Name of sign appears in angiography?
3-What is the incidence reported to be familial
4-Write the name of structures labelled by arrows ?

Answer

1-Carotid body tumor

2-Lyre’s sign

3- 10 %

4-Internal and external carotid artery


450

Station 161

1-Name of the procedure?


2-Principles for this procedure?
3-Type of incision in this technique ?

Answer

1-Submucous septoplasty.

2- Principles: mobilisation, reorientation and fixation.

3-Cottle incision.
451

Station 162

1-Type of this image?


2-Define ?
3-What type of surgery indicated in this patient?

Answer

1-Ct scan of the nose and paranasal sinuses.

2-Coronal CT scan of the paranasal sinuses shows mucosal swelling in


relatively large ethmoid bullae on both sides, and mucosal swelling in
both maxillary sinuses. With bilateral concha bullosa.

3-Functional endoscopic sinus surgery(FESS).


452

Station 163

1-Charactrastic feature of picture A:


2-Charactrastic feature of picture B:
3-Most likely diagnosis?
4-Type of inheretance?

A B

Answer

1- Heterochromia iridis.

2- White forlock and distiopia canthorum.

3- Waardenberge syndrome.

4- Autosomal domninant.
453

Station 164

35 years old patient with history of recurrent sinusitis and purulant


discharge after first molar extraction.

1-Most likely diagnosis?


2-Common surgery indicated for this pathology?
3-Others 3 eitiology for this pathology?

Answer

1-Oroantral fistula.

2-Caldwell luc operation.

3-Trauma ,surgery like caldwell luc ,maxillary sinus tumor


454

Station 165

35 years old male complaining of left upper neck swelling , tender with
high fever with left tonsil pushed medially with moflid voice.

1-Most likely diagnosis?


2-Most common cause of this infection?
3- The most common signs and symptoms?
4-Imaging of choice for diagnosis?

Answer

1-Infection in the parapharyngeal space.

2- Pharyngitis and dental infections .

3-Painful/tender neck mass and fever.


- odynophagia, dysphagia
- trismus, and dyspnea.

4-CT scans with intravenous contrast have become the cornerstone of


diagnosis.
455

Station 166

Laryngoscopic photograph of neonate with sever respiratory distress


and stridor since birth.

1-Most propable diagnosis?


2-Whats grading system?
3-Whats grading of this case?
4-Line of management ?

Answer

1-Subglottic stenosis.
2-Cotton –Myer grading system.
3-Grad III (71-99 %).
4/a-Endoscopic approach
• Cold knife lysis
• Laser excision
• Dilation (balloon dilation is the most common)
• Stents.
b-Open approaches:
• Patch tracheoplasty: rib cartilage and pericardium
• Segmental resection with primary anastomosis
• Anterior wedge resection
• Tracheal autograft
• Slide tracheoplasty
• Transplant (autograft or allograft)
456

Station 167

65 years old woman presened with a nonhealing ,bleeding lesion in Rt


lower lid close inspection reveals superficial telangiectasias that blanch
on palpation.

1-Most propable diagnosis ?


2-Most common site of location ?
3-Name 3 types of this lesion?
4-Risk factors for this lesion ?
5-Treatment ?

Answer

1-Basal cell carcinoma.

2-Inner canthus.

3-Nodular , superficial and pigmented.

4-Sun exposur.
-Immunosuppression.
-Prior injury.
-Genetic diseases.

5-Treatment ?
-Surgical excision with safity margin.
457

Station 168

1-Name of this test ?


2-What the diagnosis in graph A ?
3-What the diagnosis in graph B ?
4-Whats the amplitude reflecting activity of the hair cells?
5-Whats the amplitude reflecting whole nerve activity?
6-Others diseases causes this abnormality?

Answer

1-Electrocochleography.

2- Normal ear.

3- Meniere's disease.

4- The summating potential.

5- The compound action potential.

6- Perilymph fistula , autoimmune inner ear disease, and superior


semicircular canal dehiscence.
458

Station 169

30 years old patient with history of old nasal surgery presents with
recurrent nasal bleeding and crustation .

1-Most likely diagnosis ?


2-Others most common causes ?
3-Most common presenting features?
4-What tests would you do ?
5-Lines of treatment ?

Answer

1-Nasal septal perforation.


2-Nasal trauma(nose picking) or surgery.
-Bilateral electrical cautery.
-Systemic diseases like Wegner granuloma, sarcoid,syphilis.
-Cocaine abuse
3-Asymptomatic ,Crustation ,bleeding ,nasal obstruction and whistling.
4-Blood tests(ESR,cANCA,ACE) ,chest X-ray ,CT scan .
5-Line of treatment ?
-Conservative:-Nasal saline irrigation.
-Lubrication with antibiotic ointement.
-25% glucose in glyserin.
-Surgical :Silastic button ,mucoperichonderial rotational flap or
composit flap(cartilage grafts harvested from pinna)
459

Station 170

Femal Patient presents with thickens pinna and collapses with saddle-
nose deformity and hoarseness of voice since 6 months.

1-Most propable diagnosis?


2-Etiology ?
3-Commonlly affected areas?
4- Imaging can be helpful in the diagnosis?
5-Management ?

Answer

1-Relapsing polychondritis.
2-Unknown etiology. that causes episodic inflammation of cartilaginous
structures throughout the body.
3- Ears, nose, eyes, larynx, bronchi, costal cartilages, and articular
joints.
4-Yes.
5-Management ?
-Medical : managed with high-dose corticosteroids. Maintenance
therapy consists of methotrexate and low-dose corticosteroids.
Azathioprine, cyclophosphamide, cyclosporine, and dapsone have
been tried in refractory cases.
- Surgical measures.
460

Station 171
Dr.sajad Al-helo

1-Name 3 differntial diagnosis of swelling in this area ?


2-Define the CT scan?
3-Most common likely diagnosis ?

Answer

1-Ethmoid mucocel.
-Dermoid cyst.
-Sebeceous cyst

2-Sagital Ct scan of the nose and paranasal sinuses showning cystic


lesion with hypodense center in the soft tissue of the inner canthus,
with intact anterior frontal plate.

3-Retention cyst of the inner canthus.


461

Station 172
Dr.sajad Al-helo
1-Describe this photograph and CT scan ?
2-Differntial diagnosis ?
3-Most propable diagnosis ?
4-Common presenting features ?
5-How you can differentiat it from EAC osteoma?

Answer

1-Photograph of female shown the Rt post auricular smooth swelling


with normal skin colour.
-Axial CT scan of temporal bone 6 sections demonstarate bone density
regular swelling of Rt mastoid bone.

2- Temporal bone osteoma.


-Sebeceous cyst.
-Post auricular lymphadenopathy.
-Mastoid abscess.
-Eosinophil granuloma.

3-Temporal bone osteoma


4-Asymptomatic post auricular swelling.
5-EAC osteoma its pedinclated and at osteo-cartilegenous junction.
462

Station 173
Dr.sajad Al-helo

23 years old male presents with left nasal vestibule swelling


and nasal obstruction with flaring of lateral ala.

1-Define the CT scan for this patient ?


2-Differential diagnosis ?
3-How can present clinically?
4-Treatment ?

Answer

1-Axial CT scan of the nose and paranasal sinuses shown the soft tissue
swelling involving the left vestibule and lateral nasal wall with intact
bony structure.
2-Nasoalveolar cyst.
-Maxillary retention cyst.
-Nasolabial cyst.
-vestibular abscess.
-Sebeceous cyst.
3-Nasoalveolar cyst present as a smooth compressible swelling arising
from a lateral portion of the floor of nasal vestibule.
4-Treatment ?
-Surgical removal.
463

Station 174
Dr.sajad Al-helo

13 years old girl presents with slowly growning swelling in the left
inner canthus push the orbit laterally with proptosis .

1-Define the photograph and CT scan for this


patient?
2-Most likely diagnosis ?
3-Most common presenting features ?

Answer

1-Left orbital proptosis and eyelid edema with fullness of left inner
canthus.
-Axial CT scan 4 sections at the level of orbit demonsterate Lf proptosis
with soft tissue cystic swelling at ethmoid sinus.

2-Ethmoid mucocel.

3-Asymptomaic.
-Proptosis.
-Diplopia.
-Pain and edema if infected
464

Station 175
Dr.sajad Al-helo

3 years old girl with history of recurrent nasal infection and discharge.

1-Describe the picture and MRI of this child?


2-Differential diagnosis ?
3-Ist manditory todo surgical excision and why?
4-Whats the percent of intracranial connection?

Answer

1-Picture of child shown broad nasal dorsam with midline pit.


-Axial T1 w MRI with contrast of the nose shown high signal intensity
of midline nasal lesion.

2-Dermoid cyst.
-Glioma.
-Encephalocoele.

3-yes ,to prevent intracranial complications ,osteomylitis and nasal


deformity from recurrent infections.

4- 20 -40 %
465

Station 176

1- What are common primary sources for cystic nodal metastases?


2- What is the differential diagnosis?

Answer

1-Tonsil and thyroid gland.

2-Branchial cleft cysts, lymphangioma, abscess, and cystic nodal


metastases.
466

Station 177

50 yrs old female presents with a pulsatile, compressible mass that


refills rapidly on the release of pressure.

1-Most likely Diagnosis ?


2- Origin of this mass ?

Answer

1-Carotid body tumor.

2- Originate from small chemoreceptive and baroreceptive organs


located at the adventitia of the common carotid artery bifurcation
467

Station 178

38 y/o Male/Chief complaint: Oral ulcer/2x2cm ulcer, lower gingiva near


the retromolar trigone/6 months duration.last 2 months present with 2
level neck mass .
1-Differential diagnosis ?
2-Most likely diagnosis ?
3-Common presenting Symptoms ?
4-What is your next plan of action?

Answer

1-TB adenopathy
-Metastatic carcinoma from oral cavity cancer
-Lymphoma
-Lymphadenitis from aphthous ulcer
2- Metastatic carcinoma from oral cavity cancer
3- Non healing wound,
- pain, “on and off” bleeding ,
- pain in swallowing, ear pain, a change in speech, uncoordinated
swallowing, or a lump in the neck.
4- Thorough Physical Examination.
-Endoscopy.
-Imging /CT scan for oral cavity,neck and chest
-Biopsy.
468

Station 179
Dr.Sundus Al-Sedra

5 years old child presented with Rt auricular swelling after one day of
trauma.

A-What is the diagnosis?


B-Describe the pathophysiology ?.
C-What is the likely long –term complication if left untreated?

Answer

A-Pinna hematoma.

B-The blood /blood clot(or pus in case of an abscess formation)


separates the perichondrium from the cartilage causing
necrosis. Fibrous tissue form in the overlying skin.

C-Untreated pinna hematoma may lead to the development of a


deformed ear, so called cauliflower ear`.
469

Station 180
Dr.Sundus Al-Sedra

4 year old is brought to emergency department with a history of


inserting a FB up his nose.

A. What type of imaging is showing?


B. Describe what you can see.
C. Diagno.sis?
D. What are the main risks to the patient?
E. Management?

Answer

A-Lateral sinus X-ray.

B-There is circular radio-opaque object with a distinctive double-circle


enhancement at the edge at the level of the inferior nasal meatus.
C- It is likely to represent a button (disc) battery.

D- 1-local tissue damage, eg.septal perforation.


2-aspiration (& more tissue damage).

E.This FB should be removed immediately (patient may require GA),


because batteries are known to rapidly cause local necrosis. Alkaline
content of battery causes liquefaction necrosis, which results in
denaturation of proteins & saponification of fats (penetration of base is
not limited like in case of coagulation necrosis, caused by acid).
470

Station 181
Dr.Sundus Al-Sedra

23 year old patient presents with a sore throat.


A. Describe what you are able to visualize on examination of this
patients throat?
B. Diagnosis?
C. Provide an anatomical description of the pathology.
D. What other signs/symptoms can you expect? (list 5).
E. What are the key steps in an immediate management of this
patient?

Answer

A. There is unilateral, right sided tonsillar swelling & the uvula is


pushed towards the left.
B. Right sided peritonsillar abscess (quinsy).

C. There is acollection of pus in the peritonsillar space- a potential


space between a tonsilar capsule (formed on lateral aspect of
the tonsil by condensation of pharyngobasilar fascia) & the
superior consitrictor muscle.
D. Odynophagia, halitosis, trismus, fever, malaise, referred otalgia,
muffled/ `hot potato` voice, dribbling.
E. 1-resuscitation & pain control- I.V. access & blood sampling,
analgesia, I.V. fluid.
2-abscess drainage (needle aspiration & /or incision).
3-I.V. AB & steroids.
4-antiseptic mouthwash.
471

Station 182
Dr.Sundus Al-Sedra

A. Name the structures shown in first figure.


B. Which procedure does the second figure depict?
C. Indication of procedure?
D. Under local or GA done ?
E. If no response what is next step?

Answer

A- a-Superior crus of the lateral canthal tendon.


b-Lateral canthal tendon.
c-Inferior crus of lateral canthal tandon.

B. Inferior cantholysis.

C. Retroorbital hematoma/heamorrhage.

D. Under local A.

E. If the eye still proptosed, UGA, a superior canthotomy &/or


medial orbital decompression /ligation of any bleeding vessels
may be indicated.
472

Station 183
Dr.Sundus Al-Sedra

A. Name the item & its type.


B. Advantages.
C. Disadvantages.

Answer

A- It is a cuffed portex tracheostomy tube with inner tube.


B- 1-Inflated cuff enables positive pressure ventilation (reduces the
risk of the air /oxygen mix leaking backwards past the outside of
the tracheostomy tube).
2-Inflated cuff decreases the risk of any aspirate entering the lung.

C. 1-The risk of pressure trauma resulting in tracheal ulceration,


stenosis ( tracheostomy tubes are usually made with high
volume, low-pressure cuff which should reduce the risk).
2-If the tracheostomy tube lumen is occluded (e.g. decanulation
cap, mucous plug) & the cuff is inflated, the patient will not be able to
breath since the tracheostomy acts as FB obstructing the trachea.
3-Swallowing can be impaired with the cuff inflated (pressure against
the esophagus).
473

Station 184
Dr.Sundus Al-Sedra

A. Name the device shown .


B, What is its function?

Answer

A. It is a heat moisture exchanger (HME).

B. It provides humidification of the airway & is used when no extra


oxygen is needed. It is very useful when patient is out of bed &
mobilizing.
474

Station 185
Dr.Sundus Al-Sedra

A. What instrument is shown in figure?


B. Which procedure is it used for?
C. Name two indications for this procedure.
D. Name five possible complications of this procedure.

Answer

A. Tilley Lichwitz Antral trocar & cannula.


B. Maxillary antral washout/ maxillary sinus lavage. (the tip of the
trocar is carefully passed underneath the inferior turbinate & the
point of puncture is about 2cm behind the anterior end of the
inferior turbinate . The trocar is directed superolaterally towards
the ipsilateral outer canthus). The sinus is aspirated & irrigated.
C. a-Diagnostic to obtain fluid for microbiology.
b-Theraputic in acute infection, to remove the secretions.
D. a-Bleeding.
b-Misplacement of the cannula & false passage. into a cheek
(resulting in pain, swelling &emphysema of the cheek)
Penetration of the orbital floor (causing proptosis & periorbital
swelling).
c-Infection.
d-Damage to nasolacrimal duct resulting in epiphora.
e-Air embolism.
MCQs MCQs

Chapter four
MCQs

MCQs
MCQ
OTOLARYngology
s

HEAD & NECK SURGERY


475

1-Pathogen commonly implicated in acute suppurative otitis media


inclued, excipet:
A-streptococcus pyogenes.
B-hemophilus influenzae.
C-moraxella catarrhalis.
D-pseudomoneus aerogenosa.

2-Which test reflects the vestibulo-spinal function ?


A-Headshake nystagmus.
B-Electronystagmography.
C-Rotatary chair testing.
D-Tandem walking test.
E-Positional test.

3-Following is true regarding rhinitis medicamentosa


A-It is caused by steroid drops
B-There is atrophy of inferior turbinate
C-It is caused by oral pseudoephidrine
D-Septoplasty is treatment of choice
E-It is caused by vasoconstrictive drops

4-Following are causes of epistaxis except


A-Trauma
,
B-Ostler s disease
C-Nasal polyp
D-Septal perforation
E-Idiopathic

5-A patient has been put on IV heparin for pulmonary embolism .He
nose bleed . His PT is 50 seconds. His treatment is:started
A-Give adrenaline
B-Give normal saline
C-Give protamine sulphate
D-Give blood transfusion
E-Give transamine
476

6-A young boy of ten years age went for swimming. Next day he
developed nasal congestion, fever and headache .He deteriorated over
next twenty four hours and developed redness and edema of both
eyelids on left side. His likely diagnosis is :
A-Acute maxillary sinusitis
B-Cavernous sinus thrombosis
C-Meningitis
D-Erysipelas
E-Acute ethmoiditis

7-Allergic rhinitis
A-Is a type 1 IgE mediated hypersensitivity reaction
B-Is a type 2 IgM mediated reaction
C-IgA antibodies are produced in desensitization process which block
IgE
D-Total serum IgE levels have to be checked before starting
immunotherapy
E-Is very common in old age.

8-A young woman was on contraceptive pills due to which she


developed deep vein thrombosis .She was placed on warfarin .She
started bleeding from nose. The best test to check her coagulation
status is :
A-Bleeding time
B-Clotting time
C-Platelet count
D-Activated partial thromboplastin time
E-Hess test.

9-A young boy was involved in road traffic accident and broke his nose.
Next day he noticed a watery fluid coming from his left nostril.
Which test will you do to confirm the nature of fluid.

A-Blood sugar
B-Blood sugar in the fluid collected from nose
C-Leukocyte count in fluid
D-Do biochemical analysis of fluid and match the findings with findings
of specimen collected by lumbar puncture
E-Protein levels in nasal fluid
477

10-A 40 year old man who is on steroids developed boil in the nose
.Two days later he developed swelling of eye lids on one side with
chemosis and opthalmoplegia of extraoccular muscles . He also has
fever. Which investigation will clinch the diagnosis .
A-Total leukocyte count
B-ESR
C-X-ray Para nasal sinuses
D-MRI scan
,
E-Quickenstead s test

11-A young girl has developed saddling of nose after trauma .The best
thing to do is
A-Septoplasty
B-Tip plasty
C-SMR
D-Augmentation rhinoplasty
E-septorhinoplasty

12-Following are measures for house dust mite avoidance in allergic


rhinitis except.
A-Do dry dusting
B-Avoid carpets in bed room
C-Use polythine bedcovers for mattress
D-Wash bed linen in boiling water
E-Do not do dusting in the morning.

13-The skeleton of nasal tip is formed by.


A-Caudal end of the septal cartilage
B-Lower lateral cartilage
C-Upper lateral cartilage
D-Sesamoid cartilage
E-Hyaline cartilage

14-A two year old child is having unilateral nasal discharge which is foul
smelling and purulent the likely diagnosis is.
A-Adenocarcinoma of nose
B-Foreign body nose
C-Sinusitis
D-Nasal polyp
E-Rhinitis
478

15-The best treatment for recurrent nasal polyps is?


A-Intranasal polypectomy
B-Transantral ethmoidectomy
C-Functional endoscopic sinus surgery
D-Steroids
E-Transantral ethmoidectomy with postoperative inhaled steroids.

16-Major portion of nasal cavity is lined by?


A-Columnar epithelium (respiratory type)
B-Olfactory epithelium
C-Squamous epithelium
D-Skin
E-Cuboidal epithelium

17-Following structures open into middle meatus?


A-Frontal sinus
B-Nasolacrimal duct
C-Sphenoid sinus
D-Posterior ethmoid cells
E-Hiatus semilunaris

18-The middle turbinate?


A-Is part of ethmoid bone
B-Is part of maxilla
C-Is a separate bone
D-It is only mucosal structure
E-It contains cartilage and bone covered with mucosa

19-CSF formation is a process of:


a) Transudation.
b) Exudation.
c) Ultrafiltration.
d) Filtration.
e) Translocation.
479

20-The patient has developed traumatic CSF rhinorrhoea .The best way
to localize the site of leak is?
A-CT scan
B-MRI scan
C-Radioisotope scan
D-Use florescence dye intrathecally and examine nose with blue
nasoendoscope
E-Examine nose with endoscope

21-Which cranial nerve lies in cavernous sinus as lateral relation of


sphenoid sinus?
A-Trochlear nerve
B-Facial nerve
C-Vidian nerve
D-Sphenoplatine nerve
E-Mandibular division of maxillary nerve

22-Skeletal support to columella is provided by?


A-Caudal end of quadrilateral cartilage
B-Middle crura of allay cartilage
C-Medial crura of allar cartilage
D-Medial crural feet of allar cartilage
E-Anterior nasal spine.

23-The floor of maxillary sinus is closely related to?


A-Canine teeth
B-Molar teeth
C-Premolar teeth
D-Second premolar and first molar
E-All of above

24-A young boy was involved in road traffic accident . He developed


vertigo when he tried to move about . On examination he had blue ear
drum and facial palsy on right side. Audiogram done two weeks later
confirmed sensorineural hearing loss. The likely diagnosis is?
A-Transverse fracture of temporal bone
B-Longitudinal fracture of temporal bone
C-Ossicular chain disruption
D-Disruption of spiral ganglion
E-Brain contusion
480

25-A two year old child is suffering from acute otitis media. His pain is
not relieved in spite of adequate treatment .The best treatment in this
situation is?
A-Change antibiotic
B-Increase analgesic dose
C-Do hot fomentation
D-Give lignocaine ear drops
E-Do myringotomy.

26-Which of the following is not a clinical feature of baurotraumatic


otitis media?
A-Otalgia
B-Deafness
C-Tinnitus
D-Blue ear drum
E-Purulent ear discharge

27-Which of the following is not included in the treatment of


perichondritis of pinna?
A-Local antibiotics
B-Systemic antibiotics
C-Incision and drainage
D-Removal of necrotic cartilage
E-Subsequent plastic surgery

28-A thirty year old lady had flue which was followed by severe otalgia
and tinnitus. One day later she had blood stained discharge from ears
and on otoscopy had reddish vesicles on ear drum. She is suffering
from :

A-Malignant otitis externa


B-Otitis externa hemorrhagica
C-Ramsay-Hunt syndrome
D-Hemotympanum
E-Acute otitis media
481

29-A two year child is very slow in developing language. On otoscopy


he has dull ear drum. Investigation of choice is?
A-Audiogram
B-X-ray mastoids
C-Tympanogram
D-Serum bilirubin
E-Nystagmogram

30-Following are characteristics of discharge in unsafe chronic


supurative otitis media except?
A-Thick discharge
B-Scanty discharge
C-Odorless discharge
D-Blood stained discharge
E-Purulent discharge

31-A 20 year old boy had long standing foul smelling ear discharge.
Recently he has developed otalgia, fever, headache and vomiting .The
likely diagnosis is?

A-Brain abscess
B-Bezolds abscess
C-Perilabyrinthine fistula
D-Otitis externa
E-Meningitis.

32-Which of the following is not a cause of hoarseness?


A-Carcinoma of right bronchus
B-Acute laryngitis
C-Thyroidectomy
D-Hypothyroidism
E-Carcinoma of larynx

33-Which of the following is not true regarding tracheobronichial


foreign body in the children?
A-The child presents with breathing difficulty
B-The foreign body usually lodges in left bronchus
C-The foreign body is best removed using rigid bronchoscope
D-X-ray chest will show expiratory emphysema on involved side
E-Cheat X-ray will show Mediastinal shift to involved side
482

34-A thirty year old lady has five children. She is complaining of
hoarseness for last one month which is not improving. She never had
any surgery .Her probable diagnosis is?
A-Carcinoma of larynx
B-Vocal cord nodules
,
C-Reinke s edema
D-Hypothyroidism
E-Recurrent laryngeal nerve palsy.

35-A six months old baby has strider which gets worse on crying and
improves on lying in prone position .He is suffering from?
A-Laryngeal web
B-Subglottic stenosis
C-Laryngomalscia
D-Congenital paralysis of vocal cord
E-Vocal cord polyp

36-Absolute indication for tonsillectomy is?


A-Chronic tonsillitis
B-Rheumatic fever
C-Diphtheria carrier
D-Obstructive sleep apnea
E-Glassopharyngeal neurectomy

37-Which of the following statements is not rue regarding


oropharyngeal herpes zoster?
A-Vesicles appear in the distribution of ninth and tenth cranial nerves
B-Vesicles are unilateral
C-Pharyngeal condition may be associated with geniculate herpes
D-Healing occurs leaving scarring
E-Acyclovir when used early is helpful.

38-Which of the following is not a clinical feature of adenoid


hypertrophy?
A-Nasal obstruction
B-Snoring
C-Eustachian tube obstruction
D-Nocturnal cough
E-Chronic suppurative otitis media
483

39-Which of the following is not true regarding acute reteropharyngeal


abscess?
A-It is limited to one side of the midline
B-It forms due to suppuration in reteropharyngeal lymph node
C-Produces difficulty in breathing and suckling
D-It is common in children
E-Incision and drainage is done from outside trough carotid sheath.

40-Surgical emphysema after trachestomy is corrected by:


a)taking more stitches of the wound
b)cold compresses
c)widening of the wound by removal of some stitches
d)antihistaminic intake.

41-Saddle nose may be due to the following except:


a)overresection of septal cartilage
b)nasal trauma
c)septal abcess
d)rhinosceleroma

42-The most serious complication after tonsillectomy:


a)respiratory obstruction
b)reactionary haemorrhage
c)incomplete removal
d)infection.

43-Otoscopic manifestation of chronic secretory otitis media may


include the following except:
a)perforation at pars flaceida
b)transverse handle of malleous
c)absent cone of light
d)air bubbles behind the tympanic membrane.

44-A newly born infant with respiratory distress & difficult feeding is
more likely to be due to:
a)laryngeal web at the anterior half of vocal cords
b)bilateral posterior choanal atresia
c)congenital subglottic stenosis
d)congenital meatal atresia
484

45-The most common cause of oroantral fistula is:


a)acute sinusitis
b)car accident
c)dental extraction of upper second premolar tooth
d)radical antrum operation.

46-The most common cause for posterior septal perforation is:


a)TB
b)syphilis
c)leprosy
d)scleroma.

47-Moure's sign is:


a)presence of laryngeal click
b)absence of laryngeal click
c)external neck swelling
d)internal pharyngeal swelling.

48-The following symptoms are true of primary atrophic rhinitis except:


a)bad odour felt by the patient
b) bad odour felt by others
c)epistaxis
d)sense of nasal obstruction

49-Unilateral malignant tumor of the vocal cord with subglottis


ectension & lymph node metastasis is best treated by:
a)unilateral cordectomy
b)total laryngectomy with neck dissection
c)cheomotherapy
d)tracheostomy only.

50-For assessment of the skull base and intracranial extension of


cancer,
which combination of studies is most informative?
A. PET and CT
B. CT and MRI
C. PET and MRI
D. MRI and MR angiography
E. PET/CT and MRI .
485

51-The MR findings of glomus jugulare paraganglioma include a:


A. T1 and T2 “salt and pepper” appearance.
B. T1 and T2 “ground glass” appearance.
C. T1 and T2 high signal for cystic necrosis.
D. T1 and T2 high signal within the petromastoid air cells.

52-A jugular foramen schwannoma


A. Has no “pepper” flow voids, even if it is a large tumor.
B. Has characteristic flow voids.
C. Is a benign tumor of sympathetic nerves in the jugular foramen.
D. Has a high T1 signal.

53-The classic imaging finding in clival meningioma is


A. “Dural tails.”
B. Flow voids.
C. Central calcification.
D. Peripheral necrosis.

54-The classic CT appearance of fibrous dysplasia of the skull base


includes
A. Pagetoid mixed pattern.
B. Sclerotic pattern.
C. Cystic pattern.
D. All of the above.

55-Langerhans histiocytosis of the skull base has also been known as


A. Hand-Schüller-Christian disease.
B. O’Neill-Morris-Wreesman syndrome.
C. Schüller Siwe syndrome.
D. Delkargios syndrome .

56-All of the following structures extend through the foramen ovale


except the?
A. Mandibular nerve.
B. Cranial nerve (CN) V3.
C. Meningeal branch of the mandibular nerve.
D. Accessory meningeal artery.
486

57-All of the following structures extend through the jugular foramen


except the
A. Inferior petrosal sinus.
B. Glossopharyngeal nerve.
C. Middle meningeal artery.
D. Sigmoid sinus.

58-All of the following structures extend through the foramen magnum


except the
A. Meninges.
B. Vertebral arteries.
C. Posterior meningeal artery.
D. Medulla oblongata.

59-The agger nasi air cell is


A. The largest and most consistent ethmoid air cell.
B. Positioned anterior to the insertion of the inferior turbinate.
C. Positioned anterior to the insertion of the middle turbinate.
D. Consistently pneumatized.

60-The uncinate process may be attached to any of the following


except the
A. Skull base.
B. Lamina papyracea.
C. Nasal septum.
D. Middle turbinate.

61-On a coronal CT scan, the ethmoid roof is noted tobe 7 mm above


the cribriform plate, which makes this a Keros type
A. I.
B. II.
C. III.
D. IV.
487

62-The sphenoid sinus ostium is generally located


A. Superior to the superior turbinate 1 to 1.5 cm above the roof of the
choana.
B. Inferior to the superior turbinate 1 to 1.5 cm above the roof of the
choana.
C. Medial to the superior turbinate 1 to 1.5 cm above the roof of the
choana.
D. Lateral to the superior turbinate 1 to 1.5 cm above the roof of the
choana.

63-Mandibular swing should not be performed alone when


A. The tumor is lateral to the ICA.
B. The tumor extends beyond the floor of the middle cranial fossa.
C. The tumor involves intracranial structures.
D. Any of the above is true.

64-Which of the following is not a risk factor for oral cancer?


A. Tobacco
B. Alcohol
C. Fanconi anemia
D. Herpes simplex virus
E. Human immunodeficiency virus (HIV) infection.

65-Postoperative chemoradiation therapy is indicated in patients with


A. Positive neck nodes.
B. Vascular invasion.
C. Perineural invasion.
D. T4 tumors.
E. Extracapsular lymph node spreading.

66-In nasopharyngeal cancer the most common site of metastases is


the
A. Bone.
B. Liver.
C. Lung.
D. Brain.
.
488

67-NPC is highly sensitive to


A. Doxorubicin (Adriamycin).
B. Methotrexate.
C. Cyclophosphamide.
D. Platinum-based regimens.

68-The preferred imaging study for evaluating the intracranial or


perineural extent of nasopharyngeal tumors is.
A. CT.
B. Enhanced magnetic resonance imaging (MRI).
C. Enhanced CT.
D. Ultrasonography.

69-The parapharyngeal space extends from the skull base to the


A. Hyoid.
B. Level of the epiglottis.
C. Level of the angle of the mandible.
D. Level of the postcricoid region.

70-The masticator space includes the following muscles, except the


A. Medial pterygoid.
B. Lateral pterygoid.
C. Posterior digastric.
D. Temporalis.

71-A nasopharyngeal primary SCC with extension to the oropharynx


without parapharyngeal space extension is stage
A. T1.
B. T2.
C. T3.
D. T4.

72-Esophageal webs are associated with the following, except


A. Plummer-Vinson syndrome.
B. Celiac disease.
C. Gastroesophageal reflux disease (GERD).
D. Bullous pemphigoid .
489

73-Cancer is limited to the True vocal cord, but paralysis is present.


A. T1a
B. T1b
C. T2
D. T3
E. T4a.

74-Vocal cord fixation is most likely due to invasion of the


A. Conus elasticus.
B. Quadrangular membrane.
C. Paraglottic space.
D. Broyle tendon.

75-Which of the following would not be recommended as part of the


treatment for a T1 piriform sinus hypopharynx tumor?
A. Transoral laser resection of the primary tumor with negative margins
B. Radiation therapy alone
C. Transcervical lateral pharyngectomy
D. Bilateral neck dissection after surgery for the primary tumor
E. Concomitant chemoradiation therapy.

76-A 75-year-old woman presents with an incidentally found T1


hypopharynx tumor of the medial piriform sinus. CT and PET findings
are negative for cervical disease. She undergoes surgical resection of
the primary lesion. Which of the following is the best next step in
treatment of the neck?
A. No neck dissection necessary
B. Ipsilateral neck dissection only
C. Bilateral neck dissection
D. Ipsilateral radiation therapy to the neck
E. Bilateral radiation therapy to the neck.

77-The radiological demarcation between level I and level II in the


neck is:
A. A vertical line through the posterior border of the submandibular
gland.
B. The posterior belly of the digastric muscle.
C. The mylohyoid muscle.
D. A vertical line through the tip of the greater cornu of the hyoid bone.
490

78-Which of the following is most accurate at staging the neck in


patients with SCC of the larynx who have not received any previous
treatment?
A. Clinical palpation
B. MRI
C. PET-CT
D. US-FNA.

79-The treatment of choice for lymphadenopathy caused by atypical


mycobacteria in children is
A. Rifampicin-based antimicrobial chemotherapy.
B. Sulfa-based antimicrobial chemotherapy.
C. Complete surgical excision without antimicrobials.
D. Surgical curettage and debulking followed by antimicrobial therapy.

80-Sacrifice of which of the following structures constitutes an


extended MRND?
A. Accessory nerve, hypoglossal nerve, internal jugular vein
B. Sternomastoid muscle, omohyoid muscle, and internal jugular vein
C. Accessory nerve, sternomastoid muscle, and internal jugular vein
D. Accessory nerve, internal jugular vein, and submandibular gland.

81-Which of the following statements regarding second arch branchial


cysts is true?
A. The peak age of presentation is in the teenage years.
B. Most cases will have a rudimentary tract extending deeply to the
tonsillar fossa.
C. Approximately 10% to 30% are bilateral.
D. These cysts may become inflamed in patients with acute tonsillitis
because of lymphoid tissue in the wall.

82-In patients with differentiated thyroid cancermetastatic to the lateral


side of the neck, which preoperative investigation should be considered
routine?
A. Ultrasound examination
B. PET scan
C. Contrast-enhanced CT of the neck
D. Diagnostic radioiodine scan
.
491

83-Which of the following side effects is associated with the


administration of radioactive iodine?
A. Dysphagia
B. Xerostomia
C. Secondary malignancy
D. All of the above.

84-What is the rate of incidental malignancy in multinodular goiters?


A. Less than 5%
B. 5% to 10%
C. 10% to 30%
D. 30% to 50%.

85-Which of the following areas is a common site for nodal metastases


from thyroid cancers?
A. The perifacial lymph nodes
B. Level IV posterior to the great vessels
C. Level II superior to the accessory nerve
D. The parapharyngeal space

86-Children who are seen with papillary thyroid cancer are at high risk
of
A. Locally advanced primary tumors.
B. Bony metastases.
C. Pulmonary metastases.
D. Local recurrence.

87-The most important differential diagnosis in patients with suspected


anaplastic thyroid cancer is
A. Poorly differentiated thyroid cancer.
B. Lymphoma.
C. Sarcoma.
D. Metastatic melanoma.

88-The next step in managing a patient with normal clinical


examination results and a rising thyroglobulin level after surgery for
thyroid cancer is.
A. Elective neck dissection.
B. Ultrasound imaging.
C. Therapeutic radioiodine.
D. Tyrosine kinase inhibitor therapy.
.
492

89-The most reliable landmark for the facial nerve in parotid surgery is
the
A. Tragal pointer.
B. Posterior belly of the digastric.
C. Tympanomastoid suture line.
D. Bony cartilaginous ear canal.

90-Characteristic MRI findings for pleomorphic adenoma include


A. Hyperintensity on T1.
B. Hyperintensity on T2.
C. Enhancement with gadolinium.
D. A and B.
E. B and C.

91-Which of the following statements about recurrent minor salivary


gland cancer is correct?
A. Local recurrences require chemoradiation therapy.
B. Local recurrences indicate a palliative course.
C. Local recurrences do not necessarily portend a decreased chance of
survival.
D. Surgical intervention is not a therapeutic option for local recurrences

92-Paragangliomas often have demonstrable feeder vessels that most


commonly arise from the
A. External carotid artery.
B. Internal carotid artery.
C. Ascending pharyngeal artery.
D. Lingual artery.
E. Superior thyroid artery.

93-The parapharyngeal space is bounded by the following structures,


except the
A. Base of the skull.
B. Lateral pharyngeal wall.
C. Masseter muscle.
D. Pterygoid muscles.
E. Prevertebral muscles..
.
493

94-A 35-year-old man presents with a 3-cm schwannoma of the


cervical plexus. The best next step is
A. Observation with repeated imaging in 6 months.
B. Radiation therapy.
C. Chemotherapy.
D. Early surgical intervention.

95-The differential diagnosis of a 3-cm mass on the right side of the


neck located in the carotid space should include all of the following,
except a
A. Glomus vagale tumor.
B. Carotid body tumor.
C. Schwannoma of the vagus nerve.
D. Pleomorphic adenoma of the deep lobe of the parotid gland .

96-Which vein is not a reliable recipient for venous drainage of a free


flap?
A. External jugular vein
B. Internal jugular vein
C. Facial vein
D. Anterior jugular vein
E. Middle thyroid vein.

97-Which nerve is used in the sensate radial forearm free flap?


A. The lateral antecubital cutaneous nerve is used.
B. The sensory branch of the radial nerve is used.
C. The radial forearm flap cannot be raised as a sensate flap.
D. The medial antecubital cutaneous nerve is used.
E. The posterior cutaneous nerve of the forearm is used.

98-Which is true regarding the deltopectoral flap?


A. The donor site can be closed primarily.
B. It is the preferred choice for pharyngeal reconstruction.
C. The blood supply is from the axillary artery.
D. It should be raised as a musculocutaneous flap to preserve the blood
supply.
E. The distal end of the flap has the most tenuous blood supply.
494

99-The blood supply to the skin paddle of the fibula osteocutaneous


flap
A. Is highly unreliable.
B. Is always anterior to the fibula.
C. Is always in the posterior intermuscular septum.
D. Is usually located in the proximal one third of the bone.
E. Can be better protected with harvesting of acuff of soleus or flexor
hallucis longus muscle.

100-Which is true regarding the pectoralis major muscle?


A. The blood supply is from the pectoral branch of the thoracoacromial
artery.
B. The nerve supply is from the long thoracic nerve.
C. The main blood supply to the muscle is from the internal mammary
artery.
D. The lateral thoracic artery should be preserved.
E. The nerve supply is from the axillary nerve.

101-Which one is not a pivotal flap?


A. Bilobed flap
B. Rhombic flap
C. V-Y flap
D. Z-plasty
E. Paramedian forehead flap.

102-Which one of these defects is better reconstructed with a free


flap?
A. A glossectomy defect less than 30%
B. A lateral marginal mandibulectomy defect in a nonirradiated patient
with more than 1 cm of thickness of remaining mandible
C. A mucosal defect of the floor of the mouth
D. A mucoperiosteal defect of the hard palate
E. A hemiglossectomy defect
495

103-Which vessel is not a first choice option in microvascular


anastomosis in the neck?
A. Superior thyroid artery
B. Facial artery
C. Lingual artery
D. Transverse cervical artery
E. Occipital artery.

104-Which tumor is most likely to have retropharyngeal lymph node


metastasis?
A. Nasopharynx
B. Oral cavity
C. Glottic larynx
D. Parotid .

105-Which of the following is the most common late sequela after


radiation therapy to the head and neck?
A. Osteoradionecrosis
B. Feeding tube dependence
C. Hypothyroidism
D. Brachial plexopathy.

106-After radiation therapy, which of the following is not a suspicious


feature of a persistent metastatic lymph node?
A. Size greater than 1.5 cm
B. Focal enhancement on CT
C. Well-circumscribed node
D. Calcification.

107-The presence of peripheral cysts around the intracranial


component of a sinonasal lesion is generally associated with which of
the following tumors?
A. Sinonasal undifferentiated carcinoma
B. Melanoma
C. Esthesioneuroblastoma
D. Squamous cell carcinoma
E. Adenoid cystic carcinoma.
496

108-Which of the following tumors is not commonly associated with


lateral retropharyngeal lymphadenopathy?
A. Nasopharyngeal carcinoma
B. Paranasal sinus carcinoma
C. Tonsil carcinoma
D. Papillary carcinoma of the thyroid
E. Pyriform sinus carcinoma.

109-Hypopharyngeal cancers can cause paralysis of the ipsilateral vocal


cord by all of the following mechanisms except
A. Direct invasion of the paraglottic space.
B. Direct invasion of the cricoarytenoid joint.
C. Perineural involvement of the recurrent laryngeal nerve.
D. Direct invasion of the recurrent laryngeal nerve.
E. Involvement of the recurrent laryngeal nerve by paratracheal
lymphadenopathy.

110-Insulin Diabetic Patient undergoing total thyroidectomy, should


a. Be admitted 1 hour before surgery
b. Receive pre operative insulin pre-
c. Receive intravenous normal saline preoperatively
d. Receive a glucose, insulin and potassium infusion preoperatively
e. Should receive intravenous insulin intraoperatively.

111-Total serum thyroxine is reduced by


a. Oral contraceptive pills
b. Pregnancy
c. Nephrotic Syndrome
d. Propanolol.

112-Facial Nerve Is Not Fully Developed Untill A Childe Is:


A. 1 Year
B. 2 Year
C. 4 Year
D. 6 Year
E. 8 Year.
497

113-The Differential Diagnosis Of Acute Facial Nerve Parlysis , Should


Not Include:
A. Sarcoidoisis
B. Herbes Zoster
C. Kawasaki’s Disease
D. Lyme Disease
E. Shwanoma.

114-The most common source of infection of the lateral pharyngeal


space is from the
A. Floar of the mouth
B. Neck
C. Mastoid
D. Soft palate
E. Tonsils.

115-lateral pharyngeal space is devided into two compartments by the


A. 11th cranial nerve
B. Carotid sheath
C. Styloid process
D. Carotid artery
E. Jugular artery .

116-parotid gland is separated from submandibular gland by


A. Mylohyoid
B. Stylohyoid
C. Stylomandibular ligament
D. Posterior belly of digastric.M
E. Anterior belly of digastric,m.

117-A congenital choanal atresia is:


A. Most commonly a membranosious closure
B. Mostly aunilateral
C. Commoner in female
D. Of early presentation in unlilateral cases.
E. Not a life threatening condition jn bilateral cases
498

118-The interaural difference in wave V of ABR must not exceed :


A-0.2 msec
B-1 msec
C-0.1 msec
D-2.4 msec

119-Which of the following is MOST specific for otosclerosis:


A-Hyperacusis Willichii
B-Normal tymponometry with lost reflexes.
C-Carhart's notch.
D-Schwart's sign.
E-None of the above

120- 3 years old child with sinusitis , you expect affected sinuses are :
A-Frontal and maxillary.
B-Frontal and ethmoidal.
C-Sphenoidal and maxillary
D-Maxillary and ethmoidal.

121-Patient with right complete facial paralysis is working in a noisy for


20 years,you can expect:
A-Right ear hearing loss is less at 4kHz.
B-Left ear hearing is worse than right.
C-Both ears got equal hearing.
D-Right ear hearing loss is more at 4kHz .

122-Cochlear microphonics originate from:


A-Cochlear nerve.
B-Basilar membrane.
C-Outer hair cells.
D-Inner hair cells.

123-Compoud action potential originates from:


A-Auditory nerve.
B-Basilar membrane.
C-Outer hair cells.
D-Inner hair cells..
.
499

124-The endocochlear potential originates from:


A-Basilar membrane.
B-Outer hair cells.
C-Stria vascularis.
D-Inner hair cells .

125-The 4th branchial arch nerve develops into:


A-The glossopharyngeal nerve.
B-Vagus nerve.
C-Mandibular nerve.
D-Superior laryngeal nerve.

126-The most common complication of FESS:


A-Bleeding.
B-Cranial complictions.
C-Anosmia.
D-Synaechia..

127-The MOST common route of spread of supraglottic carcinoma :


A-Lymphatic.
B-Blood spread.
C-Anteriroly through fenestra of cartilage to pre-epiglottic space.
D-Inferiorly to the lung.

128-Early signs of respiratory failure:


A-Cyanosis.
B-Tachypnea.
C-Subcostal retraction.
D-Biphasic stridor.
E-Irritability and gasping.

129-Exostosis is characterised by all except:


A-Multiple.
B-Bilateral.
C-Peduculated.
D-Usually in cold water divers.
E-Lamellated bone .
500

130-In a patient with right paralytic labyrinth, which of the following is


correct:
A-Right beating nystagmus.
B-Left beating nystagmus.
C-Vertical nystagmus.
D-Total hearing loss.

131-In acoustic neuroma, you can find:


A-High frequency SNHL.
B-Low frequency SNHL.
C-Flat audiogram.
D-Notch at 2 KHZ.

132-In modified radical neck dissection type I , which of the following


will be preserved:
A-Thoracic duct.
B-Spinal accesory nerve.
C-Internal jugular vein.
D-Sternomastoid muscle.
E-2 & 3.

133-Trotter' triad consisit of all of the following except:


A-Bilateral conductive hearing loss.
B-Otalgia and facial pain.
C-Unilateral immobile soft palate.
D-Unilateral ear effusion.

134-The nasopalatine nerve supplies sensation to the mucosa of the?


A-Anterior premaxillary palate.
B-Soft palate.
C-Lateral nasal passage.
D-Anterior cheek.

135-The most commonly injured branch of the facial nerve in


rhytidectomy is the?
A-Temporal.
B-Marginal mandibular.
C-Buccal.
D-Cervical.
E-Zygomatic .
501

136-Infection spread from the maxillary sinus intracranially most


likely travels through the ?
A-Angular veins.
B-Ophthalmic artery.
C-Superior labial veins.
D-Pterygoid plexus .

137-Which virus is especially associated with unilateral sensorineural


hearing loss?
A. cytomegalovirus
B. rubella
C. rubeola
D. adenovirus
E. mumps.

138-Susceptibility to noise-induced hearing loss may be potentiated by


A. genetic predisposition.
B. smoking.
C. the gender of the subject.
D. excessive alcohol intake.
E. heart disease.

139-Which is the most common traumatic ossicular abnormality?


A. necrosis of the long process of the incus
B. incudostapedial joint disarticulation
C. fracture of the stapes suprastructure
D. fixation of the incus
E. dislocation of the stapes footplate.

140-Which of the following is untrue about Waardenburg’s syndrome


(WS)?
A. It may be associated with Hirschsprung’s disease.
B. Dystopia canthorum occurs in WS1 and WS3.
C. Hearing loss is typically mixed in nature.
D. Profound hearing loss is common.
E. It may be associated with heterochromia iridis.
502

141-Branchio-oto-renal syndrome may be associated with the


following, except
A. dilated vestibular aqueduct.
B. autosomal recessive inheritance.
C. mutations in EYA1.
D. renal anomalies.
E. marked phenotypic variability.

142-Which of the following statements is most accurate about


congenital hearing loss?
A. The incidence is decreasing.
B. Fifty percent or more is inherited.
C. It is always profound.
D. It may be late in onset.
E. It is nearly always sensorineural in nature.

143-The most reliable means of improving hearing in the patient with


otosclerosis is a
A. hearing aid.
B. fenestration operation of Lempert.
C. stapedectomy.
D. stapedotomy
E. partial stapedectomy.

144-The principal objective in stapes surgery is to


A. protect the inner ear from intense sound.
B. increase round window membrane mobility.
C. restore ossicular continuity.
D. restore sound pressure transmission to the inner ear.
E. increase fluid movement in the inner ear.

145-Which of the following does not describe Carhart’s notch?


A. ranges in magnitude up to 15 dB
B. is centered at 2 kHz
C. is an apparent sensorineural hearing loss
D. usually disappears after stapes surgery
E. occurs without a conductive hearing loss.
503

146-The most frequent site of first occurrence of otosclerosis is in the


A. oval window rim.
B. fossula post fenestram.
C. round window niche.
D. fissula ante fenestram.

147-Computed tomography scanning is superior to magnetic resonance


imaging in establishing which one of the following diagnoses?
A. Bezold’s abscess
B. coalescent mastoiditis
C. epidural abscess
D. cerebellar abscess
E. subdural empyema.

148-Critical steps in constructing a canal wall down mastoid cavity


include all of the following except
A. removal of the mastoid tip.
B. obliteration of the mastoid with soft tissue.
C. wide saucerizarion of the cavity.
D. always drilling parallel to the facial nerve and using copious irrigation
to identifythe sheath before exposing it.
E. removal of conchal cartilage when performing the meatoplasty.

149-The most critical factor in avoiding injury to the facial nerve during
mastoid operations is
A. use of a facial nerve monitor.
B. a preoperative CT scan.
C. drilling parallel to the nerve.
D. using a diamond bur.
E. exposing the sheath of the nerve by removing the overlying bone.

150-Therapy that has been proven to help in otitis media (OM) includes
all of the following except
A. antibiotics.
B. antihistamines.
C. corticosteroids.
D. tympanostomy tube placement.
E. analgesics.
504

151-Advanced malignancies of the temporal bone are best treated with


a. Surgery only
b. Radiation only
c. Surgery and radiation
d. Chemotherapy

152-What is the most common cause of inoperability in congenital


aural atresia?
a. Absence of the oval window
b. A facial nerve overlying the oval window
c. Poor mastoid pneumatization

153-One of these complications is not seen with underlay


tympanoplasty grafting:
a. Reperforation
b. Blunting
c. Infection
d. Myringitis of the graft.
d. Unilateral atresia.

154-Which one of following statements is correct?


a. Blunting is a complication of underlay tympanoplasty.
b. The rate of take for overlay tympanoplasty is greater than for
underlay tympanoplasty.
c. Lateralization is a complication of underlay tympanoplasty.
d. Myringitis is not seen after underlay tympanoplasty.

155-Cartilage should be considered as a graft material in the following


situations:
a. The atelectatic ear
b. Cholesteatoma
c. A perforation anterior to the annulus
d. A draining perforation
e. All of the above

156-Contraindications to canal wall reconstruction


tympanomastoidectomy include
a. Mastoid cholesteatosis
b. Sinus tympani involvement
c. Facial paralysis
d. Tegmen defect with meningoencephalic herniation
505

157-The most common intraoperative finding associated with revision


stapedectomy is
a. Perilymph fistula
b. Prolapsed facial nerve
c. Displaced prosthesis
d. Fibrosis of oval window tissue.

158-Presence of acoustic reflexes on a preoperative audiogram should


prompt the operating surgeon to
a. Cancel the surgery.
b. Order a CT scan.
c. Consider superior semicircular canal dehiscence as a cause of the
conductive hearing impairment.
d. Do all of the above.

159-What segment of the facial nerve is most commonly damaged as a


result of temporal bone trauma?
a. Intracranial
b. Meatal
c. Labyrinthine/perigeniculate
d. Tympanic

160-Which of the following is considered to be an absolute


contraindication to cochlear implantation?
a. Duration of deafness greater than 30 years
b. Auditory neuropathy
c. Enlarged vestibular aqueduct
d. Michel aplasia
e. All of the above

161-An absolute contraindication to middle fossa vestibular nerve


section is
a. An only hearing ear
b. Bilateral Ménière’s disease
c. Incapacitating vertigo
d. Diuretic allergy
e. Age less than 60 years
506

162-The most common form of BPPV results from


a. Anterior canal canalithiasis
b. Lateral canal cupulolithiasis
c. Lateral canal canalithiasis
d. Posterior canal cupulolithiasis

163-What is the most common serious complication of superior canal


dehiscence (SCD) plugging surgery in the initial 24 hours after surgery?
a. Aphasia
b. Hematoma
c. Seizure
d. Meningitis
e. Posterior canal canalithiasis

164-Gradenigo’s syndrome usually includes all of the following except


a. Retro-orbital pain
b. Fourth cranial nerve palsy
c. Otorrhea
d. Hearing loss
e. Sixth nerve palsy.

165-Which of the following drugs is most likely to cause permanent


hearing loss?
A. erythromycin
B. quinine
C. cisplatin
D. penicillin G

166-A 30 years old patient with recurrent attacks of vertigo, hearing


loss and tinnitus associated with nausea and vomiting has
a- benign paroxysmal positional vertigo
b-vestibular neuronitis
c-Meniere’s disease

167-The inner ear is ebmbryologically derived from


A- ectoderm
B-entoderm
C- mesoderm
D- all of the above
507

168-All of the following are examples of absorbable suture, except


which of the following?
A. Polyglactin
B . Poliglecaprone 2 5
c. Polypropylene
D . Polydioxanone
d-acoustic neuroma.

169-In a diagnostic workup of headache, further radiologic evaluation


with MRI or cr scan is indicated in which of the following
circumstances?
A. Pulsatile headaches
B. Headache awakening one from sleep
C. Onset of headache in childhood
D. Unilateral headache

170-Which drug is most likely to be effective for the treatment of


migraine-associated vertigo?
A. Sumatriptan
B . Meclizine
c. Nortriptyline
D . Diazepam

171-Which of the following descriptions is characteristic of carcinoma in


situ?
A. Pushing borders with mild atypia along the basal layer
B. Mild atypia, but with violation of the underlying basement membrane
C. Full-thickness cellular atypia with an intact basement membrane
D. Moderate atypia extending into the upper third of the mucosa .

172-Robotic surgery benefits from:


A. 360° wristed motion
B . Scaled motion with tremor suppression
C. Binocular magnification
D. Both B and C
E. All of the above
508

173-A cerebrospinal fluid leak during ethmoidectomy is most likely to


occur at:
A. The junction of fovea ethmoidalis and lamina papyracea
B. The medial fovea ethmoidalis
C. The insertion of the uncinate process
D. The planum sphenoidale

174-To best determine whether the lamina papyracea is intact, which


test or procedure should be used?
A. Intraoperative CT scan
B. Fat float test
C. Bulb press test
D. Endoscopic dissection and search for orbital fat

175-Which of these is the most commonly identified organism in


subperiosteal orbital abscess due to sinusitis?
A. Pseudomonas aeruginosa
B. Haemophilus influenzae
C. Streptococcus pneumoniae
D. Streptococcus viridans .

176-What is the initial radiographic study of choice in a patient with


suspected cerebrospinal fluid (CSF) leak from accidental trauma?
A. High-resolution CT scan
B. MR imaging
c. cr cisternogram
D. Radionuclide cisternogram
E. MR cisternogram

177-Which of the following is not considered part of the nasal valve?


A. Head of the inferior turbinate
B. Bony piriform aperture
C. Nasal floor
D . Membranous septum
509

178-Which of the following most commonly causes nasal valve


obstruction?
A. Previous rhinoplasty
B. Turbinate hypertrophy
C. Nasal polyposis
D. Congenital

179-Which of the following glands produce the majority of the


unstimulated saliva?
A. Parotid glands
B. Minor salivary glands
C. Sublingual glands
D. Submandibular glands .

180-Based on the multicellular theory of tumorigenesis,


mucoepidermoid carcinoma arises from:
A. Excretory duct cells
B. Acinar cells
C. Striated duct cells
D. Intercalated duct cells

181-Workup for a new diagnosis of vocal fold paralysis does not


routinely include:
A. Cf scan
B . Laryngeal electromyography (LEMG)
C. Swallowing assessment
D. Serology

182-What is the first-line treatment for recurrent respiratory


papillomatosis?
A. Surgical removal
B. Inhaled cidofovir
C. High-dose oral steroid burst
D. Azathioprine
E. Radiation therapy
510

183-Stroboscopy allows assessment of the following except:


A. Vocal fold closure
B. Vocal fold mobility
C. Mucosal pliability
D . Vocal fold level during phonation
E. Vocal fold sub-epithelial pathology .

184-Continuous positive airway pressure (CPAP) is effective for


obstructive sleep apnea because it:
A. Supports the soft palate
B. Compresses the tongue base
C. Stimulates activity of dilating muscles
D. Prevents pharyngeal collapse during expiration
E. Forces the pharynx open after obstruction

185-Temporary vocal fold augmentation materials include:


A. Silas tic
B. Hyaluronic acid
c. Titanium
D . Gore-Tex
E. Teflon

186-Which of the following is the most common cause of iatrogenic


bilateral vocal fold paralysis?
A. Cervical spine surgery
B . Lung surgery
c. Thyroid surgery
D . Carotid surgery

187-Where are septal fractures most commonly seen?


A. Above the interface with the maxillary crest
B. At the caudal septum
C. At the junction of the cartilage with the perpendicular plate of the
ethmoid bone
D. Right where the cartilage interfaces with the maxillary crest
E. Mediastinoscopy
511

188-Which of the following is one of the most important factors in


successful frontal sinus obliteration?
A. Choosing the correct obliteration material
B. Complete removal of all sinus mucosa
C. Choosing the appropriate plate size for posterior table reconstruction
D . Use of atraumatic technique for elevation of the pericranial flap

189-Which of the following nasal bone fractures are most likely to be


associated with other facial fractures?
A. Displaced nasal fractures
B. Fractures of the caudal aspect of the nasal bones
C. Fractures of the cephalic end of the nasal bones

190-Which is not a vertical buttress of the face?


A. Zygomaticomaxillary
B. Zygomatic arch
C. Nasomaxillary
D. Pterygomaxillary D. Fractures of the nasal sidewall

191-Robin sequence is characterized by:


A. Micrognathia, cleft palate, and glossoptosis
B . Micrognathia, cleft palate, and airway obstruction
C. Cleft palate, airway obstruction, and glossoptosis
D. Micrognathia, glossoptosis, and airway obstruction.

192-Which of the following is the second most commonly occurring


type of tracheoesophageal fistula (TEF) ?
A. Proximal TEF with distal esophageal atresia (EA)
B. Proximal EA with distal TEF
C. H-type TEF
D . Proximal and distal TEF with EA

193-Which of these is the most likely diagnosis in a neonate with


expiratory stridor and a brassy cough?
A. Congenital subglottic stenosis
B. Tracheomalacia
C. Robin sequence
D . Bilateral vocal fold paralysis
512

194-. Imaging for pediatric rhinosinusitis is most accurately


accomplished with:
A. Plain films
B. CT scans
C. MRI
D . Maxillary sinus transillumination

195-A child with bilateral second branchial cleft anomalies {BCA) may
have all of the following except:
A. Bilateral preauricular pits
B . Sensorineural hearing loss
C. Renal disease
D. Microtia
E. Autosomal recessive inheritance.

196-Recurrent respiratory papillomatosis (RRP) demonstrates a


predilection for which of the following sites?
A. Junction of ciliated and squamous epithelium
B. The undersurface of the vocal folds
C. The laryngeal ventricle
D. All of the above

197-Which of the following is currently the most common indication for


tracheotomy in the pediatric population?
A. Providing access for pulmonary toilet
B. Airway obstruction from subglottic stenosis
C. Airway obstruction from craniofacial abnormalities
D. Respiratory failure and ventilator dependence

198-All of the following are appropriate associations on radiologic


evaluations except:
A. Steeple sign-laryngotracheitis
B. Thumbprint sign-epiglottitis
C. Air trapping on chest radiograph-vocal cord immobility
D. Thickened soft tissue overlying C-spine-retropharyngeal
abscess/phlegmon
513

199-The most common presenting symptoms in children with recurrent


respiratory papillomatosis (RRP) are:
A. Dysphagia and poor weight gain
B. Dyspnea with exertion
C. Progressive hoarseness and stridor
D. Odynophagia

200-Which laryngeal site is the most at risk of developing


radionecrosis?
A. Cricoid
B . Thyroid lamina
c. Arytenoid
D . Epiglottis

201-Which of the following salivary cancers is most associated with


pain and perineural spread of tumor?
A. Acinic cell cancer
B. Adenoid cystic cancer
C. High-grade mucoepidermoid cancer
D. Squamous cell cancer

202-Fibrous dysplasia is commonly treated with:


A. Observation
B . Recontouring and esthetic surgery
C. Wide local excision
D. Chemotherapy.

203-The most common acute toxicity observed after oral radiation


therapy are:
A. Mucositis, osteonecrosis, and xerostomia
B . Mucositis, carotid endarteritis, and xerostomia
C. Mucositis, dysphagia, and xerostomia
D. Mucositis, fibrosis, and xerostomia

204-The commonest presenting symptom of nasopharyngeal (NP)


carcinoma is
A. Epistaxis
B. Hearing loss due to serous otitis media
C. Enlarged cervical lymph node
D. Nasal obstruction
514

205-Which of these is the most common presenting symptom( s) for


tracheal adenoid cystic carcinoma?
A. Hemoptysis
B . Dyspnea
c. Fever
D . Dysphagia

206-What is the only absolute contraindication to primary


tracheoesophageal puncture?
A. Obesity
B. Prior radiation
C. Partial pharyngectomy
D. Separation of trachea and esophageal party wall
E. Neck mass E. Abducent nerve palsy.

207-Accurate diagnosis of obstructive sleep apnea {OSA) can best be


made by:
A. A careful history with a complete review of systems
B. Bed partner history
C. Physical examination of the upper airway
D . A home sleep study or an in-lab polysomnogram

208-Clinical outcomes in the assessment of obstructive sleep apnea


(OSA) treatment include:
A. Apnea-hypopnea index
B. Daytime sleepiness
C. Airway collapsibility
D. Oxygen desaturation

209-What is considered the gold standard for treatment of obstructive


sleep apnea in adults?
A. Nasal septoplasty
B. Positive airway pressure therapy
C. Uvulopalatopharyngoplasty
D. Tongue base reduction therapy
515

210-Most cases of sudden sensorineural hearing loss (SSNHL) are


caused by:
A. Temporal bone trauma
B. Genetic predisposition
C. Neurologic disease
D. No identifiable source \ediopathic .

211-Spontaneous hearing recovery after a sudden sensorineural


hearing loss, if it is to occur, occurs within what time frame?
A. 3 days
B. 2 weeks
C. 2 months
D. 6 months

212-Which of the following indications in congenital aural atresia is the


most important determinant of surgical candidacy by CT scan?
A. Degree of mastoid pneumatization
B . Size and position of ossicles
C. Thickness of atretic bone
D. Size of middle ear

213-Which of the following is true regarding the presence of


semicircular canal fistula from erosion by cholesteatoma?
A. Most patients complain of vertigo .
B. The maj ority involve the posterior semicircular canal .
C. Removal of the matrix from a fistula inevitably results in an
anacoustic ear.
D. It is best managed with topical neomycin.
E. In an extensive fistula with a contacted mastoid, a canal-wall-down (
CWD) procedure is best .

214-Which of the following define the superior boundary of the sinus


tympani:
A-Subiculum.
B-Fossula of the cochlear window.
C-Meatal fossa.
D-Ponticulus.
E-Fossaovalis..
516

215-The most common congenital ossicular anomalies is :


A-Stapes anomalies.
B-Malleus anomalies.
C-Incudo-stapedeal joint anomalies.
D-Incudo-malleal joint anomalies.
E-Incus anomalies.

216-Acute sppurative otitis media associated to facial palsy is adequatly


treated with:
A- Myringotomy and systemic AB.
B- Cortical mastoidectomy with systemic AB.
C-Cortical mastiodectomy with facial n. decompression.
D- Myringotomy with facial n.decompression.
E- Systemic AB .

217-The best line of treatment of uncontroled unilateral Meniere’s


disease with sever Vertigo and profound SNHL is ?
A-intrtympanic injection of corticisteroids.
B- intratympanic injection of gentamycin.
C- Labrinthectomy.
D- Vestibular nerve neuroectomy.
E- Ultrasound destruction of semicircular canals.

218-CSF is produced by a process of ultrafiltration mainly from the


following cells:
A-Ependymal cells.
B- Epithelial cells of choroid plexus.
C- Glial cells.
D- Intimal cells of IC. blood vessels.
E- Ventricular supporting cells.

219-The most common cause of unilateral proptosis in adults is:


A-orbital pseudotumour
B-Graves ophthalmopathy
C-paranasal isnus tumours
D-lymphomas
E-meningioma

.
517

220-In salivary gland tumours all the following are true Except
A- The commonest parotid tumour is pleomorphic adenoma.
B- The adenolymphoma (Warthin’s tumour) is commoner in young
women, is painful and grows rapidly.
C- A submandibular tumour is more likely to be malignant than a parotid
tumour.
D- In mucoepidermoid carcinoma, recurrence rates and survival
with histological grade. correlate
E- Distant metastases after many years are characteristic of
adenoidcystic carcinoma.

221-Malignant changes of long standing recurrent pleomorphic


adenoma of the parotid are suspected:

A-If the recurrence is a single module of tumor.


B-If the recurrence is multiple small modules.
C-When the recurrence is in the deep lobe of parotid.
D-When previous surgery was a simple lumpectomy.
E-If patient had previous surgery plus radiotherapy .

222-The most reliable diagnostic tool for lateral sinus thrombosis is:
A-MRI.
B-MRA.
C- CT scan.
D-Echo of high neck.
E-Lumbar puncture

223-The auriculo temporal nerve


A-supplies the skin of the upper 2/3 of the lateral surface of the auricle
B- is a branch of maxillary nerve
C- supplies the middle ear mucosa through the tympanic plexus
D- non of the above

224-The Eustachian tube is opened by contraction of


A- tensor tympani muscle
B-levator palati muscle
C-tensor palati muscle
D-Salpingopharyngeus muscle
518

225-The sensory end-organ of the semicircular canal is :


a- the organ of Corti
b- the macula
c- the crista
d- non of the above .

226-All of the following are diagnostic of tympanic membrane


retraction except
a- fore-shortened handle of malleus
b- prominent lateral process of malleus
c- Schwartz sign
d- distorted cone of light

227-Bezold abscess is a collection of pus :


a- above and in front of the auricle
b-behind the auricle
c-in the upper part of the neck deep to the sternomastoid
d- in the peritonsillar space

228-The most common complication of myringotomy operation is


a- injury of facial nerve
b-dislocation of the incus
c-injury of the jagular bulb
d- residual perforation

229-Intermittent fever with rigors and headach in a patient with


cholesteatma may be due to :
a-otogenic meningitis
b-otogenic brain abscess
c- lateral sinus thrombophlebitis
d-extradural abscess

230-A persistent profuse ear discharge after acute otitis media is


a- cholesteatoma
b- secretory otitis media
c-mastoiditis
d- diffuse otitis externa
519

231-In otitic barotrauma, the following statements are correct except:


a-occurs during airplane ascent
b- occurs during airplane rapid descent
c- can cause rupture of the tympanic membrane
d- occurs during diving

232-The commonest cause of vertigo is


a- meniere’s disease
b-labyrinthitis
c-benign paroxysmal positional vertigo
d-ototoxicity

233-Most cases of extradural abscess of the temporal lobe


a- are asymptomatic and discovered accidentally during mastoidectomy
b- present with persistent ipsilateral temporal headach
c- present with vertigo
d- present with pulsating discharge,hearing loss and tinnitus .

234-The type of hearing loss in otosclerosis may be


a- conductive
b- sensorineural
c-mixed
d-all of the above

235-A 30 years old patient with recurrent attacks of vertigo, hearing


loss and tinnitus associated with nausea and vomiting has
a- benign paroxysmal positional vertigo
b-vestibular neuronitis
c-Meniere’s disease
d-acoustic neuroma

236-In lower motor neurone facial paralysis with intact taste sensation
at the anterior 2/3 of the tongue, the level of the lesion is :
a- in the internal auditory canal
b-in the horizontal tympanic part
c- in the vertical part above the stapes
d- in the stylomastoid foramen
520

237-By modified radical mastoidectomy we mean


a- removal of mastoid air cells and all middle ear contents
c- removal of diseased mastoid air cells
c- removal of mastoid air cells and all middle ear contents with
preservation of healthy remnants of tympanic membrane and ossicles
d- non of the above .

238-Which of the following is associated with objective tinnitus


a-Menière's disease.
b-Ear wax impaction.
c-Acoustic neuroma.
d-Palatal myoclonus.
e-Middle ear effusion

239-THE SECOND COMMONEST CPA MASS LESION IS:


A. GLIOMA
B. ANEURYSM
C. MENINGIOMA
D. EPIDERMOID

240-External auditory canal receives blood supply from all the arteries
except:
A. Posterior auricular
B. Superficial temporal
C. Facial
D. Maxillary

241-Lymphatic drainage of pinna goes to:


A. Parotid node
B. Retroauricular node
C. Superficial cervical node along external jugular vein
D. All of the above .

242-Tegmen tympani is formed by:


A. Petrous part
B. Squamous part
C. Both of the above
D. Mastoid part
521

243-glossopharyngeal nerve enters the middle ear through:


A. Roof
B. Floor
C. Anterior wall
D. Posterior wall

244-Processus cochleariformis contains:


A. Basal turn of cochlea
B. Tensor tympani tendon
C. Stapedius tendon
D. Apex of the cochlea

245-Central axis of cochlea is known as:


A. Spiral lamina
B. Modiolus
C. Processus cochleariformis
D. Crus commune .

246-Which of the following involve the floor of the mouth:


a- Ludwig's angina.
b- Infectious mononucleosis.
c- Peritonsillar abscess.
d- Glossitis.

247-Which is not correct for infectious mononucleosis:


a- it is viral.
b- Affect lymph glands.
c- Confirmed by Paul banner test.
d- There is increase in the count of monocytes.

248-all are present in the medial wall of tympanic cavity except:


a- oval window
b- Round window
c- Pyramid
d-promontory
e-tympanic part of facial nerve
e- There is increase in the count of lymphocytes
522

249-Most benign tumor of frontal sinus


a- fibroma
b- polyp
c- angioma
d- osteoma
e- papilloma .

250-Which of the following is not the site for PARAGANGLIOMA?


a . Carotid bifurcation
b. Jugular foramen
c. Promontory in middle ear
d. Geniculate ganglion

251-Greisinger’s sign means:


A· Pain over the tempromandibular joint
B· Pain in the eye
C· Pain and tenderness over the mastoid
D· Pain and tenderness over the auricle

252-Fossa of Rosenmullar is a common site for:


A· Angiofibroma
B· Lipoma
C· Adenoid
D· Nasopharyngeal carcinoma

253-Early sign of nasopharyngeal carcinoma is:


A· Unilateral secretory otitis media
B· Trotter’s triad
C· Petrositis
D· Nasal obstruction

254-Reactionary bleeding after adenoidectomy is treated by:


A· Removal of the remnants and posterior pack
B· Antibiotics
C· Anterior nasal pack
D· Ligation of the bleeding vessel
523

255-The usual cause of reactionary post-tonsillectomy bleeding is:


A· Slipped ligature
B· Wound infection
C· Tonsillar remnant
D· Early extubation

256-In multiple laryngeal papillomata, all of the following are true


accept:
A· Occurs in children
B· Affects only the glottic area
C· Recurrence is common
D· Best treated by laser surgery

257-Trotter’s triad is a clinical diagnosis of:


A· Nasopharyngeal angiofibroma
B· Nasopharyngeal cyst
C· Petrositis
D· Nasopharyngeal carcinoma

258-Posterior nasal packing is indicated in all of the following situations


except:
A· Severe posterior epistaxis
B· Antro-choanal polyp
C· Reactionary post-adenoidectomy bleeding
D· After removal of nasopharyngeal angiofibroma .

259-Griessinger’s sign is positive in:


A· Acute petrositis
B· Acute sinusitis
C· Acute labyrinthitis
D· Lateral sinus thromophelbitis

260-During total laryngectomy for malignancy, which of the following


maneuvers is not performed?
A. The strap muscles are divided and removed with the larynx.
B. The inferior constrictor is separated from the thyroid cartilage.
C. The vallecula is entered below the hyoid bone.
D. The trachea and esophagus are separated in an areolar plane,
which may beentered superiorly or inferiorly.
E. Tracheal transection occurs at least 2 cm below the tumor
524

261-Total laryngogectomy for laryngeal cancer was first performed in


1873 by
A. Lange (United States).
B. Watson (Scotland).
C. Billroth (Austria).
D. Czerny (Austria).
E. Trotter (England).

262-Pharyngoesophageal reconstruction
A. is usually achievable with skin or dermal grafts.
B. seldom results in the resumption of swallowing.
C. is incompatible with speech rehabilitation.
D. is best accomplished by gastric transposition when the
esophagectomy extends low in the thorax.
E. cannot include neck skin reconstruction

263-Contraindications to total laryngopharyngectomy include


A. laryngeal spread of hypopharyngeal tumors.
B. extrapharyngeal spread.
C. superior mediastinal node metastasis.
D. deep invasion of the vertebral bodies.
E. vocal cord fixation

264-Which of the following has little effect on recorded auditory brain


stem response potentials during acoustic tumor removal?
A. filter types and settings
B. near- versus far-field recordings
C. depth of anesthesia
D. acoustic interference
E. rate of stimulation

265-Before beginning the removal of a cerebellopontine angle tumor,


the functional integrity of the facial nerve monitoring system should be
checked. Which method is preferred?
A. tapping the electrode tip
B. observing the stimulus artifact
C. stimulating the facial nerve in the mastoid segment
D. stimulating the muscle directly
E. measuring electrode impedances
525

266-Which of the following cochlear components in most sensitive to


acoustic overstimulation?
A. inner hair cells
B. outer hair cells
C. spiral ganglion cells
D. dendritic endings of the cochlear nerve

267-The major effect of facial nerve monitoring during acoustic tumor


removal surgery is
A. decreased surgical time.
B. fewer postoperative complications.
C. increased proportion of cases with House-Brackmann grade I or II
results postoperatively
D. decreased incidence of House-Brackmann grade V and VI results
postoperatively.
E. reduced medicolegal risks.

268-Which of the following components of the auditory pathway is


suspected of contributing to the susceptibility of an individual to sound
overexposure effects?
A. cochlear efferent system
B. medial geniculate
C. auditory cortex
D. middle ear

269-The frequency region first affected by habitual exposure to


excessive sound is typically ___ kHz.
A. 0.5 to 2
B. 3 to 6
C. 6 to 8
D. 8 to 10

270-Herpes zoster oticus (Ramsay-Hunt syndrome)


A. is caused by primary varicella zoster virus infection.
B. produces bilateral vertigo and hearing loss mainly in the elderly.
C. produces unilateral facial weakness, ear pain, and occasional vertigo
or hearing loss mainly in the elderly.
D. produces unilateral loss of facial sensation, ear pain, vertigo, and
hearing loss mainly in the elderly.
E. none of the above
526

271-Rubella causes the greatest fetal cochlear damage when the


maternal infection occurs
A. during the first trimester.
B. during the second trimester.
C. during the third trimester.
D. anytime during the pregnancy

272-The modern stapedectomy was first described in 1956 by


A. Toynbee.
B. Rosen.
C. Shea, Jr.
D. House

273-At times early stapes fixation of otosclerosis can be diagnosed


A. by seeing stapes fixation on physical examination.
B. by seeing an abnormality of the tympanic membrane.
C. when a maximum conductive hearing loss is present on audiogram.
D. when an on-off stapedial reflex can be seen on impedance
audiometry

274-What is the most common site of fixation of the stapes?


A. annular ligament
B. posterior crus
C. anterior crus
D. entire footplate

275-The facial recess provides access to the


A. internal auditory canal.
B. retrolabyrinthine air cells.
C. epitympanic space.
D. posterior mesotympanum.
E. mastoid antrum

276-Surface landmarks used for cortical mastoidectomy include the


A. sinodural angle of Citelli.
B. petrosquamous (Korner's) septum.
C. temporal line.
D. scutum
527

277-Which of the following is an advantage of the modified radical


mastoidectomy over the intact wall procedure?
A. the ability to maintain the normal anatomy of the external ear canal
B. the ability to easily identify recurrent cholesteatoma
C. fewer episodes of postoperative otorrhea
D. superior hearing results
E. the fact that long-term follow-up is not necessary .

278-Which of the following is the most significant variable in successful


hearing restoration following ossiculoplasty?
A. size of the mastoid
B. presence or absence of middle-ear mucosa
C. eustachian tube function
D. type of prosthesis used

279-Which of the following is never required in the medial surface


tympanic membrane grafting technique?
A. deepithelialization of the surface of the tympanic membrane
remnant
B. lateralization of a medially rotated malleus handle
C. gelatin sponge packing in the eustachian tube
D. removal of a prominent bony protrusion of the anterior canal wall

280-A maximal conductive hearing loss occurs with which combination


of conditions?
A. ossicles intact, perforation centered over the round window
B. ossicles intact, total perforation
C. ossicular discontinuity, total perforation
D. ossicular discontinuity, intact tympanic membrane.

281-Which of the following temporal bone injuries carries the most


urgent need for surgical exploration?
A. transverse petrosa fracture with severe vertigo and nystagmus
B. longitudinal fracture with cerebrospinal fluid leakage and bleeding
from the external auditory canal C. transverse temporal bone fracture
with immediate complete facial paralysis
D. penetrating tympanic membrane injury to the posterosuperior
quadrant with severe vertigo and nystagmus
E. triangular, trapdoor depressed tympanic membrane perforation
528

282-The most useful radiographic study to evaluate the majority of


patients with trauma to the temporal bone is
A. axial computed tomography.
B. coronal computed tomography.
C. magnetic resonance imaging without contrast enhancement.
D. magnetic resonance imaging with contrast enhancement.
E. digital subtraction angiography .

283-Bezold's abscess results when mastoid infection


A. erodes the outer cortex of the mastoid tip.
B. in the zygomatic air cell erodes through the cortical bone at the
zygoma.
C. perforates the medial aspect of the mastoid tip.
D. perforates the outer cortex at the root of the zygoma into the
mandibular fossa
E. erodes the cortex through the area perforata

284-Which of the following is the poorest choice for first-line


antimicrobial therapy in acute otitis media?
A. erythromycin/sulfisoxazole
B. trimethoprim–sulfamethoxazole
C. clarithromycin or azithromycin
D. ciprofloxacin
E. amoxicillin

285-Infection spreads from the middle-ear cavity by which of the


following pathways?
A. periphlebitis or thrombophlebitis
B. bony erosion
C. direct extension through preexisting pathways
D. all of the above
E. a and c

286-In cholesteatomas, most bone resorption occurs because of


A. pressure necrosis.
B. the action of osteoblasts.
C. the action of osteoclasts.
D. the action of mononuclear phagocytes.
E. alkaline phosphatase
529

287-Patients with aural cholesteatomas most commonly seek medical


advice because of
A. imbalance and vertigo.
B. chronic purulent drainage.
C. conductive hearing loss.
D. pain.
E. facial tics.

288-Children with a history of chronic otitis media with effusion have


mastoids that are
A. well pneumatized and sclerotic.
B. well pneumatized but not sclerotic.
C. poorly pneumatized and sclerotic.
D. poorly pneumatized but not sclerotic.
E. no different than those of normal children

289-Valsalva's and Politzer's tests of eustachian tube function assess


A. active tubal opening function but not tubal patency.
B. active tubal opening function and tubal patency.
C. tubal patency but not active tubal opening function.
D. tubal closing function but not tubal opening function.
E. tubal protective function but not tubal closing function

290-The infant eustachian tube differs developmentally from that in


adults in that the tube in infants is
A. longer.
B. shorter.
C. less compliant.
D. stiffer.
E. more efficient

291-The only muscle that actively opens the eustachian tube is the
A. levator veli palatini.
B. tensor veli palatini.
C. salpingopharyngeus.
D. lateral pterygoid.
E. medial pterygoid.
530

292-Which of the following statements represents a characteristic of


the loudness of tinnitus?
A. Loudness matching with pure tones shows a close relationship to
subjective tinnitus loudness.
B. Loud tinnitus only occurs at pitch-matched frequencies with hearing
loss greater than 30 dB.
C. Loudness frequently fluctuates daily.
D. Parauditory tinnitus is perceived as louder than sensorineural
tinnitus.
E. Distressingly loud tinnitus is more characteristic of middle-aged
individuals than of the elderly

293-An ipsilateral retroauricular thrill in a patient with tinnitus suggests


A. a venous hum.
B. a carotid–cavernous fistula.
C. a glomus jugulare tumor.
D. a glomus vagale tumor.
E. an occipital arteriovenous malformation

294-Which of the following drugs is mostly likely not ototoxic?


A. bisphosphonate
B. vancomycin
C. carboplatin
D. deferoxamine

295-Which of the following drugs has the greatest vestibulotoxicity?


A. neomycin
B. netilmicin
C. streptomycin
D. kanamycin

296-Which of the following medications is known to cause cochlear


outer hair-cell damage?
A. furosemide
B. vancomycin
C. cisplatin
D. erythromycin
531

297-In addition to facial nerve paralysis, the most common finding in


Ramsay-Hunt syndrome is
A. otalgia.
B. internuclear ophthalmoplegia.
C. downbeat nystagmus.
D. conductive hearing loss.
E. pulsatile tinnitus

298-Which of the following statements regarding otomycosis is true?


A. Systemic treatment is necessary.
B. Aspergillus and Candida are the most common responsible genera.
C. Invasive fungal infections are common in immunocompetent
patients.
D. The most common complaint is severe otalgia.
E. It occurs most commonly in diabetic

299-Which of the following statements regarding necrotizing external


otitis are true?
A. Staphylococcus is the predominant pathogen.
B. It occurs only in elderly, diabetic patients.
C. Diagnosis is made by gallium-67, technetium-99m, and high-
resolution computed tomography.
D. Surgical intervention is usually limited to minimal debridement.
E. Mortality is 75%.

300-The optimum surgical approach for the auditory brain stem


implant is the
A. middle cranial fossa.
B. retrosigmoid.
C. suboccipital.
D. transcochlear.
E. translabyrinthine

301-The target region for the auditory brain stem implant is the
A. interstitial nucleus of Cajal.
B. dorsal and ventral cochlear nuclei.
C. zona inserta of the cochlear nerve.
D. roof of the fourth ventricle.
E. superior and medial vestibular nuclei.
532

302-A 30-year-old woman in good health with which of the following


would be the best candidate for an Audiant bone-anchored hearing aid?
A. bilateral sloping moderate to profound sensorineural hearing loss
B. bilateral flat moderate sensorineural hearing loss
C. bilateral 60-dB conductive hearing loss from EAC atresia
D. bilateral moderate CHL from ossicular discontinuity
E. bilateral severe mixed hearing loss from otosclerosis

303-In children 6 to 24 months of age, the technique most commonly


used to assess hearing is
A. behavioral observation audiometry.
B. visual reinforcement audiometry.
C. conditioned play audiometry.
D. conventional audiometry.
E. auditory brain stem response audiometry

304-Facial nerve stimulation during cochlear implant use is most likely


to occur in patients with deafness from
A. large vestibular aqueduct syndrome.
B. aminoglycoside ototoxicity.
C. temporal bone fracture.
D. congenital cytomegalovirus infection.
E. autoimmune inner-ear disease

305-The most accurate information needed for planning reconstruction


of the ossicular chain is provided by
A. tympanometry.
B. computed tomography scanning.
C. pure tone and speech audiometric testing.
D. impedence testing.
E. intraoperative ossicular assessment

306-Which of the following should raise the greatest suspicion that a


CHL on the audiogram is not, in fact, conductive at all?
A. an absent ipsilateral acoustic reflex
B. a word understanding score of 64%
C. a type A tympanogram
D. a type Ad tympanogram
E. a type C tympanogram
533

307-The transmission of sound energy through the middle ear in


humans is influenced most by the
A. areal ratio of the round-to-oval window.
B. ossicular lever ratio.
C. pars flaccida.
D. peripheral third of the pars tensa.
E. central two thirds of the pars tensa.

308-The use of intratympanic aminoglycoside antibiotics for the control


of vertigo in Ménière's disease
A. is beneficial only with complete ablation of ipsilateral peripheral
vestibular function.
B. has a lower incidence of postoperative hearing loss.
C. decreases endolymph volume secondary to toxic effects on the
dark cells of the vestibular labyrinth.
D. is restricted to bilateral disease.
E. is contraindicated in the elderly.

309-Which of the following is thought to be of greatest importance in


the etiology of Ménière's disease?
A. failure of the resorption limb of radial endolymph flow.
B. failure of the resorption of longitudinal endolymph flow.
C. immune complex reactions in the stria vascularis.
D. enlargement of the vestibular aqueduct and endolymphatic sac.
E. decreased endolymph production by the dark cells of the vestibular
labyrinth .

310-Shrapnell's membrane is attached directly to the squama of the


temporal bone at the
A. suprameatal spine.
B. petrosquamous suture.
C. tympanic sulcus.
D. notch of Rivinus.
E. cribriform area

311-The most common cause of inherited congenital deafness is


A. the oculoauriculovertebral spectrum.
B. autosomal recessive inheritance.
C. Stickler syndrome.
D. Usher's syndrome.
E. branchio-otorenal syndrome
534

312-Which test reflects vestibulospinal function?


A. headshake nystagmus
B. electronystagmography
C. rotatory chair testing
D. tandem walking
E. positional testing

313-Thyroid cancer
A. does not occur in multinodular goiter.
B. is frequently undetectable by palpation.
C. constitutes 50% of endocrine malignancies.
D. is found as an occult malignancy in 1% of autopsies.
E. is more frequent than benign thyroid lesions in irradiated thyroid
glands .

314-What is the most common ectopic location for a missing superior


parathyroid gland?
A. intrathymic
B. carotid sheath
C. intrathyroidal
D. superior mediastinal
E. retroesophageal–retropharyngeal area

315-Which pharyngeal pouch gives rise to the superior parathyroid


glands?
A. first
B. second
C. third
D. fourth
E. fifth

316-Inspiratory stridor tends to occur when there is an obstruction


A. above the level of the vocal cords.
B. at the level of the vocal cords.
C. below the level of the vocal cords.
D. in the nasal airway.
E. all of the above
535

317-Otolaryngology patients with reflux disease appear to differ from


most gastroenterology patients in that they
A. are obese.
B. have dysphagia.
C. have heartburn.
D. have regurgitation.
E. have daytime reflux

318-Which test is the most sensitive for the diagnosis of


extraesophageal reflux?
A. barium esophagography
B. esophagoscopy and biopsy
C. radionuclide scanning
D. ambulatory pH monitoring
E. Bernstein acid perfusion test

319-Which technique can be used for cricopharyngeal spasm after total


laryngectomy to improve voice?
A. pharyngeal plexus neurectomy
B. cricopharyngeal myotomy
C. botulinum toxin injection of the pharynx
D. all of the above .

320-The only absolute contraindication to performing a secondary


tracheoesophageal puncture for speech rehabilitation is
A. stomal stenosis.
B. pharyngeal stenosis.
C. decreased vision.
D. manual dexterity.
E. severe chronic obstructive airway disease

321-In rheumatoid arthritis involving the larynx,


A. onset is slow and progressive, without airway emergencies.
B. cricoarytenoid joint arthritis occurs in the majority.
C. laryngeal muscles are unaffected.
D. rheumatoid nodules do not occur.
E. symptoms are often a presenting feature of the disease
536

322-The best indication for use of the Co2 laser in laryngeal surgery is
A. excision of benign vocal fold polyps.
B. the treatment of recurrent respiratory papillomas.
C. vaporization of a vocal fold lesion.
D. excision of an anterior commissure vocal fold carcinoma .

323-Which pharyngoesophageal reconstruction technique results in the


highest rateof postoperative pharyngocutaneous fistula?
A. free jejunal transposition
B. pectoralis major myocutaneous flap
C. gastric pull-up
D. radial forearm free flap
E. deltopectoral flap

324-Which technique is most useful for reconstruction after total


laryngopharyng
-oesophagectomy?
A. free jejunal transposition
B. radial forearm free flap
C. pectoralis major myocutaneous flap
D. lateral thigh fasciocutaneous flap
E. gastric pull-up

325-In treating laryngeal granulomas,


A. it is essential to remove the precipitating factor.
B. speech therapy is usually not required.
C. patients should not be treated for reflux if radiographic studies are
negative.
D. excision with the carbon dioxide laser should be avoided

326-Which of the following statements regarding surgical treatment of


vocal fold cysts is true?
A. Vaporization of the cyst and overlying mucosa with the carbon
dioxide laser is the best treatment.
B. Marsupialization using microlaryngeal scissors is the best treatment.
C. Endoscopic microaspiration of cysts is the easiest and most
effective treatment.
D. Excision preserving the overlying mucosa is the ideal treatment.
E. Rate of recurrence is low regardless of the surgical treatment
537

327-Preepiglottic space involvement by supraglottic carcinoma is


associated with
A. cricoid cartilage involvement.
B. a positive Delphian node.
C. thyroid cartilage involvement.
D. level IV node involvement.
E. bilateral cervical metastases.

328-In treating early vocal fold cancer,


A. carbon dioxide laser excision offers a higher cure rate than does
radiotherapy.
B. carbon dioxide laser excision is more effective than is radiation
therapy if the tumor involves the anterior commissure.
C. radiation therapy is associated with less morbidity than is endoscopic
excision.
D. laser excision should not be attempted in cases of recurrence.
E. endoscopic excision offers a one-session treatment as opposed to
multiple sessions with radiotherapy

329-The best initial treatment of infantile subglottic hemangiomas is


A. carbon dioxide laser vaporization.
B. tracheotomy and open excision
C. conservative treatment with steroids and observation.
D. the Nd:YAG laser.

330-The most sensitive test to assess arytenoid mobility in patients


with unilateal or bilateral vocal fold motion impairment is
A. laryngeal electromyography.
B. flexible fiberoptic laryngoscopy.
C. flow–volume loop and spirometry.
D. dynamic laryngeal computed tomography.
E. direct laryngoscopy and arytenoid palpation .

331-Which of the following is the least acceptable treatment for vocal


nodules?
A. precise vaporization with the microspot Co2 laser set a 0.5 W
B. vocal fold stripping
C. precise removal with forceps and microscissors
D. voice therapy from a speech pathologist
538

332-Which kind of laryngeal cyst has been reported in association with


larynx cancer?
A. anterior saccular
B. lateral saccular
C. epidermoid inclusion
D. mucus retention

333-The best method to assess both the etiology and degree of


aspiration is
A. videolaryngoscopy.
B. pharyngeal manography.
C. scintigraphy.
D. dynamic magnetic resonance imaging.
E. videofluoroscopic swallowing study

334-Vocal nodules
A. often require surgical therapy.
B. always result in dysphonia.
C. are congenital.
D. are synonymous with vocal cord cysts.
E. usually respond to medical and behavioral therapy
335-Optimal results after vocal fold surgery may be best achieved by
including
A. postoperative absolute voice rest.
B. smoking cessation.
C. antireflux therapy.
D. preoperative and postoperative voice therapy.
E. perioperative steroids

336-Which of the following is a contraindication for MRI ?


A. cardiac pacemaker
B. metallic cochlear implant
C. cerebral aneurysm clip
D. all of above

337-Retropharyngeal space enlargement can occur in


A. retropharyngeal abscess.
B. lymphoma.
C. cystic hygroma.
D. all of the above
539

338-Laryngeal stroboscopy is
A. unable to assist in the diagnosis of glottic cancer.
B. unable to reveal vocal scanning effects.
C. a series of light flashes coordinated with fundamental vocal
frequency.
D. very useful in patients with vocal fold paralysis.
E. only able to demonstrate the vertical mucosal wave.

339-A chyle leak can be treated by all of the following except


A. bed rest.
B. continued suction drainage.
C. a long-chain fatty-acid diet.
D. pressure dressings.
E. A chyle leak can be treated by all of the above

340-Which structure is situated between the aryepiglottic fold and the


vestibular fold?
A. conus elasticus
B. saccule
C. triangular membrane
D. ventricle
E. quadrangular membrane

341-Patients with cricothyroid joint arthritis frequently complain of


A. dysphagia.
B. aspiration.
C. hoarseness.
D. odynophagia.
E. neck pain

342-Lymphomas of the head and neck present in all of the following


areas except
A. the cervical nodes.
B. Waldeyer's ring.
C. the thyroid gland.
D. the subdermal layer of skin.
E. the orbit.
540

343-Which of the following lymph node groups is removed in


posterolateral neck dissection?
A. suboccipital
B. perilateral
C. postauricular
D. periparotid
E. a and c

344-Which nerves are routinely preserved in supraomohyoid neck


dissections?
A. motor branches of the cervical plexus
B. greater auricular nerve
C. ansa cervicalis
D. sensory branches of the cervical plexus
E. a, b, and d

345-Neurofibromas differ from schwannomas in that


A. schwannomas are not encapsulated.
B. neurofibromas can be easily resected from the nerve of origin.
C. neurofibromas do not undergo sacomatous transformation.
D. neurofibromas are not encapsulated

346-von Recklinhausen's disease is characterized by


A. autosomal dominant inheritance.
B. more than five café-au-lait spots on the body.
C. positive family history in around 50% of cases.
D. possible association with spinal bifida.
E. all of the above

347-Which nodes are most frequently involved in squamous cell


carcinoma of the upper aerodigestive tract?
A. external jugular
B. upper jugular
C. submandibular
D. supraclavicular
E. preauricular
541

348-The best incision to explore the carotid sheath for a unilateral


penetrating neck injury is a(n)
A. modified Conley incision.
B. lateral cervical incision along the anterior sternocleidomastoid
muscle.
C. MacFee incision.
D. H incision.
E. Schobinger incision

349-Which zone of the neck has the most difficult surgical access?
A. base of the skull region
B. midcervical region
C. lower cervical region below the cricoid
D. posterior neck triangle
E. anterior cervical triangle

350-The anatomic boundaries of zone III of the neck are from the
A. hyoid to the base of the skull.
B. superior border of the thyroid cartilage to the base of the skull.
C. hyoid to the mandible angle.
D. mandible angle to the base of the skull.
E. clavicle to the cricoid

351-The prevertebral fascia envelops all of the following except the


A. vagal nerve.
B. brachial plexus.
C. sympathetic trunk.
D. cervical nerve plexus.
E. phrenic nerve

352-Which of the following surgical principles for dealing with deep


neck infection is false?
A. Establishing an airway is the first priority.
B. Wide debridement of necrotic fascia and skin is necessary in
treating patients with cervical fasciitis.
C. External drainage of all deep neck abscesses is necessary.
D. Normal anatomy is often distorted.
E. Ligation of thrombosed vessels may be necessary
if thrombosis has occurred
542

353-Trismus associated with deep neck infection is most indicative of


A. peritonsillar abscess.
B. parapharyngeal abscess.
C. parotitis.
D. submandibular abscess.
E. infection tracking along the carotid sheath

354-Nasopharyngeal angiofibroma ?
A. is a common tumor of the head and neck.
B. arises from the posterolateral wall of the roof of the nose.
C. is seen only in patients in their teenage years or younger.
D. requires angiography for diagnosis.
E. is unlikely to be resectable through a lateral rhinotomy approach

355-Apnea is defined as a cessation of air flow at the nostrils and


mouth for _ seconds.
A. 5
B. 10
C. 15
D. 20

356-What is the predominant blood supply to the platysmal


myocutaneous flap?
A. submental branch of the facial artery
B. transverse cervical artery
C. platysmal branch of the occipital artery
D. ascending cervical artery

357-Nodal metastases in nasopharyngeal carcinoma


A. are rarely the presenting symptom.
B. indicate a worse prognosis if bilateral.
C. indicate a worse prognosis if low in the neck.
D. are not treated surgically.
E. should be confirmed by open biopsy prior to definitive therapy.
543

358-Nasopharyngeal carcinoma
A. commonly presents as serous otitis media.
B. behaves more aggressively in Chinese populations.
C. has strong human leukocyte antigen associations in non-Chinese
populations.
D. may be excluded on nasopharyngeal examination if the mucosa is
macroscopically normal.
E. invades the skull base in only 10% of cases.

359-Residual disease after resection of nasopharyngeal angiofibroma


A. may remain asymptomatic.
B. has a natural history of relentless progression.
C. will eventually require a secondary form of treatment
D. is often fatal.
E. has a worse outcome in older patients

360-Otalagia may occur via cranial nerve


A. IX, from the glottic larynx.
B. IX, from the anterior lateral tongue.
C. X, from the hard palate.
D. IX, from the retromolar trigone.
E. V, from the oropharynx

361-What is the appropriate treatment for a T1N0 midline soft palate


lesion?
A. complete local excision and no postoperative radiation therapy if
margins are clear
B. complete local excision and postoperative radiation therapy to the
primary lesion if the margins are positive
C. radiation therapy to the primary lesion and both necks
D. radiation therapy to the primary lesion only
E. radiation therapy to the primary lesion and one side of the neck

362-The factor that is least likely to influence prognosis in squamous


cell carcinoma of the oropharynx is
A. tumor size.
B. depth of invasion.
C. histologic differentiation.
D. blood transfusion.
E. decreased proportions of cytotoxic suppressor cells
544

363-Which type of malignant tumor may have a bilateral presentation?


A. mucoepidermoid carcinoma
B. adenoid cystic carcinoma
C. expleomorphic carcinoma
D. adenocarcinoma
E. acinic cell carcinoma

364-What is the most common presenting symptom of a malignant


parotid tumor?
A. facial nerve paralysis
B. skin ulceration
C. nodal metastasis
D. painless swelling
E. pain

365-Which of the following does not influence the timing and


performance of facial reanimation?
A. donor site morbidity
B. proximal and distal nerve integrity
C. age
D. inability to close the eyelid
E. patient expectations

366-What is the most successful approach for repair of most cribriform


cerebrospinal fluid leaks?
A. intranasal
B. external nasal
C. transantral
D. intradural
E. extradural

367-Cerebrospinal fluid leak may be confidently diagnosed by all of the


following means except
A. B2 transferrin.
B. computed tomography cisternography.
C. intrathecal dye injection.
D. intrathecal radio-tracer injection.
E. glucose oxidase test paper
545

368-Which of the following would not explain the development of a


delayed
cerebrospinal fluid leak?
A. delayed increase in intracranial pressure
B. lysis of a clot in areas of dehiscence
C. resolution of edema
D. neural degeneration of olfactory fibers
E. wound maturation

369-The technique of removal of the intrasinus septum of the frontal


sinus is termed
A. the Ridell procedure.
B. the Lothrop (Chaput-Mayer) procedure.
C. Killian's procedure.
D. Lynch frontoethmoidectomy.
E. Kummel-Beck trephination

370-One contraindication to the Caldell-Luc operation is


A. foreign body in the antrum.
B. mucocele.
C. unerupted teeth in children.
D. antral cysts

371-During inspiration the main current of airflow in a normal nasal


cavity is through:
A. Middle part of the cavity in middle meatus in a parabolic curve
B. Lower part of the cavity in the inferior meatus in a parabolic curve
C. Superior part of the cavity in the superior meatus
D. Through olfactory area

372-Which o fthe following is the most common etiological agent in


paranasal sinus mycoses?
a) Aspergillus spp
b) Histoplasma
c) Conidiobolus coronatus
d) Candida albicans
546

373-audiogram in early meniere's disease shows:


a-notch at 2 kHz in bone conduction.
b-notch at 4 kHz in air conduction.
c-a flat curve.
d-a rising curve

374-Melkersson-Rosenthal syndrome includes all except:


a-facial paralysis.
b-fissured tongue.
c-circumoral oedema.
d-sarcoidosis

375-In ramsay hunt syndrome,vesicular eruptions are seen in all the


following areas except?
a-concha.
b-posteromedial surface of pinna.
c-soft palate.
d-tragus and surrounding skin

376-recurrent facial paralysis is seen in all except:


a-acoustic neuroma.
b-diabetes.
c-sarcoidosis.
d-cholesteatoma.

377-landmark used for identification of geniculate ganglion of facial n.


is:
a-oval window.
b-processus cochleariformis.
c-pyramid.
d-digastric ridge
547

378- Commonest complication of CSOM is

a) conductive deafness

b) Meningitis
c) temporal lobe abscess
d) Cholesteotoma

379-Unilateral nasal obstruction in a child is most


often due to
a) foreign body
b) chaonal atresia
c) polyps
d) fungal infection

380- Regarding multiple papillomas in the larynx which of the following


statements are correct:
a) common in infants and children
b) viral in origin
c) vocal cords usual site
d) treatment is removal by direct laryngoscopy
e) All are correct
548

381- Hard elongated swelling in the tonsillar fossa


after tonsillectomy is:

a) thrombosed vein
b) tonsilolitth
c) elongated styloid process
d) calcified esophagus

382-Treatment of choice for otosclerosis is

a) Stepedectomy

b) fenestration

c) Hearing aid

d) taympanoplasty

383- Multi perforations of the tympanic membrane is


characteristic of otitis media due to
a) Staphylococcal

b) Strepta

c) tuberculous

d) Pneumococcal
549

384- Carcinoma larynx with no neck secondaries is


treated by

a) laryngectomy
b) Laryngectomy and radical neck dissection
c) Radiation
d) Radiation & laryngectomy

385- Treatment of cholesteatoma of the middle ear


in a 6 year old
a) Radical mastoidectomy

b) modified radical mastoidectomy


c) simple mastoidectomy

d) None

386- The earliest symptom of acoustic neuroma is

a) deafness

b) Tinnitus

c) Vertigo

d) facial weakness
550

387- Tonsils develop embryologically from

a) First pharyngeal pouch

b) Second pharyngeal pouch

c) Third pharyngeal pouch

d) Fourth phafyrfgea4pouch

388-Lancinating pain around the tonsils during eating


is indicative of:

a) Trigeminal neuralgia

b) Glossophyngeal neuralgia

c) facial neuralgia

d) None of the above

389- Polyp which has a mulberry appearance and


bleeding on touch is due to
a) Malignancy

b) Rhinosporiodisis

c) Antrochoanal polyp

d) Nasopharyngeal angiofibroma
551

390-Which type of diverticulum is common in the


oesophagus

a) traction type
b) pulsion type
c) rolling hernia
d) none

391-Which collagen disease most commonly affects


the esophagus

a) Scleroderma

b) SLE

c) Polyarteritis nodosa

d) Wegners

392- Unsafe otitis media in a child with high fever


and convulsion, the diagnosis is
a) temporal lobe abscess
e) cerebellar abscess
b) meningitis
c) lateral sinus thrombosis
552

393-Treatment of maxillary antral carcinoma (T3 No)


is

a) Only surgery

b) surgery and radiotherapy

c) chemotherapy of radiotherapy

d) radiotherapy only

394-Labyrinthine artery is a branch of


a) Internal carotid artery

b) Basilar artery
c) Posterior cerebellar artery
d) Anterior inferior cerebellar artery

395- Apple-jelly nodules on the nasal septum are


found in cases of:
a) Tuberculosis
b) Lupus vulgaris
c) Syphilis

d) Scleroma
e) None of the above
553

396-Costen's syndrome refers to neuralgia


originating in the:

a) Temporomandibular joint

b) Optic foramen

c) Meatal foramen

d) Sphenopalatine ganglion

397-Areas of carcinoma of oral mucosa can be


identified by staining with:
a) 1 % zinc chloride

b) 2 % silver nitrate

c) Gentian violet blue

d) 2 % toluidine blue

398- Gustatory sweating and flushing (Frey's


syndrome) follows damage to the:
a) Trigeminal nerve
b) Facial nerve
c) Glossopharyngeal nerve
d) Vagus nerve
e) Auriculotemporal nerve
554

399-Hyposthesia of the posterior aspect of the


external auditory canal may be an early sign of:

a) Trigeminal neuralgia
b) Costen’s syndrome

c) Lateral sinus thrombosis

d) Multiple sclerosis
e) Acoustic neuroma

400- Hyposthesia of the posterior aspect of the


external auditory canal may be an early sign of:
a) Trigeminal neuralgia
b) Costen’s syndrome
c) Lateral sinus thrombosis

d) Multiple sclerosis
e) Acoustic neuroma

401- Medication which may prevent rapid progress of


cochlear otosclerosis is
a) steroids
b) antibiotics
c) fluorides
d) vitamins
555

402-Conditions that may cause esophageal strictures include all of the


following except
A. Behçet’s syndrome.
B. gastroesophageal reflux.
C. caustic ingestion.
D. polymyositis.
E. Crohn’s disease.

403-A white boy presents with a history of chronic sinusitis, pulmonary


infection, and malabsorption. Which of the following tests is the most
specific for his disease?
A. sweat chloride
B. c-ANCA
C. erythrocyte sedimentation rate
D. angiotensin converting enzyme level
E. rheumatoid factor

404-Which of the following therapeutic agents can be most


successfully used in the acute management of hereditary angioedema?
A. epinephrine
B. corticosteroid
C. antihistamine
D. C1-INH concentrate
E. attenuated androgen

405-The treatment of choice for verrucous carcinoma is


A. surgical resection followed by chemotherapy.
B. surgical resection followed by radiation therapy.
C. chemotherapy followed by radiation therapy.
D. radiation therapy followed by chemotherapy.
E. surgical resection.

406-A patient presents with bilateral cauliflower ear deformity, saddle


nose deformity, and stridor. Which of the following therapeutic agents
may be useful?
A. cidofovir
B. interferon
C. dapsone
D. attenuated androgens
E. C1-INH concentrate
556

407-The “gold standard” for treatment of advanced Wegener’s


granulomatosis is glucocorticoids and
A. trimethoprim and sulfamethoxazole.
B. methotrexate.
C. quinolone antibiotics.
D. cyclophosphamide.
E. dapsone.

408-The virus most commonly causing mumps (epidemic parotitis) is a


A. coxsackievirus.
B. poxvirus.
C. paramyxovirus.
D. herpesvirus.
E. papovavirus.

409-The fascia covering the parotid and submandibular glands


originates from which layer?
A. the superficial cervical fascia
B. the visceral or middle layer of the cervical fascia
C. the superficial layer of the deep cervical fascia
D. the superficial musculoaponeurotic system
E. the carotid sheath

410-Regional metastases to cervical lymph nodes decrease 5-year


survival rates by
A. 10%.
B. 20%.
C. 50%.
D. 80%.
E. 90%.

411-The most common synchronous secondary primary tumor in


patients with laryngeal carcinoma is
A. floor of the mouth carcinoma.
B. bronchogenic carcinoma.
C. base of the tongue carcinoma.
D. lymphoma.
E. esophageal carcinoma
557

412-Which procedure is not indicated for vocal fold paralysis with


aspiration?
A. tracheostomy
B. laryngectomy
C. arytenoidectomy
D. medialization thyroplasty
E. Teflon injection

413-Isolated laryngeal candidiasis is usually secondary to


A. prolonged antimicrobial use.
B. systemic corticosteroid use.
C. inhaled corticosteroid use.
D. insulin-dependent diabetes.

414-Which section of the tracheobronchial system exhibit seromucous


glands?
A- Segmental bronchi
B- bronchioles
C- bronchioles terminales
D- bronchioles respiratory
E- alveolar ducts

415-Alveolar type II cells contain many lamellar bodies throughout


their cytoplasm containing:
A-Lysosomes
B-Phagocytosed debris
C-Secretory material
.E. infection with human immunodeficiency virus
D-Mitochondria

416-The function of the ciliated epithelial cells of the respiratory


mucosa is:
A-secrete mucus
B-move mucus toward the pharynx
C-move macrophages toward the alveoli
558

417-Concerning the thymus:


A-Lymphoid nodules form thymic cortex.
B-Epithelial reticular cells form Hassal’s corpuscles.
C-T cells migrate into medulla to become immunologically competent
D-Macrophage are essential component of blood thymus barrier.

418-Which of the following is thought to function in preventing


immune response against self antigens:
A-B cells.
B-T memory cells.
C-T helper cells.
D-T suppressor cells.

419-Which of the following are important factors in considering a


temporalis muscle transposition procedure?
a. Airway structure
b. Appearance of nasolabial structures
c. Type of smile on unaffected side
d. b and c
e. All of the above D-are sensory cells

420-The best treatment of postoperative trismus is


a. Temporomandibular joint (TMJ) resection
b. TMJ prosthesis
c. Stretching therapy
d. Laser scar ablation
e. Steroid injection into the pterygoid muscles

421-Sagging of the postero-superior wall of the EAC occurs in:


a. Acute mastoiditis.
b. Furunculosis of the external ear.
c. Otitis media.
d. Petrositis.

422-Gradinigo syndrome occurs in:


a. Acute mastoid abscess.
b. Acute petrositis.
c. Chronic otitis media.
d. Secretory otitis media.
559

423-Tobey-Ayer's test is a characteristic sign in:


a. Brain abscess.
b. Lateral sinus thrombosis.
c. Extradural abscess.
d. Meningitis.
e. Cavernous sinus thrombosis.

424-The fluids presents in secretory otitis media is:


a. Mucopurulent.
b. Serosanguinous.
c. Exudates.
d. Transudates.
e. Mixture of exudates & transudates.

425-Longitudinal fracture of the temporal bone may be associated with


all of the following EXCEPT:
a. LMNL facial palsy.
b. Traumatic perforation of the tympanic membrane.
c. Conductive hearing loss.
d. Profound hearing loss.

426-ABR "Auditory Brain stem Response" is used in:


a. Test of hearing in malingering.
b. Test of hearing in retrochoclear lesion.
c. Detection of acoustic neuroma.
d. All of the above.

427-The most accurate diagnostic test to detect degeneration of the


facial nerve:
a. Nerve excitability test.
b. Electromyography.
c. Electroneurography.
d. Stapedial reflex.
560

428-The most common vertigo is:


a. Acoustic neuroma.
b. Ototoxicity.
c. Meniere's disease.
d. Benign paroxysmal positional vertigo.

429-The commonest organism in malignant otitis externa:


a. Streptococci.
b. Pneumococci.
c. Pseudomonas aeroginosa.
d. Moraxella catarrhalis.

430-Mixed hearing loss may be caused by one of the following:


a. Otosclerosis.
b. Meniere's disease.
c. Ear wax.
d. Acoustic neuroma.

431-Fluctuant SNHL usually occurs in:


a. Presbyacusis.
b. Meniere's disease.
c. Otosclerosis.
d. All of the above.

432-Nystagmus & vertigo induced by pressure on the tragus is a sign


of:
a. Fistula complicating cholesteatoma.
b. Benign paroxysmal vertigo.
c. Vestibular neuritis.
d. Cholesteatoma only.

433-The earliest symptom in a case with cholesteatoma that indicates


intracranial complication is:
a. Persistent headache.
b. Facial palsy.
c. SNHL.
d. Squint.
561

434-A false +ve fistula test is due to:


a. Labyrinthine fistula with dead ear.
b. Cholesteatoma bridging an inner ear fistula.
c. Hyper mobile footplate of the stapes.
d. All of the above.

435-The concept that the facial nerve supplies the auricle is related to:
a. Ramsy-Hunt syndrome.
b. Jugular foramen syndrome.
c. Horner's syndrome.
d. Bell's palsy.

436-Topognostic test is used in the assessment of facial paralysis


include all the following EXCEPT:
a. Schirmer test.
b. Stapedial reflex.
c. Electroneurography.
d. Gustatory test.

437-All are true about rhinoscleroma EXCEPT:


a. It is endemic in Egypt.
b. It is a disease of low immunity & low resistance.
c. It runs in families due to genetic inheritance.
d. It is sub-epithelial inflammatory granuloma.

438-Mikulicz cell is a characteristic histological finding in:


a. Rhinoscleroma.
b. Rhinosporodosis.
c. Aspergillosis.
d. Sarcoidosis.

439-Perforation of bony part of the nasal septum occurs in:


a. Sarcoidosis.
b. Rhinoscleroma.
c. Tuberculosis.
d. Syphilis.
562

440-Spontaneous recovery is usual in:


a. Rhinosporodosis.
b. Sarcoidosis.
c. Tuberculosis.
d. Syphilis.

441-The causative agent of rhinoscleroma is:


a. Sporozoon.
b. Low virulent T.B bacillus.
c. Treponema Ballidum.
d. Gram –ve short capsulated diplobacillus.

442-The causative agent of lupus vulgaris is:


a. Sporozoon.
b. Low virulent T.B bacillus.
c. Treponema Ballidum.
d. Gram –ve short capsulated diplobacillus.

443-All the following lines of treatment could be applied in


rhinoscleroma EXCEPT:
a. Rifampicin.
b. Cytotoxic drugs.
c. Surgery to canalize the stenosed canal.
d. Laser surgery.

444-Unilateral polypoidal mass arising from the lateral wall of the nose
in 55 years old man is most probably:
a. Inverted papilloma.
b. Rhinoscleroma.
c. Allergic nasal polyp.
d. Antrochoanal polyp.

445-Rhinoscleroma characterized by the following EXCEPT:


a. Hard-like nodules.
b. Histopathology shows Mikulicz cells.
c. Responses to rifampicin.
d. Ulceration of the surrounding tissue.
563

446-Resistant epistaxis from below the middle turbinate requires


ligation of:
a. The anterior ethmoidal artery.
b. The sphenopalatine artery.
c. The maxillary artery.
d. The internal jugular vein.

447-Nasopharyngeal carcinoma cause Horner's syndrome as a result of


infiltration of:
a. 3rd cranial nerve.
b. 5th cranial nerve.
c. 7th cranial nerve.
d. Cervical sympathetic chain.

448-Surgery is contraindicated for ca.of maxillary sinus if X-ray refeal


of invovement of:
A-Floor of the orbit.
B-Foramen ovale.
C-Lateral antral wall.
D-Lamina papyracea.
E-Lateral ptyregoid plate

449-The best results in the treatment of advance ca.maxilla are


achieved by:
A-Surgery alone.
B-Radiation alone.
C-Surgery and radiation.
D-Surgery and chemotherpy.

450-In sudden recent unilateral hearing loss with vertigo:


A-Betahestine is the treatment of choice.
B-should be treated as an emergency.
C-Injectable steroid should be given for all patients.
D-The patient should be refered for out patient for investigations.
E-Antimicrobial treatment is of no value.
564

451-Primary atrophic rhinitis is characterized by all of the following


except:
A- Recurrent epistaxis
B- Anosmia
C- Bad odor smelled by the neighbors
D- Affects male only
E-Laser resection

452-Functional endoscopic sinus surgery is the operation of


choice in all of the following except:
A- Chronic sinusitis
B- Mucocele of the paranasal sinuses
C- Nasal polyposis
D-Twisted nose

453-An adult presented with acute onset dysphagia, edema


of the uvula and palate, medially pushed tonsil and trismus is
most likely to have:
A- Parapharyngeal abscess
B- Acute retropharyngeal
C- Quinsy
D- Chronic retropharyngeal abscess

454-The following are signs of chronic tonsillitis except:


A- Enlarged cervical lymph nodes
B- Inequality of the size of the tonsils
C- Pus in the tonsillar crypts
D- Edema of the uvula

455-In multiple laryngeal papillomata, all of the following are


true accept:
A- Occurs in children
B- Affects only the glottic area
C- Recurrence is common
D- Best treated by laser surgery
565

456-T.B of the larynx mostly affects:


A- Posterior part of the larynx
B- Supraglottis
C- Anterior part of the larynx
D- Subglottis

457-Safe type of chronic suppurative otitis media is characterized by:


A- Scantly offensive ear discharge
B- Profuse mucopurulent ear discharge
C- Marginal perforation
D- All of the above

458-Absent laryngeal click may suspect:


A- Supraglottic carcinoma
B- Pyriform fossa tumor
C- Subglottic cancer
D- Post-cricoid carcinoma

459-Myringotomy is indicated in:


A- Secretory otitis media
B- Acute otitis media with bulging drum
C- Otitic barotrauma
D- All of the above

460-The most serious complication following adenotonsillectomy is:


A- Reactionary hemorrhage
B- Respiratory obstruction
C- Respiratory infection
D- Non of the above

461-All the following are signs of chronic tonsillitis except:


A- Inequality of size
B- Congested anterior pillars
C- Presence of crypts over the medial tonsillar surface
D- Enlarged cervical lymph nodes
566

462-Posterior nasal packing is indicated in all of the following situations


except:
A- Severe posterior epistaxis
B- Antro-choanal polyp
C- Reactionary post-adenoidectomy bleeding
D- After removal of nasopharyngeal angiofibroma

463-Shwartz’s sign may be positive in:


A- Chronic suppurative otitis media
B- Atelactatic middle ear
C- Meniere’s disease
D- Otosclerosis

464-Griessinger’s sign is positive in:


A- Acute petrositis
B- Acute sinusitis
C- Acute labyrinthitis
D- Lateral sinus thromophelbitis

465-Recurrent laryngeal nerve is motor to all of the intrinsic muscles of


the larynx except:
A- Vocalis muscle
B- Posterior cricoarytenoid muscle
C- Lateral cricoarytenoid muscle
D- Cricothyroid muscle

466-The only abductor muscle in the larynx is:


A- Sternothyroid muscle
B- Lateral cricoarynoid muscle
C- Cricothyroid muscle
D- Posterior cricoarynoid muscle

467-Cortical mastoidectomy is indicated in:


A- acute mastoiditis not responding to medical treatment
B- Bezold’s abscess
C- mastoid abscess
D- all of the above
567

468-Facial nerve gives secretomotor supply to:


A- parotid gland
B- submandibular salivary gland
C- pituitary gland
D- thyroid gland

469-All the following are possible complications of acute sinusitis


except:
A- temporal lobe brain abscess
B- cavernous sinus thrombosis
C- orbital cellulitis
D- osteomylaitis

470-One of the signs of otogenic cerebellar abscess is:


A- hemiplegia
B- hemianethesia
C- aphasia
D- dysdiodokokainesia

471-Acute pulmonary edema occurring on opening of the trachea


during tracheostomy is due to:
A- sudden drop of pressure in the tracheobronchial tree and lungs
B- sudden wash of carbon dioxide
C- injury of the esophagus
D- non of the above

472-Hump nose is managed by:


A- Reduction rhinoplasty
B- Augmentation rhinoplasty
C- Nasal tip reconstruction
D- All of the above

473-Rhinoscleroma is treated by the following except:


A- Streptomycin
B- Rifampicin
C- Radiotherapy
D- Puncture and lavage
568

474-Apnea immediately after tracheostomy is treated by:


A- Multiple skin incisions
B- Inhalation of oxygen
C- Temporary closure of the tracheostomy opening
D- Intubation

475-Kernig’s sign is present in:


A- Otitic meningitis
B- Otitic brain abscess
C- Otitic hydrocephalus
D- All of the above

476-The main site of ossicular necrosis in chronic suppurative otitis


media is:
A- Incudomalleolar joint
B- Long process of incus
C- Stapedial arches
D- Handle of malleus

477-On tracheostomy, apnea developed after incising the trachea is


due to:
A- Sudden wash of CO2 in the trachea
B- Sudden release of the pressure in the trachea
C- Reflex vagal stimulation
D- Non of the above

478-The most common cause of esophageal stricture in children is:


A- Esophageal carcinoma
B- Diphtheria
C- Achalasia of the cardia
D- Corrosive intake

479-Fossa of Rosenmullar is a common site for:


A- Angiofibroma
B- Lipoma
C- Adenoid
D- Nasopharyngeal carcinoma
569

480-Early sign of nasopharyngeal carcinoma is:


A- Unilateral secretory otitis media
B- Trotter’s triad
C- Petrositis
D- Nasal obstruction

481-All of the following are removed in radical neck dissection EXCEPT:


A) sternocleidomastoid muscle
B) external carotid artery
C) internal jugular vein
D) spinal accessory nerve
E) submaxillary gland

482-Regarding laryngeal cancer, which of the following is FALSE?


A) hoarseness appears early
B) involved nodes are not palpable in 35% of cases
C) distant metastasis appears early
D) direct extension is common
E) it is 90% five-year curable when limited to one cord

483-Which of the following factors is NOT associated with squamous


cell carcinoma
of the larynx?
a) male sex
b) age in fifth and sixth decades
c) history of woodworking
d) large ethanol intake
e) tobacco smoking

484-In LeFort I fractures, the fragment consists of all of the following


EXCEPT
a) upper teeth and palate
b) lower portions of the pterygoid processes
c) portions of the walls of both maxillary antra
d) nasal spine
e) bridge of the nose
570

485-In general, traumatic perforations of the tympanic membrane:


a) are a surgical emergency
b) will heal spontaneously in most cases
c) usually require operative repair
d) require microsurgical repair
e) require a graft for repair

486-The MOST common organism in acute otitis media of older


children and adults is:
a) Staphylococcus
b) Streptococcus
c) Hemophilus influenzae
d) Klebsiella pneumoniae
e) Pseudomonas

487-Small, malignant tumours of the larynx that are intrinsic in origin


and have not spread beyond the larynx are BEST treated by:
a) irradiation
b) laryngofissure
c) total laryngectomy
d) total laryngectomy and radical neck dissection
e) radium needle implants

488-Mixed tumours of the salivary gland:


a) are most common in the submaxillary gland
b) are usually malignant
c) are most common in the parotid gland
d) usually cause facial paralysis
e) are associated with calculi

489-Which of the following is NOT a cause for conductive hearing loss?


a) otitis media
b) otosclerosis
c) noise-induced hearing loss
d) perforation of the tympanic membrane
e) ossicular chain disruption
571

490-Conductive hearing losses are usually reversible. Which of the


following conditions is reversible by surgical treatment?
a) otosclerosis
b) presbycusis
c) sudden hearing loss
d) ototoxicity
e) meningitis

491-What is the BEST treatment for most cases of sensorineural


hearing loss associated with aging (presbycusis)?
a) nothing
b) hearing aid
c) ear trumpet
d) diuretic therapy
e) labyrinthectomy

492-The MOST common benign lesion of the external ear is:


A) melanoma
B) chondrodermatitis nodularis chronicus helicus
C) cerumenoma
D) actinic keratosis
E) exostosis of the canal

493-MOST of the infectious and/or inflammatory diseases involving the


middle ear space are secondary to:
A) ciliary dyskinesia
B) resistant pathogens
C) eutstachian tube dysfunction
D) tobacco abuse
E) allergic diathesis

494-Acute otitis is:


A) a rare condition
B) the most common reason ill children visit the doctor
C) usually not accompanied by pain and fever
D) caused by coliform bacteria
E) treated by placing ventilating tubes
572

495-The MOST common cause for infant stridor, accounting or 60% of


the cases, is:
A) subglottic hemangioma
B) vocal cord paralysis
C) laryngomalacia
D) congenital webs
E) laryngeal cleft

496-What is the MOST common cause of acquired subglottic stenosis?


A) motor vehicle trauma
B) prolonged endotracheal intubation
C) chronic bronchitis
D) tracheoesophageal fistula
E) previous tracheal surgery

497-A 5-year-old child has persistent serous effusions in both ears for
6 months after a routine acute infection. He has a 40-dB condutive
heraring loss in bothears and has been having trouble in school. What
would be the BEST treatment for this child?
A) observe the child for another 3 months
B) prescribe amoxicillin for 10 days
C) recommend hearing aids
D) place ventilating tubes
E) prescribe prophylactic antibiotics for 3 months

498-The following clinical entities are common causes for tinnitus


EXCEPT:
A) high-frequency hearing loss
B) Ménière’s disease
C) ototoxic drugs
D) loud noise exposure
E) acute otitis media

499-Vertigo is very common in all of the following conditions EXCEPT:


A) vestibular neuritis
B) Ménière’s disease
C) presbycusis
D) viral labyrinthitis
E) benign paroxysmal positional vertigo
573

500-The fastest, safest means of establishing a surgical airway is:


A) endoscopic intubation
B) tracheotomy under local anesthesia
C) tracheotomy under general anesthesia
D) cricothyrotomy
E) puncture through the thyroid membrane

501-The most common cause of unilateral proptosis in adults is:


A-orbital pseudotumour
B-Graves ophthalmopathy
C-paranasal isnus tumours
D-lymphomas
E-meningioma

502-The most common cause of unilateral proptosis in children is:


A-dermoid
B-orbital cellulitis
C-rhabdomyosarcoma
D-lymphangioma
E-capillary haemangioma

503-The most common otolaryngologic manifestation of Wegener’s


granulomatosis at onset is:
A-subglottic stenosis
B-septal perforation
C-hearing loss
D-sinusitis
E-otitis media

504-Excision of the vocal cord and a segment of underlying thyroid


cartilage with or without the ipsilateral arytenoid. Which of the
following is the best description of such maneuver?
A-laryngofissure with cordectomy .
B-supracricoid laryngectomy
C-vertical partial laryngectomy .
D-vestibulectomy
E-cordectomy
574

505-The proper placement of a Teflon injection in the vocal fold is:


A-as far lateral as possible
B-in Reinke”s space
C-superior to level of the glottis
D-inferior to the level of the glottis
E-in one mass at the level of the vocal process

506-The most common cause of vocal cord paralysis in the adult is


A-surgical trauma
B-Malignancy
C-neurological disorder
D-Idiopathic
E-infections

507-A patient with gastroesophageal reflux a lesion of the middle


portion of the true vocal cord. This finding most likely suggest:
A-Reinke’s oedema
B-contact ulcer
C-Granuloma
D-Carcinoma
E-glottic stenosis

508-A 28 year old patient is diagnosed as having an intracordal vocal


fold cyst. The operative technique of choice is:
A-needle aspiration
B-steroid injection
C-marsupillization
D-vaporization with CO2 laser
E-excision with mucosal preservation

509-Which of the following vocal cord lesions is most appropriate for


CO2 laser excision?
A- polyp
B- nodule
C-intracordal cyst
D-sulcus vocalis
E-papilloma
575

510-Intubation granuloma can be seen after endotracheal intubation.


On indirect laryngoscopy, the lesion is seen in the following anatomic
position on the vocal cords
A-anterior commisure
B-junction of anterior 1/3, posterior 2/3
C-mid vocal cords
D-posterior 1/3
E-vocal process of the arytenoids

511-A patient referred for evaluation of vertigo has slurred speech.


Speech evaluation reveals ataxic dysarthria. The diagnosis most likely :
A-parkinsonism
B-cerebral palsy
C-brainstem stroke
D-myasthenia gravis
E-multiple sclerosis

512-The most accurate diagnosis of a superior laryngeal nerve


paralysis is made by:
A-- history
B-- physical examination
C- - electroglottography
D- - electromyography
E- - phonoanalysis

513-Bilateral recurrent laryngeal nerve paralysis as the result of


external trauma is most commonly associated with:
A-cricotracheal separation
B-thyroid cartilage fracture
C-arytenoid cartilage dislocation
D-epiglottic prolapse
E-anterior commissure avulsion

514-The sensitivity of a test is defined by which one of the following?


A-true positive/ (true positive + false positive)
B-true positive/ (true positive + false negative)
C-true negatives/ (true negatives + false positive)
D-true negatives/ (true negative + false negative)
E-true positives/ (true negatives + false positives)
576

515-The specificity of a test is defined by which one of the following?


A-true positive/ (true positive + false positive)
B-true positive/ (true positive + false negative)
C-true negatives/ (true negatives + false positive)
D-true negatives/ (true negative + false negative)
E-true positives/ (true negatives + false positives)

516-Of the following complications of parotidectomy, the most common


is
A-Haematoma
B-Seroma
C-flap necrosis
D-infection
E-salivary fistula

517-What is the most characteristic presentation of sarcoidosis?


A- lymphadenopathy
B- facial paralysis
C- airway obstruction
D- characteristic chest X-ray
E- parotid swelling

518-Distant metastasis are most likely to occur with which of the


following parotid malignancy?
A-acinous cell carcinoma
B-malignant oncocytoma
C-high grade mucoepidermoid carcinoma
D-adenocarcinoma
E-adenoid cystic carcinoma

519-Which of the following factors increases the likelihood of


pleomorphic adenoma recurrence following tumour resection and
postoperative radiation therapy for recurrent tumour?
A-patient age
B-presence of gross tumour following surgical resection
C-total delivered radiation dose
D-number of previous recurrences
E-number of radiation fractions per day
577

520-Treatment of recurrent pleomorphic adenoma of the parotid


should include:
A-radiation therapy
B-tumour enucleation
C-observation until the tumour becomes symptomatic
D-tumour resection, removal of residual gland and facial nerve
preservation
E-enbloc resection of the tumour with surrounding tissue including the
facial nerve with immediate grafting

521-The presence of lung metastases in a patient with a history of


adenoid cystic carcinoma of the head and neck is associated with:
A-an average survival of less than one year
B-an average survival of greater than 2 years
C-an average survival shorter than with metasteses in other
organs
D-long term cure with a regimen of chemotherapy and radiartion
therapy
E-regional lymph node disease

522-Of the following, the most common primary sarcoma of the major
salivary glands is:
A-malignant fibrous histiocytoma
B-leiomyosarcoma
C-neurosarcoma
D-angiosarcoma
E-osteosarcoma

523-The most common therapeutic option in the management


of patients with carcinoma of unknown primary is:
A-radiotherapy
B-modified neck dissection
C-chemoradiotherapy
D-radiotherapy with salvage surgery
E-selective neck dissection
578

524-Of biopsy-proven granulomatous inflammation of a cervical lymph


node in a child, the most common cause is:
A-Mycobacterium tuberculosis
B-S. aureus
C-anaerobes
D-cat-scratch disease
E-non-tuberculous mycobacteria

525-A barium swallow of a patient who experienced a sensation of


food sticking, heartburn and episodic choking reveals a marked
cricopharyngeal impression. Which of the following steps would be
most helpful in making a diagnosis?
A-Upper oesophageal sphincter manometry
B-24-hour ambulatory pH monitoring
C-Pharyngeal plexus block
D-Trial of H2 blockers
E-oesophagoscopy
526-What is the correct treatment of an asymptomatic Zenker’s
diverticulum detected accidentally on a barium study?
A-diverticulectomy
B-diverticulopexy
C-observation
D-cricopharyngeal myotomy
E-Dohlman procedure with CO2 laser

527-A 51-year-old male is sent for a barium swallow &the report comes
back as a peristalsis, esophageal dilatation, and failure of the
lower oesophageal sphincter to relax with retention of ingested
material.This patient probably has:
A-achalasia of the cardia
B- diffuse esophageal spasm
C- oesophageal carcinoma
D-pharyngeal pouch
E- trachea-oesophageal fistula

528- The most common cause of a platelet-related clotting disorder is


A. Idiopathic thrombocytopenic purpura.
B. Megaloblastic anemia
C. Von Willebrand’s disease
D. Aspirin therapy
579

529-The inner ear has become damaged in the following diseases


except
A. Autoimmune inner ear disease
B. Relapsing polychondritis
C. Mixed connective tissue disease.
D. Polyarteritis nodosa

530-Which of the following lung capacities cannot be measured by a


spirometer?
A. Total lung capacity
B. Inspiratory capacity
C. Vital capacity
D. Functional residual capacity.

531-Which of the following statements is false?


A. OSAS has no effect on intellectual performance.
B. OSAS is more common in males than females.
C. OSAS is associated with increased incidence of myocardial infarction
and stroke.
D. Diminished pharyngeal muscle tone will aggravate symptoms of
snoring and sleep apnea.
E. The most frequently occurring daytime symptom of OSAS is
sleepiness.

532-All of the following are amide local anesthetics except


A. Lidocaine
B. Bupivacaine
C. Ropivacaine
D. Tetracaine.
E. Mepivacaine

533-Which of the following drugs is most likely to cause permanent


hearing loss?
A. erythromycin
B. quinine
C. cisplatin.
D. penicillin G
580

534-In an otherwise healthy patient with advanced laryngeal cancer,


the treatment regimen that is most likely to result in preservation of the
larynx at 2 years is
A. concurrent chemotherapy and radiation therapy
B. induction chemotherapy followed by radiation.
C. radiation followed by adjuvant chemotherapy
D. total laryngectomy followed by radiation
E. radiation alone

535-An example of a gene identified as playing a role in the molecular


generation of head and neck cancer is
A. Tumor necrosis factor _ (TNF-_)
B. Tubulin
C. p53
D. DNA polymerase

536-Which of the following is an infectious etiology of


laryngotracheobronchitis, or “croup”?
A. Haemophilus influenzae
B. Staphylococcus aureus
C. Branhamella catarrhalis
D. Parainfluenza type 3
E. Varicella

537-The narrowest portion of an infant’s upper airway is


A. The carina
B. The vocal cords
C. The subglottic space
D. The nasal airway
E. The pharynx

538-A patient is noted to have a draining sinus tract deep in the


external canal. Surgical resection will include a dissection of
A. The jugular vein
B. The superficial temporal artery
C. The hyoid bone
D. The stapes
E. The facial nerve
581

539-The central portion of the hyoid bone should be removed when


removing a
A. Dermoid
B. Thyroglossal duct cyst
C. Thyroid nodule
D. Branchial cleft cyst
E. Thymic cyst
540-The removal of a second branchial cleft sinus tract may require
removal of
A. The thyroid gland
B. The tonsil
C. The parotid gland
D. The hypoglossal nerve
E. The hyoid bone

541-Which of the following types of radiographic evaluations may be


used to evaluate patients with airway noise?
A. Barium swallow study
B. CT
C. MRI
D. Ultrasound
E. All of the above
542-The p53 gene encodes for a protein that
A. Enhances hearing acuity
B. Increases tumor metastases
C. Arrests the cell cycle at G1 in a cell with damaged DNA
D. All of the above

543-Usher’s syndrome type 1 affects


A. The visual system only
B. The auditory system only
C. The vestibular system only
D. All of the above

544-The dorsal (pars superior) portion of the otocyst forms


A. The cochlea
B. The bony portion of the labyrinth
C. The semicircular ducts and the utricle
D. The ossicular chain
E. The tympanic membrane
582

545-Rubella infection of the inner ear can result in


A. Scheibe-type dysplasia of the cochlea and saccule
B. Agenesis of the stria vascularis
C. Malformation of the anterior crista
D. Agenesis of a cochlear duct
E. Duplication of the lateral semicircular duct

546-Von Hippel-Lindau disease may cause otologic symptoms


mimicking
A. Meniere’s disease
B. Benign positional vertigo
C. Presbycusis
D. Sudden idiopathic deafness

547-The appropriate behavioral assessment procedure for a typically


developing 12-month-old infant is
A. Visual reinforcement audiometry
B. Play audiometry
C. Immittance audiometry
D. Otoacoustic emissions

548-For evaluation of tinnitus, the optimal imaging modality is


A. Computed tomographic (CT) scan with intravenous (IV) contrast
B. Positron emission tomographic (PET) scanning
C. Angiography
D. T2-weighted magnetic resonance imaging (MRI)
E. MRI with IV contrast

549-Temporal bone trauma causing facial nerve paralysis is best


evaluated by
A. Surgical exploration
B. CT scan with IV contrast
C. MRI
D. CT scan without contrast high resolution
550-Penetration of which structure allows infection to spread from the
paranasal sinuses to the orbit?
A. Basal lamella
B. Fovea ethmoidalis
C. Uncinate process
D. Lamina papyracea
583

551-What is a postnatal derivative of the first branchial cleft?


A. The footplate of the stapes
B. The external acoustic meatus
C. The cervical sinus (of His)
D. The incudomalleolar joint
E. The middle ear cavity

552-What is the nerve that innervates the muscle derived from the
third branchial arch?
A. The trigeminal nerve
B. Cranial nerve VII
C. The glossopharyngeal nerve
D. The hypoglossal nerve
E. The recurrent laryngeal branch of the vagus nerve

553-Most acoustic neuromas arise from which of the following nerves?


A. Superior vestibular nerve
B. Acoustic portion of the eighth nerve
C. Inferior vestibular nerve
D. Nervus intermedius

554-Which of the following is not a contraindication for a patient to


undergo magnetic resonance imaging (MRI) scanning?
A. Titanium reconstruction plates
B. Cardiac pacemaker
C. Intracranial aneurysm clip
D. Cochlear implant

555-The best study to evaluate for metastatic nodal disease in the


neck is
A. Ultrasound
B. MRI scan
C. Computed tomographic (CT) scan
D. Plain films
584

556-The most specific sign of metastatic disease within a lymph node


is
A. Central necrosis
B. Oblong shape
C. Enhancement
D. High signal on T1-weighted image

557-Laryngeal stroboscopy is the least likely to benefit patients who


have:
A. spasticity
B. nonvibrating segment
C. intracordal mass lesion
D. early vocal cord cancer
E. vocal cord at different level

558-a patient with Gastroesophageal reflux has a lesion of the middle


portion of the true vocal cord. This finding most likely suggests:
A. reinke’s edema
B.glottic stenosis
C. contact ulcer
D. granuloma
E. carcinoma

559-In patients with spastic dysphonia, botulinum toxin works by:


A. an unkown mechanism.
B. preventing calcium-dependent release of acetylcholine.
C. increasing calcium-dependent release of acetylcholine
D. increasing level of dopamine in the centeral nervous system.
E. decreasing level of dopamine in the centeral nervous system

560-One stage laryngotracheoplasty for repair of subglottic stenosis in


children avoids all of the following postoperative sequelae except
A. multiple procedures
B. ongoing tracheotomy care
C. need for prologed antibiotic therapy
D. problems of prolonged indwelling laryngeal stent
E. pulmonary atlectasis and neuromuscular weakness
585

561-Increased strain in muscles used for vocalization may be


associated with all of the following except:
A. asthma
B. bruxism
C. beta adrenergic blockade
D. ventral abdominal hernia
E. sensorineural hearing loss

562-Which of the following anatomic sites of the respiratory tract is at


least risk for the development of recurrent papillomatosis?
A. mid area of the laryngeal surface of the epiglottis
B. nasopharyngeal surface of the soft palate
C. midthoracic tracheobronchial tree
D. false vocal cords
E. true vocal cords

563-Which of the following factors is the most associated with a


decreased likelihood of complication with bronchoscopic removal of a
foreign body?
A. type of foreign body
B. location of foreign body
C. use of telescopic -guided forceps
D. administration of halothane anesthesia
E. bronchoscopic experience of the surgeon

564-A patient undergoing videostroboscopy has normal vocal fold


mucosal waves. This finding is most consistent with a diagnosis of:
A. tuberculosis of the larynx
B. cricoarytenoid joint ankylosis
C. neurogenic vocal fold paralysis
D. submucosal fibrosis of the vocal fold
E. invasive squamous cell carcinoma of the vocal fold

565-A patient referred for evaluation of vertigo has slurred speech.


Speech evaluation reveals ataxic dysarthria. The diagnosis most likely
is:
A. Parkinsonism
B. cerebral palsy
C. brainstem stroke
D. multiple sclerosis
E. myasthenia gravis
587

571-Which is of the following antihistamines has the longest half-life?


A. astemazole
B. loratidine
C. terfenadine
D. diphenhydramine
E. chlorpheniramine

572-Which of the following symptoms is least likely to be associated


with a cluster headache?
A. Unilateral nasal congestion and rhinorrhea
B. reddening and tearing of one eye
C. sever uniocular pain
D. extreme restlessness
E. nausia and vomiting

573-the most common site of occurrence of nasal polyps is the:


A. nasal mucosa
B. anterior ethemoid
C. posterior ethemoid
D. inferior turbinate
E. osteomeatal region

574-patients who have the highest serum level of bacteria specific IgE
are likely to have:
A. asthma
B. nasal polyosis
C. allergic rhinitis
D. atrophic rhinitis
E. sphenoid sinusitis

575-After endoscopic sinus surgery significant postoperative bleeding


following removal of packing is usually best controlled by:
A. cautery
B. embolization
C. turbinate resection
D. repaking the nose
E. transantral maxillary artery ligation
588

576-when injury to the lacrimal system is suspected during endoscopic


sinus surgery, the surgeon should :
A. immediatly perform a dacrocystogram
B. perform an external dacrocystorhinostomy
C. perform an endoscopic dacrocystorhinostomy
D. place silastic tubing stents in the lacrimal drainage system
E. completes the procedure, taking care to avoid additional injury

577-Intranasal corticosteroid treatment of allergic rhinitis may have all


of the following effects except:
A. decrease rhinorrhea
B. decrease nasal congestion
C. decrease subjective symptoms of asthma
D. decrease bronchial sensitivity to metacholine challenge
E. increased peak expiratory flow rates

578-the most common cause of recurrent epistaxis after diagnostic


angiography and embolization of the internal maxillary artery is:
A. incomplete embolization
B. recanalization of the artery
C. systemic bleeding diathesis
D. inability to locate the bleeding site
E. continued bleeding from the ethmoid arteries.

579-the most important test in the definitive diagnosis of a 4 year old


child with nasal polyposis and frequent refractory sinusitis is:
A. sweat chloride
B. biopsy of polyp
C. HIV antibody titer
D. pulmonary function
E. quantitative immunoglobulin
589

580-Absolute bone conduction is reduced in

(a) Otosclerosis

(b) Otitis media with effusion

(c) Endolymphatic hydrops

(d) Traumatic ossicular dislocation

581-Most useful test to diagnose ossicular discontinuity is


(a) Evoked response audiometry

(b) Impedance audiometry

(c) Pure-tone audiometry

(d) Rinne test


590

582-Which operation is indicated in coalescent


mastoiditis?
(a) Schwartz mastoidectomy
(b) Bondy's operation
(c) Fenestration operation
(d) Combined approach

583-Blue drum is seen in


(a) Acute otitis media
(b) Congenital cholesteatoma
(c) Glomus tumour
(d) Tympanosclerosis

584-Trismus in peritonsillar abscess is due to spasm of


(a) Superior constrictor muscle

(b) Medial pterygoid

(c) Masseter

(d) Temporalis muscle


591

585-Body of incus is lodged in


(a) Epitympanum
(b) Mesotympanum
(c) Hypotympanum
(d) Protympanum

586-Rehabilitation of the potential for voice after


laryngectomy includes all except
(a) Electrolarynx
(b) Tracheo-oesophageal puncture and voice
prosthesis
(c) Oesophageal voice
(d) Tracheostomy tube with a speaking valve

587-Facial nerve supplies all of the following muscles


except
(a) Stapedius

(b) Anterior belly of digastric


(c) Buccinator
(d) Stylohyoid
592

588-Trautmann's triangle is a landmark for


(a) Mastoid antrum
(b) Facial recess
(c) Posterior cranial fossa
(d) Endolymphatic sac

589-Best pre-operative measure to reduce blood


loss at surgery in angiofibroma is
(a) Hormonal therapy
(b) Pre-operative radiotherapy
(c) Embolisation
(d) Cryotherapy

590-Type of tympanoplasty in which graft is placed


directly on the head of stapes is called
(a) Type II
(b) Type III
(c) Type IV
(d) Type V
593

591-Preferred incision for a radical neck dissection


in an irradiated patient is
(a) Haynes-Martin
(b) Mc Fee
(c) Schobinger
(d) Sorenson

592-Which of the following is primarily affected


in Meniere's disease?
(a) Middle ear
(b) Cochlea
(c) VIIIth cranial nerve
(d) Tubal function

593-Which of the following sites is the origin of


laryngocele
(a) Aryepiglottic fold
(b) Laryngeal saccule
(c) Ventricle of Morgagni
(d) Vestibular fold
594

594-Gradenigo's syndrome is characterized by all


of the following except
(a) Retrobulbar pain

(b) Paralysis CN VI

(c) Spread of middle ear infection to petrous


apex
(d) Cavernous sinus thrombosis

595-Bilateral abductor paralysis is treated by


(a) Thyroplasty type I

(b) Injection of gelfoam into the cord

(c) Lateralisation of arytenoid

(d) Injection of fat into the cord

596-Mickulitz cell is seen in

(a) Rhinoscleroma

(b) Rhinosporidiosis

(c) Salivary gland disease

(d) Thyroiditis
595

597-Decreased lacrimation on Schirmer's test


indicates a lesion of CN VII
(a) Proximal to geniculate ganglion

(b) In tympanic segment

(c) Vertical segment above the nerve to


stapedius
(d) Vertical segment above the origin of chorda
tympani

598-Meniere's disease is characterised by all


of the following except
(a) Low-pitched tinnitus

(b) Stabbing pain in the ear

(c) Intolerance to loud sounds

(d) Positive Rinne test

599-A 40-year-old man presented with severe attacks


of vertigo. He had history of upper respiratory infection
a week before. No complaints of hearing loss, tinnitus
or facial paralysis. Likely diagnosis in this case is
(a) Benign paroxysmal positional vertigo
(b) Vestibular neuronitis
(c) Meniere's disease
(d) Serous labyrinthitis
596

600-surgical emphysema after trachestomy is corrected by:


a)taking more stitches of the wound
b)cold compresses
c)widening of the wound by removal of some stitches
d)antihistaminic intake.

601-60 years old male smoker presents with a unilateral, asymmetric


nodular neck mass. gave him a 2 week trial of antibiotics without
effect.What should be done first?
A – patient should be followed for an additional 2 weeks
B – physical exam followed by FNA of the node
C – open incisional biopsy
D – schedule patient for modified neck dissection.

602-If neck biopsy occurs prior to definitive treatment which of the


outcomes are worse with metastatic cervical carcinoma?
A-wound necrosis
B- regional neck recurrence
C- distant metastasis
D- all of the above

603.During general anesthesia for a parathyroidectomy for relief of


hyperparathyroidism, you should monitor the ECG closely for?
A Prolonged QT Interval
B. Shortened QT interval
C. Shortened PR Interval
D. None of the above

604.You are performing a general anesthetic for a patient undergoing


dissection of the lower neck for malignancy when you notice a 20% drop
in oxygen saturation, diminished ECG amplitude, a drop in blood
pressure, and diminished breath sounds. Based on these findings, you
suspect?
A. Inadvertent migration of the endotracheal out of the trachea and into
the pharynx
B. Pulmonary embolus
C. Pneumothorax
D. Malfunction of the anesthesia monitors
597

605. Which induction agent is the best choice for the moderate
hypothyroid patient?
A. Propofol
B. Thiopental
C. Etomidate
D. ketamine.

606-The histopathologic finding associated with the greatest degree of


residual/ recurrent conductive hearing loss after stapedotomy is:
A-Resorptive osteitis of the incus.
B-Prosthesis abutting the bony margin of the oval window.
C-New bone formation in the oval window.
D-Presence of the prosthesis lying on a footplate fragment.
E-Obliteration of the round window by

607-Acute sppurative otitis media associated to facial palsy is adequatly


treated with:
A- Myringotomy and systemic AB.
B- Cortical mastoidectomy with systemic AB.
C-Cortical mastiodectomy with facial n. decompression.
D- Myringotomy with facial n.decompression.
E- Systemic AB.
otosclerosis.

608-Which of the following should raise the greatest suspicion that a


conductive hearing loss on the audiogram is not in fact,conductive at
all?:
A-An absent ipsilateral acoustic reflex.
B- A word understanding score of 64%.
C-Atype A tympanogram.
D- A type Ad tympanogram.
E-Atype C tympanogram.
598

609-The best investigation to confirm the diagnosis of nasopharyngeal


angiofibroma is
A- biosy
B-CT scan
C-carotid angiography
D- non of the above.

610-The best line of treatment of uncontroled unilateral Meniere’s


disease with severe vertigo and profound SNHL is?
A-Intratympanic injection of corticosteroids.
B-Intratympanic injection of gentamycin.
C-Labrinthectomy.
D-Vestibular nerve neurectomy.
E-Ultrasound destruction of the semicircular canals.

611-The most common age when a thyroglossal cyst usually present


is?
A-At birth.
B-5-8 years.
C-8-15 years.
D-15-18 years.
E-More than 18 years.

612. The buccinator muscle .all are true except:


a. is attached to both jaws opposite the molar teeth
b. is supplied by the facial nerve
c. is continuous with the superior constrictor of the pharynx
d. is a muscle of mastication
e. is pierced by the parotid duct opposite the second upper molar tooth.

613-The parotid gland .all are true except:


a. extends behind the temporamandibular joint
b. is separated from the submandibular gland by sphenomandibular
ligament
c. receives secrotomotor fibres which emerge from the brain in the
glossopharyngeal nerve
d. when acutely enlarged results in pain sensation being transmitted
along the great auricular nerve
e. has a palpable duct
599

614-The best line of treatment of an infant with subglottic


harmangioma and mild stridor is?
A-Laser coagulation.
B-Tracheostomy.
C-Systemic steroid.
D-Observation.
E-Surgical excision.

615- What type of frontal cell is characterized by a large single cell


pneumatizing cephalad into the frontal sinus?
A. I
B. II
C. III
D. IV.

616- The Messecklinger approach to sinus surgery is ___ to __


dissection.
A. anterior; posterior
B. posterior; anterior
C. lateral; medial
D. medial; lateral

617-What is the most posterior ethmoid cell?


A. agger nasi.
B. Haller.
C. Onodi.
D.Semon .

618- Which of the following is not a major complication of endoscopic


sinus surgery?
A. orbital hematoma
B. blindness
C. diplopia.
D. periorbital emphysema.
600

619- Which of the following is not a minor complication of endoscopic


sinus surgery?
A. anosmia
B. hyposmia
C. epistaxis
D. periorbital ecchymosis

620-Revision surgery of the paranasal sinus for persistent maxillary


disease may be attributed to any of the following except
A. inadequate removal of the uncinate process.
B. middle turbinate destabilization and lateralization.
C. overwidening of the natural ostium.
D. creation of the posterior fontanelle ostium.
E. refractory rhinitis.

621- One contraindication to the Caldwell-Luc operation is


A. foreign body in the antrum.
B. mucocele.
C. unerupted teeth in children.
D. antral cysts

622- The technique of removal of the intrasinus septum of the frontal


sinus is termed
A. the Ridell procedure.
B. the Lothrop (Chaput-Mayer) procedure.
C. Killian's procedure.
D. Lynch frontoethmoidectomy.
E. Kummel-Beck trephination.

623-The technique of creating a vascularized anterior frontal sinus


table by preserving periosteal integrity when approaching the frontal
sinus is termed
A. the Ridell procedure.
B. the Lothrop (Chaput-Mayer) procedure.
C. frontal osteopathic flap.
D. Lynch frontoethmoidectomy.
E. Killian's procedure.
601

624- Which of the following is not a complication of external


ethmoidectomy?
A. cranial nerve II injury
B. lacrimal sac injury
C. supraorbital nerve injury
D. infraorbital nerve injury
E. cerebrospinal fluid leak.

625-Which component of the nasal septum may be bilaminar?


A. quadrangular cartilage
B. vomer
C. perpendicular plate of the ethmoid
D. none of the above
E. all of the above

626- The nasal septum and which of the following make up the nasal
valve angle?
A. lower lateral cartilages
B. head of the inferior turbinate
C. upper lateral cartilages
D. pyriform aperture
E. nasal bones

627- Which of the following is not a complication of septal surgery?


A. palatal hypesthesia
B. cerebrospinal fluid leak
C. airway obstruction
D. intranasal synechiae
E. All are recognized complications.

628-Which of the following cannot reduce the incidence of septal


perforations?
A. avoidance of the submucous resection
B. closure of mucosal flap tears
C. replacement of morselized cartilage
D. application of tight septal splints and packing
E. quilting suture to coapt mucosal flaps and eliminate dead
space
.
602

629- Blood supply to the cartilaginous nasal septum is directly via the
A. anterior ethmoid artery.
B. posterior ethmoid artery.
C. labial artery.
D. sphenopalatine artery.
E. overlying mucoperichondrium.

630- The risk of sinonasal cancer is increased with exposure to all of


the following except
A. hydrocarbons.
B. wood particles.
C. thorium dioxide.
D. salted fish.

631-What is the commonly accepted point of origin of juvenile


nasopharyngeal angiofibroma?
A. superior/posterolateral nasal cavity
B. nasopharynx
C. posterior nasal septum
D. superoposterior base of the skull

632- Juvenile nasopharyngeal angiofibroma is classically associated


with all of the following except
A. epistaxis.
B. nasal obstruction.
C. retrobulbar pain.
D. adolescent boys

633- Which of the following statements regarding nasal cavity


malignant melanoma is true?
A. The probability of death does not decrease with time as it
does with other tumors.
B. Cure rates are high.
C. Incidence of local recurrence is low.
D. It originates more commonly in the sinuses than in the nasal
cavity.
603

634-What are the histologic features of esthesioneuroblastoma?


A. well-differentiated cells with abundant cytoplasm
B. poorly differentiated cells with multiple mitotic figures
C. undifferentiated small neuroepithelial cells with densely
staining nucleoli and scant cytoplasm
D. granular cells in various stages of differentiation with a high
mitotic rate

635- Which of the following cochlear components in most sensitive to


acoustic overstimulation?
A. inner hair cells
B. outer hair cells
C. spiral ganglion cells
D. dendritic endings of the cochlear nerve

636- The type of hearing loss caused by sound overexposure with the
most insidious nature is
A. temporary threshold shift.
B. acoustic trauma.
C. noise-induced hearing loss.
D. all of the above

637-The frequency region first affected by habitual exposure to


excessive sound is typically ________ kHz.
A. 0.5 to 2
B. 3 to 6
C. 6 to 8
D. 8 to 10

638- Which recently discovered objective measure of cochlear function


promises to be capable of detecting noise-induced hearing loss at an
early stage of development that precedes detection by the pure-tone
audiogram?
A. auditory brain stem response
B. middle-latency evoked response
C. electrocochleography
D. evoked otoacoustic emissions
604

639- Which of the following has been proposed as the mechanism by


which the sensory cell degenerates following sound overexposure?
A. metabolic exhaustion due to overuse
B. activity-induced ischemia
C. ionic poisoning due to microbreaks in the cytoarchitecture of
the organ of Corti
D. all of the above

640-Which of the following components of the auditory pathway is


suspected of contributing to the susceptibility of an individual to sound
overexposure effects?
A. cochlear efferent system
B. medial geniculate
C. auditory cortex
D. middle ear

641- Which of the following agents interacts with loud sounds


synergistically to produce noise-induced hearing loss?
A. some medicinal drugs
B. some industrial chemicals
C. segmental or whole-body vibration
D. all of the above

642- What noise level (in dBA) requires industries to implement a


hearing-conservation program?
A. 80
B. 85
C. 90
D. 100

643-Which entity holds that if one sound contains half as much energy
as a second sound, but lasts twice as long, both sounds are capable of
producing the same amount of damage to the ear?
A. dBA scale
B. linear decibel scale
C. distortion-product otoacoustic emissions
D. equal-energy principle
605

644- What role does the otolaryngologist play in dealing with noise-
induced hearing loss?
A. diagnose the condition
B. counsel patients on preventive measures
C. provide medicolegal testimony in workmen's compensation
cases
D. all of the above

645- What is the most common site of fixation of the stapes?


A. annular ligament
B. posterior crus
C. anterior crus
D. entire footplate

646-At times early stapes fixation of otosclerosis can be diagnosed


A. by seeing stapes fixation on physical examination.
B. by seeing an abnormality of the tympanic membrane.
C. when a maximum conductive hearing loss is present on
audiogram.
D. when an on-off stapedial reflex can be seen on impedance
audiometry

647- The modern stapedectomy was first described in 1956 by


A. Toynbee.
B. Rosen.
C. Shea, Jr.
D. House.

648- Measurement of the distance from the incus to the footplate is


important in order to determine
A. if stapes fixation is the cause of hearing loss.
B. the proper length of prosthesis needed to avoid damage to
the contents of the vestibule.
C. the type of prosthesis required.
D. all of the above
606

649-A major advantage of using a microdrill is that it


A. creates a perfect round 0.7-mm fenestra.
B. is easier to use than a laser.
C. is atraumatic.
D. Any type of prosthesis can be used

650-Which of the following is an advantage of the modified radical


mastoidectomy over the intact wall procedure?
A. the ability to maintain the normal anatomy of the external ear
canal
B. the ability to easily identify recurrent cholesteatoma
C. fewer episodes of postoperative otorrhea
D. superior hearing results
E. the fact that long-term follow-up is not necessary

651-Surface landmarks used for cortical mastoidectomy include the


A. sinodural angle of Citelli.
B. petrosquamous (Korner's) septum.
C. temporal line.
D. scutum.
E. Ponticulus .

652-The tegmen tympani


A. separates the mastoid air cells from the middle fossa.
B. lies just medial the descending facial nerve and is difficult to
visualize directly.
C. is also known as the posterior fossa dural plate.
D. is the mastoid cortex landmark that corresponds to the
mastoid antrum.
E. is an important landmark for locating the facial nerve

653-The facial recess provides access to the


A. internal auditory canal.
B. retrolabyrinthine air cells.
C. epitympanic space.
D. posterior mesotympanum.
E. mastoid antrum
.

.
607

654-Careful opening of the subarcuate cell tract


A. will allow the surgeon to clean out the sinus tympani.
B. is an alternate route to the antrum.
C. allows drainage of the petrous apex.
D. will expose the internal auditory canal.
E. exposes the horizontal facial nerve .

655-Following a properly performed radical mastoidectomy,


A. there should never be any more episodes of otorrhea.
B. there should be a second-stage ossicular reconstruction
within 12 months.
C. it is relatively easy to reconstruct the posterior canal wall.
D. there is a maximum conductive hearing loss.
E. it is impossible to use a hearing aid on that side.

656-The advisability of extracranial-to-intracranial bypass during


extirpative skull-base surgery for lesions involving the internal carotid
artery is best determined by
A. physical findings.
B. anamnestic information.
C. cerebral angiography.
D. magnetic resonance angiography.
E. the xenon–computed tomography balloon occlusion test.

657-Which of the following skull-base surgical approaches provides


direct access to the petrous apex and infralabyrinthine areas?
A. infratemporal
B. midface degloving
C. basal subfrontal
D. anterior craniofacial
E. extended maxillotomy

658-For tumors involving the middle cranial fossa and the more inferior
portions of the infratemporal fossa, the best approach is the
A. lateral facial.
B. lateral transtemporal sphenoid.
C. transparotid.
D. midface degloving.
E. transmandibular.
608

659-For large lesions of the clivus that extend into the parapharyngeal
space, adequate exposure is obtained by
A. transethmoidal–sphenoidotomy.
B. transseptal–sphenoidotomy.
C. lateral rhinotomy.
D. transantral.
E. transoral with mandibulotomy.

660-Good exposure of the ethmoid block is obtained by


A. lateral rhinotomy.
B. transseptal sphenoid.
C. transpalatal.
D. transoral.
E. Mandibulotomy

661-Which of the following is not an indication for open rhinoplasty?


A. severe asymmetry of the lateral bony walls
B. congenital anomalies
C. use of a columellar strut
D. extremely wide bony or cartilaginous hump
E. nasal deformity that is difficult to assess

662-The fascial graft technique


A. is best accomplished through an open rhinoplasty approach.
B. softens implant irregularities.
C. uses temporalis fascia or synthetic sheeting.
D. has minimal donor site morbidity.
E. all of the above.

663-Which of the following statements is false?


A. In a previously operated nose, mucoperiostial flap elevation
should begin at the maxillary crest and proceed upward.
B. Release of septal angulations and fibrous contractions is
important in septal framework surgery.
C. In correction of septal curvature, cartilage wedges are
removed from the concave side of the septum while cartilage
scoring is used on the convex side.
D. Most angulations and deviations of the septum occur at the
bony–cartilaginous junction
609

664-Major characteristics of the unilateral cleft-lip nasal deformity do


not include
A. ipsilateral retrodisplacement of the lower lateral cartilage.
B. lateral and inferior displacement of the ipsilateral alar base.
C. ipsilateral maxillary underdevelopment.
D. ipsilateral domal depression .

665-Which of the following statements regarding spreader grafts is


false?
A. Spreader grafts are placed between the septal cartilage and
the lower lateral cartilages.
B. Spreader grafts may be used unilaterally or bilaterally.
C. Auricular or septal cartilages may be used.
D. Spreader grafts are most easily placed via a closed
rhinoplasty approach.
E. Ideally, spreader grafts should be placed submucosally

666-The relationship between the function of the upper and lower


lateral cartilage in the majority of humans is
A. interlocked scroll.
B. overlap.
C. end-to-end contact.
D. opposed scroll.

667-Reflex lacrimal gland secretion is mediated by parasympathetic


fibers of cranial nerve
A. V.
B. VII.
C. III.
D. IX.

668-The sphenopalatine ganglion is


A. a special sensory afferent.
B. sympathetic.
C. parasympathetic.
D. a somatic sensory afferen
610

669-The nasopalatine nerve supplies sensation to the mucosa of the


A. anterior premaxillary palate.
B. soft palate.
C. lateral nasal passage.
D. anterior cheek.

670-The anatomic boundaries of zone III of the neck are from the
A. hyoid to the base of the skull.
B. superior border of the thyroid cartilage to the base of the skull.
C. hyoid to the mandible angle.
D. mandible angle to the base of the skull.
E. clavicle to the cricoid .

671-Which zone of the neck has the most difficult surgical access?
A. base of the skull region
B. midcervical region
C. lower cervical region below the cricoid
D. posterior neck triangle
E. anterior cervical triangle

672-A 40-year-old man who sustained a stab wound to zone III of the neck is
hemodynamically stable but has an acute hypoglossal nerve paralysis. What is
the next diagnostic step?
A. four-vessel angiography
B. computed tomography
C. magnetic resonance imaging
D. lateral soft tissue of the neck
E. direct laryngoscopy

673-The best incision to explore the carotid sheath for a unilateral penetrating
neck injury is a(n)
A. modified Conley incision.
B. lateral cervical incision along the anterior sternocleidomastoid muscle.
C. MacFee incision.
D. H incision.
E. Schobinger incision

t.
611

674-What injured area is the most commonly missed when evaluating a


penetrating neck injury during endoscopy?
A. esophageal inlet
B. distal cervical esophagus
C. nasopharynx
D. vallecula
E. supraglottic area

675-Recognised complications of acute suppurative otitis media


include except?
A-Temporal bone abscess.
B-Facial nerve palsy.
C-Abducent nerve palsy.
D-Bartholin abscess.
E-Cholesterol granuloma.
612

ANSWERS
1:D 44:B 87:B 130:B 173:B 216:A
2:D 45:C 88:B 131:A 174:C 217:C
3:E 46:B 89:C 132:B 175:D 218:B
4:C 47:B 90:E 133:A 176:A 219:B
5:C 48:A 91:C 134:A 177:D 220:B
6:E 49:B 92:C 135:B 178:B 221:C
7:A 50:B 93:C 136:D 179:D 222:A
8:D 51:A 94:D 137:E 180:A 223:A
9:D 52:A 95:D 138:A 181:D 224:C
10:D 53:A 96:D 139:B 182:A 225:C
11:D 54:D 97:A 140:C 183:B 226:C
12:A 55:A 98:E 141:B 184:D 227:C
13:B 56:C 99:E 142:B 185:B 228: D
14:B 57:C 100:A 143:A 186:C 229:C
15:E 58:C 101:C 144:D 187:A 230:C
16:A 59:C 102:E 145:E 188:B 231:A
17:A 60:C 103:E 146:D 189:C 232:C
18:A 61:B 104:A 147:B 190:B 233:A
19:C 62:C 105:C 148:B 191:D 234:D
20:D 63:D 106:C 149:C 192:C 235:C
21:A 64:D 107:C 150:B 193:B 236:D
22:C 65:E 108:B 151:C 194:B 237:C
23:D 66:A 109:C 152:C 195:E 238:D
24:A 67:D 110:D 153:B 196:D 239:C
25:E 68:B 111:C 154:B 197:D 240:C
26:E 69:C 112:C 155:E 198:C 241:A
27:E 70:C 113:C 156:A 199:C 242:C
28:B 71:A 114:E 157:C 200:C 243:B
29:C 72:C 115:C 158:D 201:B 244:B
30:C 73:D 116:C 159:C 202:A 245:B
31:E 74:C 117:C 160:D 203:C 246:A
32:A 75:E 118:A 161:A 204:C 247:D
33:B 76:C 119:D 162:E 205:B 248:C
34:B 77:A 120:D 163:B 206:D 249;D
35:C 78:D 121:D 164:B 207:D 250:D
36:D 79:C 122:C 165:C 208:B
37:D 80:A 123:A 166:C 209:B
38:E 81:D 124:C 167:A 210:D
39:E 82:A 125:D 168:C 211:B
40:C 83:D 126:D 169:B 212:D
41:D 84:C 127:C 170:C 213:E 613
42:A 85:B 128:B 171:C 214:D
43:A 86:C 129:C 172:E 215:B
251:C 293:E 335:D 377:B 420:C 463:D
252:D 294:B 336:D 378:A 421:A 464:D
253:A 295:C 337:D 379:A 422:B 465:D
254:A 296:C 338:C 380:E 423:B 466:D
255:A 297:A 339:C 381:C 424:E 467:D
256:B 298:B 340:E 382:A 425:D 468:B
257:D 299:D 341:D 383:C 426:D 469:A
258:B 300:E 342:D 384:C 427:C 470:D
259:D 301:B 343:E 385:B 428:D 471:A
260:C 302:C 344:E 386:A 429:C 472:A
261:C 303:B 345:D 387:B 430:A 473:C
262:D 304:C 346:E 388:B 431:B 474:C
263:D 305:E 347:B 389:B 432:A 475:A
264:C 306:B 348:B 390:B 433:A 476:B
265:C 307:E 349:A 391:A 434:C 477:A
266:B 308:C 350:D 392:A 435:A 478:D
267:D 309:B 351:A 393:B 436:C 479:D
268:A 310:D 352:C 394:D 437:C 480:B
269:B 311:B 353:B 395:B 438:A 481:B
270:C 312:D 354:B 396:A 439:D 482:C
271:A 313:B 355:B 397:D 440:B 483:C
272:C 314:E 356:A 398:E 441:D 484:E
273:D 315:D 357:C 399;E 442:B 485:B
274:C 316:A 358:A 400:E 443:B 486:B
275:D 317:E 359:A 401:C 444:A 487:A
276:C 318:D 360:D 402:D 445:D 488:C
277:B 319:D 361:C 403:A 446:B 489:C
278:C 320:E 362:C 404:D 447:D 490:A
279:A 321:B 363:E 405:E 448:B 491:B
280:D 322:B 364:D 406:C 449:C 492:B
281:D 323:E 365:E 407:D 450:B 493:C
282:A 324:E 366:A 408:C 451:D 494:B
283:C 325:A 367;E 409:C 452:D 495:C
284:D 326:D 368:D 410:C 453:A 496:B
285:D 327:E 369:B 411:B 454:D 497:D
286:C 328:E 370:C 412:C 455:B 498:E
287:B 329:C 371:A 413:C 456:A 499:C
288:C 330:E 372:A 414:A 457:B 500:D
289:C 331:B 373:D 415:C 458:D
290:B 332:B 374:D 416:B 459:D
291:B 333:E 375:D 417:B 460:B 614
292:C 334:E 376:D 418:D 461:C
: 419:E 462:B
: :
501:B 544:C 587:B 630:D 673:B :
502:B 545:A 588:C 631:A 674:A
503:D 546:A 589:C 632:C 675:D
504:C 547:A 590:B 633:A
505:A 548:E 591:B 634:C
506:A 549:D 592:B 635:B
507:D 550:D 593:B 636:C
508:E 551:B 594:D 637:B
509:E 552:C 595:C 638:D
510:E 553:A 596:A 639:D
511:E 554:A 597:A 640:A
512:D 555:C 598:B 641:D
513:A 556:A 599:B 642:B
514:B 557:A 600:C 643:D
515:C 558:E 601:B 644:D
516:E 559:B 602:D 645:C
517:D 560:E 603:B 646:D
518:E 561:C 604:C 647:C
519:B 562:C 605:C 648:B
520:D 563:C 606:E 649:A
521:B 564:B 607:A 650:B
522:A 565:D 608:B 651:C
523:B 566:B 609:C 652:A
524:E 467:D 610:C 653:D
525:A 568:C 611:B 654:C
526:C 569:C 612:D 655:D
527:A 570:D 613:B 656:E
528:A 571:A 614:D 657:A
529:C 572:E 615:C 658:C
530:D 573:A 616:A 659:E
531:A 574:B 617:C 660:A
532:D 575:A 618:D 661:C
533:C 576:E 619:A 662:E
534:A 577:D 620:C 663:C
535:C 578:E 621:C 664:C
536:D 579:A 622:B 665:D
537:C 580:C 623:C 666:A
538:E 581:B 624:D 667:B
539:B 582:A 625:B 668:C
540:B 583:C 626:C 669:A
541:E 584:B 627:E 670:D 615
542:C 585:A 628:D 671:A
543:D 586:D 629:E 672:A
616

References

1-Cumming’s otolaryngology ,head and neck


surgery/sixth edition /2015.
2-Cumming’s review of otolaryngology /2017.
3-Bailey's Head and Neck Surgery-
otolaryngology/FIFTH EDITION /2014.
4-Bailey's Head & Neck Surgery-OTOlARYNGOLOGY
/REVIEW /2014.
5-Current Diagnosis & Treatment in Otolaryngology—
Head & Neck Surgery /thired edition / 2012.
6-Scott – Brown otolaryngology /2008.
7-Essential Otolaryngology head and neck
surgery/Tenth edition /2012.
8-Ballenger’s Manual of Otorhinolaryngology Head
and Neck Surgery /2002
9-Viva training in ENT /Preparation for the FRCS(ORL-
HNS) /2013.
10-Lectures of prof.Dr.Ismail Zohdi /ORL-HNS
Department/Cairo university.
11- Image-Based Case Studies in ENT and Head &
Neck Surgery /2013.
12-Doctors Academy online educational resources
/Questions for DOHNS/2015.
13-Brackmann Otologic surgery .3rd edition \2010.
14-Color atlas of otology,from diagnosis to surgery
\1999.

Potrebbero piacerti anche